Indian-history.pdf

  • Uploaded by: Vipendra Singh
  • 0
  • 0
  • March 2021
  • PDF

This document was uploaded by user and they confirmed that they have the permission to share it. If you are author or own the copyright of this book, please report to us by using this DMCA report form. Report DMCA


Overview

Download & View Indian-history.pdf as PDF for free.

More details

  • Words: 118,730
  • Pages: 369
Loading documents preview...
CONTENTS S. No

Topic

Page

1.

Indian History

03

2.

An Introduction to Indian History

03

3.

Indian History in Brief

03

ANCIENT INDIAN HISTORY 4.

A Brief Note on Ancient Indian History

5.

Ancient Indian History MCQs

04 16-153

MEDIEVAL INDIAN HISTORY 6.

A Brief Note on Medieval Indian History

7.

Medieval Indian History MCQs

154 161-217

MODERN INDIAN HISTORY 8.

A Brief Note on Modern Indian History

9.

Modern Indian History MCQs

218 228-336

PREVIOUS YEAR QUESTIONS WITH SOLUTIONS 10.

Previous Year Questions Asked in CDS Examination

337-341

11.

CDS Previous Year Questions – Solution

342-343

12.

Previous Year Questions Asked in NDA Examination

344-354

13

NDA Previous Year Questions – Solution

355-361

14.

Previous Year Questions Asked in AFCAT Examination

362-365

15.

AFCAT Previous Year Questions – Solution

366-368

INDIAN HISTORY "India is the cradle of human race, the birthplace of human speech, the mother of history, the grandmother of legend, and the great grandmother of tradition. Our most valuable and most astrictive materials in the history of man are treasured up in India only! " Mark Twain.

AN INTRODUCTION TO INDIAN HISTORY Indian History is as old as the History of Mankind. Artifacts dating back to as much as 500, 000 years have been found. India's history and culture is ancient and dynamic, spanning back to the beginning of human civilization. Beginning with a mysterious culture along the Indus River and in farming communities in the southern lands of India. The history of India is one punctuated by constant integration with migrating peoples and with the diverse cultures that surround India. Placed in the center of Asia, history in India is a crossroads of cultures from China to Europe, and the most significant Asian connection with the cultures of Africa. India's history is more than just a set of unique developments in a definable process; it is, in many ways, a microcosm of human history itself, a diversity of cultures all impinging on a great people and being reforged into new, syncretic forms. Shown below is the India timeline starting from 3000 BC of ancient Indus valley civilization and Harappa civilization to 1000 AD of Chola Dynasty of ancient history of India. Indian History in Brief: The History of India begins with the birth of the Indus Valley Civilization in such sites as MohenjoDaro, Harappa, and Lothal, and the coming of the Aryans. These two phases are usually described as the pre-Vedic and Vedic periods. It is in the Vedic period that Hinduism first arose: this is the time to which the Vedas are dated. In the fifth century, large parts of India were united under Ashoka. He also converted to Buddhism, and it is in his reign that Buddhism spread to other parts of Asia. It is in the reign of the Mauryas that Hinduism took the shape that fundamentally informs the religion down to the present day. Successor states were more fragmented. Islam first came to India in the eighth century, and by the 11th century had firmly established itself in India as a political force; the North Indian dynasties of the Lodhis, Tughlaqs, and numerous others, whose remains are visible in Delhi and scattered elsewhere around North India, were finally succeeded by the Mughal empire, under which India once again achieved a large measure of political unity. The European presence in India dates to the seventeenth century and it is in the latter part of this century that the Mughal Empire began to disintegrate, paving the way for regional states. In the contest for supremacy, the English emerged 'victors', their rule marked by the conquests at the battlefields of Plassey and Buxar. The Rebellion of 1857-58, which sought to restore Indian supremacy, was crushed; and with the subsequent crowning of Victoria as Empress of India, the incorporation of India into the empire was complete. Successive campaigns had the effect of driving the British out of India in 1947.

3|P ag e

shop.ssbcrack.com

MCQs

INDIAN HISTORY

MCQs

Indian History is devided into following three major divisions: (a) Ancient Indian History (b) Medieval Indian History (c) Modern Indian History

ANCIENT INDIAN HISTORY A Brief Note on Ancient Indian History The History of India begins with the Indus Valley Civilization and the coming of the Aryans. These two phases are generally described as the pre-Vedic and Vedic periods. The earliest literary source that sheds light on India's past is the Rig Veda. It is difficult to date this work with any accuracy on the basis of tradition and ambiguous astronomical information contained in the hymns. It is most likely that Rig Veda was composed between 1,500 B.C. and 1,000 B.C. In the fifth century, large parts of India were united under Ashoka. The 6th Century B.C. was a period of great tumult in India. The kingdom of Magadha, one of the 16 great Janapadas had become paramount over other kingdoms of the Ganges Valley. This period also saw the emergence of various heterodox sects in India. This was the time when Buddhism and Jainism emerged as popular protestant movements to pose a serious challenge to Brahmanic orthodoxy. This period was followed by the Mauryas of whom the most famous was Ashoka the Great. The boundaries of his empire extended from Kashmir and Peshawar in the North and Northwest to Mysore in the South and Orissa in the East - but his fame rests not so much on military conquests as on his celebrated renunciation of war. For the next four hundred years (after the great Mauryas), India remained politically disunited and weak. It was repeatedly raided and plundered by foreigners. Stability was restored by the Guptas. The Gupta age was the period of peace and prosperity and witnessed an unprecedented flowering of art, literature and the sciences. This period also saw the beginning of Hindu temple architecture. After the Guptas there was only a brief afterglow, in the time of Harshavardhana of Kannauj. A Chinese traveler, Huen-tsang visited India from (629 - 645 A.D.) during the reign of Harshavardhana. His account gives us an opportunity to note the changes that had taken place in the lives of the Indian people since the days of the Guptas. PREHISTORIC PERIOD • • • • •

The Prehistoric period of India is known only through reconstructions from archaeological evidence. The name India was derived from the river named Sindhu, which is also known as INDUS. The recently reported artifacts from Bori in Maharashtra suggest the presence of human beings in India around 1.4 million years ago. From their first appearance in around 3000 BC, human beings used stones as tools for various purposes. That’s why that period is called as STONE AGE.

4|P ag e

shop.ssbcrack.com

MCQs

INDIAN HISTORY

MCQs

Stone Age is categorized into 3 divisions. They are given in the table below: Stone Age

Paleolithic or Old Stone Age

Mesolithic or Middle Stone Age

Neolithic or New Stone Age

Period

5,00,000 BC – 8000 BC 8000 BC – 6000 BC

6000 BC – 1000 BC

Tools used

Unpolished rough stones

Polished stone tools like stone axes

Occupation

Hunting, food gathering

Microliths, pointed cresconic blades, scrapers etc are made up of stones Hunting, fishing, food gathering and domesticating cattle

Shelter

Cave Rock

Temporary hut

Mud house

Important Sites

Soan Valley (Punjab) Bhimbetka (Near Bhopal)

Chotta-Nagpur, J&K, Karnataka, Tamilnadu, Andhra River Krishna (South) Pradesh, Megalaya, Bihar etc

Cultivating rice, domesticating cattle and weaving cotton and wool to make clothes, Pottery

Towards the end of the Neolithic age, copper and bronze began to be used as tools; this period is called as Chalcolithic age (1800 BC- 1000 BC). INDUS VALLEY CIVILIZATION (2500 BC – 1750 BC) • • • •

Indus Valley Civilization belongs to the BRONZE AGE and it forms a part of proto-history of India. It was one of the most ancient urban civilizations in the world. This civilization developed along the banks of river Indus. This civilization covered the parts of Sind, Baluchistan, Afghanistan, West Punjab, Gujarat, Uttar Pradesh, Haryana, Rajasthan, Jammu and Kashmir, Punjab and Maharashtra. • Mohenjadaro is the largest and Allahdino is the smallest excavated site in this civilization. Town Planning • Harappan towns were divided into two parts: Citadels: Fortified settlements on the high mounds. • Lower Town: Main residential area. • The streets ran in straight lines crossing one another at right angles. • Houses stood on both sides of the streets. • Houses, drains, wells and bathing platforms were made of Kiln-burnt bricks. • The Great Bath of Mohenjodaro was used for religious bathing. • Mohenjodaro means ‘the mount of the dead’. • Lamp-posts indicate the existence of street lighting. Agriculture • Irrigation-based agriculture was the backbone of the civilization. • Sickle stone blades mounted on wooden sticks were used for cultivation purposes. • Grains were stored in granaries. 5|P ag e

shop.ssbcrack.com

MCQs

INDIAN HISTORY

MCQs

• Wheat and six-row barley; field peas, mustard, sesame seeds, etc. were grown. • People of Indus were the first to produce cotton in the world. Trade • Agriculture, industry and forest produce provided the basis for internal and external trade. • Trade was based on Barter system. Indus people had trade contacts with the Persian Gulf and Mesopotamia. • Transportation: Bullock carts, pack animals and boats were used. • Towns associated with different industries: Daimabad bronze industry, Lothal factory for stone tools and metallic finished goods, Balakot pearl finished goods, bangle and shell industry and Chanhundaro beads and bangles factory. Major imports Materials Source from Gold Kolar (Karnataka), Afghanistan, Persia Silver South India, Afghanistan, Persia Copper Khetri (Rajasthan), Baluchistan, Arabia Tin Central Asia, Afghanistan, Bihar Agates Western India Jade Central Asia Turquoise Iran Amethyst Maharashtra Art and Craft • Bead Making and jewellery of gold and precious stones were made. • Pottery: Bright or dark red, plain and painted pottery was made. • Seals: Seals were made of terracotta and most frequently represented by an animal named Unicorn. • Terracotta figurines: Fire baked clay was used to make toys, animals, birds and both male and female figurines. Religion • The chief male deity of Indus valley civilization was Pasupati Mahadeva (Epithet of Lord Shiva). • The chief female deity of Indus valley civilization was the Mother Goddess (Goddess of fertility). • They also worshipped fire, peepal trees and Unicorn. Burial Practices • Mohenjodaro: Three forms of burial were complete, fractional and post-cremation burials. • The dead body was generally placed in the North-South direction. Political System • It is speculated that the rulers might have been wealthy merchants, or powerful landlords or spiritual leader. Scripts • Scripts were pictographic and written from right to left like modern Urdu. • These scripts are found on various seals, pottery, copper tablets, tools etc.

6|P ag e

shop.ssbcrack.com

MCQs

INDIAN HISTORY

Site Harappa Mohenjodaro Sutkagendor Chanhudaro Rangpur Kalibangan Ropar Lothal Surkotada Banawali

Important Discoveries Discovered by Dayaram Sahni R. D. Banerjee R. L. Staine N. G. Majumdar M. Vats A. Ghosh Y. D. Sharma S. R. Rao I. Joshi R. S. Bisht

MCQs

Year 1921 1922 1927 1931 1953 1953 1955-56 1957 1972-75 1973-74

Decline of the Indus Valley Civilization • After 700 years, the Harappan cities began to decline. • Reason: A shift in the course of the river and natural disasters such as drought, floods, etc or longterm climate change. • Some scientists also believe that the war with the Aryan civilization can also be a cause of their decline. ARYAN AND VEDIC AGE (1500 BC – 1000BC) • • • •

Vedic Age is the period of the Aryans in India. The word ARYAN literally means of high birth. During the early part of the Vedic period, the Indo-Aryans settled on the Indo-Gangetic Plains. The word ‘Veda’ is derived from ‘Vid’ which means mantra, slokas and also knowledge. Vedic Age is divided into the Early Vedic Period and Later Vedic Period.

Early Vedic Period (1500 BC – 1000 BC) • The People of the early Vedic period lived in Sapta Sindva which means land of seven rivers. • The seven rivers are Jhelum, Ravi, Chenab, Beas and Sutlej along with the Indus and Saraswati. Economy • Aryans followed a mixed economy consisting of both plough based agriculture and pastoralism. • The Cow was the standard unit of exchange. Gold coins like Nishka, Krishna and Satmana also were used. Polity • Some of the political terms used by Vedic people were: ➢ Samiti: National Assembly of the people. ➢ Sabha: Assembly of Elders. ➢ Purohita: Religious advisor of the King. ➢ Bali: Voluntary offerings to the chief by the people. ➢ Weapons: Made of stone, wood, bone and metal were used. Society • Jana: People who gave their faithfulness to the tribe (Vis). • Categories of Vis: Grama and Sangrama. • Kula: It means family in Rig Veda. • Varna: Caste of the People based on occupation. The four varnas were: ➢ Brahmins: Teachers and Priests 7|P ag e shop.ssbcrack.com

MCQs

INDIAN HISTORY

MCQs

➢ Kshatriyas: Rulers and Administrators ➢ Vaishyas: Merchants and Bankers ➢ Sudras: Artisans and Labourers. Religion • Nature was worshipped by Vedic Aryans - earth, fire, wind, rain and thunder. • No temples and no idol worship during this period. Economic Condition • Rig Vedic Aryans were pastoral people. • Main occupation: Cattle rearing. They finally settled in Northern India, started agriculture, cleared forests and cultivated land. • Other occupations: Carpentry and spinning. Carpenters made chariots and ploughs made with copper and bronze. • Cotton and woolen fabrics also were made. Rig Vedic Gods • Indra: Aryan warlord and controlled the weather. • Called as Purandhar or the destroyer of efforts. • Varuna: Administrator of cosmic law. • His worship gives signs of Bhakti • Agni: Intermediary between Gods and Men. Lives in domestic hearth. • Yama: A type of Adam. • Soma: God of plants. Special God of Brahmanas. Society • Based on relationship, early Aryans were tribal and democratic. • The Family was ancient and the birth of a son was desirable. The Cows • Most important form of wealth. • Duhitri – word for daughter, meaning one who milks cows. • Goghana – Indicates beef offered to guests. Women’s Position • Respectable position in society. • No child marriage and marriageable age for girls – 16 to 17 years. • There is proof of widow remarriage and the practice of Niyoga. • Monogamy, polygamy and polyandry were also known. Female Deities • Usha- Goddess of dawn • Aditi- Mother of God • Prithivi- Goddess of earth • Aryani- Goddess of forest • Saraswati-River deity

8|P ag e

shop.ssbcrack.com

MCQs

INDIAN HISTORY

MCQs

Important Rituals • Rajasuya- king’s influence was strengthened by rituals. Conferred supreme power on him. • Asvamedha – Ceremony lasted for 30 days and ended with horse sacrifice. • Vajapeya – chariot race was performed by the king. Royal chariot competed against kinsmen in a race. Types of Marriage • Brahma – In same class, a suitable dowered girl to a man. • Daiva- Father gives his daughter to a priest as a part of his fee. • Arsa – Token bride - gives cow and bull as dowry. • Prajapatya - Father gives his girl without any dowry. • Gandharva- Love marriage. • Asura- Marriage in which bride is bought from her father. • Rakshasa - Marriage by warriors. • Paishacha – Marriage by seduction. Vedic Literature Aryans developed the Vedic culture based on Vedas. There are four Vedas in Vedic literature. Vedas Description Rig Veda Oldest one. It Consists of 1028 hymns sung in honour of the Gods. Yajur Veda It consists of detailed rules to be followed at the time of sacrifice. Sama Veda

Atharva Veda

The Book of Songs. It contains a set of melodies for singing during sacrifices It contains the details of rituals of worship

PRE-MAURYAN AGE • Age period is about 300 years from 600 BC to 300 BC. • Age of Buddhism hence called Buddhan age. • Anguttra Nikaya- Source of 16 kingdoms written in Pali language. • Pali or Prakrit was spoken by the common man. • Buddhism adopted Pali and Jainism adopted Prakrit. • During the Medieval period, both languages Pali and Prakrit disappeared. • During the time of Buddha, most important kingdoms were as follows: ➢ Vatsa: Capital Kausambi ➢ Avanti: Capital Ujjayini ➢ Kosala: Capital Ayodhay ➢ Magadha: Capital Rajagriha.

9|P ag e

shop.ssbcrack.com

MCQs

INDIAN HISTORY

MCQs

MAGADHA EMPIRE • • • •

The period of Magadha Empire was 6th century to 4th century BC. Four important royal dynasties – Haryankas of Magadha, Ikshvakus of Kosala, Pauravas of Vatsa and Pradyotas of Avanti. A new dynasty named Haryanka was founded in Magadha by King Bimbisara. By defeating the Brihadrathas, Bimbisara founded the dynasty.

King Bimbisara of Magadha (544 BC-492 BC) • Bimbisara, a contemporary of Buddha. • Rajgir (Giriv Raja) was his capital. Strengthened his position with the ruling families of Kosala, Vaisali and Madra (3 Wives). • Rajgir was impregnable because his capital was surrounded by 5 hills. History Ajatshatru (492 BC-460 BC) • Bimbisara’s son killed Ajatshatru’s father and captured the throne. • Joined Vaisali and Kosala. At that time, Kosala was ruled by Prasenajit. • Buddha died during his rule and thus the first Buddhist council was arranged. History of Udayin (460-444BC) • Udayin founded the Pataliputra, new capital. • Situated at the junction of the Ganga and Son. Shishunaga Dynasty (412-344BC) • It was founded by Minister Shishunaga. • Succeeded by Kalasoka (Second Buddhist Council). • Destruction of power of Avanti was the greatest achievement. • Nandivardhan, was the last ruler of Shishunaga dynasty. Nanda Dynasty (344 – 321 BC) • Considered as the first of the non-Kshatriya dynasties. • Established by Mahapadma Nanda, a powerful empire. • He had the titles of Ekarat and Sarvakshatrantaka. • It had a tremendous army consisting of infantry – 2, 00,000 lakh, cavalry – 2,000, war chariots – 2,000 and war elephants – 3,000. • Dhanananda, was the last ruler of Nanda dynasty. • Dhanananda was contemporary of Alexander. • During Dhanananda’s rule, Alexander invaded India in 326 BC. Persian Invasion • Darius, King of Persia (522-486 BC) captured some of the territories in the east of Sindhu in 218 BC. • The Persian ruler, Xerses enlisted Indians in his army. • Kharoshti script was brought to India by Persians. Alexander’s Invasion • Alexander was born in 356 BC. • He was the son of King Philip II of Macedonia, Epirus or Olympias was his mother and Aristotle was his teacher. 10 | P a g e

shop.ssbcrack.com

MCQs

INDIAN HISTORY

MCQs



In 336 BC, he became King. • He founded the city Alexandria in Egypt. • In 326 BC, he defeated Porus and captured Taxila through the battle of Hydaspes on the Jhelum River. • The ruler of Taxila, Ambhi invited Alexander to India. • At the age of 33, he died due to malaria. • Eudamas, was the last general of Alexander in India. • Selucus Nikator was the first general of Alexander in India. • Alexander IV succeeded Alexander as the king of Macedonia. • His teacher Aristotle is considered the “Father of Politics, Biology, Taxonomy and science of Logic”. JAINISM • Jainism was founded by Rishabhadeva. • There were 24 Tirthankaras. Rishabhadeva was the first Tirthankara. Life of Mahavira • The last Tirthankara was Mahavira. • Mahavira was born in 599 BC, in Kundagram. • Siddhartha, the father of Mahavira was head of Jnatrika clan and his mother Trishala, was a Princess of Lichchavi, Yasoda was his wife and Priyadarshana also called Anojja was his daughter. • He left his home at the age of 30 and visited the court of Bimbisara and Ajatashatru. Way to Nirvana • Right faith • Right knowledge • Right conduct Five main teachings • Non-Injury • Non-lying • Non-stealing • Non-possession • Observe continence Sacred Literature • Ardhamagathi, the sacred literature of the Svetambaras written in a form of Prakrit. • Classified as follows: ➢ The twelve Angas ➢ The twelve Upangas ➢ The ten Parikarnas ➢ The six Chheda-sutras ➢ The four Mulasutras Jain councils History • First Council: At the beginning of 3rd century BC, it was held at Pataliputra by Sthulabhadra. It resulted in the compilation of 12 Angas to replace 14 Purvas. • Second Council: In the 5th century AD, it was held at Vallabhi (Gujarat) by Devridhihgani. It resulted in the compilation of 12 Angas and 12 Upangas. 11 | P a g e

shop.ssbcrack.com

MCQs

INDIAN HISTORY

MCQs

Followers of Jainism • The Digambaras (sky-clad or naked) • The Svetambaras (White-clad) BUDDHISM •

Originated in the 6th century BC. • Founded by Gautama Buddha. Buddha History • Buddha, also known as Siddhartha, Sakyamuni and Tathagata. • He was born in 563 BC, on the Vaishakha Poornima day at Lumbini, near Kapilavastu in Nepal. • His father Suddhodana, the Saka ruler. • His mother Mahamaya of Kosala dynasty, died on the seventh day of his birth. So he was taken care of by stepmother Gautami. • His wife Yoshodhara and his son named Rahula. • At the age of 29, he left his palace and attained Nirvana and Bodh Gaya at the age of 35. • In Sarnath, he delivered his first sermon. • In 483 BC, at Kusinara he attained Mahaparinirvana at the age of 80. Buddhist Councils • First Council: Held in 483 BC at Sattapanni cave near Rajagriha. Divided into two Pitakas as Sutta Pitaka and Vinaya Pitaka. • Second Council: In 383 BC, held at Vaisali under Sabakami (king was Kalasoka). Followers divided into Sthavirmadins and Mahasanghikas. • Third Council: In 250 BC, at Pataliputra by Mogaliputta Tissa (king was Ashoka). Third part of the Tripitaka was coded in the Pali language. • Fourth Council: IN 72 AD, at Kashmir (kundalvan) under Vasumitra (king was Kanishka). Buddhism divided into two sects as Mahayana and Hinayana. Buddhist literature • Tripitakas (Three hold basket) – Buddhist scriptures in Pali language. • Vinaya Pitaka: Rules of Discipline in Buddhist monasteries. • Sutta Pitaka: Largest collection of Buddha’s sermons. • Abhidhamma Pitaka: Explains the philosophical teachings of Buddhist religion. Types of Buddhism • Mahayana: Followers believed in heavenliness of Buddha. They believed in idol worship. • Hinayana: Followers believed in the original teachings of Buddha. They did not believe in idol worship. • Vajrayana: Followers believed the salvation attained by the magical power, which they called Vajra. Popular in Eastern India, particularly Bihar and Bengal. MAURYAN EMPIRE (322-185 BC) •

Mauryan Empire was founded by Chandragupta Maurya with the help of Chanakya, who was the author of Arthashastra. • Chandragupta Maurya defeated King Dhanananda and captured Kingdom Magadha and stated Pataliputra (now as Patna) as the capital. • At that time, the Maurya empire was the largest empire. 12 | P a g e shop.ssbcrack.com

MCQs

• • • • • •

INDIAN HISTORY

MCQs

Seleucus I sent ambassador namely Megasthenes, who was the author of Indika to Chandragupta Maurya’s court. With the help of his son Bindusara he succeeded and Bindusara extended their empire to southern regions. After that, Bindusara followed his son, Asoka (ruled 272-232 BC). Asoka proved himself as one of the best kings in world history. After the brutal war against Kalinga (261 BC), he converted to Buddhism. Brihadratha was the last emperor of the Mauryan Empire.

POST- MAURYAN AGE Sunga Dynasty (185 -71 BC) • Founder – Pushyamitra Sunga. • He was the commander-in-chief of the last Mauryan king Brihadratha. • Malavikagnimitram, Kalidasa’s drama tells about Pushyamitra’s son Agnimitra and Malavika’s love story. • Devabhuti – Last king of Sunga dynasty. Kanva Dynasty (72 BC – 27 BC) • Founder – Vasudeva Kanva in 72 BC • He ruled after defeating the last Sunga ruler. • Ruled for a period for 45 years. • Rulers of Kanva dynasty – Vasudeva, Bhumimitra, Narayana and Susuman. Cheta Dynasty of Kalinga • Founder- Maha Meghavahana. • Hatigumbha inscription of Kharavela gives details about the Chedis of Kalinga. • Kharavela – follower of Jainism. Satavahana Dynasty (235 – 100 BC) • After Mauryans, most powerful ruling dynasty. • Also known as Andhras. • Gautamiputra Satakarni – important ruler. • Satavahanas were Brahmanas. • Chaitya – Two common structures of Satavahanas and monastery called Vihara. • Prakrit – official language. • Issued lead coins. Indo Greeks • They were the first to invade India. • Menander –famous Indo-Greek ruler. • They were the first to issue gold coins in India. • Menander was converted to Buddhism by Nagasena (Buddhist Monk). • Demitrius – king of Bactria invaded India about 190 BC. • Demitrius is considered as Second Alexander. The Parthians (19 – 45 AD) • Known as Pahalavas, Iranian people. • Gondophernes- Greatest Parthian ruler. • St. Thomas came to India for the propagation of Christianity during the reign of Gondophernes. The Sakas (90 BC – 1st AD) • Known as Scythians. 13 | P a g e

shop.ssbcrack.com

MCQs

INDIAN HISTORY

MCQs

• • • •

Maues or Moga - First Saka king in India. Rudra Daman I - Famous Saka ruler in western India. First Inscription – Junagarh Inscription of Rudradaman in Sanskrit. Ujjayini – Rudradharman Capital. Kushans • Known as Yuch-chis or Tocharians. • Came to India from north central Asia. • Kujala Kadphises – First Kushana king. • Kanishka – famous kushana ruler. • He became ruler in 78 AD. • Peshawar – Kanishka’s capital. • ‘The Gandhara School of Art’ received royal patronage. • Kanishka is also known as ‘Second Ashoka’ • He started Saka Era in 78 AD. • Vasudeva – Last king of Kushana dynasty. • Kushana School of art is also called Mathura School. THE GUPTA EMPIRE (320-540 AD) •

Founded by Sri Gupta. • The second ruler was Ghatotkacha. • Gupta Empire is called as “The Golden Age of India” Chandragupta I (319-335 AD) • He was the first important king of Gupta dynasty. • He came to rule in 320 AD. • By marrying Kumara Devi, princess of Lichchavi Clan of Nepal he increased his power and prestige. • He was the first ruler who acquired the title of Maharajadhiraj. Samudragupta’s History (335-375 AD) • In 335 AD, Samudragupta succeeded Chandragupta I. • Harisena, his court poet composed the Allahabad Pillar inscription (known as Prayagaprasati) that contains Samudragupta’s conquests’ information. • V.A. Smith described Samudragupta as “Indian Napoleon” • Samudragupta composed “Vahukabita” and named as “Kaviraja”. • His minister was “Vasubandhu”, a Buddhist scholar. Chandragupta II (380-413 AD) • Samudragupta was succeeded by Ramagupta but Chandragupta II killed him and married his queen. • Chandragupta II was also known as Vikramaditya (Sun of power). • Took the title of Simhavikrama, by defeating Rudrasimha III. • First ruler to issue silver coins. Kumaragupta – I (413-455 AD) • Adopted the title Mahendraditya. • Founded Nalanda University and worshipper of Lord Kartikeya. • He died during the war with Hunas. Skandagupta (455-467 AD) • Kumaragupta I was followed by Skandagupta. 14 | P a g e

shop.ssbcrack.com

MCQs

INDIAN HISTORY

MCQs



Established Sudharsana Lake. • After his death, the powerful days of Guptas came to an end. Gupta Religion • During this time Bhagavad Gita was written. Buddhism decreased. • The last ruler was Vishnugupta, who died in 570 AD. • Vishnu temple at Deogarh and a brick temple at Bhitragaon belong to the architecture of Gupta period. HARSHAVARDHANA’S HISTORY (606-647 AD) • • • • • • • • • •

Harshavardhana – last Hindu emperor of North India. He was the son of Prabhakar Vardhan and belonged to the family of the Pushyabhuti dynasty (Vardhana dynasty). He started Harsha Era in 606 AD. Siladitya was his original name. Punjab, Kannauj, Bengal, Bihar and Orissa were under his control. Hieun Tsang, the Chinese traveler visited India during his rule. At the end of every five years, he celebrated a solemn festival at Prayag (Allahabad). He was a poet and dramatist. His biography “Harshacharita” was written by Banabhatta, his court poet. He also wrote “Kadambari”.

Harsha is the author of three plays Priyadharshika, Nagananda and Ratnavali. After his death in 647, once again the empire was divided into small states. SANGAM AGE • Sangam age is called the Golden period, which began in 300 BC and lasted till 300 AD. • Sangam literature is made up of three kingdoms, namely Chola, Chera and Pandiyan. • Corresponds to the post-Mauryan and pre-Gupta period. • First Sangam was held in Madurai. Agastya was the chairman. • Second Sangam held at Kapatpuram and its chairman was Tolkappiyar (author of Tolkappium). • Silappadikaram by Ilango Adigal (story of a married couple) and Manimegalai by Sattanar are the famous epics. Pandiyan History • Their emblem was Fish and the capital was Madurai. • First mentioned by Megasthanes, who said that the kingdom was famous for Pearls. • Pandiyan territory included the districts of Tirunelveli, Ramanad and Madurai. • They gained from trade with Roman Empire and sent embassies to Augus, the Roman emperor. • Mudukudumi, who was the earliest Pandiyan ruler accused Kovalan of theft. • As a result, the city was laid under a curse of Kannagi (Kovalan’s wife). Chola History • Their emblem was Tiger and kingdom was called Cholamandalam or Coromondal. • Kauveripattanam/Puhar was their capital. • Chief center was Uraiyur, famous for cotton trade. • Elara, the Chola king conquered Srilanka and ruled it for fifty years. • Karikala was their famous king. • Their main source was from trade in cotton cloth and they also maintained well organized Navy. Chera History • Their emblem was Bow and Vanji was their capital. • Muzris and Tondi were main ports. • Romans setup two regiments to protect their interests. • They fought against Cholas about 150 AD • Senguttuvan was the greatest king in Chera history. 15 | P a g e

shop.ssbcrack.com

MCQs

INDIAN HISTORY

MCQs

ANCIENT INDIAN HISTORY (MULTIPLE CHOICE QUESTIONS) 1. Who among the following was propounder of the Madhyamika Philosophy? [A]Bhadrabahu [B]Parshwanath [C]Sheelbhadra [D]Nagarjuna Answer [D] Nagarjuna- Nagarjuna (ca. 2nd or 3rd century) was a patriarch of Mahayana and the founder of Madhyamika School. 2. Which of the following Rigvedic deities represented Storm? [A]Indra [B]Maruts [C]Varun [D]Apas Answer [B] Maruts -Aryans, The Vedic Age – Religion: They personified force of nature and worshipped them as Vayu (air), Maruts (storm), Indra (rain), Varuna (water), Surya (sun), Agni (fire), Prithvi (earth), Aranyani (forest) etc. 3. The place Dharanikota in South India is related to which ancient dynasty? [A]Chola [B]Satavahana [C]Pandya [D]Chera Answer [B] Satavahana- Dharanikota is a town near Amaravati in the Guntur district of Andhra Pradesh in India, It is the site of the ancient Dhanyakataka which was the capital of the Satavahana kingdom which ruled in the Deccan around the 1st to 3rd centuries A.D. 4. Pushkalavati was capital of which of the following acient Kingdoms? [A]Gandhara [B]Kosal [C]Magadha [D]Kashi Answer [A) Gandhara- Pushkalavati was capital of ancient Kingdom of Gandhara. Its ruins are currently located in Charsadda, Pakistan, located in Peshawar valley in the Khyber Pakhtunkhwa province (formerly NWFP). Pushkalavati was home of the Sanskrit grammarian Panini. 5. “_____________is believed to mark the main site of Hastinapur, which was capital of Kauravas and Pandava The imperial gazetteer of India quotes about which of the following places? [A]Delhi [B]Ghaziabad [C]Meerut [D]Faridabad Answer [C] Meerut

16 | P a g e

shop.ssbcrack.com

MCQs

INDIAN HISTORY

MCQs

6. “Revere all the Buddhas, revere the dharma, revere the sangha” Which among the following quotes this _________? [A]Allahabad Pillar Inscription [B]Asoka Edict XII [C]Asoka Edict X [D]Mathura Lion capital Answer [D] Mathura Lion capital 7. What was the capital of Surasena Mahajanpada? [A]Viratnagar [B]Mathura [C]Taxila [D]Kashi Answer [B] Mathura 8. In which of the following Russo-Persian wars Russia was defeated? [A]first [B]second [C]third [D]none Answer [D] none- There was a series of 4 Russo-Persian wars in 1722-23, 1796, 1804, 1826-28 and all of the Russia was victorious. 9. World’s first oil paintings have been found in which of the following countries? [A]India [B]Pakistan [C]Afghanistan [D]Iran Answer [C] Afghanistan 10. Begram in Afghanistan was a capital of which of the following kings? [A]Kanishka [B]Asoka [C]Chandragupta Maurya [D]Samudragupta Answer [A] Kanishka 11. Which of the following state is the home to Bharat Dynasty? [A] Punjab [B] Haryana [C] Gujarath [D] Uttar Pradesh Correct Answer: B [Haryana] 12. “Manas Khand” which is mentioned in various ancient hindu scriptures belongs to /refers to which of the following modern states of India? [A] Gujarath [B] Punjab 17 | P a g e shop.ssbcrack.com

MCQs

INDIAN HISTORY

MCQs

[C] Himachal Pradesh [D] Uttrakhand Correct Answer: [D] Uttrakhand 13. Twang is an area in Arunanchal Pradesh in India, recently in news papers. It is world famous for which of the following? [A] India China Nathula Pass [B] India’s largest Buddhist monastery [C] Tibetan Museum [D] School for Buddhist Studies Correct Answer: B [India’s largest Buddhist monastery] 14. Ayodhya temples are located on the banks of which of the following rivers? [A] Sarayu [B] Ganga [C] Yamuna [D] Gomti Correct Answer: A [Sarayu] 15. Which among the following statements is incorrect regarding Ajanta Caves? [A] Paintings of Padmapani and Vajrapani have been found in Ajanta [B] These caves are part of UNESCO World Heritage Sites since 1983 [C] These paintings are Buddhist, Jain and Hinduism religious art [D] They were rediscovered in 1819 Correct Answer: C [These paintings are Buddhist, Jain and Hinduism religious art] Explanation: They only have Buddhist paintings 16. During whose reign Gandhāran style of art flourished? [A] Guptas [B] Hunas [C] Satavahanas [D] Kushans Correct Answer: D [Kushans] 17. Which among the following kingdom in South India was known for an excellent Navy ? [A] Pandyas [B] Cholas [C] Pallavas [D] Chalukyas Correct Answer: B [Cholas] 18. Fourth Buddhist Council was held at? [A] Kapilvastu [B] Kashmir [C] Sarnath [D] Bodhgaya Correct Answer: B [Kashmir] 18 | P a g e

shop.ssbcrack.com

MCQs

INDIAN HISTORY

MCQs

Explanation: First Rajgir, Second vaishali, Third patliputra Fourth Buddhist Council is the name of two separate Buddhist council meetings first in Sri Lanka and another at Sarvastivada school, in Kashmir around the First Century AD. 19. During whose reign among the following the Mahayan emerged? [A] Ashoka [B] Kanishka [C] Bimbisara [D] Harsha Correct Answer: B [Kanishka] Explanation: Kanishka regarded Mahayana as his state religion. The state religion of Kanishka determined preaching the divinity of Gautama Buddha 20. Hemis festival is dedicated to Lord Padmasambhava (Gur Rimpoche) venerated as the representative reincarnate of Buddha. In which part of India, it is held every year? [A] Himachal Pradesh [B] Assam [C] Ladakh [D] Orissa Correct Answer: C [Ladakh] Explanation: The Hemis festival is dedicated to Lord Padmasambhava (Gur Rimpoche) venerated as the representative reincarnate of Buddha. He is believed to have been born on the 10th day of the fifth month of the Monkey year as predicted by the Shakia Muni Buddha 21. Which of the following dynasties was founded by Mayur Sarman? [A] Gang [B] Pallava [C] Kadamba [D] Chalukya Correct Answer: C [Kadamba] 22. Who among the following granted permission to Buddhist King of Ceylon Meghavarman to build a Monastery at Bodh Gaya? [A] Chandra gupta Virkramaditya [B] Samudra Gupta [C] Skand Gupta [D] Kumara Gupta Correct Answer: B [Samudra Gupta] 23. Who among the following erected a monastery stupa at Peshawar? [A] Asoka [B] Kanishka [C] Bimbisara [D] Sri Gupta Correct Answer: B [Kanishka] 24. The 13th Rock edict of Asoka which is longest among all the rock edicts describes which of the following? 19 | P a g e shop.ssbcrack.com

MCQs

INDIAN HISTORY

MCQs

[A] Policy of Dhamma [B] Kalinga war [C] Toleration among sects [D] Purpose of rock edicts Correct Answer: B [Kalinga war] Explanation: 13th Rock edict located in Orissa describes Kalinga war 25. Which pillar edict of Asoka is longest of all pillars? [A] 2nd [B] 5th [C] 6th [D] 7th Correct Answer: D [7th] Explanation: Seventh Pillar Edict mentions about Dhamma mahamattas 26. Which of the following religious text is written in the form of questions and answers? [A] Vinaya Pitaka [B] Sutta Pitaka [C] Abhidhamma Pitaka [D] Prakirnas Correct Answer: C [Abhidhamma Pitaka] 27. In which of the following Buddhist Councils , the compilation of Dhamma Pitaka and Sutta Pitaka took Place? [A] 1st [B] 2nd [C] 3rd [D] 4th Correct Answer: A [1st] Explanation: First Buddhist Council in 483 at Satparni caves near Rajgriha 28. Who among the following belonged to Sakya clan of Kshatriyas? [A] Gautam Buddha [B] Mahavir jain [C] Chandragupta Maurya [D] Rishabhnath Correct Answer: A [Gautam Buddha] 29. Which among the following is a ritualistic Veda? [A] Rigveda [B] Yajurveda [C] Samaveda [D] Atharvaveda Correct Answer: B [Yajurveda] 30. Painted Grey ware” was used in which of the following era? [A] Early Vedic Era [B] Later Vedic Era 20 | P a g e

shop.ssbcrack.com

MCQs

INDIAN HISTORY

MCQs

[C] Harappan Era [D] None of the above Correct Answer: B [Later Vedic Era] 31. Which among the following was referred to as Buvan Chakshu in Early vedic era? [A] Earth [B] Fire [C] Water [D] Moon Correct Answer: B [Fire] 32. Which of the following two towns of Indus valley Civilization were located on the bank of Ghagghar River? [A] Harappa & Rojari [B] Harappa & Mohen jo Dero [C] Kalibanga and Banwali [D] Rangpur and Surkotda Correct Answer: C [Kalibanga and Banwali] 33. Which of the following two towns of Indus valley Civilization were located on the bank of Ghagghar River? [A] Harappa & Rojari [B] Harappa & Mohen jo Dero [C] Kalibanga and Banwali [D] Rangpur and Surkotda Correct Answer: C [Kalibanga and Banwali] 34. We have read that the writing in the Indus valley Civilization was Boustrophedon. This means that ______? [A] It was written left to right [B] It was written right to left [C] It was written left to right and right to left in alternate lines [D] It was written up to down Correct Answer: C [It was written left to right and right to left in alternate lines] 35. At which of the following harappan sites, an Inkpot has been discovered [A] Harappa [B] Lothal [C] Kalibangan [D] Chanhudaro Correct Answer: D [Chanhudaro] 36. Kavi Vatsal was a title assumed by a king belonging to which of the following dynasties? [A] Pushyabhuti [B] Pallava [C] Maurya [D] Satavahana Correct Answer: D [Satavahana] 21 | P a g e shop.ssbcrack.com

MCQs

INDIAN HISTORY

MCQs

Explanation: The name of the king is Hala who assumed the title of Kavivatsal 37. Which among the following is not correctly matched? (Maurya Kings and contemporary Travelers/ Ambassadors) [A] Chandra Gupta Maurya – Megasthenes [B] Bindusara – Deimakos [C] Harshavardhana – Hiuen Tsang [D] All are correct Correct Answer: D [All are correct] 38. Which of the following was used in house making in Indus Valley Civilization? [A] Stone [B] Brics [C] Wood [D] Bamboo Correct Answer: B [Brics] 39. Who was the founder of the Pala Dynasty of Bengal? [A] Dharmapala [B] Gopala [C] Mahendrapala [D] Mahipala Correct Answer: B [Gopala] 40. Which of the following pala ruler founded the University of Vikramsila? [A] Devapala, [B] Mahipala I [C] Dharamapala [D] Gopala Correct Answer: C [Dharamapala] 41. Which among the following pairs is NOT correctly matched? (Traveler – During reign of King) [A] Megasthenes – Chandragupta Maurya [B] Deimachos – Asoka [C] Hsuan -Tsang – Harshavardhana [D] Fa-Hien – Chandragupta II Correct Answer: B [Deimachos – Asoka] Explanation: Deimachos was a Greek ambassador who visited the court of Bindusara 42. Partha was the name of which of the following character in Mahabharta? [A] Arjuna [B] Bhima [C] Krishna [D] Yudhisthara Correct Answer: A [Arjuna] 43. Saka Year is how many years behind the Christian Era? [A] 50 [B] 75 22 | P a g e

shop.ssbcrack.com

MCQs

INDIAN HISTORY

MCQs

[C] 78 [D] 100 Correct Answer: C [78] 44. Bagh caves near Gwalior represent the paintings of which of the following era? [A] Maurya Era [B] Gupta Era [C] Kushana Era [D] Rajput Era Correct Answer: B [Gupta Era] 45. Nalanda University was founded by which of the following? [A] Kumaragupta [B] Skandagupta [C] Samudragupta [D] None of the above Correct Answer: A [Kumaragupta] 46. Mudra Rakshas is a work of Vishakhadutta. This work was depicted which of the following? [A] Chadragupta Maurya & Dhruva Devi [B] Mauryas & Nandas [C] Campaigns of Chandragupta Maurya [D] Campaigns of Ashoka Correct Answer: B [Mauryas & Nandas] 47. Who among the following is famous for saving the empire from the Huna tribe, which had overran Asia and Europe. [A] Samudra Gupta [B] Chandra Gupta II [C] Skandagupta [D] None of the above Correct Answer: C [Skandagupta] 48. The ancient city of Raj Griha which is modern Rajgir near Patna was built by which of the following kings? [A] Bimbisara [B] Ajatshatru [C] Prasenjit [D] Ashoka Correct Answer: A [Bimbisara] 49. “The greatest law giver” of ancient India was who among the following? [A] Banabhatta [B] Kautilya [C] Manu [D] Chanukya Correct Answer: C [Manu] 23 | P a g e

shop.ssbcrack.com

MCQs

INDIAN HISTORY

MCQs

50. Which of the following was an Indian sage cum Guru and is said to have transmitted Tantric Buddhism to Bhutan and Tibet in the 8th century? [A] Asanga [B] Nagarjuna [C] Padmasambhava [D] Mahavir Correct Answer: C [Padmasambhava] 51. Which among the following is Not correctly matched? [A] Takshshila – Gandhar [B] Viratanagara – Matsya [C] Sursen – Kausambi [D] All are correct Correct Answer: C [Sursen – Kausambi] Explanation: Sursen was capital of Mathura and Capital of Kausambi was Vatsa 52. Which among the following is not correctly Matched? [A] Wife of prophet Mohammed – Khadija [B] Daughter of Prophet Mohammad – Fatima [C] Place where prophet Mohammed Buried – Mecca [D] All are correct Correct Answer: C [Place where prophet Mohammed Buried – Mecca] Explanation: prophet Mohammed was buried at Madina 53. Amarsimha whose work Amarkosha is related to which of the following? [A] Poetry [B] Lexicography [C] Grammar [D] wars Correct Answer: B [Lexicography] 54. Who was the creator of Ravanabadha? [A] Kalidasa [B] Bhatrahari [C] Batsabhatti [D] None of the above Correct Answer: C [Batsabhatti] 55. What was the meaning of Visthi in the Gupta Era? [A] Religious Functionaries [B] Forced Labor [C] Prostitutes [D] Police Correct Answer: B [Forced Labor] 56. Which school of art is also called as mature School? [A] Keshena School [B] Andhra School 24 | P a g e

shop.ssbcrack.com

MCQs

INDIAN HISTORY

MCQs

[C] Arafat School [D] Mathura school Correct Answer: A [Keshena School] 57. Which among the following is true? [A] the gold coins issued by Guptas were having more Gold Content than Kushanas [B] The gold coins issued by Kushana were having more Gold content than Guptas [C] Both were having the same fraction of Gold [D] None of the above Correct Answer: B [The gold coins issued by Kushana were having more Gold content than Guptas] 58. Gandhar Kingdom of Later vedic phase was mostly located at which of the following places? [A] Afghanistan [B] Rawalpindi and Peshawar districts of Pakistan [C] Kashmir [D] Harappa Correct Answer: B [Rawalpindi and Peshawar districts of Pakistan] 59. In which of the following marriages in Vedic era, a girl was given by her father to the sacrificial priest in as a part of fee or Dakshina? [A] Brahma marriage [B] Daiva Marriage [C] Gandharva Marriage [D] None of the above Correct Answer: B [Daiva Marriage] 60. Who among the following was the most important divinity during the early Vedic era? [A] Agni [B] Indra [C] Vayu [D] Varuna Correct Answer: B [Indra] 61. The famous Gayatri mantra is addressed to which of the following vedic goddess? [A] Aditi [B] Saraswati [C] Savitri [D] Laxmi Correct Answer: C [Savitri] 62. Which among the following is Incorrect regarding the early Vedic religion? [A] They didn’t believe in Idol worship and believed in one supreme god indeed [B] The Vedic god were classified into three categories- Terrestrial, Atmospheric and Celestial [C] The people used to worship of spiritual reasons [D] None of the above Correct Answer: C [The people used to worship of spiritual reasons] Explanation: They used to worship for welfare of praja and pashu 25 | P a g e

shop.ssbcrack.com

MCQs

INDIAN HISTORY

MCQs

63. The regions where the Aryans settled in India was called Saptasindhu , which was pronounced as haptahindu as well. What was the other term used for this area? [A] Mahabharata [B] Aryavarta [C] Pataliputra [D] Brahmavarta Correct Answer: D [Brahmavarta] 64. Which among the following is incorrect about Devdasis? [A] According to Paes, devdasis held a highly respectable position in Vijayanagar society [B] Devdasis system came into existence during vijaynagar era only [C] Devdasis means the girls were married to deities [D] All are correct Correct Answer: B [Devdasis system came into existence during vijaynagar era only] 65. which among the following sentence is incorrect about Rajadhiraja Chola I? [A] He was a born fighter son of Rajendra Chola I [B] He died in the battle of Koppan fighting with Chalukyas [C] He assumed the title of Parakesari [D] All are correct Correct Answer: C [He assumed the title of Parakesari] Explanation: Parakesari and Yuddhamalla were the titles of Rajendra 1 Rajadhiraja Chola assumed the title of Rajakesari 66. Which among the following are correct about Chola King Rajendra Chola1? [A] He built a temple for Siva at Gangaikonda Cholapuram [B] He also built Tanjore Brihadisvara temple [C] He built Tanjore Brihadisvara temple for a tribute to his father [D] None of the above Correct Answer: A [He built a temple for Siva at Gangaikonda Cholapuram] Explanation: Tanjore Brihadisvara temple was created by father of Rajendra Chola-1 , Rajraja-1 67. Meander who was an Indo Greek king accepted which of the following religion? [A] Janism [B] Buddhism [C] Jainism [D] Islam Correct Answer: B [Buddhism] Explanation: Before becoming a Buddhist, Menander (an Indo-Greek king) asked Nagasena many questions relating to Buddhism.These questions and Nagasena’s answers are recorded in a form of book known as ‘Milindapanho (or The Questions of Milinda). 68. Which among the following is incorrect regarding Kushanas? [A] They originated from Central Asia [B] They were derivative of Yuezhi tribe [C] The Ancestors of Kushanas were easternmost speakers of Indo-Greek languages [D] All are correct Correct Answer: C [The Ancestors of Kushanas were easternmost speakers of Indo-Greek languages] 26 | P a g e shop.ssbcrack.com

MCQs

INDIAN HISTORY

MCQs

Explanation: As per the Chinese sources, the Kushanas (mentioned in Chinese texts as Guishuang) were one of the 5 tribes of Yueh-Chi or Yuezhi. Kushanas were also known as Tocharians. They were nomadic tribes and easternmost speakers of the Indo-Europeans Languages which were called “Tocharian languages”. 69. Who among the following is considered to organize a confederacy of 10 tribes or Kings against Sudas? [A] Vashishta [B] Vishawamitra [C] Puru [D] Agatsya Correct Answer: B [Vishawamitra] Explanation: The battle of ten kings was fought between Sudas, a Bharata king of the Tritsu family and the confederacy of ten well-known tribes- Puru, Yadu, Turvasa, Anu, Druhyu, Alina, Paktha, Bhalanas, Shiva and Vishanin. In the bloody and decisive battle on the banks of River Purushni, the Bharatas emerged victorious. The reason behind the war is the rivalry between Vishwamitra and Vasishtha. 70. The earliest epigraphic evidence for the existence of the Bhagavata cult is found in Madhya Pradesh which is the Garuda pillar inscription of Besnagar. It was erected by whom? [A] Heliodorus [B] Vijayamitra [C] Kanishka [D] Ashoka Correct Answer: A [Heliodorus] 71. During the reign of which of the following some Yavans’ or Greeks were converted to the Vaishnava Cult? [A] Shungas [B] Nandas [C] Shakas [D] Satavahans Correct Answer: A [Shungas] 72. In which of the following the philosophical and spiritual thoughts are contained? [A] Vinaya Pitak [B] Suddha Pitak [C] Abhidhamma Pitak [D] None of the above Correct Answer: C [Abhidhamma Pitak] 73. Nigashtha, Natputra, Nirgranth & Saatputra are various names of which of the following? [A] Mahavira [B] Gautam Buddha [C] Chandra gupta Vikramaditya [D] Ashoka Correct Answer: A [Mahavira]

27 | P a g e

shop.ssbcrack.com

MCQs

INDIAN HISTORY

MCQs

74. The three sangams were organized under the chairmanship of which of the following? [A] Rishi Agastya, Nakkirar, Rishi Agastya [B] Rishi Agastya, Rishi Agastya & Nakkirar [C] Nakkirar, Rishi Agastya, Rishi Agastya [D] None of the above Correct Answer: B [Rishi Agastya, Rishi Agastya & Nakkirar] 75. Who among the following was last satavahana ruler? [A] Simukha [B] Krishna [C] Yagnashathkarni [D] Pulomavi Correct Answer: C [Yagnashathkarni] 76. Which of the following group represents right combination of Triratnas of Jainism? [A] Right Faith, Right Knowledge, Right Thought [B] Right Thought, Right Action, Right Conduct [C] Right Faith, Right Knowledge , Right Conduct [D] None of the above Correct Answer: C [Right Faith, Right Knowledge, Right Conduct] 77. Katyotsarga pose is a gesture of which of the following [A] Vishnu [B] Shiva [C] Mahavira [D] Gautam Buddha Correct Answer: D [Gautam Buddha] 78. Mehrgarh in the Indus valley civilization is known for which of the following? [A] Red sandstone [B] Terracotta Cart [C] Copper Rhinoceros [D] None of the above Correct Answer: B [Terracotta Cart] Explanation: Red Sandstone- MohenjoDero Copper Rhinoceros-diamabad 79. Nude male torso has been found at which of the following place? [A] Mehergarh [B] MohanJodro [C] Harappa [D] Lothal Correct Answer: C [Harappa] 80. What were the names of Asoka’s mother, wife & daughter? [A] Mahadevi, Subhadrangi, Sanghmitra [B] Subhadrangi, Mahadevi, Sanghmitra [C] Mahadevi, Sanghamitra, Subhadrangi [D] None of the above 28 | P a g e

shop.ssbcrack.com

MCQs

INDIAN HISTORY

MCQs

81. What was the correct rank of” Baladhikarana” in the Gupta Age? [A] Head of Army [B] Head of State Police Department [C] Head of cavalry [D] Head of intelligence Correct Answer: A [Head of Army] 82. The Taxila or Takshashila was located between following two rivers? [A] Indus & Jhelum [B] Jhelum & Chenab [C] Chenab & Ravi [D] Chenab & Beas Correct Answer: A [Indus & Jhelum] 83. Which of the following place of Andhra Pradesh is well known for a Buddhist Stupa? [A] Amaravati [B] Srikakulam [C] Rajahmundry [D] Chittoor Correct Answer: A [Amaravati] Explanation: Amravati, is known as Dhanyakataka/Dharanikota was the site of a great Buddhist Stupa built in pre-Mauryan times 84. Giak & Kiari are located in which of the following? [A] Ladakh [B] Orissa [C] Kuchh [D] Assam Correct Answer: A [Ladakh] Explanation: Giak & Kiari are located in Ladakh. They are examples of Neolithic sites 85. Navanitakam is a work on which of the following? [A] Architecture [B] Medicine [C] Astrology [D] Maths Correct Answer: B [Medicine] 86. During the reign of Guptas in North India, who were their contemporaries in South India? [A] Cholas [B] Rasthrakutas [C] Satvahanas [D] Kakatiyas Correct Answer: B [Rasthrakutas] 87. The following were the immediate successors of imperial Mauryas? [A] Pandyas [B] Kushans 29 | P a g e

shop.ssbcrack.com

MCQs

INDIAN HISTORY

MCQs

[C] Nandas [D] Sungas Correct Answer: D [Sungas] 88. Who has written the biography of harshavardhana? [A] Banbhatta [B] Harsha Himself [C] Kalhana [D] Kalidasa Correct Answer: A [Banbhatta] 89. Madurai was a capital of which of the following dynasties? [A] Pandyas [B] Pallavas [C] Cholas [D] Cheras Correct Answer: A [Pandyas] 90. Which of the following authorities lays down a priority of inheritance which places the wife, followed by daughters immediately after sons? [A] Brihaspati [B] Yajnavalkya [C] Narada [D] None of the above Correct Answer: B [Yajnavalkya] Explanation: Yajnavalkya lists widow after son and daughters following her in case of no son 91. The meaning of Mohenjo dero is ‘mound of the dead’. The same meaning is expressed by which of the following names? [A] Harappa [B] Lothal [C] Amri [D] Kalibangan Correct Answer: B [Lothal] 92. Charak was a contemporary of which of the following Kings? [A] Chandra Gupta Maurya [B] Kanishka [C] Ashoka [D] Akbar Correct Answer: B [Kanishka] Explanation: Some of the scholars in the Court of Kanishka were Parsva, Vasumitra, Asvaghosa, Nagarjuna, Charaka and Mathara. Charaka has been called the Court Physician of Kanishka, though it is very much disputed. 93. In which of the following age human shifted from food gathering stage to food producing stage ? [A] Paleolithic age [B] Neolithic age 30 | P a g e shop.ssbcrack.com

MCQs

INDIAN HISTORY

MCQs

[C] Mesolithic age [D] None of the above Correct Answer: B [Neolithic age] 94. Queen Trishala was the name of mother of ________ ? [A] Mahavira [B] Gautam Buddha [C] Kanishka [D] Ashoka Correct Answer: A [Mahavira] 95. Kumarila Bhatta, who was a Hindu philosopher and scholar from Prayag, was an acharya of which of the following? [A] Mimansa [B] Samkhya [C] Vedanata [D] None of the above Correct Answer: A [Mimansa] 96. Kumārasambhava is an Sanskrit epic poem written by Kalidasa. It literally means birth of Kumara. Who was Kumara in this epic? [A] Kartikeya [B] Abhimanyu [C] Sanatahkumara [D] Arjuna Correct Answer: A [Kartikeya] 97. Which was the second capital of Kanishka? [A] Mathura [B] Sanchi [C] Banaras [D] Pataliputra Correct Answer: A [Mathura] Explanation: Kanishka’s main capital was at Peshawar (Purushpura) and regional capitals is at Taxila in Pakistan, Begram in Afghanistan and Mathura in India. Mathura had a status of almost a second capital. 98. What is the meaning of kalibangan in Rajasthan which shows the prehistoric and pre-Mauryan character of Indus Valley Civilization? [A] Black River [B] Black Bangles [C] Black forest [D] Black land Correct Answer: B [Black Bangles] 99. The Jivaka which is mentioned in Buddhist literature was one among the following ? [A] Mercchant [B] King 31 | P a g e

shop.ssbcrack.com

MCQs

INDIAN HISTORY

MCQs

[C] Physician [D] Police Correct Answer: C [Physician] Explanation: Jivaka is the name of doctor during Buddha’s time 100. The bilingual Inscriptions of Asoka involved which of the following two languages? [A] Brahmi & Aramaic [B] Brahmi, Greek & Aramaic [C] Brahmi & hieroglyphics [D] None of the above Correct Answer: B [Brahmi, Greek & Aramaic] Explanation: Asoka used 4 scripts The scripts used for the Prakrit inscriptions were Brahmi and Kharoshthi, and for the others, Greek and Aramaic 101. Which of the following was a common way of Chola kings Raj raja-1 and Rajendra 1 to commemorate their victories? [A] Erecting Pillars [B] Erecting Temples [C] Erecting inscriptions [D] issuing coins Correct Answer: B [Erecting Temples] 102. Which among the following is related to history of Kashmir? [A] Rajatarangini [B] Ashokavadana [C] Vikramorvashiyam [D] Arthashastra Correct Answer: A [Rajatarangini] 103. What was the job of Rupadarshaka in Mauryan Era as per Arthashashtra of Kautilya? [A] Royal examiner of minted coins [B] Royal Superintendent of Courtesans [C] Manager of the Stage [D] Police Correct Answer: A [Royal examiner of minted coins] 104. Which among the following has been used in the sculptures of Mathura school of Art? [A] Redsand stone [B] Marble [C] Granite [D] Clay Correct Answer: A [Redsand stone] 105. Who among the following assumed the title of ” Mahendraditya”? [A] Chandra Gupta II [B] Kumaragupta I [C] Bhanugupta [D] Samudragupta Correct Answer: B [Kumaragupta I] 32 | P a g e

shop.ssbcrack.com

MCQs

INDIAN HISTORY

MCQs

106. Which among the following is not Correctly matched? [A] Milinda Panho – Buddhism [B] Tatwarthadhigam – Vaishnavism [C] Mahapurana – Jainism [D] All are correct Correct Answer: B [Tatwarthadhigam – Vaishnavism] Explanation: Tatwarthadhigam is of Jainism composed by Uma Swami 107. Which among the following is Not Correctly matched? (Vedas & Brahman) [A] kaushitaki – Rigveda [B] Jaiminia – Samveda [C] Gopatha – Yajurveda [D] All are correct Correct Answer: C [Gopatha – Yajurveda] Explanation: Gopatha is a Brahman of Atharvaveda 108. Which among the following is NOT correct? [A] The picture of Yogi is found in many Muhar in Harppa which who has three heads and sits in crossed legs indicating Shiva Worship [B] The people of Harappa didn’t believe in Witchcraft [C] The dogs and cats were the favorite domesticated animals of Harppan people [D] All are correct Correct Answer: B [The people of Harappa didn’t believe in Witchcraft] Explanation: A lot of talismans made of bronze and copper have been found which indicate that People of Harappa believe in witchcrafts and dead souls 109. Who among the following is seemed to have known the importance of zero? [A] Bhaskaracharya [B] Aryabhatta [C] Budhayana [D] chanukya Correct Answer: A [Bhaskaracharya] 110. Which among the following is Not Correctly matched? [A] Maurya Empire – Chandragupta Maurya [B] Shunga Dynasty – Pushyamitra Shunga [C] Satvahana Dynasty – Gautamiputra satkarni [D] All are correct Correct Answer: C [Satvahana Dynasty – Gautamiputra satkarni] Explanation: The founder of Satvahana Dynasty was Simuk and not Gautamaiputra Satakarni 111. Which of the following places is known for painted rock shelters belonging to Paleolithic and Mesolithic ages? [A] Utnur [B] Uyyur [C] Bhimbetka [D] Bairath Correct Answer: C [Bhimbetka] 33 | P a g e

shop.ssbcrack.com

MCQs

INDIAN HISTORY

MCQs

112. Sarnath is associated with which aspect of Buddha’s life? [A] Birth [B] Residence [C] First sermon [D] Death Correct Answer: C [First sermon] Explanation: Buddha gave his first sermon at the Sarnath which is known as “Dhammachakra Parivartan” 113. Which Buddhist council was held during the reign of Kanishka? [A] 1st [B] 2nd [C] 3rd [D] 4th Correct Answer: D [4th] Explanation: The Fourth Buddhist Council was held at Kundalvana, Kashmir in 72 AD under the patronage of Kushan king Kanishka and the president of this council was Vasumitra, with Aśvaghosa as his deputy. This council distinctly divided the Buddhism into 2 sects Mahayan & Hinayan. 114. After death of Bindusara, who among the following is considered to help Asoka to capture the throne? [A] Vishnugupta [B] Radhagupta [C] Tishya [D] Bairam khan Correct Answer: B [Radhagupta] 115. Shatapatha Brahmana belongs to which of the following Vedas? [A] Krishna Yajurveda [B] Shukla Yajurveda [C] Rigveda [D] Samaveda Correct Answer: B [Shukla Yajurveda] 116. Kena Upanishad is of which of the following Vedas? [A] Rig Veda [B] Samaveda [C] Yajurveda [D] Atharva veda Correct Answer: B [Samaveda] Explanation: “Ken” literally means ‘by whom’. It belongs to the Talavakara Brahmana of Sama Veda and is therefore also referred to as Talavakara Upanishad. 117. Four Noble Truths are one of the most fundamental truths of which of the following religion? [A] Jainism [B] Buddhism [C] Islam [D] Hinduism Correct Answer: B [Buddhism] 34 | P a g e shop.ssbcrack.com

MCQs

INDIAN HISTORY

MCQs

Explanation: There are four noble truths of Buddhism: a. The world is full of sorrow b. Desire is the cause of sorrow c. If desire is conquered all sorrow is won over d. Desire can be conquered by following eight-fold paths (Ashtangirka Marg) 118. Which river was called Vipasa in Vedic India? [A] Ganga [B] Jhelum [C] Beas [D] Indus Correct Answer: C [Beas] 119. Which among the following philosophy deals with Logic? [A] Yoga [B] Samkhya [C] Nyaya [D] None of the above Correct Answer: C [Nyaya] 120. Gopatha Brahman is a part of which of the following Vedas? [A] Rigveda [B] Yajurveda [C] Samaveda [D] Atharva veda Correct Answer: D [Atharva veda] 121. Brahma Sutra of Badrayan are the logical point of starting of which of the following philosophies? [A] Vaisheshika [B] Nyaya [C] Vedanta [D] None of the above Correct Answer: C [Vedanta] 122. ” Karmara” in the vedic age was referred to the people occupied with the profession of ______? [A] Blacksmith [B] Potter [C] Ratha (Chariot) Makers [D] Goldsmith Correct Answer: A [Blacksmith] 123. In Vedic age which among the following grains was referred as vrihi? [A] Barley [B] Wheat [C] Rice [D] Maize Correct Answer: C [Rice] 35 | P a g e

shop.ssbcrack.com

MCQs

INDIAN HISTORY

MCQs

124. Which among the following was the most important grain grown in the vedic Age? [A] Wheat [B] Barley [C] Maize [D] Rice Correct Answer: B [Barley] 125. which among the following Brahman provides details of cultivation process? [A] Aitarey [B] kaushitaki [C] satpatha [D] None of the above Correct Answer: C [satpatha] 126. Which among the following have been proved to be the best source of info regarding ancient India? [A] Religious literature [B] inscriptions [C] coins [D] monumnets Correct Answer: B [inscriptions] 127. Who among the following is compiled the work ” Ravanavadham”? [A] Bhatti [B] Kalidasa [C] Bhasa [D] Banabhatta Correct Answer: A [Bhatti] 128. Which among the following is the most important work of Tamil Literature of Sangam Period (sometimes also referred as Bible of the Tamil land)? [A] Thirukkural [B] Purananuru [C] Malaipatukatam [D] None of the above Correct Answer: A [Thirukkural] Explanation: Thiruvalluvar was the author of the book ‘Tirukkural (also known as the Kural). It is a classic Tamil sangam literature consisting of 1330 couplets or Kurals. The book is also called as the fifth Veda or ‘Bible of the Tamil Land’. 129. Who among the following is said to have crossed the Vindhyas in the later Vedic age and aryanise the south ? [A] Parashara [B] Agatsya [C] Jamadagni [D] Vasistha Correct Answer: B [Agatsya] 36 | P a g e

shop.ssbcrack.com

MCQs

INDIAN HISTORY

MCQs

130. The capital of Chola empire was Thanjavur (Tanjore), where Brihadeshwara temple was constructed by ___________? [A] Rajednra Chola I [B] Rajaraja Chola I [C] Krishnan Raman [D] Venkata Raman Correct Answer: B [Rajaraja Chola I] Explanation: Brihadeeswarar Temple or Peruvudaiyar Kovil or Rajrajeshwaram temple at Thanjavur is the world’s first complete “granite” temple. It was built by Rajraja Chola-I and is a part of UNESCO’s world Heritage sites. The Vimana or the temple tower (known as Raja Gopuram) is 216 Feet in height and is one of the tallest buildings of its kind. The Nandi is carved out of a single rock. This temple has completed 1 millennium in 2010. It was dictated by lord Shiva to Rajraja Chola I, when he triumphed Ilam (Sri Lanka) Island. 131. Which among the following place is related to coronation of Chola Kings? [A] Tanjaur [B] Vellur [C] Chidambaram [D] Madras Correct Answer: C [Chidambaram] 132. Who among the following has been described by Banabhatta as “a lion to the Huna deer’? [A] Harshavardhana [B] Prabhakarvardhana [C] Grahavarmana [D] None of the above Correct Answer: B [Prabhakarvardhana] Explanation: An important ruling family to gain prominence after the fall of the Gupta was that of the Pushyabhutis who had their capital at Thanesar (Thanesvara in Kurukshetra). The dynasty became influential with the accession of Prabhakarvardhana, who was able to defeat the Hunas and strengthen his position in the regions of Punjab and Haryana. After his death, his elder son Rajyavardhana came to the throne but he was treacherously killed by Shashanka, the king of Bengal and Bihar. Harshavardhana then ascended the throne in AD 606. He was only sixteen years of age at that time. Still he proved himself to be a great warrior and an able administrator 133. Aihole is called the cradle of Indian temple architecture. Which among the following dynasty is related to the construction of magnificent temples in Aihole? [A] Chalukya [B] Chola [C] Chera [D] None of the above Correct Answer: A [Chalukya] 134. In ancient India, which dynasty issued the largest number of Gold coins? [A] Guptas [B] Mauryas

37 | P a g e

shop.ssbcrack.com

MCQs

INDIAN HISTORY

MCQs

[C] Cholas [D] Nandas Correct Answer: A [Guptas] 135. In which period of the Indian History , the number zero as well as decimal system were invented? [A] Gupta [B] Maurya [C] Nanda [D] Chedi Correct Answer: A [Gupta] 136. ‘Saptanga’ Theory which deals with the 7 limbs (sapta anga) of ancient political structure was propounded by ? [A] Manu [B] Kautily [C] Panini [D] Bhaskara Correct Answer: B [Kautily] Explanation: Saptanga theory of state was given by Kautilya in Arthashashtra. The seven limbs are King, Amatya (Bureaucrats) , Janapada (territory), Durga (Fort), Kosa (Treasure), Danda (coercive authority) and Mitra (ally). 137. Narasimhavarman-I was one of the most famous Tamil kings of the Pallava dynasty who ruled South India and founded Mahabalipuram. His another name is _______? [A] Mamalla [B] Mahedravarman [C] Apapar [D] None of the above Correct Answer: A [Mamalla] 138. Mahabalipuram was a port city of which of the following dynasties? [A] Chola [B] Pallava [C] Chalukya [D] Kakatiya Correct Answer: B [Pallava] 139. Chandella Kings are related to which of the following temples? [A] Khajuraho [B] Hampi [C] Mahabalipuram [D] Ajanta Correct Answer: A [Khajuraho] 140. Konark Temples are related to which of the following dynasties ? [A] Pratihara [B] Pallavas 38 | P a g e

shop.ssbcrack.com

MCQs

INDIAN HISTORY

MCQs

[C] Chandelas [D] Cholas Correct Answer: A [Pratihara] 141. The ancient places of Badami, Dwarasamudra and Warangal are related to which of the following dynasties? [A] Chalukyas, Hoysalas, Kaktiyas [B] Cholas, Hoysalas, Kaktiyas [C] Pallavas, Hoysalas, Kaktiyas [D] None of the above Correct Answer: A [Chalukyas, Hoysalas, Kaktiyas] 142. Sankara is related to which of the following? [A] Advaita Philosophy [B] Samkhya Philosophy [C] Nyaya Philosophy [D] Dharma Philosophy Correct Answer: A [Advaita Philosophy] 143. Who among the following is propounded of Madhyamika Philosophy? [A] Sankara [B] Nagarjuna [C] Basava [D] None of the above Correct Answer: B [Nagarjuna] 144. Who among the following king is related to Repair of Sudarsana Lake? [A] Rudradaman [B] Gautmi Putra Satkarni [C] Kharavela [D] Ashoka Correct Answer: A [Rudradaman] Explanation: The Junagarh Rock Inscription mentions that the Saka ruler Rudradaman (150-151 BC) had taken up the task of restoration of lake Sudarsana. 145. Which among the following comprises ” Code of Laws” ? [A] Sutras [B] Manu smruthi [C] Vedas [D] Up Vedas Correct Answer: B [Manu smruthi] 146. ”Kaviraja” or King of poets was a title of _________? [A] Chandra Gupta Maurya [B] Samudra Gupta [C] Skand Gupta [D] Ashoka Correct Answer: B [Samudra Gupta] 39 | P a g e

shop.ssbcrack.com

MCQs

INDIAN HISTORY

MCQs

147. Algebra was invented by which of the following Ancient Indian Mathematicians? [A] Bhaskara [B] Aryabhatta [C] Medhatithi [D] None of the above Correct Answer: B [Aryabhatta] 148. Battle of the Hydaspes was fought between Alexander and King Porus in 326 BC. Hydapses means which river________? [A] Jelum [B] Indus [C] Beas [D] Ravi Correct Answer: A [Jelum] Explanation: Alexander invaded India in 326 BC. In 326 BC, the Battle of Hydaspes was fought between Alexender the great and Porus along the banks of Haydaspes river( Jhelum river) in Punjab. Porus, the King of Paurava, ruled the area between the rivers Hydaspes (modern Jhelum) and Acesines (Chenab). Though, Porus was defeated, Alexendar was impressed by Porus’s might so asked Porus to rule his dynasty under his name. 149. What was used normally in the construction of Roya Houses during Mauryan period? [A] Wood [B] Mud & Lime [C] Burnt Bricks [D] None of the above Correct Answer: A [Wood] 150. Gandhar art is a combination of ____________ ? [A] Indian Style + Greek Style [B] Greek Style + Kushan Style [C] Indian Style + Kushan Style [D] Indian Style + Persian Style Correct Answer: C [Indian Style + Kushan Style] 151. Evidence of a game which was very similar to chess was found at which of the following sites at Indus valley civilization? [A] Harappa [B] Lothal [C] Mohen Jo dero [D] Kalibangan Correct Answer: B [Lothal] 152. There is no evidence of cultivation of which among the Barley, Sugarcane, Rice & Field pea at any of the sites of the Indus valley? [A] Sugarcane, Rice & Field pea [B] Rice & Field pea

40 | P a g e

shop.ssbcrack.com

MCQs

INDIAN HISTORY

MCQs

[C] Sugarcane Only [D] Rice only Correct Answer: C [Sugarcane Only] 153. Which among the following is best example of Vijaynagar Art? [A] Hampi [B] Pattadakal [C] Amaravati [D] Puri Correct Answer: A [Hampi] 154. Fa-Hien who came to visit India during Gupta Period had the main motive of ___________? [A] Establish trade relations between India and China [B] Write a book on the life of Indian People [C] Study the teachings of Gautam Buddha [D] To spread Buddhism Correct Answer: C [Study the teachings of Gautam Buddha] 155. Which among the following sites gives an evidence of maritime trade in harappan era? [A] Mohen-jo dero [B] Lothal [C] Kalibangan [D] Harappa Correct Answer: B [Lothal] 156. Devichandraguptam was a work of _________? [A] Vishakhadatta [B] Dandin [C] Bharavi [D] Kalidas Correct Answer: A [Vishakhadatta] 157. The Royal seal of Guptas bore the emblem of _________? [A] Hasti (Elephant) [B] Mayura ( Peacock) [C] Garuda (Eagle) [D] Tiger Correct Answer: C [Garuda (Eagle)] 158. Milinda-Panho is a pali text which record discussion between Milinda and Nagsena. Milinda was a ________? [A] Satvahan King [B] Shunga King [C] Indo Greek Ruler [D] Nanda king Correct Answer: C [Indo Greek Ruler]

41 | P a g e

shop.ssbcrack.com

MCQs

INDIAN HISTORY

MCQs

159. Kharvela who constructed caves in the Udaigri for jain Monks was a ruler belonging to which dynasty? [A] Cheti [B] Satavahana [C] Shunga [D] Nanda Correct Answer: A [Cheti] 160. The railings of the Sanchi Stupa were made during? [A] Maurya Period [B] Sunga Period [C] Kushana Period [D] Gupta Period Correct Answer: B [Sunga Period]

161. Which among the following was main item of revenue during Maurya Period? [A] Bhaga [B] Pindikara [C] Hiranya [D] None of the above Correct Answer: A [Bhaga] Explanation: Bhaga (it was main item of revenue levied @ 1/4 to 1/6 of crop 162. In which of the following works is mentioned the conspiracy hatched by Pushyamitra Shunga to eliminate the last Mauryan King Brihadratha? [A] Mahabhasya of patanjali [B] Rajtarangini of Kalhan [C] Harshacharita of Baanbhatta [D] Malvikagnimitra of Kalidasa Correct Answer: C [Harshacharita of Baanbhatta] 163. King Prasenjita was a contemporary and friend of Lord Buddha. He was a king of ________? [A] Kasi [B] Kosal [C] Anga [D] Magdha Correct Answer: B [Kosal] 164. Which among the following is true about Buddha? [A] He accepted existence of God [B] He didn’t accept existence of God [C] He neither accepted nor rejected the existence of god [D] None of the above Correct Answer: C [He neither accepted nor rejected the existence of god] 165. In which of the following modern State, most of the 24 Tirthankar of Jainism attained nirvana? [A] Bihar [B] Uttar Pradesh 42 | P a g e

shop.ssbcrack.com

MCQs

INDIAN HISTORY

MCQs

[C] Rajasthan [D] Odisha Correct Answer: A [Bihar] 166. Which among the following Mahajanpada was located on the junction of two famous ancient Indian Trade routes – Uttarpatha and Dakshinpatha? [A] Matsya [B] Surasena [C] Avanti [D] Anga Correct Answer: B [Surasena] 167. Earth, Water, Fire, Air, and Ether (sky) these are the panchtatvas (5 elements) and when they combine they give rise to new objects. This is a principle of which of the Vedic philosophies? [A] Sankhya [B] Yoga [C] Vaisheshika [D] Advita Correct Answer: C [Vaisheshika] 168. Which among the following system in the Vedic Age has provided for a marriage of a childless widow with the brother of blood relation of her deceased husband for the sake of progeny ? [A] Amaju [B] Niyoga [C] Arsa [D] none of the above Correct Answer: B [Niyoga] 169. Which among the following veda has been described as “Braham Veda”? [A] Rigveda [B] Samaveda [C] Yajurveda [D] Atharavaveda Correct Answer: D [Atharavaveda] 170. “Gayatri Mantra” has been mentioned in which of the Vedas? [A] Rigveda [B] Samaveda [C] Yajurveda [D] Atharvaveda Correct Answer: A [Rigveda] Explanation: Gayatri Mantra comes from the third Mandala of the Rigveda. This Mandala has 62 hymns, mainly to Agni and Indra. It is one of the “family books” (mandalas 2-7), the oldest core of the Rigveda. This Mandala also mentions Kikats, a non-Aryan tribe, which most scholars agree that was resident of somewhere in near Magadha in modern Bihar. 171. Bhimbethka in Madhya Pradesh is an ancient Indian site where art prehistoric age are found preserved in rock-shelters. These arts belong to: 43 | P a g e shop.ssbcrack.com

MCQs

INDIAN HISTORY

MCQs

1. Paleolithic age 2. Mesolithic age 3. Chalcolithic age Choose the correct option: [A] Only 1 [B] Only 1 & 2 [C] Only 2 & 3 [D] 1,2 & 3 Correct Answer: D [1,2 & 3] 172. Which among the following are directly linked to Vedic Literature: 1. Dharamasutra 2. Acaranga Sutra 3. Avatamsaka Sutra Choose the correct option: [A] Only 1 [B] 1 & 2 [C] 1 & 3 [D] 1, 2 & 3 Correct Answer: A [Only 1] 173. Which among the following was / were the reasons behind the Rise of Magadha as imperial power in Ancient India: 1. Strategic position surrounded by five hills and aggressive policy of rulers 2. Rich and fertile zone with good communication channels 3. Association with Buddhism and Gautam Buddha Choose the correct option: [A] Only 1 [B] 1 & 2 [C] 2 & 3 [D] 1,2 & 3 Correct Answer: B [1 & 2] 174. Consider the following statements: 1. The abundant female images suggest that goddesses played a central role in the Indus Valley civilization 2. Female God Figurine were abundant but male rare in the Chalcolithic communities Which among the above statements is / are correct? [A] Only 1 [B] Only 2 [C] Both 1 & 2 [D] Neither 1 nor 2 Correct Answer: C [Both 1 & 2] 175. In context with the “Village Headman” of Gupta Era in ancient India, consider the following statements: 1. The Village headman used to manage the village daily affairs with the consent of the King 2. No land transaction could be effected without the consent of the Village headman 44 | P a g e shop.ssbcrack.com

MCQs

INDIAN HISTORY

MCQs

Which among the above statements is / are correct? [A] Only 1 [B] Only 2 [C] Both 1 & 2 [D] Neither 1 nor 2 Correct Answer: B [Only 2] Explanation: First statement is incorrect because, they used to manage village affairs with the consent of elders of the village 176. In which year, collector in each District of Bengal was made responsible for settling the revenue and collecting it? [A] 1780 [B] 1786 [C] 1790 [D] 1792 Correct Answer: B [1786] 177. Consider the following statements: 1. The rebels in the revolt 1857 had no clear perspective 2. Many leaders of the revolt were defeated because of betrayal Which among the above statements is / are correct? [A] Only 1 [B] Only 2 [C] Both 1 & 2 [D] Neither 1 nor 2 Correct Answer: C [Both 1 & 2] 178. The Pasupati seal, discovered at Mohenjodaro archaeological site by Sir John Marshall, which has given rise to the Proto-Shiva theory, has the following characteristics: 1. Yogic Posture in possibly Hath Yoga 2. Surrounded by animals 3. Sitting on a lotus Choose the correct option: [A] Only 1 [B] 1 & 2 [C] 2 & 3 [D] 1, 2 & 3 Correct Answer: B [1 & 2] Explanation: Figure is sitting on throne and the sitting posture is referred as lotus posture 179. At which among the following places, Harshavardhana established a large Buddhist monastery? [A] Nalanda [B] Pataliputra [C] Vaishali [D] Nasik Correct Answer: A [Nalanda]

45 | P a g e

shop.ssbcrack.com

MCQs

INDIAN HISTORY

MCQs

180. During the period of Harshavardhana the fields were watered by the pots which were called Ghati yantra. The writing of which among the following authors records this? [A] Hiuen-tsang [B] Subhandhu [C] Dandin [D] Banabhatta Correct Answer: A [Hiuen-tsang] 181. Savatthi was capital of which among the 16 Mahajanpadas? [A] Kasi [B] Kosala [C] Matsya [D] Anga Correct Answer: B [Kosala] 182. Which among the following Buddhist Canon is related to dealing with rules for monks and nuns ? [A] Vinaya Pitaka [B] Sutta Pitaka [C] Abhidhamma Pitaka [D] None of the above Correct Answer: A [Vinaya Pitaka] Explanation: Tripitaka or Three Baskets is a traditional term used for various Buddhist scriptures. It is known as pali Canon in English. The three pitakas are Sutta Pitaka, Vinaya Pitaka and Abhidhamma Pitaka. Sutta Pitaka: It contains over 10 thousand suttas or sutras related to Buddha and his close companions. This also deals with the first Buddhist council which was held shortly after Buddha’s death, dated by the majority of recent scholars around 400 BC, under the patronage of king Ajatasatru with the monk Mahakasyapa presiding, at Rajgir. It is divided into various sections as shown in following graphics: Vinaya Pitaka: The subject matter of Vinay Pitaka is the monastic rules for monks and nuns. It can also be called as Book of Discipline. Its three books are Suttavibhanga, Khandaka and Parivara. Abhidhammapitaka deals with the philosophy and doctrine of Buddhism appearing in the suttas. However, it does not contain the systematic philosophical treatises. There are 7 works of Abhidhamma Pitaka which most scholars agree that don’t represent the words of Buddha himself. 183. The word” Vihara” in Buddhism means____________? [A] Residence of Monks [B] Hall of Worship [C] Congregation Hall [D] Garden Correct Answer: A [Residence of Monks] Explanation: Viharas were for the purpose of living, Chaityas were assemblies for the purpose of discussions. Further, Chaityas were with Stupas, Viharas did not have stupas. 184. The first Indian Mathematician who treated Mathematics as a different discipline? [A] Varahmihira [B] Aryabhatta [C] Ramanujan [D] Baudhayan Correct Answer: B [Aryabhatta] 46 | P a g e shop.ssbcrack.com

MCQs

INDIAN HISTORY

MCQs

185. Tanjore, Anhilwada, Dhara & Kanauj were the capitals of respectively which among the following dynasties? [A] Pratihara, Parmara, Solanki, Chola [B] Chola, Solanki, Parmara, Pratihara [C] Pandya, Solanki, Pratihara, Chola [D] Chola, Parmara, Pratihara, Solanki Correct Answer: B [Chola, Solanki, Parmara, Pratihara] 186. What is the correct Chronological Order of Nandas, Sungas, Mauryas & Hayankas dynasties of Ancient India? [A] Nandas, Sungas, Mauryas, Hayankas [B] Mauryas, Hayankas, Sungas, Nandas [C] Hayankas, Nandas, Mauryas, Sungas [D] Nandas, Mauryas, Sungas, Haranyakas Correct Answer: C [Hayankas, Nandas, Mauryas, Sungas] 187. Who among the following kings was a contemporary of both Mahavira and Buddha? [A] Ajatshatru [B] Nandivardhana [C] Bimbisara [D] Ashoka Correct Answer: C [Bimbisara] Explanation: Bimbisara was contemporary of both Mahavira and Buddha. The Jain texts mention that he was a disciple of Lord Mahavira. The Buddhist texts mention that he met Buddha before enlightment. Buddha promised him to visit his capital after he gets enlightment. He was a patron of Buddha and as a mark of goodwill; he presented the Bamboo Grove (karanda venu vana) to the Sangha. 188. Ashvaghosha, who has written the marvelous work “Buddhacharita” was a court poet of which among the following kings? [A] Ashoka [B] Kanishka [C] Samudra gupta [D] Ajatshatru Correct Answer: B [Kanishka] 189. Who among the following was King of Magadha at the time of Mahaparinirvana of Gautam Buddha? [A] Bimbisara [B] Udayi [C] Ajatshatru [D] Chandragupta Maurya Correct Answer: C [Ajatshatru] 190. Ceylon became a stronghold of which among the following branches of Buddhism? [A] Mahayan [B] Hinayan [C] Mahnendra [D] Theravada Correct Answer: B [Hinayan] 47 | P a g e shop.ssbcrack.com

MCQs

INDIAN HISTORY

MCQs

191. At which among the following places, Chandragupta Maurya spent his last days ? [A] Nalanda [B] Ujjain [C] Shravana Belgola [D] Kashi Correct Answer: C [Shravana Belgola] 192. There is only one edict, in which Ashoka refers himself as King of Magadha (Laja Magadhe). Identify the edict from the following: [A] Minor Rock Edict of Mashi [B] The Queens edict [C] Bhabhru Edict [D] Dhauli Pillar Inscription Correct Answer: A [Minor Rock Edict of Mashi] 193. Pushyamitra Shunga, who was originally a Senapati of the Mauryan Empire and assassinated the last Mauryan Emperor Brihadrath in 185 BC was a ____? [A] Brahaman [B] Kshatriya [C] Vaishya [D] Shudra Correct Answer: A [Brahaman] 194. “All Indians are Free and not even one of them is a slave”. Who among the following has observed the above about the Mauryan Empire? [A] Megasthenes [B] Hiuen Tsang [C] Ashoka [D] Strabo Correct Answer: A [Megasthenes] 195. What was the function of Samharta in Mauryan Administration? [A] To Collect Revenue [B] To Work as a Judge [C] Leadership of Army [D] Delivering Justice Correct Answer: A [To Collect Revenue] 196. Which among the following kings have been linked to the Sudaman Caves ? [A] Ashoka [B] ChandraGupta Maurya [C] Bindusara [D] Bimbisara Correct Answer: A [Ashoka] Explanation: Sudaman Caves were built by Ashoka and were dedicated to Ajivikas

48 | P a g e

shop.ssbcrack.com

MCQs

INDIAN HISTORY

MCQs

197. Who among the following is ascribed for the work Mudrarakshaha? [A] Kalidasa [B] Vishakhadatta [C] Kautilya [D] Bhatrihari Correct Answer: B [Vishakhadatta] Explanation: Kautilya`s Arthashastra, Ashoka`s Inscriptions, Megasthanese Indica and Vishakhadatta`s Mudrarakshash are the main sources of Mauryan History 198. For the first time in Indian History, the description of the State Administration was found for which among the following periods? [A] Gupta Period [B] Maurya Period [C] Shunga Period [D] Satavahana period Correct Answer: B [Maurya Period] 199. Rigveda was written during which among the following ages? [A] Stone age [B] Copper age [C] Iron age [D] All the above Correct Answer: C [Iron age] Explanation: Rig-Veda is known as the oldest religious text in the world. It is also known as “First testament” of mankind. It was composed around 1700 BC. Last hymns were composed between 15001200 BC. It’s a collection of hymns by a number of priest families. It is organized in 10 books which are called Mandalas 200. Which among the following dynasties is known to have given the largest grant of villages to the temples and Brahamans? [A] Prathihara [B] Guptas [C] Pala [D] Satavahana Correct Answer: B [Guptas] 201. Vedic mythology refers to an ancient settlement called Pushkalavati after Pushkal, who was the son of King Bharata in the epic Ramayan. What is the modern location of Pushkalavati? [A] Panipat [B] Pushkar [C] Peshawar (in Pakistan) [D] Punjab Correct Answer: C [Peshawar (in Pakistan)] Explanation: Pushkalavati was capital of ancient Kingdom of Gandhara. Its ruins are currently located in Charsadda, Pakistan, located in Peshawar valley in the Khyber Pakhtunkhwa province (formerly NWFP). Pushkalavati was home of the Sanskrit grammarian Panini.

49 | P a g e

shop.ssbcrack.com

MCQs

INDIAN HISTORY

MCQs

202. In ancient India, the term Nishka was related to which among the following? [A] Crop [B] Tax [C] Currency [D] Tool Correct Answer: C [Currency] 203. Who among the following is the author of Dharmasastra? [A] Manu [B] gautama [C] kasyap [D] Many Correct Answer: D [Many] 204. Yavanpriya in ancient Hindu Texts refers to which among the following? [A] Peppar [B] Clove [C] Silk [D] Muslin Correct Answer: A [Peppar] 205. Kotada timba, which contains ruins of an ancient Harappan city is located in which among the following states? [A] Punjab [B] Rajasthan [C] Uttar Pradesh [D] Gujarath Correct Answer: D [Gujarath] 206. Hieroglyphic Writing is a part of which ancient civilization? [A] Indus Valley [B] Egyptian [C] Chinese [D] Mesopotamia Correct Answer: B [Egyptian] Explanation: Between 4000 and 3000 BC, Egyptians developed a kind of picture writing known as hieroglyphics. The first writings consisted of pictures of objects, such as a house. Gradually, they started using ideas as well as objects. For example, a picture of an eye could mean sight or eye. In time, the writers also used picture signs to indicate sound 207. The silver coins of which among the following kings carried portraits and bilingual legends, which were inspired by the Kshatrapa types? [A] Satavahanas [B] Kushanas [C] Guptas [D] Mauryas Correct Answer: A [Satavahanas ] 50 | P a g e

shop.ssbcrack.com

MCQs

INDIAN HISTORY

MCQs

208. Who among the following was the first famous and powerful king of Sen dynasty? [A] Ballal Sen [B] Vijay Sen [C] Samanta Sen [D] Vikram Sen Correct Answer: C [Samanta Sen] 209. As per the Hindu Philosophy, what is the number of articles under the restraints and practices (Yamas and Niyamas)? [A] 5 [B] 15 [C] 20 [D] 36 Correct Answer: C [20] 210. As per Vedic rituals, which among the following is the divine messenger who receives the prayers and conveys them to the heavenly spheres? [A] Agni [B] Soma [C] Varuna [D] Indra Correct Answer: A [Agni] 211. Which among the following was the state emblem of Pandyas? [A] Fish [B] Bow & Arrow [C] Sun [D] Horse Correct Answer: A [Fish] 212. Simuka was an Indian king and the founder of which among the following dynasty? [A] Satavahana [B] Kadamba [C] Ganga [D] Kakatiya Correct Answer: A [Satavahana] Explanation: Simuka was the founder of the Satavahana Dynasty and he is believed to have destroyed the Shunga Power. He did so with the aid of the Rathikas and Bhojakas. He reigned for around 23 years and was beheaded by his brother Kanha, who succeeded him. 213. With which among the following branches of Mathematics, Sulva Sutras are related to__? [A] Algenbra [B] Geometry [C] Calculus [D] Game Theory Correct Answer: B [Geometry]

51 | P a g e

shop.ssbcrack.com

MCQs

INDIAN HISTORY

MCQs

214. Which among the rulers of Magadh founded the city of Patliputra (Patna) on the bank of river Ganges? [A] Chandragupta Maurya [B] Ajatshatru [C] Bimbisara [D] Bindusara Correct Answer: B [Ajatshatru] 215. A civilization (mature period 2600–1900 BCE) which was centred mostly in the western part of the Indian Subcontinent? [A] Vedic civilization [B] Mahajanpada [C] Kushan Empire [D] Indus Valley Civilization Correct Answer: D [Indus Valley Civilization] 216. The philosophy of “Nyaya Darshan” has been ascribed to who among following? [A] Kanad [B] Gautam [C] Panini [D] Bhartṛhari Correct Answer: B [Gautam] 217. At which among the following places, the famous Bull Seal of Indus Valley Civilization was found? [A] Harappa [B] Mehrgarh [C] Mohenjo-Daro [D] Adamagarh Correct Answer: C [Mohenjo-Daro] 218. Which among the following mahajanpada was situated on the bank of river Godavari? [A] Kamboja [B] Asaka [C] Avanti [D] Malla Correct Answer: B [Asaka] Explanation: Mahajanpada Asaka Located on the banks of the Godavari River. Its capital was Potali, Potana or Podana, which now lies in the Nandura Tehsil, Buldana district in the Indian state of Maharashtra and it was the only Mahajanapada situated to the south of the Vindhya Range, and was in Dakshinapatha. 219. The Brihadishwara Temple (Rajarajeswaram) is situated at? [A] Madhya Pradesh [B] Thanjavur [C] Lucknow [D] Andhra Pradesh Correct Answer: B [Thanjavur] 52 | P a g e

shop.ssbcrack.com

MCQs

INDIAN HISTORY

MCQs

Explanation: Brihadeeswarar Temple or Peruvudaiyar Kovil or Rajrajeshwaram temple at Thanjavur is the world’s first complete “granite” temple. It was built by Rajraja Chola-I and is a part of UNESCO’s world Heritage sites. The Vimana or the temple tower (known as Raja Gopuram) is 216 Feet in height and is one of the tallest buildings of its kind. The Nandi is carved out of a single rock. This temple has completed 1 millennium in 2010. It was dictated by lord Shiva to Rajraja Chola I, when he triumphed Ilam (Sri Lanka) Island. 220. Which dynasty Asoka the Great was emperor of ? [A] Maurya Dynasty [B] Kakatiya dynasty [C] Nanda dynasty [D] Pradyota dynasty Correct Answer: A [Maurya Dynasty] 221. Which among the following sites near Pondicherry gives best support to the view on an ancient trade between Rome and the ancient Tamil country of India? [A] Villianur [B] Suttukeni [C] Arikamedu [D] Andiarpalayam Correct Answer: C [Arikamedu] 222. Which among the following was capital of Mitrakas? [A] Kosambi [B] Vallabhi [C] Odantapuri [D] Vikramsila Correct Answer: B [Vallabhi] 223. In which state of India is located the ancient trading depot of Bharukaccha? [A] Rajasthan [B] Gujarath [C] Maharastra [D] Karnataka Correct Answer: B [Gujarath] 224. Indus Valley site at Surkotada is located in which among the following states ? [A] Rajasthan [B] Haryana [C] Gujarath [D] Punjab Correct Answer: C [Gujarath] Explanation: The site at Surkotada is located 160 km north-east of Bhuj, in the district of Kutch, Gujarat. In the Indus Valley Civilization, the remains of Horse bones have been found from Surkotada site. 225. Dhauli Hills, which have major Edicts of Ashoka engraved on a mass of rock, by the side of the road leading to the summit of the hills are located in which among the following states of India? 53 | P a g e

shop.ssbcrack.com

MCQs

INDIAN HISTORY

MCQs

[A] Bihar [B] Madhya Pradesh [C] Rajasthan [D] Odisha Correct Answer: D [Odisha] 226. The name of which among the following king of the Hellenistic Seleucid Kingdom has been inscribed on the edicts of Asoka? [A] Antiochus I Soter [B] Seleucus II [C] Antiochus II Theos [D] Seleucus IV Philopator Correct Answer: C [Antiochus II Theos] Explanation: Ashoka’s Hellenistic contemporaries are 1. Antiochus II Theos –Syria 2.

Ptolemy II philadelpus –Egypt

3.

Mages –Cyrne

4.

Antigonus Gonatas –Macedonia

5.

Alexander -Epirus

227. The construction of the Delhi Iron Pillar, now located at the Quwwat-ul-Islam mosque in New Delhi was constructed in which period? [A] Maurya [B] Satavahana [C] Gupta [D] Kushana Correct Answer: C [Gupta] Explanation: As per Natya-darpana (by Vishakadata), the Iron pillar had been put up by Chandragupta II himself after defeating Vahilakas. He designated that pillar as a memory of the victory. The pillar is made of 98% wrought iron and has stood more than 1,600 years without rusting or decomposing. 228. After the victory on which among the following, the Vikrama Samvat was founded by the Tuar Rajput emperor Vikramaditya of Ujjain? [A] Shakas [B] Hunas [C] Greeks [D] Kushans Correct Answer: A [Shakas] 229. Which among the following ruler is known for Junagarh Rock Inscription? [A] Rudradaman I [B] Jivadaman [C] Damajadasri [D] Jayadaman Correct Answer: A [Rudradaman I] 54 | P a g e

shop.ssbcrack.com

MCQs

INDIAN HISTORY

MCQs

Explanation: The Junagarh Rock inscription of Rudradaman is considered as an early example of chaste Sanskrit, written in mid second century AD. It mentions that one of Chandragupta Maurya’s governors, Pushyagupta, was responsible for building a dam on Sudarshana Lake near Girnar in Kathiawar. From another inscription of Skandgupta we came to know that this very dam was repaired during his reign, almost 800 years after it was built 230. Bhandi was a chief secretary of which among the following Indian Kings? [A] Samudragupta [B] Chandragupta Vikramaditya [C] Harshavardhan [D] Skandgupta Correct Answer: C [Harshavardhan] Explanation: Bhandi was a leading noble of Kannauj and on advice of the political leaders of Kannauj; he offered the crown of Kannauj to Harsha after death of Grahavarmana. Bhandi was later described as one of the chief officers of Harsha. When Harsha chased Shashanka for release of his sister, through Bhandi only Harsha could know that his sister has been released and Shashanka has escaped 231.As per Asoka’s inscriptions, which among the following place was declared tax free and proclaimed only 1/8th part as taxable? [A] Kushinagar [B] Lumbini [C] Kathmandu [D] Sarnath Correct Answer: B [ Lumbini] Explanation: At the 20th anniversary of his enthronement, Asoka announced Lumbini as tax-free and proclaimed only 1/8th part as taxable. Description of this fact is found in the inscriptions of Nigliva and Rumindei. 232.Which among the following places have given the earliest evidence of agriculture in Indian subcontinent? [A] Pratapgarh [B] Mehrgarh [C] Quetta [D] Kalat Correct Answer: B [ Mehrgarh] Explanation: The earliest evidence of Agriculture in Indian subcontinent is found at Mehrgarh, which is located in Baluchistan state of Pakistan. 233.Which among the following Vakataka ruler performed all the seven sacrifices viz. Agnishtoma, Aptoryama, Ukthya, Shodasin, Atiratra, Vajapeya, Brihaspatisava, Sadyaskra and four Asvamedhas? [A] Rudrasena-I [B]] Pravarsena-I [C] Prithvisena-I [D] ]Narendrasena-I Correct Answer: B [ Pravarsena-I ] Explanation: Vakataka ruler, Pravarsena-I, is said to have performed all the 7 sacrifices.viz .Agnishtoma, Aptoryama, Ukthya, Shodasin, Atiratra, Vajapeya, Brihaspatisava, Sadyaskra and four Asvamedhas. 55 | P a g e

shop.ssbcrack.com

MCQs

INDIAN HISTORY

MCQs

234.Who among the following was the 23rd Jain Tirthankara? [A] Nemi Natha [B] Mahavira [C] Parshvanath [D] Malinath Correct Answer: C [ Parshvanath] Explanation: Parshvanath was the twenty third Jain Tirthankara. He was a Kshatriya and son of Ashvasena, King of Banaras (Varanasi). 235.The Satapatha Brahmana and Taitriya Brahmana are the Brahmana texts of ___: [A] Rigveda [B] Yajurveda [C] Samaveda [D] Atharavaveda Correct Answer: B [ Yajurveda] Explanation: Satapatha Brahmana and Taitriya Brahmana are the Brahmana texts of Yajurveda. 236.Which among the following is the most unique feature of Dholavira site of the Indus Valley Civilization? [A] Its cities were horizontally divided in multiple parts [B] Its cities were unevenly divided [C] Its cities were divided into 3 parts [D] None of these Correct Answer: C [ Its cities were divided into 3 parts] Explanation: The most unique features of Dholavira, an Indus valley site in Gujarat, is its division. While other Indus Valley sites were divided into two parts viz. Citadel and Lower Town, Dholavira was divided into three divisions. 237.Which of the following Kingdoms were associated with the life of the Buddha? 1. Avanti 2. Gandhara 3. Kosala 4. Magadha Select the correct answer using the code given below: [A] 1, 2 and 3 [B] 2 and 4 [C] 3 and 4 only [D] 1, 3 and 4 Correct Answer: C [ 3 and 4 only ] Explanation:Avanti was not directly related to life of Buddha, so this needs to be opted out. We have read that Pasenadi (Prasenajit), king of Kosala, was the Buddha’s contemporary and is frequently mentioned in Pali texts. Kosala and Magadha were linked through matrimonial ties. Buddha wandered through the towns and villages in the kingdoms of Kosala and Magadha teaching his philosophy. Gandhara is not directly associated with the life of Buddha. It was expansion of Maurya empire that Gandhara received much Buddhist influence, notably during the reign of Asoka.

56 | P a g e

shop.ssbcrack.com

MCQs

INDIAN HISTORY

MCQs

238.Which of the following is not a correct feature of Rig-Vedic economy? [A] Primarily pastoral economy [B] Cultivation of multiple cereals [C] Abundance of wage earners [D] Sedentary agriculture society Correct Answer: C [ Abundance of wage earners] Explanation: here are no evidences of wage earners in agriculture in Rig-Vedic Economy. 239.Which of the following term is used for a “school” of learning and teaching the branches of Vedas? [A] Shakha [B] Charna [C] Ratha [D] Yajna Correct Answer: B [ Charna ] Explanation: Charana refers to the Guru-pupil lineage or school for teaching and learning of Vedas in ancient India. 240.Who among the following is the author of oldest surviving Prakrit grammar Prākrita Prakāśa? [A] Pingala [B] Yaska [C] Vararuchi [D] Shaunaka Correct Answer: C [ Vararuchi ] Explanation: The oldest surviving Prakrit grammar is Vararuchi’s Prākrita Prakāśa, whose date is debated. 241. Which of the following Pala Kings founded the Vikramshila University? [A] Gopala [B] Dharmapala [C] Devapala [D] Mahendrapala Correct Answer: B [Dharmapala] Explanation: The Vikramshila University was set up by the Pala dynasty king Dharmapala in the late 8th or early 9th century at Bhagalpur, Bihar. The ancient Vikramshila University was intended to complement the existing world class universities at Nalanda and Takshila. It lasted four centuries before being destroyed during an attack by Bakhtiyar Khilji of the Delhi Sultanate. 242. Which among the following text of Buddhism speaks about the sixteen Mahajanapadas? [A] Digh Nikaya [B] Anguttar Nikaya [C] Maha Parinibbana Sutta [D] Majjhima Nikaya Correct Answer: B [Anguttar Nikaya] 243. “Mahamaya”, the mother of Guautam Buddha belonged to which among the following dynasties? [A] Shakya [B] Koliyan

57 | P a g e

shop.ssbcrack.com

MCQs

INDIAN HISTORY

MCQs

[C] Lichchavi [D] Maurya Correct Answer: B [Koliyan] Explanation: Gautma Buddha’s mother ‘Mahamaya’ belonged to Koliya tribe and she married Suddhodhana, who was a king of Sakya clan. The Sakya and Koliya ruled on opposite banks of the Rohni river in current Rupendehi District of Lumbini, Nepal. Both of them were republics. 244. Which among the following site has yielded the cultural remains from Neolithic to Harappan era? [A] Dholavira [B] Mohenjo-Daro [C] Harappa [D] Mehrgarh Correct Answer: D [Mehrgarh] Explanation: Mehrgarh is the oldest agricultural settlement in the Indian subcontinent Agriculture-based Neolithic settlements. Despite being the agriculture settlement, it used only stone tools, so is why placed in Neolithic Era. It flourished in the seventh millennium B.C. Mehrgarh is located on the Bolan River, a tributary of the Indus, at the eastern edge of the Baluchistan plateau overlooking the Indus plain. 245. Ashoka called the third buddhist council at which of the following places? [A] Magadha [B] Pataliputra [C] Burma [D] Therevada Correct Answer: B [Pataliputra] Explanation: The Third Buddhist council was convened in about 250 BCE at Asokarama in Pataliputra, supposedly under the patronage of Emperor Ashoka. The council is recognized and known to both the Theravada and Mahayana schools, though its importance is central only to the Theravada. 246. Apart from Madurai, which among the following was also a capital of Pandyas? [A] Korkai [B] Kottaikkadu [C] Thirukkoloor [D] Pragasapuram Correct Answer: A [Korkai] Explanation: Korkai, which is now an insignificant village in Tamil Nadu, was the commercial capital and important port of the Pandya Kingdom. Korkai has been a cradle of South Indian Civilization and as per traditions; it is considered the home of three brothers who were supposed to have founded the Pandya, Chera and Chola Kingdoms. 247. Which among the following kings assassinated Brihadrath, the last Maurya King? [A] Agnimitra [B] Pushyamitra Shunga [C] Vasujyeshtha [D] Vasumitra Correct Answer: B [Pushyamitra Shunga] Explanation: Brihadratha was a weak ruler and his Senapati Pushymitra Shunga while a parading the entire Mauryan army before Brihadratha to show him the strength of the army assassinated him and this was the end of the Mauryas. Pushyamitra Shunga founded the Shunga dynasty in around 185-183 BC. 58 | P a g e shop.ssbcrack.com

MCQs

INDIAN HISTORY

MCQs

248. Who among the following was the first Tirthankar of Jainism? [A] Rishabh dev [B] Parshwanath [C] Neminath [D] Sambhavanath Correct Answer: A [Rishabh dev] Explanation: Lord Rishabhdev was the first Tirthankara of Jainism. He was born at Ayodhya in the Ikshwaku Kula or clan. In Hinduism he is known to be an avatara or incarnation of Vishnu. The name of Rishabh’s parents has been mentioned in the Bhagvata Purana. 249. Where is Largest Monastery in India? [A] Jetavana [B] Kardang Monastery [C] Tawang Monasatery [D] Zang Dhok Palri Phodang Correct Answer: C [Tawang Monasatery] Explanation: Tawang monastery is the largest of its kind in the country and is one of the largest monasteries in Asia. It is said to be one of the biggest Buddhist monasteries in the world outside of Lhasa, Tibet. It is also known by another Tibetan name, ‘Galden Namgey Lhatse’, which means ‘celestial paradise in a clear night’. 250. Which century Gautama Buddha was born in? [A] 4th century BC [B] 5th century BC [C] 6th century BC [D] 7th century BC Correct Answer: C [6th century BC] Explanation: Gautam Siddhartha was the name of Buddha and he was the founder of Buddhism also known as Gautam Buddha. Gautam Buddha was born in 563 BC in Lumbini (now in Nepal) in Sakya Kshatriya clan of Kapilvastu. He died in 483 BC. 251. Which among the following sites in Haryana, is the place as per legends where Lord Krishna delivered the sermon of Bhagavad Gita? [A] Jyotisar [B] Pehowa [C] Shahabad [D] Ladwa Correct Answer: A [Jyotisar] 252. At which among the following places, Gautam Buddha preached his last sermon? [A] Sarnath [B] Rajgir [C] Vaishali [D] Pavapuri Correct Answer: C [Vaishali]

59 | P a g e

shop.ssbcrack.com

MCQs

INDIAN HISTORY

MCQs

253. Which among the following is NOT a Vedanga? [A] Kalpa [B] Sankya [C] Nirkuta [D] Chhanda Correct Answer: B [Sankya] 254. The Excavated Buddhist Site, Lalitgiri, which was notified as Centrally Protected in the year 1937, is located in which among the following states of India? [A] Andhra Pradesh [B] Odisha [C] West Bengal [D] Karnataka Correct Answer: B [Odisha] 255. Every five years, Harshavardhana used to donate all his possessions at an assembly at which among the following places? [A] Ujjain [B] Banaras [C] Prayag [D] Kannauj Correct Answer: C [Prayag] 256. During the reign of which among the following Kings, the Gandhar Style of Art developed? [A] Kushana [B] Shaka [C] Indo-Parthian [D] Indo-greeks Correct Answer: A [Kushana] 257. History of which among the following parts of India has been described in the “Rajtarangini” of Kalhana? [A] Kashmir [B] Malwa [C] Magadha [D] Gandhar Correct Answer: A [Kashmir] 258. Consider the following statements about Lion Capital of Asoka which has been now adopted as India’s emblem: 1.It was erected around 250 BC at Sarnath 2. It was engraved with “Satyamev Jayate” 3. Its abacus is engraved with 4 animals viz. lion, elephant, deer and bull Which among the above statements is/ are correct? [A] Only 1 [B] 1 & 2 [C] 2 & 3 [D] 1, 2 &3 Correct Answer: A [Only 1] 60 | P a g e

shop.ssbcrack.com

MCQs

INDIAN HISTORY

MCQs

Explanation: Satyamev Jayate was later adopted from Mundkopnishada. The four animals are guardians of the four directions: the lion of the north, the elephant of the east, the horse of the south and the bull of the west 259. Which among the following site in Bihar is a Buddhist as well as Janism pilgrim place? [A] Bodhgaya [B] Rajgir [C] Nalanda [D] Vaishali Correct Answer: D [Vaishali] 260. Which king of Ancient India has been given the title” Indian Nepoleon” by VA Smith? [A] Samudragupta [B] Kumaragupta [C] Chandragupta Vikramaditya [D] Skandagupta Correct Answer: A [Samudragupta] Explanation: Samudragupta believed in the policy of war and conquest and because of his bravery and generalship he is known as the “Napoleon of India” by historian Vincent. A. Smith. 261.Works of 11th century Sanskrit poet Jayadeva have become the basis of which among the following dance forms of India? [A] Odissi [B] Bharatnatyam [C] Kathak [D] Manipuri Correct Answer: A [Odissi] 262. Yājñavalkya Smṛti was written in which among the following era? [A] Vedic Period [B] Later Vedic Period [C] Maurya Period [D] Gupta Period Correct Answer: D [Gupta Period] 263. In which period, “Arthshastra” was written ? [A] Gupta [B] Maurya [C] Shaka [D] Kushan Correct Answer: B [Maurya] Explanation: Arthshastra was written by Kautilya or Vishnugupta or Chanakya, who was a professor at Taxila University and later the prime minister of the Maurya Empire. The manuscript was discovered by Arya Sharma Shashtri in 1904. Arthshastra has been divided in 15 Adhikarans and 180 Prakarans. It is related to money and politics and contains 6000 shlokas

61 | P a g e

shop.ssbcrack.com

MCQs

INDIAN HISTORY

MCQs

264. Who has written “Kalpsutra”? [A] Haribhadra [B] Bhadrabahu [C] Sudharma Swami [D] Nemichandra Correct Answer: B [Bhadrabahu] Explanation: The Kalpasutra is a Jain text containing the biographies of the Jain Tirthankaras, most notably Parshvanath and Mahavira, including the latter’s Nirvana. The author of this text is Bhadrabahu. 265. Agnimitra, who is the hero of Kalidasa’s ‘Malvikagnimitram’ was a king of which of the following dynasties? [A] Sunga [B] Kanva [C] Satavahana [D] Maurya Correct Answer: A [Sunga] Explanation: Malavikagnimitram is the Sanskrit play, which depicts Agnimitra as its hero. Malvika is a maid servant whom Agnimitra falls in love. This was known to his chief queen, who imprisions her. Later it was known that Malvika was of a royal birth and she was accepted as queen of Agnimitra. Malavikagnimitram gives account of Rajsuya Yajna of Pushyamitra Shunga, father of Agnimitra. 266. Kushinara or Kushinagar, where Lord Buddha attained parinirvana was located in which among the following provinces of ancient India? [A] Anga [B] Magadha [C] Malla [D] Vatsa Correct Answer: C [Malla] Explanation: The Buddha belonged to a small gana known as the Sakya gana, and was a kshatriya. When he was a young man, he left the comforts of his home in search of knowledge. He wandered for several years, meeting and holding discussions with other thinkers. He finally decided to find his own path to realisation, and meditated for days on end under a peepal tree at Bodh Gaya in Bihar, where he attained enlightenment. After that, he was known as the Buddha or the Wise One. He then went to Sarnath, near Varanasi, where he taught for the first time. Buddha attained Nirvana at Kusinara a village some 180km from Varanasi and it was in the state of Mallas. 267. Sarnath, is related to which among the following events in the life of Buddha? [A] First Sermon [B] Last Sermon [C] Mahaparinibbana [D] Dhammachakkapavattana Correct Answer: A [First Sermon] 268. In which among the following metals, largest number of coins were issued in Gupta Era? [A] Gold [B] Silver [C] Copper [D] Brass Correct Answer: A [Gold] 62 | P a g e shop.ssbcrack.com

MCQs

INDIAN HISTORY

MCQs

269. The deity in most of the Chola temples was__? [A] Vishnu [B] Shiva [C] Brahma [D] Krishna Correct Answer: B [Shiva] 270. The term “Vitasta” in Rig Veda refers to which among the following rivers? [A] Indus [B] Beas [C] Jhelum [D] Chenab Correct Answer: C [Jhelum] Explanation: Rigvedic Name and Modern Names of Indian Rivers: Sindhu- Indus Vitasta- Jhelum Askini- Chenab Vipasa- Beas Purushini- Ravi Satudri- Sutlej Gomal- Gomati Drishvadati- Ghaggar 261.Works of 11th century Sanskrit poet Jayadeva have become the basis of which among the following dance forms of India? [A] Odissi [B] Bharatnatyam [C] Kathak [D] Manipuri Correct Answer: A [Odissi] 262. Yājñavalkya Smṛti was written in which among the following era? [A] Vedic Period [B] Later Vedic Period [C] Maurya Period [D] Gupta Period Correct Answer: D [Gupta Period] 263. In which period, “Arthshastra” was written ? [A] Gupta [B] Maurya [C] Shaka [D] Kushan Correct Answer: B [Maurya] Explanation: Arthshastra was written by Kautilya or Vishnugupta or Chanakya, who was a professor at Taxila University and later the prime minister of the Maurya Empire. The manuscript was discovered by Arya Sharma Shashtri in 1904. Arthshastra has been divided in 15 Adhikarans and 180 Prakarans. It is related to money and politics and contains 6000 shlokas 63 | P a g e shop.ssbcrack.com

MCQs

INDIAN HISTORY

MCQs

264. Who has written “Kalpsutra”? [A] Haribhadra [B] Bhadrabahu [C] Sudharma Swami [D] Nemichandra Correct Answer: B [Bhadrabahu] Explanation: The Kalpasutra is a Jain text containing the biographies of the Jain Tirthankaras, most notably Parshvanath and Mahavira, including the latter’s Nirvana. The author of this text is Bhadrabahu. 265. Agnimitra, who is the hero of Kalidasa’s ‘Malvikagnimitram’ was a king of which of the following dynasties? [A] Sunga [B] Kanva [C] Satavahana [D] Maurya Correct Answer: A [Sunga] Explanation: Malavikagnimitram is the Sanskrit play, which depicts Agnimitra as its hero. Malvika is a maid servant whom Agnimitra falls in love. This was known to his chief queen, who imprisions her. Later it was known that Malvika was of a royal birth and she was accepted as queen of Agnimitra. Malavikagnimitram gives account of Rajsuya Yajna of Pushyamitra Shunga, father of Agnimitra. 266. Kushinara or Kushinagar, where Lord Buddha attained parinirvana was located in which among the following provinces of ancient India? [A] Anga [B] Magadha [C] Malla [D] Vatsa Correct Answer: C [Malla] Explanation: The Buddha belonged to a small gana known as the Sakya gana, and was a kshatriya. When he was a young man, he left the comforts of his home in search of knowledge. He wandered for several years, meeting and holding discussions with other thinkers. He finally decided to find his own path to realisation, and meditated for days on end under a peepal tree at Bodh Gaya in Bihar, where he attained enlightenment. After that, he was known as the Buddha or the Wise One. He then went to Sarnath, near Varanasi, where he taught for the first time. Buddha attained Nirvana at Kusinara a village some 180km from Varanasi and it was in the state of Mallas. 267. Sarnath, is related to which among the following events in the life of Buddha? [A] First Sermon [B] Last Sermon [C] Mahaparinibbana [D] Dhammachakkapavattana Correct Answer: A [First Sermon] 268. In which among the following metals, largest number of coins were issued in Gupta Era? [A] Gold [B] Silver [C] Copper [D] Brass Correct Answer: A [Gold] 64 | P a g e shop.ssbcrack.com

MCQs

INDIAN HISTORY

MCQs

269. The deity in most of the Chola temples was__? [A] Vishnu [B] Shiva [C] Brahma [D] Krishna Correct Answer: B [Shiva] 270. The term “Vitasta” in Rig Veda refers to which among the following rivers? [A] Indus [B] Beas [C] Jhelum [D] Chenab Correct Answer: C [Jhelum] Explanation: Rigvedic Name and Modern Names of Indian Rivers: Sindhu- Indus Vitasta- Jhelum Askini- Chenab Vipasa- Beas Purushini- Ravi Satudri- Sutlej Gomal- Gomati Drishvadati- Ghaggar 271. Which among the following is the correct meaning of a word “Manigramam” used in medieval India? [A] Association of Villages [B] Complex of various markets [C] Association of Merchants [D] Unit of administration in a small village Correct Answer: C [Association of Merchants] Explanation: In ancient times merchants formed guilds to protect their interests. There were several such guilds in south India from the eighth century onwards – the most famous being the Manigramam and Nanadesi. These guilds traded extensively both within the peninsula and with Southeast Asia and China 272. Hieun Tsang, had visited the court of which among the following rulers? [A] Harshavardhana [B] Chandragupta Maurya [C] Asoka [D] Pulkesin II Correct Answer: A [Harshavardhana] Explanation: Hieun Tsang, the Chinese pilgrim, visited India during the reign of Harshavardhana with an aim of securing authentic Buddhist scripts. The Kannauj assembly (643 AD) was held in the honour of Hieun Tsang and to popularise Mahayana sect of Buddhism. He stayed in India for about fifteen years and recorded his experience in his book ,’ Si-Yu-Ki’. The book throws lights on the religion, customs, traditions, etc. of the country. 273. Which among the following Pala rulers is known to have founded the Vikramshila Vihara? [A] Gopala [B] Devpala 65 | P a g e shop.ssbcrack.com

MCQs

INDIAN HISTORY

MCQs

[C] Dharampala [D] Mahendrapala Correct Answer: C [Dharampala] Explanation: Dharamapala, the second ruler of Pala dynasty was a pious Buddhist King and is best known for establishing the Vikramshila University. Dharampala had also built a Vihara at Somapuri, another at Paharpur and yet another Vihara at Odantapuri. The five places viz. Nalanda, Vikramshila, Somapuri, Paharpur and Odantapuri are called Five Mahaviharas. Bakhtiyar Khilji destroyed the Vikramshila University in 1200 AD. Somapura Mahavihara is now located in Bangladesh. It is one of the best known Mahaviharas of Buddhism in Indian subcontinent and is a UNESCO World Heritage Site. 274. The temples at Belur, Halebidu and Somnathpura are supreme examples of which among the following types of Temple Architecture? [A] Nagara [B] Dravida [C] Gadag [D] Vesara Correct Answer: D [Vesara] Explanation: Vesara style architecture is the hybrid architecture of Nagara and Dravida. 275. Nakoda at Rajasthan, which is famous for miracles at Nakodaji, is a temple of which among the following? [A] Vishnu [B] Krishna [C] Buddha [D] Parsvanath Correct Answer: D [Parsvanath] Explanation: Parsvanath was the 23rd of 24th Tirthankaras. 276. Which of the following rivers was known as “Drishdvati” in Vedic Era? [A] Ravi [B] Chenab [C] Jhelum [D] Ghagghar Correct Answer: D [Ghagghar] 277. In which of the following cities of India is located world’s first complete granite temple? [A] Varanasi [B] Mathura [C] Thanjaur [D] Madurai Correct Answer: C [Thanjaur] Explanation: Brihadeeswarar Temple in Thanjaur (in Tamil Nadu) was the world’s first complete granite temple 278. Harshavardhana, who was known as Uttarapathapati, was defeated on the banks of river Narmada by which among the following Kings, also having assumed the title of Dakshinapathapati? [A] Pulkesin I [B] Kirtivarman I 66 | P a g e

shop.ssbcrack.com

MCQs

INDIAN HISTORY

MCQs

[C] Pulkesin II [D] Vikramaditya I Correct Answer: C [Pulkesin II] Explanation: Harshavardhana was defeated by Chalukya king Pulakesin-II 279. Two schools, viz. Mathura School of Art and Gandhara School of Art flourished in which among the following era? [A] Shakas [B] Kushans [C] Satavahanas [D] Guptas Correct Answer: B [Kushans] 280. Who among the following were known as Tocharians? [A] Kushans [B] Shakas [C] Hunas [D] Kalabhras Correct Answer: A [Kushans] Explanation: Kushans are also known as Yuechis or Tocharians. 281. Which among the following dynasty was identified only on the basis of Coins? [A] Gupta [B] Kushana [C] Rastrakuta [D] Chalukya Correct Answer: B [Kushana] 282. Who among the following translated “Gita Govinda” in 1792? [A] Sir Willam Jones [B] Charles Wilkins [C] Sir Robert Chambers [D] H.H. Wilson Correct Answer: A [Sir Willam Jones] Explanation: Gita Goivinda is a sanskirt text composed by Jayadeva in 12th century. In the year 1792 William Jones translated “Gita Govinda’’ into English. Charles Wilkins translated Bhagavad Gita into English. 283. Who among the following legendary kings was also known as “Sarvadamana” in his childhood? [A] Asoka [B] Bharata [C] Chandragupta Maurya [D] Samudra Gupta Correct Answer: B [Bharata] Explanation: Bharta was also known as was also known as “Sarvadamana” in his childhood. Sarvadamana means the subduer of all. India has been called as Bharatavarsha after him.

67 | P a g e

shop.ssbcrack.com

MCQs

INDIAN HISTORY

MCQs

284.The ancient port-site of Godavaya is located in which country? [A] Malaysia [B] Indonesia [C] Myanmar [D] Sri Lanka Correct Answer: D [ Sri Lanka] Explanation: Godavaya is an ancient port trade site on southern coast of Sri Lanka. 285. What was Arikamedu during ancient India? [A] A coastal settlement and trade centre [B] A mountain [C] A city in Aravalli Mountains in Gujarat [D] A Lake Correct Answer: A [ A coastal settlement and trade centre] Explanation: Arikamedu, the ancient Roman trade centre is 4 Kms. South of Puducherry on the right bank of Ariyankuppam river. Arikamedu was an Indo- Roman coastal trading station and an important centre of trade and commerce with the Western world during the early centuries of the Christian era. This port was known to the author of “Periplus of the Erithrian Sea” as Padouke.%% 286.Which of the following was denoted by the term “Kahapana” or “Karshapan” during ancient India? [A] A cloth [B] A trader [C] A monk [D] A coin Correct Answer: D [ A coin ] Explanation: “Kahapana” or “Karshapan” was a common coin during Pre-Gupta period. It was made of copper and also of silver. 287.Buddhism from India was introduced to which current region by Kasyapa Matanga? [A] China [B] South East Asia [C] Sri Lanka [D] Africa Correct Answer: A [ China ] Explanation: Kasyapa Matanga is thought to have been the first to introduce Buddhism to China. According to Chinese tradition, he traveled from India to Lo- yang in China with another Indian monk, known by his Chinese name Chu Falan. They came in C.E. 67 at the invitation of Emperor Ming of the Later Han dynasty. 288.Who among the following is writer of the ‘Kalpasutra’? [A] Simuka [B] Panini [C] Bhadrabahu [D] Patanjali Correct Answer: C [ Bhadrabahu ] Explanation: The Kalpasutra is a Jain text containing the biographies of the Jain Tirthankaras, most notably Parshvanath and Mahavira, including the latter’s Nirvana. The author of this text is Bhadrabahu. 68 | P a g e

shop.ssbcrack.com

MCQs

INDIAN HISTORY

MCQs

289.The Greek Ambassador Deimachus of Plataea had visited the court of which of the following rulers of Magadha? [A] Dhanananda [B] Chandragupta Maurya [C] Bindusara [D] Asoka Correct Answer: C [ Bindusara ] Explanation: Deimachus of Plataea had come as ambassador to Bindusāra or Amitraghāta, the son and successor of Chandragupta Maurya. He along with Megasthenes provides important information about the contemporary society and polity. He was sent by Antiochus I (the son of Seleucus Nikator). 290.Which of the following rulers were contemporary of Buddha? [A] Bimbisar of Magadha [B] Prasenjit (Pansedi) of Kosala [C] Udayan of Avanti [D] All of Above Correct Answer: D [ All of Above ] Explanation: All of the mentioned rulers were contemporary of Buddha. In Magadha, both Bimbisar and Ajatsatru ruled during Buddha’s time. 291. Rummindei Pillar Inscription which talks about taxation in Maurya era has been found at which of the following places? [A] Junagarh in Gujarat [B] Ranchi in Jharkhand [C] Bhabru in Rajasthan [D] Lumbini in Nepal Correct Answer: D [ Lumbini in Nepal ] Explanation: Lumbini Pillar Edict in Nepal is known as the Rummindei Pillar Inscription .The Lumbini Pillar Edict recorded that sometime after the twentieth year of his reign, Ashoka travelled to the Buddha’s birthplace and personally made offerings. He then had a stone pillar set up and reduced the taxes of the people in that area. 292.Which of the following regions was called “Gedrosia” during times of Alexander, the great? [A] Gujarat [B] Rajasthan [C] Punjab [D] Baluchistan Correct Answer: D [ Baluchistan ] Explanation: Gedrosia corresponded to today’s Baluchistan or more specifically Makran. Alexander had lost a lots of soldiers during his march through the Gedrosian desert. 293. The Nitisara of Kamandaka, also known as the Kamandakiya-Nitisara, was composed during the period of: [A] Satavahana dynasty [B] Gupta dynasty [C] Kalachuri dynasty [D] Nanda dynasty Correct Answer: B [Gupta dynasty] 69 | P a g e shop.ssbcrack.com

MCQs

INDIAN HISTORY

MCQs

Explanation: The Nitisara of Kamandaka, also known as the Kamandakiya-Nitisara, was composed during the period of Gupta Dynasty. It was based on Kautilya’s Arthashastra 294. The Neolithic sites Kuchai and Golbai Sasan are located in which Indian state? [A] Maharashtra [B] Bihar [C] Odisha [D] Madhya Pradesh Correct Answer: C [Odisha] Explanation: The Neolithic sites in Odisha include Kuchai in Mayurbhanj district, and the recently excavated site of Golbai Sasan on the banks of Mandakini River. 295. At which of the following Harappan sites, terracotta female figures were discovered? [A] Mohenjo-daro [B] Kot Bala [C] Kot Diji [D] Khirasara Correct Answer: A [Mohenjo-daro] Explanation: Mohenjo-daro is an archaeological site in the province of Sindh, Pakistan. Built around 2500 BCE, it was one of the largest settlements of the ancient Indus Valley civilization. Various discoveries at Mohenjo-daro include- Great Bath (the biggest bath ghat), Great granary, Bronze dancing girl, Bearded man, terracotta toys, Bull seal, Pashupati seal, three cylindrical seals of the Mesopotamian type, a piece of woven cloth. 296. Jorwe culture, a Chalcolithic culture, was first discovered in which of the following states of India? [A] Maharashtra [B] Rajasthan [C] Gujarat [D] Jammu & Kashmir Correct Answer: A [Maharashtra] Explanation: Jorwe is a village and an archaeological site located on the banks of the Godavari River in Ahmednagar district of Maharashtra. This site was first was excavated in 1950-51 under the direction of Hasmukh Dhirajlal Sankalia and Shantaram Bhalchandra Deo. 297. Tat Tvam Asi, a sanskrit phrase, originally occurs in which of the following Upanishads? [A] Mundaka Upanishad [B] Chandogya Upanishad [C] Brhadaranyaka Upanishad [D] Mandukya Upanishad Correct Answer: B [ Chandogya Upanishad ] Explanation: “Tat Tvam Asi” is a Sanskrit phrase is one of the Mahāvākyas (Grand Pronouncements) in Vedantic Sanatana Dharma. It originally occurs in the Chandogya Upanishad, a Sanskrit text embedded in the Chandogya Brahmana of the Sama Veda of Hinduism. 298. Mundaka Upanishad, the Mandukya Upanishad and the Prashna Upanishad are associated with which of the following Vedas? [A] Atharva-Veda [B] Rig-Veda 70 | P a g e shop.ssbcrack.com

MCQs

INDIAN HISTORY

MCQs

[C] Sama-Veda [D] Yajur-Veda Correct Answer: A [ Atharva-Veda ] Explanation: Atharvaveda text includes three primary Upanishads, influential to various schools of Hindu philosophy. These include the Mundaka Upanishad, the Mandukya Upanishad and the Prashna Upanishad. 299. Which of the following plays of Kalidasa tells the story of mortal King Pururavas and celestial nymph Urvashi who fall in love? [A] Malavikagnimitram [B] Abhijñānaśākuntalam [C] Vikramōrvaśīyam [D] None of the above Correct Answer: C [ Vikramōrvaśīyam] Explanation: Vikramōrvaśīyam, a play by Kalidasa, tells the story of mortal King Pururavas and celestial nymph Urvashi who fall in love. As an immortal, she has to return to the heavens, where an unfortunate accident causes her to be sent back to the earth as a mortal with the curse that she will die (and thus return to heaven) the moment her lover lays his eyes on the child which she will bear him. After a series of mishaps, including Urvashi’s temporary transformation into a vine, the curse is lifted, and the lovers are allowed to remain together on the earth. 300.The famous book Sutrakritanga provides detailed information about the life of: [A] Mahavira [B] Gautama Buddha [C] Kapila [D] Basavanna Correct Answer: A [ Mahavira] Explanation: Sutrakritanga is the second agama of the 12 main Angas of the Jain canons. It provides detailed information about the life of Mahavira. It is also known as the Golden Book of Jainism. 301.Third Buddhist Council was held at which of the following places? [A] Pataliputra [B] Rajgir [C] Vaishali [D] Lumbini Correct Answer: A [ Pataliputra ] Explanation: The Third Buddhist council was convened in about 250 BCE at Asokarama in Pataliputra, supposedly under the patronage of Emperor Ashoka. The council is recognized and known to both the Theravada and Mahayana schools, though its importance is central only to the Theravada. 302.Acharanga Sutra is associated with which of the following religions? [A] Jainism [B] Buddhism [C] Sikhism [D] Ayyavazhi Correct Answer: A [ Jainism ] Explanation: Acharanga is the first of the twelve Angas, part of the agamas (religious texts) which were compiled based on the teachings of Mahavira. 71 | P a g e shop.ssbcrack.com

MCQs

INDIAN HISTORY

MCQs

303.Which Indian city was also known as Kusumapura during the ancient period? [A] Allahabad [B] Patna [C] Ujjain [D] Amaravati Correct Answer: B [ Patna] Explanation: Patna was known as Kusumapura during the ancient period. Aryabhata called himself a native of Kusumapura or Pataliputra (present day Patna, Bihar). 304.Which of the following Indus Valley sites is not a present in India? [A] Chanhudaro [B] Farmana [C] Khirasara [D] Loteshwar Correct Answer: A [ Chanhudaro ] Explanation: Chanhudaro is an archaeological site belonging to the post-urban Jhukar phase of Indus valley civilization. The site is located 130 kilometers (81 mi) south of Mohenjo-daro, in Sindh, Pakistan. 305.Who wrote the book Pancha Siddhantika? [A] Varahamihira [B] Aryabhatta [C] Brahmagupta [D] Kalidas Correct Answer: A [ Varahamihira ] Explanation: Varahamihira was one of the only renowned Indian Astronomer, Mathematician and Astrologer. Varahamihira was born in 499 A.D. into a family of Brahmins settled at Kapittha, a village near Ujjain. He wrote the book Pancha Siddhantika. 306.Devichandraguptam, a play by Vishakhadatta, tells story about which of the following rulers? [A] Sri Gupta [B] Ramagupta [C] Bhanugupta [D] Vishnugupta Correct Answer: B [ Ramagupta ] Explanation: Ramagupta was the elder son and immediate successor of Samudragupta. Earlier, he was known from the traditional narratives only, but later, three inscriptions on Jaina tirthankara images were discovered from Durjanpur near Vidisha, which mention him as the Maharajadhiraja. 307. Bring out the only incorrect statement: [A] Jorwe culture was most prominent in Maharashtra [B] Hathonora is a site where a true hominid fossil was found [C] The Earliest evidence of Rice cultivation has come from Belan Valley [D] The people of all sites in Rajasthan during the Neolithic age were aware of Bricks Correct Answer: D [ The people of all sites in Rajasthan during the Neolithic age were aware of Bricks ] Explanation: The people of all sites in Rajasthan during the Neolithic age were aware of Bricks D is the incorrect statement. People of Gilund in Rajasthan were NOT aware of bricks 72 | P a g e

shop.ssbcrack.com

MCQs

INDIAN HISTORY

MCQs

308.Which of the following are two works of Kalidasa? [A] Raghuvamsha and Kiratarjuniya [B] Kumara Sambhav and Raghuvamsha [C] Malti Madhava and Kumara Sambhava [D] Malti Madhav and Kumara Sambhava Correct Answer: B [ Kumara Sambhav and Raghuvamsha ] Explanation: Kalidasa was one of the gems of the court of Gupta king Chandragupta II (375–415 CE). The poems he wrote were usually of epic proportions and were written in classical Sanskrit. He wrote two epic poems called Kumara sambhava, which means birth of Kumara and the Raghuvamsha, which means dynasty of Raghu. There are also two lyric poems written by Kalidasa known as Meghadutta that stands for cloud messenger and the Ritusamhara that means description of the seasons. Meghadutta is one of the finest works of Kalidasa in terms of world literature. The beauty of the continuity in flawless Sanskrit is unmatched till date. The most famous and beautiful work of Kalidasa is the Shakuntalam. It is the second play of Kalidasa after he wrote Malavikagnimitra. The Shakuntalam tells the story of king Dushyant who falls in love with a beautiful girl Shakuntala, who happens to be the daughter of a saint. 309. Which of the following rivers carries maximum quantity of water into the sea? [A] Nile [B] Amazon [C] Mississippi Missouri [D] Thames Correct Answer: B [ Amazon ] Explanation:Amazon River carries maximum quantity of water into the sea and is largest river by discharge of water in the world, 310.Who was the first Indo-Greek king, who became Buddhist? [A] Antiochus II [B] Apollodotus I [C] Apollodotus II [D] Menander II Correct Answer: D [ Menander II ] Explanation:Menander II was the Indo-Greek king who established a large empire in the South Asia and became a patron of Buddhism. 311.Which of the following are two works of Kalidasa? [A] Raghuvamsha and Kiratarjuniya [B] Kumara Sambhav and Raghuvamsha [C] Malti Madhava and Kumara Sambhava [D] Malti Madhav and Kumara Sambhava Correct Answer: B [ Kumara Sambhav and Raghuvamsha ] Explanation: Kalidasa was one of the gems of the court of Gupta king Chandragupta II (375–415 CE). The poems he wrote were usually of epic proportions and were written in classical Sanskrit. He wrote two epic poems called Kumara sambhava, which means birth of Kumara and the Raghuvamsha, which means dynasty of Raghu. There are also two lyric poems written by Kalidasa known as Meghadutta that stands for cloud messenger and the Ritusamhara that means description of the seasons. Meghadutta is one of the finest works of Kalidasa in terms of world literature. The beauty of the continuity in flawless Sanskrit is unmatched till date. The most famous and beautiful work of Kalidasa is the Shakuntalam. It is the second 73 | P a g e

shop.ssbcrack.com

MCQs

INDIAN HISTORY

MCQs

play of Kalidasa after he wrote Malavikagnimitra. The Shakuntalam tells the story of king Dushyant who falls in love with a beautiful girl Shakuntala, who happens to be the daughter of a saint. 312.Who was the first Indo-Greek king, who became Buddhist? [A] Antiochus II [B] Apollodotus I [C] Apollodotus II [D] Menander II Correct Answer: D [ Menander II ] Explanation: Menander II was the Indo-Greek king who established a large empire in the South Asia and became a patron of Buddhism. 313. Who was the founder leader of ‘Muslim Faqirs’ ? [A] Majnun Shah [B] Dadu Mian [C] Tipu [D] Chirag Ali Shah Correct Answer: A [ Majnun Shah ] Explanation: Majnu Shah was a faqir (Sufi saint) of the Madariya Sufi order founded by Syed Badiuduin Qutb-ul Shah Madar. His headquarters was at the shrine of Shah Madar in Makanpur near Kanpur. He actively participated in the Fakir-Sannyasi Rebellion, and joined in many battles against the British East India Company with his ‘pious team’. He was the founder leader of ‘Muslim Faqirs’. 314.Father of renaissance of Western India was__? [A] B.M. Malabari [B] M.G. Ranade [C] R. G. Bhandarkar [D] K.T. Telang Correct Answer: B [ M.G. Ranade ] Explanation: Justice Mahadev Govind Ranade (1842-1901) was a distinguished Indian scholar, social reformer and author, sometimes called a Father of renaissance of Western India. He was one of the founding members of the Indian National Congress . He established the “Widow Marriage Association” in 1861 to encourage and popularize it Ranade founded the Poona Sarvajanik Sabha in 1870, to represent the Government, the aspiration of the people. Known to be the mentor and political guru of famous freedom fighters Gopal Krishna Gokhale, and Bal Gangadhar Tilak. He was against caste system, untouchability and was a strong supporter of widow-remarriage. 315.Who is often called as Hindu Luther of Northern India ? [A] Dayanand Saraswati [B] Ishwar Chandra Vidyasagar [C] Radhakant Dev [D] Keshav Chandra Sen Correct Answer: B [ Ishwar Chandra Vidyasagar ] Explanation: Ishwar Chandra Vidyasagar was a great 19th century Bengali scholar, reformer, writer and philanthropist, whose ideas remain relevant even in modern India. He had devoted his life to improving the status of Hindu widows and encouraging remarriage. The outcome of these efforts was the Hindu Widow Remarriage Act of 1856. That’s why he has come to be called the Hindu Luther of Northern India.

74 | P a g e

shop.ssbcrack.com

MCQs

INDIAN HISTORY

MCQs

316. In the Indus Valley Civilisation(IVC) which among the following site has a unique water harnessing system and its storm water drainage system? [A] Dholavira [B] Surkotada [C] Daimabad [D] Banawali Correct Answer: A [ Dholavira] Explanation: Dholavira located at the Khadirbet in Bhachau Taluka of Kutch District, Gujarat. The archaeological finding at Dholavira are a) A unique water harnessing system and its storm water drainage system. B) A large well & a bath (giant water reservoirs). C) Only site to be divided into 3 parts. D) Largest Harappan inscription used for civic purpose. E) A stadium. 317.In the Indus Valley civilization the Great Granary was found in which among the following sites? [A] Mohenjodaro [B] Harappa [C] Surkotada [D] Lothal Correct Answer: A [ Mohenjodaro ] Explanation: In Mohenjodaro, the Great Granary ( the largest buliding of civilization) was found. 318.What is the meaning of term “Mohenjodaro” in Indus Valley Civilization? [A] Mount of the dead [B] Mount of the life [C] ]Mount of the struggle [D] Mount of the sorrow Correct Answer: A [ Mount of the dead ] Explanation: In the Indus Valley Civilization, Mohenjodaro literally means “Mound of the dead. 319.In the Indus Valley Civilization, the style of script is: [A] Boustrophedon [B] Pictographic [C] Brahmi [D] Not deciphered yet Correct Answer: A [ Boustrophedon ] Explanation: In the Indus Valley Civilization, the style of script is Boustrophedon i.e. Written from right to left in first line and from left to right in second line. 320.At which Indus Valley Site, the remains of Horse bones have been found? [A] Daimabad [B] Harappa [C] Kalibanga [D] Surkotada Correct Answer: D [ Surkotada ] Explanation: In the Indus Valley Civilization, the remains of Horse bones have been found from Surkotada site. The site at Surkotada is located 160 km north- east of Bhuj, in the district of Kutch, Gujarat.

75 | P a g e

shop.ssbcrack.com

MCQs

INDIAN HISTORY

MCQs

321.What metals were known to the people of Indus valley civilisation? [A] Copper, bronze, silver, gold but not iron [B] Copper, silver, iron but not bronze [C] Copper, gold, iron but not silver [D] Copper, silver, iron but not gold Correct Answer: A [ Copper, bronze, silver, gold but not iron ] Explanation: The Indus valley people knew the use of copper, bronze, silver, gold but not iron. 322.Who was the priest of the Bharatas in the battle of Ten Kings? [A] Visvamitra [B] Vasishtha [C] Atri [D] Bhrigu Correct Answer: A [ Visvamitra] Explanation: The battle of ten kings was fought between Sudas, a Bharata king of the Tritsu family and the confederacy of ten well-known tribes- Puru, Yadu, Turvasa, Anu, Druhyu, Alina, Paktha ,Bhalanas,Shiva and Vishanin. In the bloody and decisive battle on the banks of River Purushni, the Bharatas emerged victorious. The reason behind the war is the rivalry between Vishwamitra and Vasishtha. 323.That the homeland of the Aryans was Arctic Region, was propounded by ___? [A] Max Muller [B] Edward Meyer [C] Bal Gangadhar Tilak [D] Herzefeld Correct Answer: C [ Bal Gangadhar Tilak ] Explanation: The Arctic Home in the Vedas is a book on the origin of Aryans by Lokmanya Bal Gangadhar Tilak The book was written at the end of 1898, but was first published in March 1903 in Pune .It propounded the theory that the North Pole was the original home of Aryans during pre-glacial period which they had to leave due to the ice deluge around 8000 B.C. And had to migrate to the Northern parts of Europe and Asia in search of lands for new settlements. In support to his theory , Tilak has presented certain Vedic hymns, East Iranian passages, Vedic chronology and Vedic calendars with interpretations of the contents in detail. 324.King Ashvapati of the Upanishadic Age was the ruler of which among the following Kingdoms? [A] Kekaya [B] Matsya [C] Panchala [D] Sursena Correct Answer: A [ Kekaya ] Explanation: LATER VEDIC AGE – The Kekaya kingdom was situated on the bank of River Beas, east of Gandhar kingdom. King Aswapati ruled Kekaya when Janak was the king of Videha. 325.With which among the following Vedas, the Shatapatha Brahmana is related to___? [A] Rigveda [B] Yajurveda [C] Samaveda [D] Atharvaveda Correct Answer: B [ Yajurveda ] 76 | P a g e

shop.ssbcrack.com

MCQs

INDIAN HISTORY

MCQs

Explanation: Yajurveda is the book of sacrificial prayers.The world’s oldest prose literature of the IndoEuropeans is contained in Yajurveda.. There are two primary versions or Samhitas of the Yajurveda : Shukla(white) and Krishna(black) This implies that the Krishna Yajurveda includes the Brahmana prose discussions within the Samhita (no Brahman) while the Shukla Yajurveda has separately a Brahmana text, Shatapatha. 326.Which of the following was not a diety of the Harappans? [A] Shiva [B] Mother Goddess [C] Peepal Tree [D] Vishnu Correct Answer: D [ Vishnu ] Explanation:Mother Goddess, Pashupati Shiva, Peepal, Pigeon, Swastik were worshipped by the Harappans. 327.Which among the following places has given the earliest evidence of agriculture in Indian Subcontinent? [A] Brahmagiri [B] Chirand [C] Mehrgarh [D] Burzahom Correct Answer: C [ Mehrgarh ] Explanation: The site of Mehrgarh is located near the Bolan Pass, in the region of Baluchistan, to the west of the Indus River valley. In 1974, Mehrgarh excavated by the French Archaeologist Jean Francois Jarrige. 328.Robert Bruce Foote, who discovered first Palaeolithic tools in India, was originally ___? [A] Palaeobotanist [B] Geologist [C] Archaeologist [D] Historian Correct Answer: B [ Geologist] Explanation: Robert Bruce Foote – Father of Indian Pre-history. He was a geologist of the Geological Survey of India, brought out publications on the tools found in the laterite formations in the then Madras and South Arcot districts, documented the antiquities of the Neolithic and Iron Age in Salem district in Tamil Nadu, wrote memoirs on the geology of the south Maharatta country and neighbouring districts, collected antiquities, painted landscape such as “View of Cape Comorin, the Kumla Kumari Pagoda…” and skilfully handled his finances. 329.Which of the following is the most common motif of the Indus Seals? [A] Unicorn [B] Bull [C] Rhinoceros [D] Elephant Correct Answer: A [ Unicorn] Explanation: The unicorn is the most common motif on Indus seals and appears to represent a mythical animal that Greek and Roman sources trace back to the Indian subcontinent.

77 | P a g e

shop.ssbcrack.com

MCQs

INDIAN HISTORY

MCQs

330.Which of the following Harappan towns is divided into three parts ? [A] Kalibanga [B] Lothal [C] Chanhudaro [D] Dholavira Correct Answer: D [ Dholavira] Explanation: The Dholavira is the largest Indus Valley Site in independent India. It is located on Khadir Beyt, an island in the Great Rann of Kutch in Gujarat.It has been excavated by R S Bisht team of ASI. It had three citadels. Each of these three citadels of Dholavira was improved than Harappa and Mohen-jo-Daro and had an inner closure as well. 331.Which among the following can best define the Neolithic Revolution ? [A] Green Revolution [B] Technology Revolution [C] Milk Revolution [D] Agricultural Revolution Correct Answer: D [ Agricultural Revolution] Explanation: The Neolithic Revolution or Neolithic Demographic Transition, sometimes called the Agricultural Revolution, was the world’s first historically verifiable revolution in agriculture. It was the wide-scale transition of many human cultures from a lifestyle of hunting and gathering to one of agriculture and settlement which supported an increasingly large population. Archaeological data indicates that various forms of plants and animal domestication evolved in separate locations worldwide, starting around 12,000 years ago. 332.Rajatarangini, a book that generally recorded the heritage of Kashmir in the 12th Century, was written by__: [A] Pravaragupta [B] Kalhana [C] Lalitapida [D] Kashyapa Correct Answer: B [ Kalhana] Explanation: Kalhana penned the book named Rajatarangini in 12th century CE. It is a metrical historical chronicle of north-western Indian subcontinent, particularly the kings of Kashmir, written in Sanskrit. Rajatarangini describe the misrule prevailing in Kashmir during the reign of King Kalash, son of King Ananta Deva of Kashmir. 333.Who among the following Kushan Emperors was the first to introduce the gold coinage in India? [A] Vima Kadphises [B] Vima Taktu [C] Vasishka [D] Kujula Kadphises Correct Answer: A [ Vima Kadphises] Explanation: Vima Kadphises was a Kushan emperor from around 90–100 AD. Vima Kadphises added to the Kushan territory by his conquests in Afghanistan and north-west Pakistan. He issued an extensive series of coins and inscriptions. He was the first to introduce gold coinage in India, in addition to the existing copper and silver coinage.

78 | P a g e

shop.ssbcrack.com

MCQs

INDIAN HISTORY

MCQs

334.The last ruler of the Mauryan Dynasty, who was assassinated by Pushyamitra Sunga, his commanderin-chief was __ ? [A] Bindusara [B] Chandragupta [C] Brihadrata [D] Ashokavardhan Correct Answer: C [ Brihadrata] Explanation: Brihadrata was assassinated in 185 BCE during a military parade, by the commander-in-chief of his guard, the Brahmin general Pusyamitra Sunga, who then took over the throne and established the Sunga dynasty. Brihadrata, the last ruler of the Mauryan dynasty, held territories that had shrunk considerably from the time of emperor Ashoka, although he still upheld the Buddhist faith. 335.Buddhacharita, the epic biography of Gautama Buddha in Sanskrit was written by__: [A] Asvaghosa [B] Indrabhuti [C] Asanga [D] Bodhisena Correct Answer: A [ Asvaghosa] Explanation: The Buddha Charita was the biography of Buddha which was penned by Ashvaghosa probably in the first century C.E. He was a renowned Sanskrit poet. The Buddha Charita is made up of 28 songs recounting events in Shakyamuni Buddha’s life up to the time of his great awakening. 336.Which among the following kings was also known as Mamallan (great wrestler)? [A] Pulakesi II [B] Mahendravarman I [C] Narasimhavarman I [D] None of the above Correct Answer: C [ Narasimhavarman I ] Explanation: Narasimhavarman I was a Tamil king of the Pallava dynasty who ruled South India from 630– 668 A.D. Narasimhavarman was also known as Mamallan (great wrestler) and Mamallapuram (Mahabalipuram) was named after him 337.Who among the following succeeded Samudragupta as the next ruler of Gupta Dynasty? [A] Chandragupta II [B] Chandragupta I [C] Vishnugupta [D] Mahendra Correct Answer: A [ Chandragupta II] Explanation: Chandragupta ll (380-412 A.D.) succeeded Samundragupta as the next ruler of Gupta dynasty. He was also known as Devgupta, Devraj and Devshree. Some titled he adopted were : Vikramank, Vikramaditya and Parambhagawat. 338.Which of the following was main capital of Kushana king Kanishka-I? [A] Purushapura [B] Pataliputra [C] Taxila [D] Mathura Correct Answer: A [ Purushapura] 79 | P a g e

shop.ssbcrack.com

MCQs

INDIAN HISTORY

MCQs

Explanation: The capital of Kanishka I was at Purushapura (modern Peshawar) and regional capitals were Taxila (Pakistan), Begram (Afghanistan) and Mathura (India). 339.Who among the following introduced Shaka era? [A] Ashoka [B] Kanishka [C] Rudraraman [D] Bimbisara Correct Answer: B [ Kanishka ] Explanation: Kushana emperor Kanishka is credited with the initiation of the Saka era on his accession to the throne in 78 A D. 340. Who was the first ruler of Kushana dynasty to issue gold coins in India? [A] Vima Kadphises [B] Kanishka [C] Vasudeva [D] Kujula Kadphises Correct Answer: A [ Vima Kadphises ] Explanation: The famous ruler of Kushana Empire, Vima Kadphises is best known for issuing large number of Gold Coins. He is known to have maintained the Silk Route and trade with all sides including the China, Alexandria and Roman Empire. 341.Who was the founder of Kushana Empire in India? [A] Vima Kadphises [B] Kujula Kadphises [C] Kanishka [D] Vasudeva Correct Answer: B [ Kujula Kadphises ] Explanation: Kujula Kadphises was the first Yueh Chi chief who crossed the Hindukush Mountains and laid down the foundation of the Kushana Empire. He adopted the epithet of Dharma-thida and Sachdharmathida. 342.Kushans belongs to which tribe? [A] Yuchi [B] Terai [C] Ikswaku [D] Jnatrika Correct Answer: A [ Yuchi] Explanation: Yuchi-Chis or Tocharians. As per the Chinese sources, the Kushanas (mentioned in Chinese texts as Guishuang) were one of the 5 tribes of Yueh-Chi or Yuezhi. They were also known as Tocharians. Their origin is connected to China and also Central Asia. 343.The ruler of Kushan dynasty, Kanishka was the follower of__? [A] Jainism [B] Hinayanism [C] Hindusim [D] Mahayanism Correct Answer: D [ Mahayanism] 80 | P a g e

shop.ssbcrack.com

MCQs

INDIAN HISTORY

MCQs

Explanation: Kanishka was the follower of Mahayanism or the new theory of Buddhism. Mahayana means the Greater Vehicle. It refers to the path of the Bodhisattva seeking complete enlightenment for the benefit of all sentient beings, also called Bodhisattva Vehicle. 344.Which of the following is the period assigned to reign of Harshavardhana? [A] 600 – 650 A.D [B] 606 – 647 A.D. [C] 500 – 550 A.D. [D] 550 – 600 A.D. Correct Answer: B [ 606 – 647 A.D. ] Explanation: The period assigned to Harsha Vardhana reign is 606-647 A.D. He is regarded as the last great Hindu emperor of India, who ruled over Northern India. 345. Who was the father of King Harshavardhan? [A] Prabhakara Vardhana [B] Bhaskar vardhan [C] Jaivardhan [D] Mahavardhan Correct Answer: A [ Prabhakara Vardhana] Explanation: Prabhakara Vardhana, the founder of Vardhana Dynasty ruled from 580-605 AD. He had two sons – Rajya Vardhan and Harshvardhan and one daughter, Rajyasri. He died fighting with the Huns in 605 A.D. 346.What was the name of the Lichchavi princess who Chandragupta-I married? [A] Kumara Devi [B] Kubernaga [C] Dhruva Devi [D] Vasu Devi Correct Answer: A [ Kumara Devi ] Explanation: The Gupta emperor Chandragupta I married a Lichchavi princess Kumara Devi, who was the heiress to the throne of the Lichchavis and by her marriage with Chandragupta the two states got united. Lichchhavi Princess Kumari Devi was the first Indian Queen featured on a coin. 347.In context of Gupta empire during whose time period was the Iron Pillar in Delhi erected? [A] Samudragupta [B] Chandragupta II [C] Kumargupta I [D] Skandgupta Correct Answer: B [ Chandragupta II ] Explanation: As per Natya-darpana ( by Vishakadata), the Iron pillar had been put up by Chandragupta II himself after defeating Vahilakas. He designated that pillar as a memory of the victory. The pillar is made of 98% wrought iron and has stood more than 1,600 years without rusting or decomposing. 348.Which of the following are the famous temples of Gupta empire? [A] Mukund Darra Temple, Kota [B] Dasavtar Temple, Jhansi [C] Vishnu Temple, Tigawa [D] All of the above 81 | P a g e

shop.ssbcrack.com

MCQs

INDIAN HISTORY

MCQs

Correct Answer: D [ All of the above ] Explanation: The famous temples of Gupta Age are as follows: a) Vishnu Temple, Tigawa(Jabalpur) b) Shiva Temple, Bhumara(Nagaud) c) Parvati Temple, Nachria Kuthara d) Dasavtar Temple, Deogarh(Jhansi) e) Shiva Temple, Koh(Nagaland) f) Bhitragaon Temple, Bhitragaon g) Lakshman Temple, Kanpur(Brick made) h) Lakshman Temple, Sirpur (Raipur) i) Mukund Darra Temple, Kota j) Dhammekh Temple, Sarnath k) Jarasangh’s Sitting, Rajgrih (Bihar) 349.Who among the following Gupta emperor made Ujjain as his second capital? [A] Samudragupta [B] Chandragupta II [C] Kumargupta I [D] Kumargupta I Correct Answer: B [ Chandragupta II] Explanation: Chandragupta II Vikramaditya made Ujjain as second capital of his empire. 350. Which of the following inscriptions mentions Chandragupta Vikramaditya’s authority over NorthWestern India? [A] Mehrauli iron pillar inscription [B] Beirut inscription [C] Junagarh inscription [D] Allahbad pillar inscription Correct Answer: A [ Mehrauli iron pillar inscription] Explanation: The Mehrauli iron pillar inscription mentions Chandragupta Vikramaditya’s authority over north western India and a good portion of Bengal. This pillar was established by Chandragupta as Vishnupada in the honor of Lord Vishnu. 351.Which of the following is / are the 9 gems of Chandragupta Vikramaditya? [A] Amarsimha [B] Harisena [C] Kahapanaka [D] All of the above Correct Answer: D [ All of the above] Explanation: The nine gems or Navratnas adorned in the court of Chandragupta II Vikramaditya are Amarsimha, Dhanvantri, Harisena, Kalidasa, Kahapanaka, Sanku, Varahamihira, Vararuchi and.Vetalbhatta 352. The gold coins issued during the reign of Chandragupta Vikramaditya were known as__? [A] Dinara [B] Couch [C] Archer [D] Chhatra Correct Answer: A [ Dinara ] Explanation: The gold coins issued during the reign of Chandragupta Vikramaditya were known as Dinara. 353. Who defeated the Saka king Rudrasimha III and annexed his kingdom? [A] Ramagupta [B] Chandragupta II 82 | P a g e

shop.ssbcrack.com

MCQs

INDIAN HISTORY

MCQs

[C] Kumargupta I [D] Samudragupta Correct Answer: B [ Chandragupta II ] Explanation: Chandragupta II defeated the Saka king Rudrasimha III and annexed his kingdom and assumed the title Vikramaditya. This brought an end to Saka-Kshatrapa rule in western India and added the regions of Gujarat, Kathiawad and west Malwa to the Gupta empire. 354.Who granted permission to Buddhist king of Ceylon Meghavarman to build a monastic at Bodh Gaya? [A] Samudragupta [B] Kumargupta [C] Skandgupta [D] Chandragupta II Correct Answer: A [ Samudragupta] Explanation: The Gupta emperor, Samudragupta granted permission to Buddhist king of Ceylon Meghavarman to build a monastry at Bodh Gaya. Hence, he was also known as Anukampavan (full of compassion). 355.The Gupta Emperor Chandragupta II was succeeded by whom? [A] Kumargupta I [B] Skandgupta [C] Buddhagupta [D] Vishnugupta Correct Answer: A [ Kumargupta I ] Explanation: The Gupta Emperor Chandragupta II was succeeded by Kumargupta I in 415 AD. He adopted the title of Mahendraditya. His rule is known for peace and prosperity. He ruled till 455 AD. 356.The Sudarshan lake repaired second time under the reign of__? [A] Skandgupta [B] Kumargupta [C] Rudradaman [D] Parnadatta Correct Answer: A [ Skandgupta] Explanation: The Junagarh inscription gives the information about the Sudharshan lake that the lake was an artificial reservoir, was built by the Mauryan emperors for checking floods. Around 150 AD, the lake was repaired by the Shaka ruler Rudradaman I. And, it was again repaired under the Skandgupta’s reign (415 AD- 455AD) by his govemor Parnadatta. 357.The Nalanda University was founded by which Gupta ruler? [A] Samudragupta [B] Kumargupta I [C] Skandgupta [D] Chandragupta I Correct Answer: B [ Kumargupta I ] Explanation: The Nalanda University was founded by Gupta emperor Kumargupta I. He adopted the title of Mahendraditya. 83 | P a g e

shop.ssbcrack.com

MCQs

INDIAN HISTORY

MCQs

358.Who among the following foreign traveller came to India during the reign of Chandragupta II? [A] Hiuen-Tsang [B] Fa Hein [C] Megasthenes [D] Ibn Batuta Correct Answer: B [ Fa Hein] Explanation: Fa-hein, the first Chinese Buddhist traveller and monk, came to India during the reign of Gupta emperor Chandragupta II (Vikramaditya). And wrote the book “Si-Yu-Ki” that gives the detail account of the political and social condition of that time. 359.Under whose reign the Chinese traveller Fa-hein came to India? [A] Samudragupta [B] Kumargupta I [C] Skandgupta [D] Chandragupta II Correct Answer: D [ Chandragupta II ] Explanation: Fa-hein, the first Chinese Buddhist traveller and monk, came to India during the reign of Gupta emperor Chandragupta II and wrote the book “Si- Yu-Ki” that gives the detail account of the political and social condition of that time. 360.Who composed Prayag-Prashasti inscription during the Gupta period? [A] Harisena [B] Veerasena [C] Amarkhaddava [D] Aswaghosa Correct Answer: A [ Harisena] Explanation: Harisena was the court poet of Samudragupta, who mentioned the achievements of Samudragupta in the Prayag-Prasasti inscription (or Allahabad Pillar inscription). 361.Who among the following was the ruler of Kanchi during the period of Samudragupta? [A] Vishnugopa [B] Harshvardhan [C] Hastivarman [D] Kanishka Correct Answer: A [ Vishnugopa] Explanation: In the Allahabad Pillar inscription, it is noted that Samdragupta defeated twelve rulers of Dakshinapatha. In the list of defeated rulers, King Vishnugopa (Pallava dynasty ) of Kanchi is also mentioned. 362.Whom did Skandgupta appointed as the the Provincial Governor of Saurashtra? [A] Virasena [B] Parnadatta [C] Harisena [D] Amarkhaddava Correct Answer: B [ Parnadatta] Explanation: In the Skandgupta’s Junagarh Inscription, it is mentioned that Skandagupta appointed Parnadatta as goptri (governor) of Surashtra (Saurashtra). 84 | P a g e

shop.ssbcrack.com

MCQs

INDIAN HISTORY

MCQs

363.Which among the following era is identical with Gupta era? [A] Vikram era [B] Vallabhi era [C] Saka era [D] Harsha era Correct Answer: B [ Vallabhi era] Explanation: Vallabhi era is identical with Gupta era and Vallabhi era followed Gupta era in A.D. 366. 364.Which among the following is / are works of Kalidasa? 1. Ritusamhara 2. Meghaduta 3. Malavikagnimitra 4. Kumarashambhava 5. Abhijnana Sakuntalam Choose the correct option from the codes given below: [A] 1, 2 & 4 [B] 1, 2, 3 & 4 [C] 1, 2, 4 &5 [D] All of the above Correct Answer: D [ All of the above ] Explanation: Kalidasa was regarded as the greatest poet and dramatist of ancient India. He wrote the Ritusamhara, the Meghadutta, the Kumarasambava, the Raghuvansa, the Viramavamsiya, the Malavikagnimitra and Abhijnana Sakuntalam. He was the court poet of Chandragupta II. 365.Who assumed the title of ‘Ksitipasatapatih’ (Lord of hundred Kings)? [A] Samudragupta [B] Skandgupta [C] Ashoka [D] Chandragupta I Correct Answer: B [ Skandgupta ] Explanation: The titles of Skandagupta are Parambhattaraka, Paramdevta, Maharajadhiraj, Kramaditya, Vikramaditya, etc. In the Kahaum inscription he is also known as KsitipaSatapatih (or lord of a hundred kings). 366.The achievements of Samudragupta are mentioned in which inscription? [A] Hathigumpha inscription [B] Allahabad Pillar inscription [C] Bairut inscription [D] Junagarh Pillar inscription Correct Answer: B [ Allahabad Pillar inscription] Explanation: The achievements of Samudragupta are composed by his court-poet Harisena in Allahabad Pillar inscription ( or Prayag-Prasasti inscription). 367.Which the following rulers is known as the ‘Napoleon of India’? [A] Samudragupta [B] Ashoka [C] Chandragupta I [D] Skandgupta Correct Answer: A [ Samudragupta ] Explanation: Samudragupta believed in the policy of war and conquest and because of his bravery and generalship he is known as the “Napoleon of India” by historian Vincent. A. Smith. 85 | P a g e

shop.ssbcrack.com

MCQs

INDIAN HISTORY

MCQs

368.Which of the following was capital of Guptas? [A] Taxila [B] Pataliputra [C] Ujjain [D] Mathura Correct Answer: B [ Pataliputra ] Explanation: Patliputra was the capital city of Gupta dynasty. 369.Which among the following was the last ruler of Gupta empire? [A] Vishnugupta [B] Chandragupta II [C] Buddhagupta [D] Kumaragupta II Correct Answer: A [ Vishnugupta ] Explanation: Vishnugupta was the last ruler of the Gupta empire. His reign lasted 10 years, from 540 to 550 CE. 370. Who among the following rulers of Gupta dynasty started Gupta Era? [A] Vishnugupta [B] Chandragupta I [C] Skandgupta [D] Samudragupta Correct Answer: B [ Chandragupta I ] Explanation: The Gupta Era was started by Chandragupta I in 319-320 AD. He was the real founder of the Gupta dynasty and was known as Maharajadhiraja (king of kings). 371.Who among the following of Gupta dynasty adopted the title ‘Kaviraja’? [A] Chandragupta I [B] Kumargupta [C] Samudragupta [D] Skandgupta Correct Answer: C [ Samudragupta ] Explanation: Samudragupta adopted the title of Kaviraj (King of poets), he was an expert ‘ Veena’ player. 372.Who was the court poet of Samudragupta? [A] Harisena [B] Asanga [C] Veerasena Saba [D] Vasubandhu Correct Answer: A [ Harisena] Explanation: Harisena was the court poet of Samudragupta, who mentioned the achievements of Samudragupta in the Prayag-Prasasti inscription. 373.Which Gupta emperor was an expert Veena player? [A] Sri Gupta [B] Samudragupta 86 | P a g e

shop.ssbcrack.com

MCQs

INDIAN HISTORY

MCQs

[C] Chandragupta I [D] Kumargupta I Correct Answer: B [ Samudragupta] Explanation: Samudragupta was an expert ‘ Veena’ player and adopted the title of Kaviraj (King of poets). He was not only a great warrior but also a great patron of art and literature. 374.Who was the founder of Gupta Dynasty? [A] Sri Gupta [B] Samudragupta [C] Chandragupta I [D] Chandragupta II Correct Answer: A [ Sri Gupta] Explanation: Sri Gupta was the founder of Gupta Dynasty and adopted the title ‘Maharaj’. 375. Which among the following were the scholars in the Court of Kanishka-I? [A] Vasumitra, Asvaghosa and Parsva [B] Nagarjuna, Charaka and Mathara [C] Charaka, Vasumitra and Asvaghosa [D] All of the above Correct Answer: D [ All of the above] Explanation: The great scholars in the Court of Kanishka I were Asvaghosa (the Buddhist poet), Nagarjuna (the philosopher), Samgharaksha (the chaplain), Mathara (the politician), Vasumitra (the Buddhist scholar), Charaka (the physician) and Agisala (the engineer). 376. The Sakas introduced Satrap system of government along with whom? [A] Parthians [B] Kushans [C] Sungas [D] Satavahanas Correct Answer: A [ Parthians ] Explanation: The Sakas introduced Satrap system of government along with Parthians that was similar to that of the Achaemenid and Seleucid systems in Iran. Under this system, the kingdom was divided into provinces each under military governor Mahakshatrapa (great satrap). The governors of lower status were called kshatrapas (satraps). These governors had the power to issue their own inscriptions and mint their own coins. 377.Which Saka ruler was responsible for important irrigation works in Saurashtra? [A] Rudraraman [B] Moga [C] Azes [D] Nahapana Correct Answer: A [ Rudraraman ] Explanation: The Saka ruler, Rudradraman was responsible for important irrigation works in Saurashtra. His military achievements, his territories and his many personal qualities are highlighted in the famous Junagadh inscription, written in 150 AD. It is the first major inscription to be written in sanskrit.

87 | P a g e

shop.ssbcrack.com

MCQs

INDIAN HISTORY

MCQs

378. Which of the following passes was used by Sakas to come to India? [A] Bolan Pass [B] Nathu La Pass [C] Shipki La Pass [D] Bara-lacha la Pass Correct Answer: A [ Bolan Pass] Explanation: The Sakas came to India through the Bolan Pass. They were a Scythian tribe or group of tribes of Iranian origin. 379.Who overthrew Saka rulers from India? [A] Gautami putra Satakarni [B] Kanishka [C] Rudraraman [D] Demetrius Correct Answer: A [ Gautami putra Satakarni] Explanation: The sakas were finally overthrown by Gautamiputra Satakarni, the ruler of Satavahana dynasty. He killed Saka ruler Nahapana and his governor Rishavadatta and restored his territories. 380.Who composed Buddhacharita? [A] Kanishka I [B] Kanishka II [C] Asvaghosha [D] Charak Correct Answer: C [ Asvaghosha ] Explanation: Asvaghosha, a poet in the court of Kaniska I, composed biography of Buddha (Buddhacharita). He was the first dramatist who used Sanskrit in composing the plays. 381. Who was the court physician during the reign of Kanishka-I? [A] Charak [B] Vasumitra [C] Nagarjuna [D] Asvaghosa Correct Answer: A [ Charak ] Explanation: Charak was the court physician of Kanishka I. He was one of the principal contributors to the ancient art and science of Ayurveda, a system of medicine and lifestyle developed in Ancient India. He is sometimes referred to as the Father of Indian Medicine. 382.Who composed Nasik Inscription in which achievements of Gautamiputra Satkarni were recorded? [A] Gautamiputra Satkarni [B] Sarkarni- I [C] Kanha [D] Gautami Correct Answer: D [ Gautami] Explanation: The achievements of Gutamiputra Satkarni were mentioned in Nasik Inscription that were composed by his mother Gautami Balasri. The Nasik Prasasti describes Gautamiputra as the ruler of the Aparanta, Anupa, Saurashtra, Kukura, Akara and Avanti and defeated the Saka King Nahapana and 88 | P a g e

shop.ssbcrack.com

MCQs

INDIAN HISTORY

MCQs

restored the prestige of his dynasty by reconquering a large part of the former dominions of the Satavahanas. 383.Who was the mother of Gautamiputra Satkarni? [A] Shubhadrangi [B] Nirjara [C] Gautami [D] Koshala Correct Answer: C [ Gautami] Explanation: The mother of the famous Satvahana ruler Gautamiputra Satkarni was Gautami Balasri. The Nasik inscription was made by her that denotes the achievements of her son, Gautamiputra Satkarni. 384.The achievements of which ruler was recorded in Nasik Inscription? [A] Ashoka [B] Bindusara [C] Devabhumi [D] Gautamiputra Satkarni Correct Answer: D [ Gautamiputra Satkarni ] Explanation: The achievements of Gautamiputra Satkarni were mentioned in Nasik Inscription, who were made by his mother Gautami Balasri. In it, Gautamiputra Satkarni was mentioned as the destroyer of the Sakas, Pahlavas and the Yavanas. 385.Which Saka king defeated by Gautamiputra Satakarni? [A] Moga [B] Rudradaman -I [C] Menander [D] Nahapana Correct Answer: D [ Nahapana ] Explanation: The famous Satvahana ruler Gautamiputra Satkarni defeated the Saka King Nahapana and restored the prestige of his dynasty by reconquering a large part of the former dominions of the Satavahanas. 386.Who was the first Satvahana King? [A] Simuka [B] Satkarni [C] Vashisthaputra [D] Gautamiputra Correct Answer: A [ Simuka ] Explanation: Simuka (230-207 BC) was the founder of Satvahana dynasty, who conquered the presentday Maharashtra and parts of Madhya Pradesh. He was succeeded by his brother Kanha, who further extended his state to the present day Andhra Pradesh. 387. Who wrote Malvikagnimitra? [A] Kalidas [B] Tulsidas [C] Agnimitra [D] Chanakya 89 | P a g e

shop.ssbcrack.com

MCQs

INDIAN HISTORY

MCQs

Correct Answer: A [ Kalidas ] Explanation: Malvikagnimitra is a five-act drama based on Sunga king Agnimitra’s love for Vidharbha princess Malvika. It was a Sanskrit play written by Kalidas. 388. Who wrote Mahabhashya? [A] Panini [B] Patanjali [C] Kalidas [D] Tulsidas Correct Answer: B [ Patanjali ] Explanation: Mahabhashya was written by Patanjali. It is a commentary on selected rules of Sanskrit grammar from Paini’s treatise, the Ashtadhyayi and Katyayana’s Varttika. 389.Pushyamitra, the founder ruler of Sunga Dynasty performed how many Ashvamedha sacrifices? [A] None [B] 1 [C] 2 [D] 3 Correct Answer: B [ 1 ] Explanation: The founder of Sunga dynasty, Pushyamitra Sunga is recorded to have performed two Ashvamedhas to promote orthodox brahamical faith during his rule. 390. Heliodorus was the Greek ambassador of which of the following Indo-Greek kings? [A] Antialcidas Nikephoros [B] Selecus Nicator [C] Alexander [D] Constantine Correct Answer: A [ Antialcidas Nikephoros ] Explanation: Heliodorus was the Greek ambassador of Indo-Greek king Antialcidas Nikephoros of Taxila in the court of Sunga King Bhagbhadra. He erected famous votive Heliodorus pillar around 110 BC near Vidisha, Madhya Pradesh. 391.Heliodorus’ a Greek ambassador of the Indo-Greek king was sent to the court of which ruler? [A] Bhagbhadra [B] Devabhut [C] Pushyamitra [D] Ghosha Correct Answer: A [ Bhagbhadra ] Explanation: Heliodorus was the Greek ambassador sent to the court of Sunga King, Bhagbhadra by the Greek King of Taxila, Antialkidas. Heliodorus erected famous votive Heliodorus pillar around 110 BC near Vidisha, Madhya Pradesh. 392.Who was contemporary of Pushyamitra Sunga? [A] Patanjali [B] Kalidas [C] Heliodorus [D] Vasudeva 90 | P a g e

shop.ssbcrack.com

MCQs

INDIAN HISTORY

MCQs

Correct Answer: A [ Patanjali ] Explanation: The great grammarian Patanjali is said to be a contemporary of Pushyamitra Sunga, who founded Sunga dynasty by assassinating the last ruler of Mauryan Empire. 393.Who was the founder of Sunga Dynasty? [A] Pushyamitra [B] Agnimitra [C] Vasumitra [D] Vajramitra Correct Answer: A [ Pushyamitra ] Explanation: Pushyamitra Sunga laid the foundation of Sunga Dynasty by assassinating Brihadratha Maurya (the last ruler of the Mauryan Empire). He was Commander-in-Chief in the army of Mauryan Emperor. Pushyamitra Sunga ruled for 36 years and was succeeded by his son Agnimitra. Devabhuti, the last ruler of the Sunga dynasty, was overthrown by Vasudeva who founded Kanva dynasty in 75 BC. 394.The greatest Chera King remembered for building a temple of ‘Kannagi’ the Goddess of Chastity? [A] Elara [B] Karikala [C] Senguttuvan [D] Mudukudumi Correct Answer: C [ Senguttuvan] Explanation: Senguttuvan (also knoown as Red Chera) was the greatest Chera king who is remembered for building a temple of ‘Kannagi’ the Goddess of Chastity and founded the famous Pattini cult. 395.Who was the earliest known Chola King who conquered Sri Lanka and ruled it ? [A] Karikala [B] Elara [C] Udiyangera [D] Nedunjelian Correct Answer: B [ Elara ] Explanation: Elara was the earliest known Chola King and conquered Srilanka and ruled over it for 50 years. 396. Who was the chairman of Third Tamil Sangam? [A] Nakkirar [B] Agastaya [C] Tiruttakkadevar [D] Chintamani Correct Answer: A [ Nakkirar ] Explanation: Nakkirar was the chairman of 3rd Tamil Sangam held at North Madurai. 397.Who is known as the Father of Tamil literature? [A] Nakkirar [B] Pushyamitra [C] Aggatiyam [D] Agastaya Correct Answer: D [ Agastaya ] Explanation: As per Tamil sources, the father of Tamil literature is ‘Agastaya’. 91 | P a g e

shop.ssbcrack.com

MCQs

INDIAN HISTORY

MCQs

398. The word ‘Sangam’ of Sangam Age was closely associated with which of the following? [A] Crowd [B] Assembly [C] Influence [D] Source Correct Answer: B [ Assembly ] Explanation: The word Sangam is associated with a College or Assembly of Tamil scholars and Poets . 399.Variyar, the Revenue term was used for which action? [A] Tax Collector [B] Extra Demand [C] Land Tax [D] Custom Duty Correct Answer: A [ Tax Collector] Explanation: Term ‘Variyar’ was used for Tax Collector. 400. In the Sangam age, ‘Karai’ the revenue term is related with__? [A] Collecting Tax [B] Land Tax [C] Custom Duties [D] None Correct Answer: B [ Land Tax] Explanation: The Term Karai was used for Land Tax in the Sangam Period. 401.The word ‘Sangam’ of Sangam Age was closely associated with which of the following? [A] Crowd [B] Assembly [C] Influence [D] Source Correct Answer: B [ Assembly] Explanation: The word Sangam is associated with a College or Assembly of Tamil scholars and Poets . 402.Variyar, the Revenue term was used for which action? [A] Tax Collector [B] Extra Demand [C] Land Tax [D] Custom Duty Correct Answer: A [ Tax Collector] Explanation: Term ‘Variyar’ was used for Tax Collector. 403.In the Sangam age, ‘Karai’ the revenue term is related with__? [A] Collecting Tax [B] Land Tax [C] Custom Duties [D] None Correct Answer: B [ Land Tax ] Explanation: The Term Karai was used for Land Tax in the Sangam Period. 92 | P a g e

shop.ssbcrack.com

MCQs

INDIAN HISTORY

MCQs

404.Uraiyur was an ancient Chola city famous for__? [A] Pearls [B] Ship Building [C] Houses [D] Grammar Correct Answer: A [ Pearls] Explanation: Uraiyur was famous for Pearls and Muslims. 405.Which among the following was the royal emblem of Pandya Kingdom? [A] Fish [B] Carp [C] Bow [D] Both a & b Correct Answer: D [ Both a & b ] Explanation: Fish / Carp was the royal emblem of Pandya Kingdom. The Pandyas were also mentioned by Megasthanese who said that then kingdom was famous for pearls. 406.Which among the following was the royal emblem of Chera Kingdom? [A] Tiger [B] Bow [C] Fish [D] Carp Correct Answer: B [ Bow ] Explanation: Bow was the royal emblem of Chera Kingdom. 407.Which among the following was the royal emblem of Chola Kingdom? [A] Bow [B] Tiger [C] Carp [D] Fish Correct Answer: B [ Tiger] Explanation: The Chola kingdom was known as Cholamandulam, it was situated to the north-east of Pandya kingdom between Pennar and Vellar rivers. Tiger was the royal emblem of Chola Kingdom. 408.In context of Sangam Age, which among the following was the ruling class in society? [A] Arasar [B] Kadaisiyar [C] Enadi [D] Pandyas Correct Answer: A [ Arasar] Explanation: In Sangam period, Arasar was the ruling class in society. 409.Which among the following was the capital of Pandyas Kingdom? [A] Tondi [B] Musiri [C] Madurai [D] Kaveripattnam 93 | P a g e

shop.ssbcrack.com

MCQs

INDIAN HISTORY

MCQs

Correct Answer: C [ Madurai] Explanation: Madurai was the capital of Pandya Kingdom. 410.Who among the following was the first person to scale Mount Everest without supplemental oxygen? [A] Terence Bannon [B] Duncan Chessell [C] Reinhold Messner [D] Jean Troillet Correct Answer: C [ Reinhold Messner ] Explanation: Reinhold Messner (German) is the first person to scale Mount Everest without supplemental oxygen, along with Peter Habeler in 1978. He is also first climber to ascend all fourteen peaks over 8,000 metres (26,000 ft) above sea level, including the Mount Everest. 411.Who among the following was the first person to scale Mount Everest without supplemental oxygen? [A] Terence Bannon [B] Duncan Chessell [C] Reinhold Messner [D] Jean Troillet Correct Answer: C [ Reinhold Messner] Explanation: Reinhold Messner (German) is the first person to scale Mount Everest without supplemental oxygen, along with Peter Habeler in 1978. He is also first climber to ascend all fourteen peaks over 8,000 metres (26,000 ft) above sea level, including the Mount Everest. 412. Which type of forests occupies the highest percentage of area in India? [A] Tropical deciduous [B] Savanna and desert vegetation [C] Equatorial evergreen [D] Tropical Rainforests Correct Answer: A [ Tropical deciduous ] Explanation: Tropical Deciduous Forests: They are also called the “Monsoon forests”. They grow in the areas with annual rainfall of 70-200 cm. Sandal wood, Shisam, Mahua, Sal, Teak and Bamboo are important species of trees. Moist deciduous and the dry deciduous forests are two major categories of these forests. 413.Export of which of the following is an Invisible Export? [A] Services [B] Prohibited goods [C] Unrecorded goods [D] Goods through smuggling Correct Answer: A [ Services] Explanation: Any export that does not have a tangible physical presence (e.g. expertise, insurance underwriting). Here, Invisible Export means export of Services 414.The Pir Panjal range is located in which of the following states? [A] Arunachal Pradesh [B] Jammu and Kashmir 94 | P a g e

shop.ssbcrack.com

MCQs

INDIAN HISTORY

MCQs

[C] Punjab [D] Uttarakhand Correct Answer: B [ Jammu and Kashmir ] Explanation: The Pir Panjal ranges lie in the Inner Himalayan region, running from east southeast to west northwest across the states of Himachal Pradesh and Jammu and Kashmir in India as well as Pakistan Administered Kashmir in Pakistan, where the average elevation varies from 1,400 m to 4,100 m. 415. The Sankosh river forms boundary between which of the following two states? [A] Bihar and West Bengal [B] Assam and Arunachal Pradesh [C] Assam and West Bengal [D] Bihar and Jharkhand Correct Answer: B [ Assam and Arunachal Pradesh] Explanation: Sankosh river forms boundary between Assam and Arunachal Pradesh 416.In which of the following regions, maize is used as staple food? [A] Western Europe [B] Russia [C] Middle Africa [D] South-East Asia Correct Answer: C [ Middle Africa ] Explanation: Maize or corn is a cereal crop that is grown widely throughout the world in a range of agroecological environments. The grains are rich in vitamins A, C and E, carbohydrates, and essential minerals, and contain 9% protein. They are also rich in dietary fiber and calories which are a good source of energy. (Staple food is the name for a food that can be stored easily, and eaten throughout the year.) It’s an important staple food for more than 1.2 billion people in Middle Africa. 417. The place Dharanikota in South India is related to which ancient dynasty? [A] Chola [B] Satavahana [C] Pandya [D] Chera Correct Answer: B [ Satavahana ] Explanation: Dharanikota is a town near Amaravati in the Guntur district of Andhra Pradesh in India, It is the site of the ancient Dhanyakataka which was the capital of the Satavahana kingdom which ruled in the Deccan around the 1st to 3rd centuries A.D. 418.Pushkalavati was capital of which of the following acient Kingdoms? [A] Gandhara [B] Kosal [C] Magadha [D] Kashi Correct Answer: A [ Gandhara ] Explanation: Pushkalavati was capital of ancient Kingdom of Gandhara. Its ruins are currently located in Charsadda, Pakistan, located in Peshawar valley in the Khyber Pakhtunkhwa province (formerly NWFP). Pushkalavati was home of the Sanskrit grammarian Panini. 95 | P a g e

shop.ssbcrack.com

MCQs

INDIAN HISTORY

MCQs

419. Who among the following was propounder of the Madhyamika Philosophy? [A] Bhadrabahu [B] Parshwanath [C] Sheelbhadra [D] Nagarjuna Correct Answer: D [ Nagarjuna ] Explanation: Nagarjuna (ca. 2nd or 3rd century) was a patriarch of Mahayana and the founder of Madhyamika School. 420.Which of the following Rigvedic deities represented Storm? [A] Indra [B] Maruts [C] Varun [D] Apas Correct Answer: B [ Maruts ] Explanation: Aryans, The Vedic Age – Religion : They personified force of nature and worshipped them as Vayu (air), Maruts (storm), Indra (rain), Varuna (water), Surya (sun), Agni (fire), Prithvi (earth), Aranyani (forest) etc. 421.The place Dharanikota in South India is related to which ancient dynasty? [A] Chola [B] Satavahana [C] Pandya [D] Chera Correct Answer: B [ Satavahana ] Explanation: Dharanikota is a town near Amaravati in the Guntur district of Andhra Pradesh in India, It is the site of the ancient Dhanyakataka which was the capital of the Satavahana kingdom which ruled in the Deccan around the 1st to 3rd centuries A.D. 422.Pushkalavati was capital of which of the following acient Kingdoms? [A] Gandhara [B] Kosal [C] Magadha [D] Kashi Correct Answer: A [ Gandhara ] Explanation: Pushkalavati was capital of ancient Kingdom of Gandhara. Its ruins are currently located in Charsadda, Pakistan, located in Peshawar valley in the Khyber Pakhtunkhwa province (formerly NWFP). Pushkalavati was home of the Sanskrit grammarian Panini. 423.“_____________is believed to mark the main site of Hastinapur, which was capital of Kauravas and Pandava The imperial gazetteer of India quotes about which of the following places? [A] Delhi [B] Ghaziabad [C] Meerut [D] Faridabad Correct Answer: C [ Meerut ] Explanation: Meerut will be the correct answer. 96 | P a g e

shop.ssbcrack.com

MCQs

INDIAN HISTORY

MCQs

424.“Revere all the Buddhas, revere the dharma, revere the sangha” Which among the following quotes this _________? [A] Allahabad Pillar Inscription [B] Asoka Edict XII [C] Asoka Edict X [D] Mathura Lion capital Correct Answer: D [ Mathura Lion capital] Explanation: Mathura Lion capital will be the correct answer. 425.What was the capital of Surasena Mahajanpada? [A] Viratnagar [B] Mathura [C] Taxila [D] Kashi Correct Answer: B [ Mathura] Explanation: Mathura will be the correct answer. 426.In which of the following Russo-Persian wars Russia was defeated? [A] first [B] second [C] third [D] none Correct Answer: D [ none] Explanation: There was a series of 4 Russo-Persian wars in 1722-23, 1796, 1804, 1826-28 and all of the Russia was victorious. 427.World’s first oil paintings have been found in which of the following countries? [A] India [B] Pakistan [C] Afghanistan [D] Iran Correct Answer: C [ Afghanistan] Explanation: World’s first oil painting were found in Bamiyan caves in Afghanistan. 428.Begram in Afghanistan was a capital of which of the following kings? [A] Kanishka [B] Asoka [C] Chandragupta Maurya [D] Samudragupta Correct Answer: A [ Kanishka ] Explanation: Kanishka will be the correct answer. 429. Which among the following is the correct location of an ancient site Deh Morsai Ghundai? [A] Northern Pakistan [B] Eastern Persia [C] Modern Turkey [D] Afghanistan Correct Answer: D [ Afghanistan] Explanation: Afghanistan will be the correct answer. 97 | P a g e shop.ssbcrack.com

MCQs

INDIAN HISTORY

MCQs

430. At which among the following sites the where first evidence of cultivation of cotton has been found? [A] Piklihal [B] Hathonora [C] Nal [D] Ghaligai Correct Answer: C [ Nal ] Explanation: Nal will be the correct answer. 431.At which of the following sites, largest variety of food grains in the Chalcolithic age has been found? [A] Navdatoli [B] Gilund [C] Adamgarh [D] Banahali Correct Answer: A [ Navdatoli ] Explanation: Navdatoli will be the correct answer. 432.Consider the following statements: 1. First human like appearance was in Australopithecus 2. First known Homo species is Homo heidelbergensis Which among the above statements is / are correct ? [A] 1 only [B] 2 only [C] both [D] none Correct Answer: A [ 1 only] Explanation: First known Homo species is Homo habilis 433.Consider the following statements: 1. Both Ramapithecus & Shivapithecus have been considered the earliest ancestors of modern human beings 2. While Ramapithecus is a male fossil, Shivapithecus is a female fossil Which among the above statements is / are correct? [A] 1 only [B] 2 only [C] Both [D] None Correct Answer: B [ 2 only ] Explanation: 2 Only will be the correct answer. 434.In which state of India, the first Paleolithic site was discovered? [A] Jammu & Kashmir [B] Rajasthan [C] Madhya Pradesh [D] Karnataka Correct Answer: D [ Karnataka ] Explanation: The first evidence of the Stone Age culture in India surfaced in Karnataka as early as in 1842 when Dr. Primrose discovered polished stone knives and arrow heads at Lingsugur in Raichur district of Karnataka. 435.Which among the following age marks a very distinct growth in population of Human beings compared to other ages? [A] Lower paleolithic age [B] Upper Paleolithic Age 98 | P a g e

shop.ssbcrack.com

MCQs

INDIAN HISTORY

MCQs

[C] Mesolithic age [D] Neolithic Age Correct Answer: C [ Mesolithic age ] Explanation: Mesolithic age will be the correct answer. 436. Which of the following subjects are dealt in by Kalpa Sutras? [A] Sacrifices [B] Family ceremonies [C] Varnas, Ashramas [D] All of the above Correct Answer: D [ All of the above ] Explanation: Under Kalpa Sutras ,Shravta Sutras deal with the sacrifices,Grihya Sutras deal with the family ceremonies,Dharma Sutras deal with Varnas, Ashramas. 437.According to the later Vedic Era beliefs, which of the following Gods were elected democratically? [A] Prajapati [B] Adityas [C] Vasus [D] Ashvins Correct Answer: A [ Prajapati ] Explanation: In the Later Vedic period, Prajapati was the important diety occupied the supreme position. Prajapatis were elected democratically. First such elected Prajapati was Lord Vishnu, followed by Lord Bràhma and Lord Shankar. There were total 26 Prajapatis mentioned in Vedas. 438.Which of the following is the equivalent term used for Raja in early vedic era? [A] Gopati [B] Gomat [C] Gopa [D] Gomitra Correct Answer: A [ Gopati ] Explanation: In the Early Vedic age, ‘Raja’ was known as Gopati. 439.In the Later Vedic period, who was the god of Shudras? [A] Pushan [B] Prajapati [C] Agni [D] Indra Correct Answer: A [ Pushan] Explanation: According to the later Vedic traditions, the gods were from different castes. For example, Agni is Brahmin; Indra and Varuna are Kshatriya; Rudra and Maruts are Vaishya while Pushan is a Sudra god. Pushan (पूषन) is a solar deity and one of the Adityas; responsible for journeys, marriage and a carrier of souls to other world. He also protected the travellers in their journeys. 440.In which Mandal of Rigveda ‘Gayatri Mantra is mentioned? [A] III [B] IV 99 | P a g e

shop.ssbcrack.com

MCQs

INDIAN HISTORY

MCQs

[C] V [D] VI Correct Answer: A [ III ] Explanation: Gayatri Mantra comes from the third Mandala of the Rigveda. This Mandala has 62 hymns, mainly to Agni and Indra. It is one of the “family books” (mandalas 2-7), the oldest core of the Rigveda. This Mandala also mentions Kikats, a non-Aryan tribe, which most scholars agree that was resident of somewhere in near Magadha in modern Bihar. 441.In which of the following Vedas, the priest is Brahma? [A] Sam Veda [B] Yajur Veda [C] Rig Veda [D] Atharva Veda Correct Answer: D [ Atharva Veda ] Explanation: In Atharva Veda, the priest is Brahma. 442.Which of the following Upanishads gives clear reference of four Ashrams of individual’s life? [A] Jabala Upanishad [B] Katha Upanishad [C] Virhadranyaka Upanishad [D] None of the above Correct Answer: A [ Jabala Upanishad ] Explanation: In Jabala Upanishad, there is a clear reference of four Ashrams i.e. Brahmachari, Grihastha, Vanaprastha and Sanyasi. 443.Which of the Brahamana says’wife is half her husband’? [A] Satapatha Brahamana [B] Aitareya Brahamana [C] Gopatha Brahamana [D] Taittiriya Brahmana Correct Answer: A [ Satapatha Brahamana] Explanation: The Satapatha Brahamana states that the wife is half her husband and completes him. 444.The ‘Battlle of Ten kings’ is depicted in which mandal of Rig Veda? [A] seventh [B] nineth [C] eighth [D] tenth Correct Answer: A [ seventh ] Explanation: The ‘Battlle of Ten kings (or Dashrajnya ) is depicted in the 7th mandal of Rig Veda. This battle led to the formation of Bharata that gradually evolved into Bharatvarsha, Hindustan and India. It was the battle between 10 major tribal chiefs against Raja Sudas of the Bharata tribe. 445.Which of the following Vedanga is related to metrics? [A] Kalpa [B] Nirkuta 100 | P a g e

shop.ssbcrack.com

MCQs

INDIAN HISTORY

MCQs

[C] Chhanda [D] Shiksha Correct Answer: C [ Chhanda] Explanation: There are six vedangas (limbs of Vedas) evolved for the proper understanding of the Vedas. These are Siksha (phonetics), Kalpa (rituals), Vyakarna (grammar), Nirukta (etymology), Chhanda (metrics) and Jyotisha (astronomy). 446.Which of the following is the oldest Smriti? [A] Manu Smriti [B] Yajnvalkya Smriti [C] Narad Smriti [D] Brihaspati Smriti Correct Answer: A [ Manu Smriti] Explanation: Manu Smriti (Pre-Gupta period) is the oldest Smriti. 447. Who was the founder of Yoga philosophy? [A] Jaimini [B] Kapila [C] Akshapada Gautam [D] Patanjali Correct Answer: D [ Patanjali ] Explanation: Patanjali Yoga philosophy is one of the six major orthodox schools of Hinduism. Yoga Sutras of Patanjali is a key text of the Yoga school of Hinduism. Patanjali was the founder of Yoga philosophy. 448.As per the Dharmashastras, what does ‘Anuloma Vivah’ mean ? [A] A marriage between a higher caste man and a lower caste woman [B] A marriage between a lower caste man and a higher caste woman [C] A marriage between a man and woman of a same caste [D] A marriage between a man and a woman of same gotra Correct Answer: A [ A marriage between a higher caste man and a lower caste woman ] Explanation: An Anuloma vivah is a marriage between a higher caste man and a lower caste woman. A Pratiloma Vivah is a marriage between a lower caste man and a higher caste woman. 449.The IX mandala of the Rigveda samhita is dedicated to? [A] Soma [B] Purushkuta [C] Gotra [D] Gayatri Mantra Correct Answer: A [ Soma] Explanation: The IX mandala of the Rigveda samhita is dedicated to Soma. 450.Which of the following are ritualistic teachings attached to the hymns of the Vedas? [A] Brahmanas [B] Upanishads [C] Aranyakas [D] Itihasas Correct Answer: A [ Brahmanas] 101 | P a g e

shop.ssbcrack.com

MCQs

INDIAN HISTORY

MCQs

Explanation: The ritualistic teachings attached to the hymns of the Vedas were known as the Brahmanas. 451. As per the Dharmashastras, what does ‘Anuloma Vivah’ mean ? [A] A marriage between a higher caste man and a lower caste woman [B] A marriage between a lower caste man and a higher caste woman [C] A marriage between a man and woman of a same caste [D] A marriage between a man and a woman of same gotra Correct Answer: A [ A marriage between a higher caste man and a lower caste woman ] Explanation: An Anuloma vivah is a marriage between a higher caste man and a lower caste woman. A Pratiloma Vivah is a marriage between a lower caste man and a higher caste woman. 452.The IX mandala of the Rigveda samhita is dedicated to? [A] Soma [B] Purushkuta [C] Gotra [D] Gayatri Mantra Correct Answer: A [ Soma ] Explanation: The IX mandala of the Rigveda samhita is dedicated to Soma. 453.Which of the following are ritualistic teachings attached to the hymns of the Vedas? [A] Brahmanas [B] Upanishads [C] Aranyakas [D] Itihasas Correct Answer: A [ Brahmanas ] Explanation: The ritualistic teachings attached to the hymns of the Vedas were known as the Brahmanas. 454.Which of the following types of Vedic priests was most responsible for making correct arrangements for vedic sacrificial rites? [A] Hotri [B] Udgatri [C] Adhvaryu [D] Nakshotri Correct Answer: C [ Adhvaryu ] Explanation: Adhvaryu In the operational aspects of Vedic fire rites, the Adhvaryu was a central figure because he was the one who measured the sacrificial ground, built all that was needed and prepared materials to be used like, articles of oblation, utensils, woods and water. He also kindled the fire for expected offerings. Thus, success of a Yajna was dependent on correct duties of the Adhvaryu priest. 455.The hymns of regveda were composed by ? [A] Hotri [B] Udgatri [C] Adhvaryas [D] Nakshotri Correct Answer: A [ Hotri ] Explanation: Hotri For Vedic priests viz. Hotri, adhvaryu, udgatri and brahman (brahmin) participated in the shrauta rites and each of them had special attributes. Hotri was the priest chanting the hymns of Rig Veda; adhvaryu chanting the hymns of Yajur Veda; Udgatri chanting the Sama Veda hymns. Brahman 102 | P a g e shop.ssbcrack.com

MCQs

INDIAN HISTORY

MCQs

priest was the most and supervisor of the entire ceremony and the one chanting the hymns of Atharva Veda. 456.Purusha sukta is a part of which among the following vedas? [A] Sam Veda [B] Rig Veda [C] Atharva Veda [D] Yajur Veda Correct Answer: B [ Rig Veda] Explanation: Rig Veda Purusha sukta is the Xth mandal of the Rig Veda that explains the origin of four varnas viz. Brahmana, Kshatriya, Vaishya and Shudra. 457.Which among the following are important functionaries of Early Vedic period? [A] Purohita [B] Senani [C] Gramini [D] All of the above Correct Answer: D [ All of the above] Explanation: The important functionaries of Early Vedic period are Purohita, Senani and Gramini. 458. Which among the following is considered to be the oldest Veda? [A] Sam Veda [B] Yajur Veda [C] Rig Veda [D] Atharva Veda Correct Answer: C [ Rig Veda ] Explanation: There are four Vedas composed by Aryans – Rig Veda, Sam Veda, Yajur Veda and Atharva Veda. Of the four Vedas, the Rig veda (collection of lyrics) is the oldest text in the world and is also known as the first testament of mankind. It contains 1028 hymns, divided into 10 mandals. 459.Which among the following places in India where all the Paleolithic, Mesolithic and Neolithic sites are found? [A] Bhimbetka [B] Belan Valley [C] Adamgarh [D] Mirzapur Correct Answer: B [ Belan Valley ] Explanation: Belan Valley is the place in India where all the Paleolithic, Mesolithic and Neolithic sites are found. 460.At which Indus Valley Site, the beared man in steatite image has been found? [A] Harappa [B] Mohenjodaro [C] Lothal [D] Kalibanga Correct Answer: B [ Mohenjodaro ] Explanation: In the Indus Valley Civilization, a beared man in steatite image has been found Mohenjodaro 103 | P a g e

shop.ssbcrack.com

MCQs

INDIAN HISTORY

MCQs

461. Which among the following Harappan site has been divided into three parts? [A] Dholavira [B] Surkotda [C] Daimabad [D] Mohenjodaro Correct Answer: A [ Dholavira ] Explanation: Dholavira located at the Khadirbet in Bhachau Taluka of Kutch District, Gujarat. The archaeological finding at Dholavira are a) A unique water harnessing system and its storm water drainage system. B) A large well & a bath (giant water reservoirs). C) Only site to be divided into 3 parts. D) Largest Harappan inscription used for civic purpose. E) A stadium. 462.Which among the following was the capital of Chera Kingdom? [A] Tondi [B] Musiri [C] Vanji [D] Uraiyaur Correct Answer: C [ Vanji ] Explanation: The capital of Chera Kingdom was Vanji. Tondi and Musiri were the important sea ports of Cheras. 463.The Pattini cult was established by which Sangam ruler? [A] Nedunjeral Adan, the Chera ruler [B] Senguttuvan, the Chera ruler [C] Karikala, the Chola ruler [D] Nedujeliyan, the Pandyan ruler Correct Answer: B [ Senguttuvan, the Chera ruler ] Explanation: The Pattini cult i.e. The worship of Kannagi as an ideal wife, was started by Senguttuvan, the ruler of Chera. 464.Which of the following was the capital of Chola dynasty? [A] Madurai [B] Karur [C] Uraiyaur [D] Kaveripattnam Correct Answer: C [ Uraiyaur ] Explanation: The capital of Chola Kingdom was Uraiyaur that was famous for cotton trade and Purhar. 465.Who built a temple of Augustus at Muzris? [A]Romans [B] Cholas [C] Pandyas [D] Cheras Correct Answer: A [ Romans ] Explanation: Romans built a temple of Augustus at Muzris. 466.Which of the following book is considered as the ‘Odyssus of Tamil poetry’? [A] Tirukkural [B] Manimekalai 104 | P a g e

shop.ssbcrack.com

MCQs

INDIAN HISTORY

MCQs

[C] Silappadikarma [D] Jivaka Chintamani Correct Answer: B [ Manimekalai ] Explanation: Manimekalai is a sequel of an epic ‘Silappatikaram’ written by Seethalai Saathanaar and is considered as the ‘Odyssus of Tamil poetry’. 467.Who was the author of the book ‘Tirukkural’? [A] Seethalai Saathanaar [B] ]Tiruttakrdeva [C] Ilango Adigal [D] Thiruvalluvar Correct Answer: D [ Thiruvalluvar ] Explanation: Thiruvalluvar was the author of the book ‘Tirukkural (also known as the Kural). It is a classic Tamil sangam literature consisting of 1330 couplets or Kurals. The book is also called as the fifth Veda or ‘Bible of the Tamil Land’. 468. Tolkāppiyam belongs to which tamil Sangam? [A] first [B] second [C] third [D] fourth Correct Answer: B [ second ] Explanation: Tolkāppiyam is a work on the grammar of the Tamil language. It deals with orthography, phonology, morphology, semantics, prosody and the subject matter of literature. It belongs to second tamil Sangam. 469.Where was the first Tamil Sangam held__? [A] Madurai [B] Kapatpuram [C] Kaveripattnam [D] Kural Correct Answer: A [ Madurai ] Explanation: The first tamil Sangam was held to the south of Madurai under the patronage of Makeerthy, the Pandyan king. Agastya was the president of first sangam. Unfortunately, no literary work of this Sangam was available. The second tamil Sangam was held in Kapatpuram, second capital of the Pandyas. The third Tamil Sangam was held in Madurai. Its chairman was a tamil poet Nakkirar. 470.In Sangam age, the book ‘Bharatam’ was written by__? [A] Perudevanar [B] Manimekalai [C] Seethalai Saathanaar [D] Tiruttakrdeva Correct Answer: A [ Perudevanar ] Explanation: The book, ‘ Bharatam’ was written by Perudevanar. The author had also composed the invocatory versus for the Sangam classics Ahnanuru, Puranamuru, Kuruntogai, Narrinai and Aingurunuru.

105 | P a g e

shop.ssbcrack.com

MCQs

INDIAN HISTORY

MCQs

471. In Sangam age, the book ‘Bharatam’ was written by__? [A] Perudevanar [B] Manimekalai [C] Seethalai Saathanaar [D] Tiruttakrdeva Correct Answer: A [ Perudevanar ] Explanation: The book, ‘ Bharatam’ was written by Perudevanar. The author had also composed the invocatory versus for the Sangam classics Ahnanuru, Puranamuru, Kuruntogai, Narrinai and Aingurunuru. 472. Which of the following book is the sequel of an epic ‘Silappadikarma’? [A] Jivaka Chintamani [B] Manimekalai [C] Bharatman [D] Kunal Correct Answer: B [ Manimekalai ] Explanation: The story of Manimekalai is a sequel of an epic ‘Silappatikaram’ that tells the story of the conversion to Buddhism of the daughter of Kovalan and Madhavi. Seethalai Saathanaar was the author of the book ‘Manimekalai’. 473. Who was the author of the book ‘Silappadikarma’? [A] Ilango Adigal [B] Seethalai Saathanaar [C] Perudevanar [D] Tiruttakrdeva Correct Answer: A [ Ilango Adigal] Explanation: The author of an epic Silappadikarma is Ilango Adigal. The epic deals with the love story of Kovalan (Son of a wealthy merchant in Puhar) and Madhavi (a beautiful courtesan dancer) of Kaveripattinam. It is also called Illiyad of Tamil Poetry. 474. Who was the author of the book ‘Manimekalai’? [A] Ilango Adigal [B] Seethalai Saathanaar [C] Perudevanar [D] Tiruttakrdeva Correct Answer: B [ Seethalai Saathanaar ] Explanation: Seethalai Saathanaar was the author of the book ‘Manimekalai’. The book is a poem in 30 cantos. Its story is a sequel of an epic ‘Silappatikaram’ that tells the story of the conversion to Buddhism of the daughter of Kovalan and Madhavi. 475. Which of the following ancient Tamil Kingdoms came to be known from Sangam Literature? [A] Chola [B] Chera [C] Pandya [D] All of the above Correct Answer: D [ All of the above ] Explanation: The history of ancient Tamil kingdoms i.e. Cholas, Cheras and Pandyas came to be known from Sangam Literature. 106 | P a g e

shop.ssbcrack.com

MCQs

INDIAN HISTORY

MCQs

476.Who was the author of the Jivaka Chintamani? [A] Perudevanar [B] Seethalai Saathanaar [C] Tiruttakrdeva [D] Ilango Adigal Correct Answer: C [ Tiruttakrdeva ] Explanation: The epic tamil poem, Jivaka Cintamani (also known as Civaka Cintamani) is a Jain religious epic authored by jain saint Tiruttakrdeva. This epic is considered one of the classic epics of Sangam Literature. The poem means “fabulous gem”and is also known as Manannul (Book of Marriages). 477.In context of Mauryan administration, who was the chief supervisor of the collection of revenue from the whole kingdom? [A] Samaharta [B] Sannidhata [C] Karmantika [D] Antapal Correct Answer: A [ Samaharta ] Explanation: As per the Arthashastra of Chankya, Samaharta was the Chief Collector general of revenue that supervises the collection of revenue from the whole kingdom.Pradeshtri was known as the Divisional Commissioner. They were the modern district magistrates and in charge of district. They were to make tours once in every 5 years to inspect the entire administration of the areas under control. Antapal was known as the Governor of the frontier. 478. In context of Mauryan administration, who was regarded as the ‘Chief treasury officer’? [A] Samaharta [B] Paura [C] Sannidhata [D] Karmantika Correct Answer: C [ Sannidhata] Explanation: As per the Arthashastra of Chankya, there were two important officers in the Central Administration. These are Sannidhata (the Chief treasury officer) and Samaharta (the Chief Collector General of Revenue). 479. Mauryan Emperor Ashoka is referred as Ashokavardhana in which of the following? [A] Junagarh Inscription [B] Puranas [C] Nitlur minor rock edict [D] Maski edict Correct Answer: B [ Puranas ] Explanation: In the Puranas, the Mauryan Emperor Ashoka is referred as Ashokavardhana. However, in the Rudradaman’s Junagarh rock edict, the Mauryan Emperor Ashoka is mentioned as Asoka Maurya. 480.In which inscription, the Mauryan Emperor Ashoka refers himself as Piyadassi laja Magadhe (Piyadassi, King of Magadha)? [A] Junagarh Inscription [B] Kandhar Inscription [C] Dipavamsa Inscription [D] Babhru Inscription 107 | P a g e shop.ssbcrack.com

MCQs

INDIAN HISTORY

MCQs

Correct Answer: D [ Babhru Inscription] Explanation: In Babhru edict, the Mauryan Emperor Ashoka refers himself as Piyadassi laja Magadhe (Piyadassi, King of Magadha). In this edict he states his faith in the Buddha, the Dhamma and the Sangha. 481.Who was Paura during the Maurya empire? [A] Governor of the city [B] Chief Judge [C] Mantri [D] Dandapal Correct Answer: A [ Governor of the city] Explanation: As per the Arthashastra of Chanakya, Paura was the Governor of the city. 482.In the Maurya empire, what was the name of department of the Chief of the Mines? [A] Dauvarika [B] Antapal [C] Karmantika [D] Paura Correct Answer: C [ Karmantika] Explanation: Karmantika was the department name of the Chief of the Mines. 483.In the Maurya empire, who was the superintendent of Ports? [A] Pattanadhyaksha [B] Manadhyaksha [C] Kosadhyaksha [D] Ganikadhyaksha Correct Answer: A [ Pattanadhyaksha ] Explanation: As per the officials mentioned in the Arthashastra, Pattanadhyaksha was the superintendent of Ports. 484.In the Maurya empire, who looked after Markets and check wrong practices? [A]Samsthadhyaksha [B] Sitadhyaksha [C] Akaradhyaksha [D] Manadhyaksha Correct Answer: A [ Samsthadhyaksha ] Explanation: In the Mauryan Empire, Samsthadhyaksha looked after the Markets and check wrong practices. 485.As per the officials mentioned in the Arthashastra, the duties related to Crown lands was performed by whom? [A] Kosadhyaksha [B] Hastyadhyaksha [C] Lohadhyaksha [D] Sitadhyaksha Correct Answer: D [ Sitadhyaksha ] Explanation: Sitadhyaksha was responsible for the maintenance of Crown lands.

108 | P a g e

shop.ssbcrack.com

MCQs

INDIAN HISTORY

MCQs

486. In the Mauryan period, the duties of Mudradhyaksha were concerned with – [A] Forest produce [B] Ports [C] Commerce [D] Passports Correct Answer: D [ Passports ] Explanation: Mudradhyaksha was responsible for the issue of passports in the Mauryan period. 487.In the Mauryan empire, the post Devatadhyaksha is related to – [A] Store House [B] Crown Lands [C] Customs [D] Religious Institutions Correct Answer: D [ Religious Institutions ] Explanation: Devatadhyaksha was appointed to perform the duties related to religious institutions. 488. The Ashoka’s Bairut inscription was brought to Allahabad by__? [A] Cunningham [B] James Princep [C] Feroj Shah Tughlaq [D] Jahangir Correct Answer: A [ Cunningham ] Explanation: The Ashoka’s Bairut inscription was brought to Allahabad by Cunningham. On the other hand, Pillars from Topra and Meerut were brought to Delhi by the Firoz Shah Tughlaq. The Kaushambi Pillar was brought to Allahabad by the Mughal Emperor Jehangir. 489.Which Greek ruler sent Dionysuis as an ambassador in the court of Patliputra? [A] Selecus Nicator [B] Antiochus I [C] Potlemy Phildelphos [D] Antigonas Correct Answer: C [ Potlemy Phildelphos ] Explanation: The Egyptian ruler, Ptolemy II Philadelphus had sent Dionysuis as an ambassador in the court of either Bindusara or Asoka. 490.The Kalinga war was fought in which year of Ashoka reign? [A] 6 [B] 7 [C] 8 [D] 9 Correct Answer: C [8] Explanation: The Magadha Emperor, Ashoka invaded Kalinga in the eight year of his reign in 261 B.C. Nearly one lakh soldiers lost their lives in that war and one and half lakh soldiers were captured. The 13th rock edict of Ashoka throws light on this war. 491.Which of the following officers were mandated by Asoka to spread the Dhamma Policy of Asoka? [A] Yuktas [B] Pradesikas 109 | P a g e shop.ssbcrack.com

MCQs

INDIAN HISTORY

MCQs

[C] Rajukas [D] All of above Correct Answer: D [ All of above ] Explanation: After 12 years of Asoka’s coronation, the Major Rock Edict III was issued. It mentions the generosity towards Brahmans. It stated that the Yuktas (subordinate officers) and Pradesikas (district Heads) along with Rajukas (Rural officers ) shall go to the all areas of kingdom every five years and spread the Dhamma Policy of Asoka. 492.Which Ashokan inscription prohibits animal slaughter? [A] Major rock edict III [B] Major rock edict I [C] Major rock edict IV [D] Major rock edict V Correct Answer: B [ Major rock edict I ] Explanation: The Major rock edict I of Ashoka prohibits animal slaughter and holidays of festive gathering. Only two peacocks and one deer were killed in Asoka’s kitchen. He wished to discontinue this practice of killing two peacocks and one deer as well. On the other hand, Major Rock Edict II mentions medical treatment for men and animals, construction of roads, wells and tree planting. 493. Which Ashokan inscription elaborates the policy of Dhamma? [A] Major rock edict IX [B] Major rock edict XI [C] Major rock edict XII [D] Major rock edict X Correct Answer: B [ Major rock edict XI ] Explanation: The Major rock edict XI elaborates the Ashoka’s policy of Dhamma. It stresses the respect for elders, abstain from killing animals and liberal towards friends. 494. Before ascending the Maurya throne, Ashoka was the viceroy of which of the following? [A] Taxilla [B] Ujjain [C] Pataliputra [D] Kandhar Correct Answer: A [ Taxilla ] Explanation: Before ascending the Maurya throne, Ashoka was the viceroy of Taxila. 495.Which among the following is the bilingual inscription? [A] Girnar Inscription [B] Kandhar Inscription [C] Maski Inscription [D] Bairut Inscription Correct Answer: B [ Kandhar Inscription ] Explanation: The Kandhar Inscription of Ashoka is bilingual (Greek and Aramaic) inscription. 496.Among all the major rock edicts of Ashoka, which one is the longest? [A] 7th major rock edict [B] 11th major rock edict 110 | P a g e

shop.ssbcrack.com

MCQs

INDIAN HISTORY

MCQs

[C] 13th major rock edict [D] 9th major rock edict Correct Answer: C [ 13th major rock edict ] Explanation: The Major Rock Edict XIII is the largest rock edict of Ashokan inscription that mentions a) Asoka’s victory over Kalinga. B) Victory of Asoka’s Dhamma over Greek Kings, Antiochus, Ptolemy, Antigonus, Magas, Alexander and Cholas, Pandyas, etc. C) Kamboj, nabhaks, Bhoja, Andhra, etc. 497.Among all the pillar edicts of Ashoka, which one is the longest one? [A] 7th pillar edict [B] 6th pillar edict [C] 5th pillar edict [D] 3th pillar edict Correct Answer: A [ 7th pillar edict] Explanation: The 7th pillar edict elaborates the works done by the Mauryan Emperor, Ashoka for Dhamma Policy. He stated that all sects desire both self control and purity of mind. 498. Which among the following major rock edicts of Ashoka are in Kharosthi script? [A] Mansehra and Sahbazgarhi [B] Sanchi [C] Sarnath [D] Bairut Correct Answer: A [ Mansehra and Sahbazgarhi] Explanation: The two major rock edicts of Ashoka viz. Mansehra and Sahbazgarhi are in Kharosthi script. These both the rock edicts are in Khyber Pakhtunkhwa, Pakistan. 499. Which Greek ruler sent Megathenes in the court of Chandragupta Maurya? [A] Seleucus Nicator [B] Antiochus [C] Potlemy [D] Magas Correct Answer: A [ Seleucus Nicator ] Explanation: Megasthenes was sent by the Greek ruler Seleucus I Nicator as an ambassador in the court of Chandragupta Maurya in Pataliputra. 500.Which among the following Ashokan inscriptions mentions the name Ashoka? [A] Bhabru Edict [B] Maski Edict [C] Puranas [D] Junagarh Inscription Correct Answer: B [ Maski Edict ] Explanation: The Mauryan Emperor Asoka has been mentioned by name ‘Ashoka’ in his inscriptions at Maski and Gujarra only. 501. Which of the following is not a correct observation as mentioned by Megasthenes about India? [A] There is abundant gold, silver, copper and iron in India [B] There is a well established caste system in India [C] There are frequent famines in India [D] Dionysus has invaded India 111 | P a g e shop.ssbcrack.com

MCQs

INDIAN HISTORY

MCQs

Correct Answer: C [ There are frequent famines in India] Explanation: The option C is not correct. Indians did not face famines because of abundant rains and assured at least one of two seasons of crops; and abundance of fruits and edible roots. 502. According to Sri Lankan chronicle Mahavamasa, Mauryan Emperor Ashoka converted to Buddhism by__? [A] Moggaliputa Tissa [B] Tivara [C] Karuvaki [D] Nigrodha Correct Answer: D [ Nigrodha] Explanation: As per the Sri Lankan chronicle Mahavamasa, Mauryan Emperor Ashoka was impressed by Nigrodha and was ultimately converted to Buddhism due to his impression only. 503.Which Ashokan pillar edict provides a list of animals and birds killing of which was prohibited? [A] Pillar edict V [B] Pillar edict VIII [C] Pillar edict XII [D] Pillar edict X Correct Answer: A [ Pillar edict V] Explanation: The Pillar edict-V of Ashoka provides a list of animals and birds which should not be killed on some days and another list of animals which have not to be killed at all occasions and also describes the release of 25 prisionars by Mauryan Emperor Asoka. 504.Which Ashokan pillar edict mentions the Dhamma Mahamattas? [A] pillar edict VII [B] pillar edict VIII [C] pillar edict VI [D] pillar edict V Correct Answer: A [ pillar edict VII] Explanation: The Ashokan Pillar Edict-VII mentions the Dhamma Mahamattas. It propagated a proper courtesy to slaves and servants, obedience to parents, respectful behaviour towards Brahmans and Sharmanas and generiosty towards friends, acquaintances and relatives. 505.Which rock edict of Ashoka appeals for tolerance among sects? [A] Major Rock Edict X [B] Major Rock Edict XI [C] Major Rock Edict XII [D] Major Rock Edict XIII Correct Answer: C [ Major Rock Edict XII] Explanation: The Major Rock Edict XII of Ashoka directed and determined request for tolerance among different religious sects. This edict reflects the anxiety the king felt because of conflict between sects and carries his plea for harmony. 506. Which rock edict of Ashoka mentions the purpose of the rock edicts? [A] Major Rock Edict XIV [B] Major Rock Edict XV [C] Major Rock Edict XI [D] Major Rock Edict XIII 112 | P a g e

shop.ssbcrack.com

MCQs

INDIAN HISTORY

MCQs

Correct Answer: A [ Major Rock Edict XIV ] Explanation: The Major Rock Edict XIV describes the purpose of the rock edicts of different parts of the country. 507. What is the number of prehistoric rock shelters in Bhimbetka? [A] 221 [B] 243 [C] 268 [D] 289 Correct Answer: B [ 243] Explanation: 243 will be the correct answer. 508. The earliest evidence of domestication of animals has been provided by Adamagarh. In which of the following states is located Adamgarh? [A] Rajasthan [B] Madhya Pradesh [C] Gujarat [D] Maharastra Correct Answer: B [ Madhya Pradesh] Explanation: Madhya Pradesh will be the correct answer. 509.The oldest evidence in human history for the drilling of teeth in a living person was found in which of the following cultures? [A] Ahar banas [B] Mehrgarh [C] Sohn Valley [D] Malwa Correct Answer: B [ Mehrgarh] Explanation: Mehrgarh wil be the correct answer. 510.The Gufkral chalcolithic site is located in which state? [A] Rajasthan [B] Jammu & Kashmir [C] Karnataka [D] Uttarakhand Correct Answer: B [ Jammu & Kashmir] Explanation: Jammu & Kashmir will be the correct answer. 511.At which of the following sites, all the Paleolithic, Mesolithic and Neolithic sites have been found in sequence? [A] Sarai Nahar Rai [B] Kurnool Valley [C] Belan Valley [D] Ahar Correct Answer: C [ Belan Valley ] Explanation: Belan Valley will be the correct answer.

113 | P a g e

shop.ssbcrack.com

MCQs

INDIAN HISTORY

MCQs

512.The script of Indus Valley people was __? [A] Desi Script [B] Pictographic Script [C] Brahmi Script [D] Kharosti Script Correct Answer: B [ Pictographic Script ] Explanation: The pictographic script was used by the Indus Valley people. 513.In the Indus Valley Civilization, Great Bath was found at which place? [A] Mohenjodaro [B] Harappa [C] Rakhi Garhi [D] Ropar Correct Answer: A [ Mohenjodaro] Explanation: The Great Bath was discovered at Mohenjodaro in Sindh, Pakistan. It measures 11.88 meters x 7.01 meters and has a maximum depth of 2.43 meters. 514.Which among the following Indus Valley site is located in Montogomery (Punjab) in Pakistan? [A] Harappa [B] Mohenjodaro [C] Kalibanga [D] Chanhudaro Correct Answer: A [ Harappa] Explanation: The harappa site is located in Montogomery district of Punjab in Pakistan. 515.Which one of the following archaeologists discovered the Harappa site of the Indus Valley civilisation? [A] Rakhal Das Bannerjee [B] Sir William Jones [C] Vishnu Shridhar Wakankar [D] Daya Ram Sahini Correct Answer: D [ Daya Ram Sahini ] Explanation: In 1921, an Indian archaeologist Raj Bahadur Daya Ram Sahini discovered the Harappa site of the Indus Valley civilisation. 516.Which of the following Indus Valley Sites was discovered by Rakhal Das Bannerjee? [A] Kali Banga [B] Harappa [C] Lothal [D] Mohenjodaro Correct Answer: D [ Mohenjodaro] Explanation: In 1922, an Indian archaeologist Mr.Rakhal Das Bannerjee discovered the Mohenjodaro site of the Indus Valley civilisation. 517.Which among the following was the main occupation of Palaeolithic (Old stone) people? [A] Agriculture [B] Farming [C] Hunting [D] Fishing 114 | P a g e shop.ssbcrack.com

MCQs

INDIAN HISTORY

MCQs

Correct Answer: C [ Hunting ] Explanation: Hunting was the main occupation of Palaeolithic (Old stone) people. 518.Who among the following is known as father of Indian Pre-History? [A] Robert Bruce foot [B] Sir William Jones [C] E.J.H Mackey [D] Sir John Marshall Correct Answer: A [ Robert Bruce foot ] Explanation: Robert Bruce foot was the father of Indian pre-history. 519.The earliest evidence of Banking transactions in India comes from __: [A] Vedic Era [B] Maurya Era [C] Gupta Era [D] Medieval India Correct Answer: A [ Vedic Era ] Explanation: Earliest evidence of Banking in India is found from the period of Vedic Civilization. During those days, loan deeds called rnapatra (ऋणपत्र) or rnalekhya (ऋणले ख्य) were prevalent. Interest rates as well as usury (सूदखोरी) was prevalent in Vedic India. The Vedic word Kusidin refers to an usurer (सूदखोर).This term is also found in Manusmriti. 520.The caves and rock-cut temples at Ellora are __: [A] Buddhist [B] Buddhist and Jain [C] Hindu and Jain [D] Hindu, Buddhist and Jain Correct Answer: D [ Hindu, Buddhist and Jain ] Explanation: Ellora, built by Rashtrakutas is located 29 km North-West of Aurangabad in Maharashtra. It is one of the World Heritage Sites. It is well known for its monumental caves viz. 12 Buddhist (caves 1– 12), 17 Hindu Caves (caves 13–29) and 5 Jain Caves (caves 30–34). These caves were built in proximity and demonstrate the religious harmony prevalent during this period of Indian history. 521.To which of the following ethnic hordes, Toramana belonged to ? [A] Scythians [B] Hunas [C] Yue-chis [D] Sakas Correct Answer: B [ Hunas ] Explanation: King of the Huns Toramana was a ruler of the Hephthalite Empire who ruled its Indian region in the late 5th and the early 6th century. Toramana consolidated the Hephthalite power in Punjab and conquered northern and central India including Eran in Madhya Pradesh. His territory also included Uttar Pradesh, Rajasthan and Kashmir 522.Vidhushaka, a common character in Sanskrit drama is ___? [A] Brahmana [B] Kshatriya 115 | P a g e

shop.ssbcrack.com

MCQs

INDIAN HISTORY

MCQs

[C] Vaishya [D] Shudra Correct Answer: A [ Brahmana] Explanation: In classical Sanskrit drama, Vidushaka (Bufoon) is essentially a Brahmin and is a confident / attendent of the Hero. Despite being educated in Sanskrit, he speaks in Prakrit, the vernacular language of the time. Bharata mentions him along with the principal characters in his Natyasastra. 523.The important trade centres of ancient India on the trade route connecting Kalyana with Vengi was__? [A] Tagara [B] Sripura [C] Tripuri [D] Tamralipti Correct Answer: A [ Tagara] Explanation: Tagara was the important trade centres of ancient India was on the trade route connecting Kalyana with Vengi. 524. At which among the following places, the Dharma-Chakra Jina Vihara was constructed by Gahadavala Queen Kumaradevi? [A] Bodha Gaya [B] Rajgrih [C] Kushinagar [D] Sarnath Correct Answer: D [ Sarnath] Explanation: Dharmachakra Jina Vihara In Sarnath, Varanasi :The last great monument in the 12th century : The temples included what is described as the Main Shrine and another entitled the, Dharmachakra Jina Vihara was the gift of Kumaradevi, the Buddhist queen of the great Gahadavala king Govindrachandra of Kannauj (CE 1114-1154). 525. The Bhamala Stupa, a ruined Buddhist Stupa, is located in which one of the following countries? [A] Nepal [B] Bhutan [C] Pakistan [D] China Correct Answer: C [ Pakistan] Explanation: Bhamala Stupa is an ancient Gandhara stupa located in Pakistan near Haripur and is a part of so called Bhamala Buddhist Complex. It is a national heritage site of Pakistan and dates back to 4th century AD. This stupa is different from other Gandhara stupas as it looks cross shaped like an Aztec Pyramid. 526.Which among the following is known as the earliest example of Panchayatana style of temple? [A] Dashavatara temple at Deogarh [B] Temple at Pathari [C] Shatrughneshwara temple at Bhubaneshwar [D] Lakshmana temple at Sirpur Correct Answer: A [ Dashavatara temple at Deogarh] Explanation: The Dashavatara Temple or Vishnu Temple also called Gupta temple at Deogarh. It was built in the Gupta Period (320 to c. 600 AD). Archaeologists have inferred that it is the earliest known 116 | P a g e shop.ssbcrack.com

MCQs

INDIAN HISTORY

MCQs

Panchayatana temple in North India. A hindu temple is a Panchayatana one when the main shrine is surrounded by four subsidiary shrines. 527.Which rock edict gives information about Asoka’s conversion to Buddhism? [A] Bhabru rock edict [B] Kalinga rock edict [C] Tarai rock edict [D] Barabar cave rock edict Correct Answer: A [ Bhabru rock edict ] Explanation: The Bhabru rock edict gives information about Asoka’s conversion to Buddhism. The rock edict stated that he has full faith in Buddha, Sangha and Dhamma. 528.Which rock edict describes Asoka’s first Dhamma Yatra to Bodhgaya and Bodhi Tree? [A] Major Rock Edict V [B] Major Rock Edict VI [C] Major Rock Edict VII [D] Major Rock Edict VIII Correct Answer: D [ Major Rock Edict VIII] Explanation: The Major Rock Edict VIII describes Asoka’s first Dhamma Yatra to Bodhgaya and Bodhi Tree. 529.Which rock edict of Ashoka requests for religious tolerance among sects? [A] Major Rock Edict VI [B] Major Rock Edict VII [C] Major Rock Edict VIII [D] Major Rock Edict V Correct Answer: B [ Major Rock Edict VII] Explanation: The Major Rock Edict VII of Ashoka requests for religious tolerance amongst all sects. It appears from the edict that tensions among the sects were intense perhaps in open antagonism. 530. Which Ashokan inscription mentions five contemporary Hellenic Kings? [A] Major Rock Edict XI [B] Major Rock Edict XII [C] Major Rock Edict X [D] Major Rock Edict XIII Correct Answer: D [ Major Rock Edict XIII] Explanation: The Major Rock Edict XIII of Ashokan inscription mentions Asoka’s victory over Kalinga and names of Greek Kings such as Antiochus, Ptolemy, Antigonus, Magas and southern Indian rulers such as Cholas, Pandyas. It has also mentioned names of Kamboj, Nabhaks, Bhoja, Andhra, etc. 531.In which rock edict Ashoka mentioned “Every Human is my child…”? [A] Major Rock Edict V [B] Major Rock Edict IV [C] Major Rock Edict VI [D] Major Rock Edict VII Correct Answer: A [ Major Rock Edict V] Explanation: In the major Rock Edict V, Ashoka mentioned “Every Human is my child”. He had concerns about the policy towards slaves. This rock edict also referes to the appointment of Dhamma-mahamatta for the first time in the twelfth year of his reign. 117 | P a g e shop.ssbcrack.com

MCQs

INDIAN HISTORY

MCQs

532.Who was the Syrian ambassador in the court of Bindusara? [A] Megasthenes [B] Dimachos [C] Dionysius [D] Amitrochates Correct Answer: B [ Dimachos] Explanation: The Syrian ruler, Antiochus I sent Dimachos as an ambassador in the court of Bindusara. 533. Who among the following is the author of ‘Indica’ ? [A] Ashoka [B] Chankya [C] Megasthenes [D] Seleucus Correct Answer: C [ Megasthenes ] Explanation: ‘Indica’ of Megasthenes is a collection of other Roman Greek travellers and describes the geography, culture, administration, prosperity, etc. Of the Mauryan cities. Megasthenes was sent by the Greek ruler Seleucus I Nicator as an ambassador in the court of Chandragupta Maurya in Pataliputra. 534.In which year, Ashoka invaded Kalinga? [A] 261 BC [B] 235 BC [C] 285 BC [D] 275 BC Correct Answer: A [ 261 BC ] Explanation: The Mauryan emperor, Ashoka invaded Kalinga in 261 BC and after a fierce battle Kalinga was conquered. The 13th rock edict of Ashoka elaborates the Kalinga war. 535.Which among the following dynasties were ruling over Magadha when Alexander invaded India? [A] Shisunagas [B] Nandas [C] Mauryas [D] Kosalas Correct Answer: B [ Nandas ] Explanation: The ruler of Nanda Dynasty, Dhanananda was the contemporary of Alexander. 536.In which year, Alexander invaded India? [A] 326 BC [B] 316 BC [C] 346 BC [D] 256 BC Correct Answer: A [ 326 BC ] Explanation: Alexander invaded India in 326 BC. In 326 BC, the Battle of Hydaspes was fought between Alexender the great and Porus along the banks of Haydaspes river (Jhelum river) in Punjab. Porus, the King of Paurava, ruled the area between the rivers Hydaspes (modern Jhelum) and Acesines (Chenab). Though, Porus was defeated, Alexendar was impressed by Porus’s might so asked Porus to rule his dynasty under his name. 118 | P a g e

shop.ssbcrack.com

MCQs

INDIAN HISTORY

MCQs

537.Who was the founder of Nanda dynasty? [A] Mahapadmananda [B] Shisunaga [C] Dhanananda [D] Nandivardhan Correct Answer: A [ Mahapadmananda ] Explanation: Mahapadmananda (345 BC – 329 BC) was the founder of Nanda dynasty. Mahapadmananda was also known as Ekarat and Sarvakshatrantaka. 538.Which among the following rulers is often described as ‘the first empire builder of Indian History’? [A] Dhanananda [B] Mahapadmananda [C] Bimbisara [D] Chandragupta Maurya Correct Answer: B [ Mahapadmananda ] Explanation: The founder of Nanda dynasty, Mahapadmananda is often described as ‘the first empire builder of Indian History’. 539.Who was the last ruler of Shisunaga dynasty? [A] Kalashoka [B] Nandivardhan [C] Naga-Dasak [D] Udayin Correct Answer: B [ Nandivardhan ] Explanation: The last ruler of Haryanka dynasty, Nagadasaka was over thrown by his amatya (or governor) Shisunaga and he established Shisunaga dynasty in 412 BC. Nandivardhan (367 BC–345 BC) was the last ruler of this Shisunaga dynasty. 540.What was the other name of Bimbisara? [A] Shrenika [B] Ekarat [C] Sarvakshatrantaka [D] Shalimal Correct Answer: A [ Shrenika ] Explanation: As per the Jain literature, Bimbisara (who was the real founder of Haryanka dynasty) was also known as Shrenika. 541.Which physician was sent by Bimbisara to treat Avanti King Pradyota? [A] Jivaka [B] Udayin [C] Kalashoka [D] Both a & b Correct Answer: A [ Jivaka ] Explanation: The royal physician, Jivaka, was sent by Bimbisara to treat Avanti King Pradyota, who was ailing with Jaundice.

119 | P a g e

shop.ssbcrack.com

MCQs

INDIAN HISTORY

MCQs

542.Viratnagar, capital of Matsya mahajanpada located in which region ? [A] Malwa [B] Peshawar [C] Jaipur [D] Bundelkhand Correct Answer: C [ Jaipur ] Explanation: Jaipur Viratnagar, capital of Matsya mahajanpada located near present Jaipur. It was used as a hiding place by Pandavas during 13th year of their exile. 543.Ancient Indraprastha was capital of which of the following Mahajanapada? [A] Kuru [B] Matsya [C] Panchal [D] Vajji Correct Answer: A [ Kuru] Explanation: Kuru Indraprastha, capital of Kuru Mahajanpada located in Meerut Delhi region. 544.Which of the following was the capital of Northern Panchalas? [A] Ahicchatra [B] Kampilya [C] Vajji [D] Panchal Correct Answer: A [ Ahicchatra] Explanation: The Northern Panchalas had their capital at Ahicchatra. On the other hand, Southern Panchalas had their capital at Kampilya. 545. Mathura, the famous city was the capital of which ancient Mahajanapada? [A] Panchal [B] Kasi [C] Kuru [D] Sursena Correct Answer: D [ Sursena ] Explanation: Sursena Mathura, the famous city was the capital of Sursena Mahajanpada 546.Which of the following cities was located at junction of Uttarapatha and Dakshinapatha? [A] Varanasi [B] Patliputra [C] Mathura [D] Champa Correct Answer: C [ Mathura ] Explanation: Uttarapatha and Dakshinapatha were two trade routes of ancient India. Mathura was located at the junction of these two famous trade routes. 547. Taxila was the capital of which ancient Mahajanpada ? [A] Gandhar [B] Anga [C] Magadha [D] Kasi 120 | P a g e

shop.ssbcrack.com

MCQs

INDIAN HISTORY

MCQs

Correct Answer: A [ Gandhar] Explanation: Gandhar Taxila, the great center for learning, was the capital of Gandhara. 548. Ujjaini located in the Malwa region, came under which Mahajanapada? [A] Anga [B] Kasi [C] Kosal [D] Avanti Correct Answer: D [ Avanti] Explanation: Ujjaini was located in the Malwa region came under Avanti Mahajanapada. 549.Who inspired the young Bengal Movement in the 19th century? [A] Ram Tanu Lahiri [B] Henry Vivian Dorozev [C] Rasik Kumar Malik [D] Piyare Chand Mitra Correct Answer: B [ Henry Vivian Dorozev] Explanation: Henry Louis Vivian Derozio (1809 –1831) was assistant headmaster of Hindu College, Kolkata, a radical thinker and one of the first Indian educators to disseminate Western learning and science among the young men of Bengal. He constantly encouraged them to think freely, to question and not to accept anything blindly. His teachings inspired the development of the spirit of liberty, equality and freedom. His activities brought about intellectual revolution in Bengal. It was called the Young Bengal Movement and his students, also known as Derozians, were fiery patriots. 550.Who is regarded as “Maker of Modern India”? [A] M. G. Ranade [B] Mahatma Gandhi [C] Keshav Chandra Sen [D] Ram Mohan Roy Correct Answer: D [ Ram Mohan Roy ] Explanation: Raja Rammohan Roy has come to be called the ‘Maker of Modern India’. He was the main force behind introduction of the western education and English language in India. He advocated the study of English, Science, Western Medicine and Technology. He spent his money on a college to promote these studies. He was the founder of the Brahmo Samaj and a great leader of social reform. It was as a result of his persistent campaign that the custom of Sati was declared illegal in Bengal in 1829 A.D. By Lord William Bentick. He was the chief advocate of the modern process of education and the scientific learning. 551.Who among the following is known as ‘Mother of Indian Revolution’ ? [A] Rani Laxmi Bai of Jhansi [B] Bhikaji Rustam Kama [C] Annie Besant [D] Sarojini Naidu Correct Answer: B [ Bhikaji Rustam Kama ] Explanation: Bhikaiji Rustom Cama,(1861-1936) was an outstanding lady of great courage, fearlessness, integrity, perseverance and passion for freedom, and is considered as the mother of Indian revolution because of her contributions to Indian freedom struggle. She was credited with designing India’s first tricolour flag with green, saffron and red stripes bearing the immortal words – Vande Matram. 121 | P a g e

shop.ssbcrack.com

MCQs

INDIAN HISTORY

MCQs

552.Who among the following built the Gomateshwara statue at Sravanabelagola ? [A] Chandragupta Maurya [B] Kharvela [C] Amoghavarsha [D] Chamundaraya Correct Answer: D [ Chamundaraya ] Explanation: Bahubali also called Gomateshwara was an Arihant. According to Jainism, he was the second of the hundred sons of the first Tirthankara, Rishabha and king of Podanpur. A monolithic statue of Bahubali referred to as “Gommateshvara” built by the Ganga dynasty minister and commander Chamundaraya is a 60 feet (18 m) monolith and is situated above a hill in Shravanabelagola, in the Hassan district of Karnataka. It was built in the 10th century AD. 553.Which of the following cities was capital of Maharaja Ranjit Singh? [A] Amritsar [B] Patiala [C] Lahore [D] Kapurthala Correct Answer: C [ Lahore ] Explanation: Maharaja Ranjit Singh (called “The Lion of the Punjab”) (1780-1839) was a Sikh ruler of the Punjab. His tomb is located in Lahore, Pakistan. He is remembered for uniting the Punjab as a strong state and his possession of the Koh-i-noor diamond. He took the title of Maharaja on April 12, 1801 (to coincide with Baisakhi day), with Lahore having served as his capital from 1799. 554.Which among the following Mudra is the gesture of Buddha as depicted in his first sermon? [A] Abhaya Mudra [B] Dhyana Mudra [C] Dharmachakra Mudra [D] Bhumisparsa Mudra Correct Answer: C [ Dharmachakra Mudra ] Explanation: The sermon Buddha gave to the five monks was his first sermon, called the Dhammacakkappavattana Sutta. The Seated Buddha from Gandhara is an early statue of the Buddha made in Gandhara, in modern Pakistan, in the 2nd or 3rd century The pose, which was to become one of a number of standard ones, shows Buddha as a teacher setting the wheel of Dharma in motion (Dharmachakra Mudra). Buddha did this following his enlightenment and after delivering his first sermon at the deer park at Sarnath near Varanasi in Uttar Pradesh. 555. Which of the following inscriptions mentions the name of Kalidasa? [A] Allahabad pillar inscription [B] Aihole inscription [C] Alapadu grant [D] Hanumakonda inscription Correct Answer: B [ Aihole inscription] Explanation: Aihole inscription written by the Badami Chalukyas King, Pulakesin II who reigned from 610 to 642 CE and was a follower of Jainism. This valuable inscription discovered in the Meguti temple near Aihole (Bijapur district in Karnataka). The inscription is in the Sanskrit language, which uses the old Kannada script. The accounts of Pulkeshi’s campaigns are provided in the Aioli inscription dated 634 AD. It was composed by his court poet Ravikirti. Bharavi, who is associated with the Pallavas of Kanchi along with Kalidasa is mentioned in the famous Aihole Inscription of Pulakesin II. 122 | P a g e shop.ssbcrack.com

MCQs

INDIAN HISTORY

MCQs

556.Who among the following built the Gomateshwara statue at Sravanabelagola ? [A] Chandragupta Maurya [B] Kharvela [C] Amoghavarsha [D] Chamundaraya Correct Answer: D [ Chamundaraya ] Explanation: Bahubali also called Gomateshwara was an Arihant. According to Jainism, he was the second of the hundred sons of the first Tirthankara, Rishabha and king of Podanpur. A monolithic statue of Bahubali referred to as “Gommateshvara” built by the Ganga dynasty minister and commander Chamundaraya is a 60 feet (18 m) monolith and is situated above a hill in Shravanabelagola, in the Hassan district of Karnataka. It was built in the 10th century AD. 557.Which among the following Mudra is the gesture of Buddha as depicted in his first sermon? [A] Abhaya Mudra [B] Dhyana Mudra [C] Dharmachakra Mudra [D] Bhumisparsa Mudra Correct Answer: C [ Dharmachakra Mudra ] Explanation: The sermon Buddha gave to the five monks was his first sermon, called the Dhammacakkappavattana Sutta. The Seated Buddha from Gandhara is an early statue of the Buddha made in Gandhara, in modern Pakistan, in the 2nd or 3rd century The pose, which was to become one of a number of standard ones, shows Buddha as a teacher setting the wheel of Dharma in motion (Dharmachakra Mudra). Buddha did this following his enlightenment and after delivering his first sermon at the deer park at Sarnath near Varanasi in Uttar Pradesh. 558.Which of the following inscriptions mentions the name of Kalidasa? [A] Allahabad pillar inscription [B] Aihole inscription [C] Alapadu grant [D] Hanumakonda inscription Correct Answer: B [ Aihole inscription] Explanation: Aihole inscription mentions the name of Kalidasa. 559.By whom Pallava King Mahendravarman-I got defeated? [A] Pulakesin II [B] Vikramaditya II [C] Yajnavarman [D] Pulakesin I Correct Answer: A [ Pulakesin II ] Explanation: The Pallava King Mahendravarman I got defeated by the Chalukya King Pulakesin II. Though, Narasimhavarman I (the son of Mahendravarman I) took the revenge of his father from Pulakesin II by defeating and killing him in the Battle of Vatapi in 642 AD and captured the Chalukyan capital, Vatapi. Thus, he assumed the title of ‘Vatapikonda’ (Conqueror of Vatapi). 560.Who was the author of Dashkumarcharitam and at whose court did he live? [A] Dandin, Nandivarman II [B] Bharavi, Nandivarman II 123 | P a g e shop.ssbcrack.com

MCQs

INDIAN HISTORY

MCQs

[C] Appar, Dandivarman I [D] Bharavi, Narsimhavarman I Correct Answer: A [ Dandin, Nandivarman II ] Explanation: Dashkumarcharitam was composed by Dandin, who was a Sanskrit author of prose romances and expounder on poetics in the 6th-7th century. Both Bharavi and Dandin, the authors of Kiratarjuniyam and Dasakumarcharitam respectively, lived in the Pallava court. 561.The famous Konark Sun temple was built by whom? [A] Narshimhadeva I [B] Angabhima deva I [C] Konkanivarman [D] Narsimhavarman II Correct Answer: A [ Narshimhadeva I ] Explanation: The 13th-century famous Konark Sun temple was built by King Narshimhadeva I (of the Eastern Ganga dynasty) at Konark, Odisha around AD1250. 562.Who was the founder of Ganga dynasty? [A] Konkanivarman [B] Narshimhadeva I [C] Narsimhavarman II [D] Vikramaditya II Correct Answer: A [ Konkanivarman ] Explanation: Konkanivarman was the founder of Ganga dynasty. He was known as Dharmamahadhiraja. 563.The Kadamba dynasty was founded by whom? [A] Konkanivarman [B] Kirtisarman [C] Mayurasharma [D] Yajnavarman Correct Answer: C [ Mayurasharma ] Explanation: The Kadamba dynasty was founded by Mayurasharma in 345 AD. It was an ancient royal dynasty of Karnataka that ruled northern Karnataka and the Konkan from their capital Banavasi ( at present that place is a Uttara Kannada district). 564.Which among the following chalukyan king performed ‘Ashwamedha yajna’ ? [A] Pulakesin I [B] Pulakesin II [C] Vishnuvardhan [D] Kirtivarman I Correct Answer: A [ Pulakesin I ] Explanation: Pulakesin I, the chalukya king, performed Ashwamedha Yajna (horse sacrifice ceremony) to access power. 565.Which among the following Pallava king wrote the famous burlesque ‘Mattavilasa Prahasana’ ? [A]Mahendravarman I [B] Narsimhavarman [C] Narsimhavarman II [D] Paramesvar Varman Correct Answer: A [ Mahendravarman I ] 124 | P a g e shop.ssbcrack.com

MCQs

INDIAN HISTORY

MCQs

Explanation: The versatile Pallava ruler and great scholar Mahendravarman I wrote the famous burlesque Mattavilasa Prahasana (the Sport of Drunkards). It is a short one-act Sanskrit play. 566.Dashkumarcharitam was composed by whom? [A] Bharavi [B] Dandin [C] Varahmihira [D] Kalidas Correct Answer: B [ Dandin ] Explanation: Dashkumarcharitam was composed by Dandin, who was a Sanskrit author of prose romances and expounder on poetics in the 6th-7th century. The book, Dashkumarcharitam is a prose romance in Sanskrit that describes the adventures of ten young men, the Kumaras, all of whom are either princes or sons of royal ministers, as narrated by the men themselves. 567.Who among the following was the author of Kiratarjuniya? [A] Bharavi [B] Dandin [C] Varahmihira [D] Kalidasa Correct Answer: A [ Bharavi ] Explanation: Bharavi, a 6th century Sanskrit poet , was the author of Kiratarjuniya (“Arjuna and the Mountain Man”). 568.Who among the following Pallava ruler built Vaikuntaperumal temple at Kanchi ? [A]Nandivarman II [B] Aparajita [C] Mahendrvarman I [D] Vikramaditta II Correct Answer: A [ Nandivarman II ] Explanation: Vaikunta Perumal Temple was built by Pallava King Nandivarman-II in the 8th century, he was a worshipper of Lord Vishnu. 569.Who among the following Pallava ruler built the Shore Temple at Mahabalipuram? [A] Narsimhavarman II [B] Nandivarman II [C] Aparajita [D] Mahendravarman I Correct Answer: A [ Narsimhavarman II ] Explanation: The Shore Temple at Mahabalipuram and Kailasanatha temple in Kanchipuram built by Pallava ruler Narasimhavarman II. 570.Who took the title of Vatapikonda ? [A] Narsimhavarman I [B] Simhavishnu [C] Mahendravarman I [D] Nandivarman II Correct Answer: A [ Narsimhavarman I ] 125 | P a g e

shop.ssbcrack.com

MCQs

INDIAN HISTORY

MCQs

Explanation: The Pallava King Narsimhavarman I assumed the title of ‘Vatapikonda’ (Conqueror of Vatapi), when he defeated and killed Pulakesin II (Chalukya King) and captured the Chalukyan capital, Badami in 642 AD. 571.What was the ruling period of Chalukyas of Badami? [A] 500 – 550 A.D. [B] 550 – 650 A.D. [C] 543 – 757 A.D. [D] 545 – 700 A.D. Correct Answer: C [ 543 – 757 A.D. ] Explanation: The ruling period of Chalukyas of Badami was 543 – 757 A.D. They ruled in Karnataka. 572.Who among the following Chalukyan king sent an ambassador to Persian king Khusrau II ? [A] Pulakesin I [B] Pulakesin II [C] Kirtivarman I [D] Vishnuvardhan Correct Answer: B [ Pulakesin II ] Explanation: The Chalukyan king, Pulakesin II sent an ambassador to the Persian king Khusrau II in 625 A.D. And received one from him. 573.Which Chalukyan ruler set up it capital at Vengi ? [A] Vishnuvardhan [B] Mahendravarman [C] Kakusthavarman [D] Pulakesin II Correct Answer: A [ Vishnuvardhan ] Explanation: The son of Pulakesin II, Vishnuvardhan established the eastern branch of the chalukyas with it’s capital first at Pistapur and later at Vengi. 574.Which of the following Pallava rulers invaded the Chalukya Kingdom and captured Badami ? [A] Narsimhavarman I [B] Kirtivarman [C] Vishnuvardhan [D] Mahendravarnan I Correct Answer: A [ Narsimhavarman I ] Explanation: In the Battle of Vatapi, the Pallava ruler Narsimhavarman I defeated and killed Pulakesin II and captured the Chalukyan capital, Badami in 642 AD. Thus, he assumed the title of ‘Vatapikonda’ (Conqueror of Vatapi). 575.Who was the author of Aihole Inscription? [A] Ravikirti [B] Hiuen Tsang [C] Bharavi [D] Dandin Correct Answer: A [ Ravikirti] Explanation: Aihole Inscription is a eulogy written by Ravikirti who was the court poet of Chalukya King Pulakesin II. 126 | P a g e shop.ssbcrack.com

MCQs

INDIAN HISTORY

MCQs

576.Who was the founder of Chalukya Dynasty? [A] Vishnuvardhan [B] Pulakesin I [C] Kirtivarman I [D] Pulakesin II Correct Answer: B [ Pulakesin I ] Explanation: The founder of Chalukya dynasty, Pulakesin I established a small kingdom with Badami (or Vatapi) as its capital in Bijapur District, Karnataka. 577.Who among the following wrote ‘Nagananda’? [A] Banabhatta [B] Simhanada [C] Pulakesin II [D] Harshavardhana Correct Answer: D [ Harshavardhana] Explanation: The emperor of Vardhana dynasty, Harshavardhana wrote a play ‘Nagananda’. It describes the story of the Jimutavahana’s self-sacrifice to save the Nagas. It has an enlightening Buddhist legend for its subject and is one of the best works of the Indian Drama. 578.During reign of Harsha, the Kannauj assembly was held to publicise the doctrines of which of the following sects? [A] Mahayana [B] Hinayana [C] Theravada [D] Svetambara Correct Answer: A [ Mahayana ] Explanation: The Kannauj assembly (643 AD) was held in the honour of Hieun Tsang (Chinese pilgrim) and to popularise Mahayana sect of Buddhism.Harshavardhana was a Mahayana Buddhist. He organised Kannauj assembly (643 AD) to popularise Mahayana sect of Buddhism. Though, he was a tolerant ruler and supported all Indic faiths viz. Buddhism, Vedism and Jainism. The scholars regarded him as the last great Hindu emperor of India, who ruled over Northern India. 579.Which of the following Chinese emperors was contemporary of Harshavardhana? [A] Taizong [B] Wu Zetian [C] Ruizong [D] Xuanzong Correct Answer: A [ Taizong] Explanation: In 641 AD Harshavardhan sent an envoy to Taizong (Tai-Tsung), the Tang emperor of China and in return, the Chinese ruler sent three embassies to Harsha court. 580.The Chalukya king, Pulakesin II defeated Harshavardhan on the banks of which river? [A] Narmada [B] Kaveri [C] Niranjana [D] Rijupalika Correct Answer: A [ Narmada] 127 | P a g e shop.ssbcrack.com

MCQs

INDIAN HISTORY

MCQs

Explanation: Chalukya king Pulakesin II had defeated Harshavardhan on the banks of River Narmada in 618 AD. Pulakeshin, who ruled from the Chalukyan capitalof Badami, challenged Harsha’s conquests. The former had established himself as ‘lord paramount’ of the south, as Harsha had of the north. Unwilling to tolerate the existence of a powerful rival in the south, Harsha had marched from Kanauj with a huge force. Such was Pulakeshin’s efficiency in guarding the passes of the Narmada that Harsha was compelled to accept the river as the demarcation and retire from the field of battle after losing a major part of his elephant force. 581.Who defeated Harsha Vardhana on the banks of river Narmada in 630 A.D.? [A] Pulakesin II [B] Mahendravarman I [C] Pulakesin I [D] Kirtivarman I Correct Answer: A [ Pulakesin II ] Explanation:As per the Aihole inscription, Harsha Vardhana was defeated by Pulakesin-II, the great Chalukya king, on the banks of river Narmada in 630 A.D. 582.Which of the following Chinese traveller came to India during the reign of Harshavardhana? [A] Hiuen-Tsang [B] Fa Hein [C] Nicolo Conti [D] Both A & B Correct Answer: A [ Hiuen-Tsang ] Explanation: Hieun Tsang, the Chinese pilgrim, visited India during the reign of Harshavardhana with an aim of securing authentic Buddhist scripts. The Kannauj assembly (643 AD) was held in the honour of Hieun Tsang and to popularise Mahayana sect of Buddhism. He stayed in India for about fifteen years and recorded his experience in his book ,’ Si-Yu-Ki’. The book throws lights on the religion, customs, traditions, etc. Of the country. 583.Which among the following play is/are written by Harshavardhan ? [A] Nagananda [B] Ratnavali [C] Priyadarshika [D] All of these Correct Answer: D [ All of these ] Explanation: Harsha wrote three sanskrit plays- Nagananda, Ratnavali and Priyadarshika. 584.What was the other name of Harshavardhana? [A] Siladitya [B] Harshaditya [C] Bhaskarvarman [D] Vishnuvardhan Correct Answer: A [ Siladitya ] Explanation: Bharshavardhan (606-647 AD) was also known as Siladitya. His kingdom extends from Punjab, West Bengal, Orissa and from Himalayas to the entire Indo-Gangetic plain north of the Narmada river. He gave equal respect to all religions and became the follower of Buddhism in his later life.

128 | P a g e

shop.ssbcrack.com

MCQs

INDIAN HISTORY

MCQs

585.Who wrote Kadambari? [A] Chankya [B] Banabhatta [C] Charak [D] Radhagupt Correct Answer: B [ Banabhatta ] Explanation: Kadambri is a romantic novel in Sanskrit, written by Banabhatta who was the court poet and close companion of Harshavardhana. 586.Who assassinated Grahavarmana, the brother in law of Harshavardhan ? [A] Shashanka of Gauda [B] Kirtivarman [C] Chandragupta [D] Narsimhavarman Correct Answer: A [ Shashanka of Gauda ] Explanation: Shashanka of Gauda assassinated Grahavarmana and imprisoned his wife Rajyasri ( who was sister of Harshavardhana). 587.Who was the brother-in-law of Harshavardhan? [A] Grahavarmana [B] Banabhatta [C] Devgupta [D] Bhaskarvarman Correct Answer: A [ Grahavarmana ] Explanation: The sister of Harshavardhan, Rajyasri was married to Grahavarmana, King of Kanauj. He was the last ruler of Kanauj belonging to the Maukhari royal dynasty. 588.Who among the following was the founder of Pushyabhuti Dynasty ? [A] Pushyabhuti [B] Prabhakar Vardhan [C] Aditya vardhan [D] Harshavardhan Correct Answer: A [ Pushyabhuti ] Explanation: According to Banabhatta, Pushyabhuti was the founder of Pushyabhuti Dynasty. 589.Who among the following is the author of ‘Harshacharita’ ? [A] Banabhatta [B] Hiuen Tsang [C] Harshavardhan [D] Bhaskarvarman Correct Answer: A [ Banabhatta ] Explanation: Banabhatta was the court poet of Harshavardhana, who wrote Harshacharita and Kadambari. Harshacharita is an important historical work narratted the incidents of the earlier part of Harsha’s reign. 590.Which among the following define the similarities between Buddhism and Jainism? [A] Both opposed Brahmanical domination and caste system [B] Both accepted the theories of rebirth 129 | P a g e shop.ssbcrack.com

MCQs

INDIAN HISTORY

MCQs

[C] Both preached truth and non-violence [D] All of the above Correct Answer: D [ All of the above ] Explanation: The similarities between Buddhism and Jainism are a) Both opposed Brahmanical domination and caste system. B) Both have accepted the theories of Karma and rebirth and Moksha. C) Both preached truth, non-violence, celibacy and detachment from material comforts.. D) Both Uphold the essence of Vedas. 591.Tirthankara Parswanath’s idol is recognized by which symbol? [A] Lion [B] Bull [C] Tortoise [D] Serpent Correct Answer: D [ Serpent ] Explanation: Tirthankara Parswanath was the 23rd Tirthankar who preached four teachings of Jainism: not to kill, not to lie, not to steal and not to own property. His idol is recognized by the symbol of serpent. 592.What was the fifth tenet added by Tirthankara Mahavira to the four basic tenets of Jainism? [A] Non injury [B] Non lying [C] Observe continence [D] Non stealing Correct Answer: C [ Observe continence] Explanation: The five main teachings of Jainism are do not commit violence, do not speak a lie, do not steal, do not acquire property and observe continence (brahmacharya). The brahmacharya was added by Lord Mahavira. 593.The grandson of Ashoka, who accepted Jainism was __? [A] Samprati [B] Kunala [C] Dasharatha [D] Salisuka Correct Answer: A [ Samprati] Explanation: Emperor Samprati was the grandson of Emperor Ashoka, who reigned from 224–274 BCE. And accepted Jainism. 594.Which ruler is credited for the spread of Jainism in Karnataka? [A] Chandragupta Maurya [B] Bimbisara [C] Ashoka [D] Samparati Correct Answer: A [ Chandragupta Maurya ] Explanation: The spread of Jainism in Karnataka is attributed to Chandragupta Maurya. The Emperor became a Jaina, gave up his throne and spent the last years of his life in Karnataka as a Jaina ascetic. 595.Under whose leadership, Jainism spread in South India? [A] Sthalabahu [B] Bhadrabahu 130 | P a g e

shop.ssbcrack.com

MCQs

INDIAN HISTORY

MCQs

[C] Ashoka [D] Chandragupta Maurya Correct Answer: B [ Bhadrabahu ] Explanation: The Jainism was spread in South India under the leadership of Bhadrabahu. 596.Under whose leadership Shvetambara sect formed? [A] Bhadrabahu [B] Sthalabahu [C] Chandragupta Maurya [D] Ashoka Correct Answer: B [ Sthalabahu ] Explanation: Sthalabahu Jainism is divided into two major sects viz. The Digambara (sky clad) sect and the Shvetambara (white clad) sect. Under the leadership of Sthalabahu, Shvetambara sect formed. Under the leadership of Bhadrabahu, the Digambara sect was formed. 597.Where was the second Jain council held? [A] Pataliputra [B] Vallabhi [C] Patliputra [D] Vaishali Correct Answer: B [ Vallabhi ] Explanation: The second Jain Council was held at Vallabhi under the chairmainship of Devaradhi Kshamasramana in 512 AD and resulted in finl compilation of 12 Angas and 12 Upangas. 598.Where was the first Jain council held? [A] Pavapuri [B] Pataliputra [C] Jimbhikagrama [D] Vaishali Correct Answer: B [ Pataliputra ] Explanation: The first Jain council was held at Pataliputra by Sthulabahu in the beginning of the third century BC and resulted in the compilation of 12 Angas to (sections or limbs) to replace the lost Purvas (former texts). 599.At which place Mahavira attained Niravana? [A] Pavapuri [B] Kundagram [C] Vaishali [D] Rajgriha Correct Answer: A [ Pavapuri] Explanation: At the age of 72 in 468 BC, Tirthankar Mahavira passed away at Pavapuri, Bihar. 600.Mahavir Swami belonged to which clan? [A] Kosala [B] Huna [C] Jantriak [D] Sakya Correct Answer: C [ Jantriak] 131 | P a g e

shop.ssbcrack.com

MCQs

INDIAN HISTORY

MCQs

Explanation: Jantriak Vardhamana Mahavir belonged an aristocratic family and Kshatriya clan called Naya or Jnatri. He was born in Kudagrama, near modern Patna and his father was King of this area. His mother, Trishala was sister of Chetak (Cetakka) an eminent Lichchvi prince of Vaisali. 601.The name of wife of Vardhaman Mahavira was __? [A] Yashoda [B] Chellna [C] Trishala [D] Mahamaya Correct Answer: A [ Yashoda] Explanation: Yashoda was the wife of Mahavir Swami, who was the daughter of Samarvira king. 602.According to some Jain traditions, who was son-in-law and first disciple of Tirthankara Mahavira? [A] Jamali [B] Jamvant [C] Jamval [D] Cant’ say Correct Answer: A [ Jamali ] Explanation: According to some Jain traditions (Shwetambara mainly), Jamali was the son-in-law and first disciple of Tirthankara Mahavira. 603.Which of the following Jain Tirthankara have been mentioned in Rigveda? [A] Rishabha and Aristanemi [B] Mahavira and Parsvanath [C] Rishabha and Malinath [D] Parsvanath and Rishabha Correct Answer: A [ Rishabha and Aristanemi ] Explanation: The Jain Tirthankaras, Lord Aristanemi (22nd Tirthankar) and Lord Rishabh Nath (1st Tirthankar) has been mentioned in Rigveda. 604.In which year, Vardhamana Mahavira was born? [A] 563 BC [B] 540 BC [C] 533 BC [D] 560 BC Correct Answer: B [ 540 BC ] Explanation: Vardhamana Mahavira was born in 540 BC at Kundaligrama, now in Bihar. 605.On the bank of which river, Mahavira attained Kaivalaya? [A] Niranjana [B] Rijupalika [C] Yamuna [D] Jamuna Correct Answer: B [ Rijupalika] Explanation: At the age of 42, Mahavira attained Kaivalaya under a Sal tree on the bank of river Rijupalika near Jrimbhikgrama in Modern Bihar. 132 | P a g e

shop.ssbcrack.com

MCQs

INDIAN HISTORY

MCQs

606. Who was the first Tirthankara in Jainism? [A] Rishabhdev [B] Parsavanath [C] Mahavira [D] Munisuvrata Correct Answer: A [ Rishabhdev ] Explanation: Lord Rishabhdev was the first Tirthankara of Jainism. He was born at Ayodhya in the Ikshwaku Kula or clan. In Hinduism he is known to be an avatara or incarnation of Vishnu. The name of Rishabh’s parents has been mentioned in the Bhagvata Purana. 607. What was the name of Gautam Buddha’s only son ? [A] Rahul [B] Channa [C] Kanthala [D] Chunda Correct Answer: A [ Rahul ] Explanation: Rahul was the only son of Gautama Buddha and his wife Yasodhara, born at 534 BC. 608.At which age Gautam Buddha got Nirvana? [A] 24 [B] 35 [C] 38 [D] 42 Correct Answer: B [ 35 ] Explanation: Gautam Buddha was born at Lumbini, Kapilvastu in 563 BC. At the age of 35 years, Gautam Buddha attained Nirvana (or enlightment) under a pipal tree at Bodh Gaya on the bank of river Niranjana. 609.During whose reign the Third Buddhist Council was organized? [A] Ashoka [B] Kalasoka [C] Ajatsatru [D] Kanishka Correct Answer: A [ Ashoka ] Explanation: The Third Buddhist Council was held at Patliputra during the reign of Emperor Ashoka. It was held under the Presidentship of Moggliputra Tissa to revise scriptures. 610.In which Buddhist council, Buddhism was divided into Sthaviravadins and Mahasanghikas? [A] Second Buddhist council [B] Third Buddhist council [C] First Buddhist council [D] Fourth Buddhist council Correct Answer: A [ Second Buddhist council ] Explanation: In the second Buddhist council, Buddhism was divided into Sthaviravadins and Mahasanghikas.

133 | P a g e

shop.ssbcrack.com

MCQs

INDIAN HISTORY

MCQs

611.Which among the following is not among Ashtamahasthanas? [A] Lumbini [B] Rajgriha [C] Vajrayana [D] Taxila Correct Answer: D [ Taxila ] Explanation: The holy places where the four principal events of the Buddha’s life took place are Lumbini (birth), Bodh Gaya (Enlightenment), Sarnath (First Sermon), Kusinagar (Death took place). Apart from these four places, there are another four holy places viz. Sravasti, Rajgriha, Vaishali and Vajrayana, these eight holy places are known as Ashtamahasthanas. 612. Which among conclusions has been derived from the debasement of the coins and gradual disappearance of gold coins during the post-Gupta period? [A] Commodities became cheap [B] Gold Mining was stalled [C] Money economy was gradually replaced by Barter Economy [D] There was a decline in trade Correct Answer: D [ There was a decline in trade ] Explanation: Debasement of the coins and gradual disappearance of gold coins during the post-Gupta period indicates the Decline of Trade. 613.In the revenue department of Mauryas, who among the following was in-charge of the collection of all revenues in the empire? [A] Sannidhatri [B] Pradeshta [C] Yukta [D] Samahatra Correct Answer: D [ Samahatra ] Explanation: Mauryan administration: Samaharta – chancellor of the exchequer; responsible for the collection of revenue. The Revenue Department had Samharta who was incharge of the collection of all revenues in the empire 614.Which among the following Kushana king adopted the epithet Dharma-thida? [A] Vima Kadaphises [B] Kuzul Kadaphises [C] Kanishka the Great [D] Huvishka Correct Answer: B [ Kuzul Kadaphises ] Explanation: Kuzul Kadaphises (or Kadphises I) was the founder of kushanas dynasty. Kujula known to have assumed in his coins the epithets Dharma-thida and Sacha-dharma-thita which evidently refer to his adherence to the Buddhist and Saiva faiths. 615.Which among the following Gupta emperors called himself “Lichchhavi-dauhitra”? [A] Shrigupta [B] Chandragupta I [C] Chandragupta II [D] Samudragupta 134 | P a g e

shop.ssbcrack.com

MCQs

INDIAN HISTORY

MCQs

Correct Answer: D [ Samudragupta ] Explanation: Chandragupta-I married the daughter of Licchhavis, a royal influential family .His son and successor Samudragupta calls himself Lichchavi- dauhitra the son of the daughter of the Lichchavis. 616.In Pali texts, who among the following is referred to as Nigantha Nataputta? [A] Gautam Buddha [B] Mahavira [C] Parshvanatha [D] Rishabha Correct Answer: B [ Mahavira ] Explanation: Nigantha Nataputta refers to Mahavira. A Nirgrantha refers to Jains. A person who has attained kevalya can be a nirgrantha. 617.At which among the following sites of Indus Valley Civilization, the rows of distinctive fire altars with provision of ritual bathing have been found ? [A] Mohen-jo-daro [B] Harappa [C] Kalibangan [D] Lothal Correct Answer: C [ Kalibangan ] Explanation: Kalibangan – is an archaeological site where Ploughed Field, Bones of Camel, Circular and Rectangular Graves, distinctive fire (Vedic ) altars with provision of ritual bathing have been found. 618.The earliest evidence of rice cultivation comes from which among the following valleys? [A] Central Ganga Valley [B] Belan Valley [C] Gomal Valley [D] Bolan Valley Correct Answer: B [ Belan Valley ] Explanation: Belan Valley, Allahabad is one of the regions the earliest evidence of rice cultivation has come. 619.The inscriptions of Ashoka and the Brahmi script were deciphered by ___? [A] Alexander Cunningham [B] James Princep [C] Max Muller [D] Mortimer Wheeler Correct Answer: B [ James Princep ] Explanation: James Princep (1799-1840) was an English scholar, orientalist and antiquary. He was the founding editor of the Journal of the Asiatic Society of Bengal and is best remembered for deciphering the Kharosthi and Brahmi scripts of ancient India. He was the first to decipher the inscriptions of Ashoka and the Brahmi script. 620.The minister of the Gahadvala King Govindachandra and author of Kalpadruma was __? [A] Apararka [B] Chandesvara 135 | P a g e

shop.ssbcrack.com

MCQs

INDIAN HISTORY

MCQs

[C] Lakshmidhara [D] Vijnanesvara Correct Answer: C [ Lakshmidhara ] Explanation: THE GAHADVALA DYANSTY – descendants of Suryavansha of Ayodhya. The Gahadvala, or Gaharwar, was a Hindu and Buddhist Indian dynasty that ruled the kingdom of Kannauj for approximately a hundred years, beginning in the late eleventh century. The founder of the Kannauj Gahadvala dynasty was Chandradeva, who took control of Kannauj at the end of the 11th century (1090A.D.). During the rule of his successor, Govindachandra, from 1114–1154, the state of Gahadvala reached the pinnacle of its power, »His minister, Lakshmidhara, was the author of several legal works, including Kalpadruma. 621.Which of the following Chola kings was the first to capture Maldives? [A] Rajaraja [B] Rajendra I [C] Rajadhiraja [D] Rajendra II Correct Answer: A [ Rajaraja ] Explanation: [Rajaraja Chola I laid the foundation for the growth of the Chola kingdom into an empire, by conquering the kingdoms of southern India and the Chola Empire expanded as far as Sri Lanka in the south, and Kalinga (Orissa) in the northeast. He conquered Sri Lanka, the Maldives, Sumatra and other places in Malay Peninsula. The naval conquest of the ‘old islands of the sea numbering 12,000’, the Maldives marked one of the conquests of Rajaraja. Rajaraja I built a strong navy with the aim of controlling the sea. 622.“The king was the ruler of all except Brahamanas”. This was the view of __? [A] Manu [B] Yajnavalkya [C] Apasthamba [D] Gautama Correct Answer: D [ Gautama ] Explanation: Gautama-dharmasutra The Dharmasutras: The Law Codes of Apastamba, Gautama, Baudhayana, and Vasistha The Dharmasutras are the four surviving written works of the ancient Indian tradition on the subject of dharma, or the rules of behavior a community recognizes as binding on its members. Gautama- dharmasutra is believed to be the oldest of the four Hindu Dharmasastras, It survives as an independent treatise. According to Gautama the ruler is the owner of everything in his realm except the brahmanas. 623.For which among the following offenses, Manu recommended higher punishment to Brahamans than the persons of other varnas? [A] Profanity [B] Murder [C] Theft [D] Treason Correct Answer: C [ Theft] Explanation: According to Manu, a Brahamana was to be awarded higher punishment than the persons of other varnas for the offence of Theft.

136 | P a g e

shop.ssbcrack.com

MCQs

INDIAN HISTORY

MCQs

624.Metronymics were borne by the later rulers of which among the following dynasties? [A] Maurya [B] Sunga [C] Kanva [D] Satavahana Correct Answer: D [ Satavahana] Explanation: Metronymics: A name derived from the name of a mother or female ancestor. The metronymics are borne by the later Satavahanas and not by the early Satavahanas. It was mainly because Satavahanas had to establish matrimonial alliance with maharathis. The metronymics of the Satavahanas was the result of such alliances. 625.One of the writers of the Dharmashashtra disapproved the practice of Sati declaring it as an act of suicide. Identify him from the given options: [A] Angiras [B] Medhatithi [C] Usana [D] Visnu Correct Answer: B [ Medhatithi ] Explanation: Medhatithi is one of the oldest and most famous commentators on the Manusmriti. He considered sati system identical to suicide, which was forbidden by the Vedas. One shall not die before the span of one’s life is run out. 626.Vallabhi era is identical with which of the following era? [A] The Vikrama era [B] The Gupta era [C] The Harsha era [D] None of the above Correct Answer: B [ The Gupta era ] Explanation: Vallabhi era is identical with Gupta era and Vallabhi era followed Gupta era in A.D. 366. 627.The cotton cloth worn by Indians had a brighter white colour than any cotton found elsewhere. This view was recorded by __? [A] Alexander [B] Megasthenes [C] Nearchus [D] Pliny Correct Answer: B [ Megasthenes ] Explanation: Megasthenes – an ancient Greek historian and diplomat, author of an account of India, the Indica, in four books. He was sent by the Hellenistic king Seleucus I on embassies to the Mauryan emperor Chandragupta Maurya. He wrote that the dress worn by the Indians was made of cotton of a brighter white colour than any cotton found elsewhere. 628.Which among the following is / are the terms used for coins of the Gupta period? 1. Dinara 2. Dramma 3. Rupaka 4. Suvarna Choose the correct answer from the codes given below: [A] 4 only [B] 137 | P a g e

shop.ssbcrack.com

MCQs

INDIAN HISTORY

MCQs

[C] 1, 4 [D] 1, 2, 3, 4 Correct Answer: C [ 1, 4 ] Explanation: The gold coins of Gupta period were called Dinara inspired by the Roman coins but later the coins were minted in the Indian style with the weight standard of 9.2 gms of gold and called Suvarna. 629.The Kushans issues the coins made of __? 1. Gold 2. Silver 3. Copper Choose the correct option from the codes given below: [A] Only 1 [B] Only 1 & 2 [C] Only 1 & 3 [D] 1, 2 & 3 Correct Answer: D [ 1, 2 & 3 ] Explanation: The Kushans issued coins of Gold, Silver and Copper . 630.The last Mauryan ruler, who was killed by his Commander-in-Chief was __? [A] Dasaratha [B] Kun-ala [C] Samprati [D] Brihadratha Correct Answer: D [ Brihadratha ] Explanation: Maurya Emperor Brihadratha was killed by his senapati (Commander-in-Chief) Pushymitra Shunga . Pushymitra Shunga founded the Shunga Dynasty in around 185-183 BC. 631.Who among the following was the author of Abhidhamma treatise Kathavatthu? [A] Mah-a kassapa [B] Mahinda [C] Milinda [D] Moggaliputta Tissa. Correct Answer: D [ Moggaliputta Tissa. ] Explanation: The Abhidhamma treatise Kathavatthu was written by Moggaliputta Tissa. He was a Buddhist monk and scholar who lived in the 3rd century BC . Moggaliputta-Tissa was the spiritual teacher of the Mauryan Emperor Ashoka, and his son Mahinda, who brought Buddhism to Sri Lanka. He also presided over the Third Buddhist Council. 632.In which among the following sacrifices, the sacrificial materials used was sura in Vedic Era? [A] Agnistoma [B] Rajasuya [C] Sautramani [D] Vajapeya Correct Answer: C [ Sautramani ] Explanation: Sautramani was the sacrifice of the asuras and was adopted by the devas. The sacrificial materials used was sura, wine. In the Sautramani sacrifice the oblations of sura are prescribed. 633.In the Rigvedic Dasrajan Yudha (Battle of Ten Kings) the Bharatas emerged winner at the bank of __? [A] The Indus River [B] The Saraswati River 138 | P a g e

shop.ssbcrack.com

MCQs

INDIAN HISTORY

MCQs

[C] The Sutlej River [D] The Parushni River Correct Answer: D [ The Parushni River ] Explanation: According to Rigveda,the famous battle of ten kings ( or Dasrajan Yudha) was between Sudas, a Bharata king of the Tritsu family and the confederacy of ten well-known tribes- Puru, Yadu, Turvasa, Anu, Druhyu, Alina, Paktha ,Bhalanas,Shiva and Vishanin.In the bloody and decisive battle on the banks of River Parushni the Bharatas emerged victorious. 634.Which of the following Pala ruler founded the Somapuri University? [A] Gopala [B] Kumarpala [C] Dharmapala [D] Ramapala Correct Answer: C [ Dharmapala ] Explanation: The Somapuri University is situated at North Bengal and was founded by Dharmapala, the Pala ruler. 635.Which among the following University was founded by Ramapala, the ruler of Pala dynasty? [A] Somapuri [B] Vallabhi [C] Jagadal [D] Odantpuri Correct Answer: C [ Jagadal ] Explanation: Jagadal is situated in Bengal and was founded by Ramapala, the Pala ruler. 636.Which among the following Bodhisattva holds thunderbolt? [A] Vajrapani [B] Manjushri [C] Maitreya [D] Amitabha Correct Answer: A [ Vajrapani ] Explanation: The Bodhisattva, Vajrapani, holds a thunderbolt like the deity Indra of Hinduism and fights against evil and sin. Avlokitesvara or Padmapani is the lotus bearer; Majushri holds a book describing 10 paramitas ( spiritual perfections); Maitreya is the future Buddha; Ksitigarbha is the bodhisattva of hellbeings and guardian of purgatories; while Amitabha is Buddha of heaven and the Buddha of Immeasurable Life and Light. 637.Gautma Buddha’s mother ‘Mahamaya’ belongs to which tribe? [A] Pala [B] Gupta [C] Kushan [D] Koliya Correct Answer: D [ Koliya ] Explanation: Gautma Buddha’s mother ‘Mahamaya’ belonged to Koliya tribe and she married Suddhodhana, who was a king of Sakya clan. The Sakya and Koliya ruled on opposite banks of the Rohni river in current Rupendehi District of Lumbini, Nepal. Both of them were republics. 139 | P a g e

shop.ssbcrack.com

MCQs

INDIAN HISTORY

MCQs

638.The Vallabhi University was set up by which ruler? [A] Kumargupta I [B] Bhattarka [C] Dharmapala [D] Gopala Correct Answer: B [ Bhattarka ] Explanation: The Maitrak King, Bhattarka established the Vallabhi University in Gujarat. 639.The Vikramshila University was set up by which ruler of Pala dynasty? [A] Dharmapala [B] Ramapala [C] Gopala [D] Kumarpala Correct Answer: A [ Dharmapala ] Explanation: The Vikramshila University was set up by the Pala dynasty king Dharmapala in the late 8th or early 9th century at Bhagalpur, Bihar. The ancient Vikramshila University was intended to complement the existing world class universities at Nalanda and Takshila. It lasted four centuries before being destroyed during an attack by Bakhtiyar Khilji of the Delhi Sultanate. 640.The University of Nalanda was set up by which Gupta ruler? [A] Kumargupta I [B] Chandragupta II [C] Samudragupta [D] Kumargupta II Correct Answer: A [ Kumargupta I ] Explanation: The University of Nalanda was established at Badagoan,Bihar in the 5th century during the reign of the Gupta emperor Kumaragupta I. 641.Which sect of Buddhism does not believe in idol-worship? [A] Hinayana [B] Mahayana [C] Vajrayana [D] None of the above Correct Answer: A [ Hinayana ] Explanation: Those who followed strictly the doctrine of Buddha and denied the existence of God were known as the followers of Lesser vehicle or Hinayana. The Hinayana sect does not believe in idol worship, nor do its followers believe that Buddha was God. 642.Which among the following Buddhist scripture deals in the moral code of conduct? [A] Abhidhamma Pitaka [B] Vinay Pitaka [C] Sutta Pitaka [D] None of them Correct Answer: B [ Vinay Pitaka ] Explanation: The Buddhist scripture, Vinay Pitaka mainly deals with rules and regulations, which the Buddha promulgated. It describes in detail the gradual development of the Sangha. In other words, it is the code of ethics to be obeyed by the early sangha, monks and nuns. 140 | P a g e

shop.ssbcrack.com

MCQs

INDIAN HISTORY

MCQs

643.Which among the following Buddhist scripture deals in the teachings of Buddhism? [A] Abhidhamma Pitaka [B] Vinay Pitaka [C] Sutta Pitaka [D] Both b & c Correct Answer: C [ Sutta Pitaka ] Explanation: The Sutta-pitaka (or basket of suttas) contains the essence of the Gautma Buddha’s teaching regarding the Dhamma. It contains more than 10,000 suttas (or teachings) attributed to the Buddha or his close disciples. It is the second of three sections of the Tripitaka, the earliest Buddhist scripture. The Sutta Pitaka is made of five nikayas ( or collections) of suttas viz. Digha Nikaya, Majjhima Nikaya, Samyutta Nikaya, Anguttara Nikaya, Khuddaka Nikaya. 644.Abhidhamma pitaka deals in which of the following? [A] members of great community [B] teachings of Buddha [C] moral code of conduct [D] matters of psychology and metaphysics Correct Answer: D [ matters of psychology and metaphysics] Explanation: The Abhidhamma pitaka deals in the matters of psychology and metaphysics. 645.During whose reign Mahayana sect of Buddhism came into existence? [A] Ashoka [B] Kanishka [C] Ajatsatru [D] Nagarjuna Correct Answer: B [ Kanishka ] Explanation: Mahayana sect of Buddhism emerged during the reign of Kanishka. Asanga was the founder of Mahayan sect of Buddhism. 646.In which Buddhist Council, Buddhism was divided into Mahayana and Hinayana sects? [A] First Buddhist Council [B] Second Buddhist Council [C] Third Buddhist Council [D] Fourth Buddhist Council Correct Answer: D [ Fourth Buddhist Council ] Explanation: Buddhism was divided into Mahayana and Hinayana in the Fourth Buddhist Council. 647.Who presided First Buddhist Council? [A] Mahakassapa [B] Vasumitra [C] Moggliputra Tissa [D] Sabakami Correct Answer: A [ Mahakassapa ] Explanation: The First Buddhist Council was held at Saptaparni cave near Rajagriha in 72 AD, during the reign of Emperor Ajatsatru. It was held under the Presidentship of Mahakassapa to compile the Dhamma Pitaka and Vinaya Pitaka. 141 | P a g e

shop.ssbcrack.com

MCQs

INDIAN HISTORY

MCQs

648.Who presided Fourth Buddhist Council? [A] Mahakassapa [B] Vasumitra [C] Moggliputra Tissa [D] Sabakami Correct Answer: B [ Vasumitra ] Explanation: The Fourth Buddhist Council was held at Kundalvana, Kashmir in 72 AD during the reign of Kushan king Kanishka. It was held under the Presidentship of Vasumitra to compose commentaries on the Tripitika. 649.Who presided Second Buddhist Council? [A] Mahakassapa [B] Vasumitra [C] Moggliputra Tissa [D] Sabakami Correct Answer: D [ Sabakami] Explanation: The second Buddhist Council took place at Vaishali in 383 BC. It was presided by Sabakami under Kalasoka of Shisunga dynatsy to settle down dispute between two opposing groups of monks of Vaishali and Patliputra and monks of Kausambi and Avanti regarding the code of discipline. 650.First Buddhist Council was held during the reign of which of the following kings? [A] Ajatsatru [B] Ashoka [C] Kanishka [D] Kalasoka Correct Answer: A [ Ajatsatru] Explanation: The First Buddhist Council was held at Saptaparni cave near Rajagriha in 72 AD, during the reign of Emperor Ajatsatru. It was held under the Presidentship of Mahakassapa to compile the Dhamma Pitaka and Vinaya Pitaka. 651.During whose reign was the Fourth Buddhist Council held? [A] Ashoka [B] Kalasoka [C] Ajatsatru [D] Kanishka Correct Answer: D [ Kanishka ] Explanation: The Fourth Buddhist Council was held at Kundalvana, Kashmir in 72 AD during the reign of Kushan king Kanishka. It was held under the Presidentship of Vasumitra to compose commentaries on the Tripitika. 652.What symbol represents nirvana of Gautama Buddha? [A] Lotus [B] Wheel [C] Horse [D] Bodhi Tree Correct Answer: D [ Bodhi Tree ] 142 | P a g e

shop.ssbcrack.com

MCQs

INDIAN HISTORY

MCQs

Explanation: Bodhi Tree is the symbol of nirvana of Gautama Buddha. On the other hand, Stupa represents the symbol of death of Gautama Buddha. Further, The symbol ‘Horse’ signifies the renunciation of Buddha’s life. 653.What symbol represents birth of Gautama Buddha? [A] Bodh tree [B] Lotus [C] Horse [D] Wheel Correct Answer: B [ Lotus ] Explanation: Lotus and bull resembles the symbol of birth of Gautama Buddha. 654.According to Buddhist traditions, who was Charioteer of Buddha? [A] Channa [B] Kanthaka [C] Devdatta [D] Chunda Correct Answer: A [ Channa ] Explanation: Channa was the name of Charioteer of Gautam Buddha. 655.Who was ‘Kanthaka’ in the context of Gautam Buddha? [A] Charioteer [B] Body-guard [C] Cousin [D] Horse Correct Answer: D [ Horse] Explanation: Kanthaka was the royal horse of Gautama Buddha. 656.Which one is the longest epic of the world? [A] Ramayana [B] Ramcharitmanas [C] Mahabharata [D] Hanuman Chalisa Correct Answer: C [ Mahabharata] Explanation: The Mahabharata by Ved vyas is the longest epic of the world consists of 1,00,000 shlokas. 657.The Boghazkoi inscription was discovered in __? [A] India [B] Iran [C] Syria [D] Turkey Correct Answer: D [ Turkey ] Explanation: Homeland of Aryans : one of the inscription proves that the Aryans are from Central Asia is Boghazkoi (Asia Minor ,Turkey ) Inscription. 658.The Aryans dedicated whole of a separate Mandala in Rig-Veda for the hymns dedicated to __? [A] Agni [B] Varuna 143 | P a g e

shop.ssbcrack.com

MCQs

INDIAN HISTORY

MCQs

[C] Indra [D] Soma Correct Answer: D [ Soma ] Explanation: Soma & Haoma : (It is the 9th Mandal of Rigveda which contains 114 hymns ) The fermented juice of the plant called Soma appears to have been the only intoxicating drink used in Vedic times. So much were the ancient Aryans addicted to this drink, that Soma was soon worshipped as a deity both in India and in Iran (under the name Haoma in the latter country), and we find one entire Mandala, or Book, of the Rig-Veda, dedicated to this deity. 659. Emperor Akbar conferred the title of Jagatguru upon which among the following Jaina Philosophers? [A] Anandasagar Suri [B] Vijaykanak Suri [C] Hira vijay Suri [D] Vijay Shantichandra Suri Correct Answer: C [ Hira vijay Suri ] Explanation: 1526–1595 C.E: Muni Hiravijayji Suri was the supreme pontiff of Tapa Gachcha order of Jain Svetambara tradition. He is famous for propounding the Jain philosophy to Emperor Akbar and turning him towards vegetarianism. Akbar was so impressed with Hiravijaya Suri that he bestowed on him the title of Jagatguru . Akbar almost denounced meat eating and almost turned to vegetarianism due to inspiration from him. 660.Which among the following village was the site of the Vikramshila Mahavihara, the renowned educational centre of Pala period? [A] Antichak [B] Aphasad [C] Basarh [D] Chandimau Correct Answer: A [ Antichak] Explanation: Antichak – Excavated site Vikramshila Vikramashila was founded by Pala King Dharmapala in response to a supposed decline in the quality of scholarship at Nalanda. [It was destroyed by Bakhtiyar Khilji ] 661.In context with the Mahayana Buddhism faith, the future Buddha is ___? [A] Krakuchanda [B] Amitabha [C] Maitreya [D] Kanak Muni Correct Answer: C [ Maitreya] Explanation: Maitreya is a future Buddha. According to scriptures, Maitreya will be a successor of the historic Sakyamuni Buddha, who in the Buddhist tradition is to appear on Earth, achieve complete enlightenment, and teach the pure dharma. 662.Which among the following two dynasties of the Northern India confronted with the Rashtrakutas? [A] The Pratiharas and the Paramaras [B] The Palas and the Chandelas [C] The Pratiharas and the Palas [D] The Chalukyas and the Chahamanas 144 | P a g e

shop.ssbcrack.com

MCQs

INDIAN HISTORY

MCQs

Correct Answer: C [ The Pratiharas and the Palas ] Explanation: North Indian Dynasties: Towards the close of the 8th century AD, there were three great powers in India – the Palas in the east, the Gurjar- Pratiharas in the north and the Rashtrakutas in the deccan. The tripartite struggle for the supremacy among the Palas, Partiharas and the Rashtrakutas was the desire to possess the city of Kannauj, which was then a symbol of sovereignty. 663.The rulers of which among the following dynasties called themselves “Brahma-Kshatriay”? [A] Palas [B] Senas [C] Pratiharas [D] Chahamanas Correct Answer: B [ Senas] Explanation: Senas Dynasty: The dynasty’s founder was Hemanta Sen. It was a Hindu dynasty that ruled from Bengal through the 11th and 12th centuries. 664.Which of the following pairs is not correctly matched? [A] Nagananda Harsha [B] Mudrarakshasa Visakhadatta [C] Mrichchhakatika Sudraka [D] Ratnavali Rajasekhara Correct Answer: D [ Ratnavali Rajasekhara] Explanation: »Mricchakatika, is a ten-act Sanskrit drama written by Sudraka. »Nagananda is one of the best Sanskrit dramas in five acts dealing with the popular story of Jimutavahana’s self-sacrifice to save the Nagas, written by king Harsha (Vardhana Dynasty). »The Mudrarakshasa (“The Signet of the Minister”) is a historical play in Sanskrit by Vishakhadatta that narrates the ascent of the king Chandragupta Maurya to power in Northern India. »Ratnavali is a Sanskrit drama about a beautiful princess named Ratnavali, and a great king named Udayana, written by Indian emperor Harsha (Vardhana dynasty). 665.Parnadatta was appointed the Provincial Governor of Saurashtra by __? [A] Chandragupta Maurya [B] Rudradaman [C] Chandragupta II [D] Skandagupta Correct Answer: D [ Skandagupta] Explanation: Skandgupta’s Junagarh Inscription: It states that Skandagupta appointed Parnadatta as goptri (governor) of Surashtra (Saurashtra). 666.The title “Paramasaugata” was adopted by ___? [A] Bhaskar Varman [B] Shashanka [C] Rajya Vardhana [D] Harsha Correct Answer: C [ Rajya Vardhana ] Explanation: Rajyavardhana has been mentioned as Parama- Saugata in Harsha’s Madhuvana and Banskhera inscriptions.

145 | P a g e

shop.ssbcrack.com

MCQs

INDIAN HISTORY

MCQs

667.Garuda was adopted as dynastic emblem after the imperial Guptas by ___? [A] Rashtrakutas [B] Western Chalukyas [C] Shilaharas [D] Chedis Correct Answer: A [ Rashtrakutas ] Explanation: Rashtrakutas adopted “Garuda”as dynastic emblem after the imperial Guptas. 668.Who among the following was the first Satavahana King to introduce the Ruler’s head on the coins ? [A] Satkarni I [B] Gautamiputra Satkarni [C] Vasishthiputra Pulumavi [D] Yajna Satakarni Correct Answer: A [ Satkarni I ] Explanation: Satavahana dynasty ruled from Pune in Maharastra to Coastal Andhra Pradesh in the 2nd century BC onwards. The coins issued by Satavahanas had Bilingual legends. The name of the Kings was mentioned in Prakrit as well as some south Indian Language. Satavahana Kings promoted Buddhism. Satkarni I was the first Satavahana King to introduce the Ruler’s head on the coins. 669.Which one of the following places was a mint centre of the Yaudheyas? [A] Bayana [B] Rohtak [C] Bareilly [D] Mathura Correct Answer: B [ Rohtak ] Explanation: The sacred town of Rohtak, also called Rohtak or mint of the Yaudheyas . Note : Karttikeya was the principal deity of the Yaudheyas. 670. Who among the following was the ruler of Kanchi during the time of Samudragupta ? [A] Hastivarman [B] Mantaraja [C] Nilaraja [D] Vishnugopa Hide Answer Correct Answer: D [ Vishnugopa] Explanation: King Vishnugopa (Pallava dynasty ) of Kanchi is mentioned in the list of rulers of the South defeated by Samudra Gupta. It is mentioned in the Allahabad Pillar inscription of Samudra Gupta. 671.Who among the following is known to have performed four Asvamedha sacrifices? [A] Pushyamitra Sunga [B] Pravarasena I [C] Samudragupta [D] Nandivarman Pallavamalla Correct Answer: B [ Pravarasena I ] Explanation: Vakataka Dynasty: 3rd Century -5th Century AD. They were the most important after the fall of Satavahanas and before the rise of Chalukyas and they ruled in Modern Maharashtra and Madhya 146 | P a g e

shop.ssbcrack.com

MCQs

INDIAN HISTORY

MCQs

Pradesh.power They were contemporary of Guptas.The dynasty was founded by Vindhyashakti. Pravarasena I was the real founder of Vakataka empire. He performed four Asvamedha Yajnas. 672.In which of the following inscriptions Ashoka made his famous declaration, “All men are my children”? [A] Minor Rock Edict (Ahraura) [B] Pillar Edict VII [C] Lumbini Pillar Edict [D] Separate Kalinga Rock Edict I Correct Answer: D [ Separate Kalinga Rock Edict I ] Explanation: Separate Edicts: They were found at sites in Kalinga »Separate Edict I : Asoka declared all people are my sons »Separate Edict II: proclamation of edicts even to a single person. 673.The beneficiaries of Asoka’s donations in the region of Barabar Hill were__? [A] Buddhists [B] Ajivikas [C] Svetambar Jains [D] Digambar Jains Correct Answer: B [ Ajivikas ] Explanation: The Edicts of Asoka are a collection of 33 inscriptions on the Pillars of Asoka. The one of the Edict of Asoka is “Barbara cave Inscription : giving away the Barbara cave to Ajivika sect. Dasharataha is known to have gifted the Nagarjuni hills to Ajivikas. 674. Who identified “Sandrokottus” of the Greco-Roman literature with Chandragupta Maurya? [A] D. R. Bhandarkar [B] Alexander Cunningham [C] R. P. Chanda [D] William Jones Correct Answer: D [ William Jones ] Explanation: i.Maurya Empire was the first and one of the greatest empire to be established in Indian history by Chandragupta Maurya. Ii.He dethronged the last Nanda ruler Dhananand and occupied Patliputra in 322 BC with the help of Kautilya(Chankya). Iii.In 305 BC ,he defeated Selecus Nikator ,who was one of the generals of Alexander The Great (Greek ruler). Iv.He is also well-known in the Greek texts as – Sandro Kottus, Androcottus, Sandokyptos. 675.Rulers of which of the following dynasties maintained diplomatic relations with distant countries such as Syria in the West? [A] Maurya [B] Gupta [C] Pallava [D] Chola Correct Answer: A [ Maurya] Explanation: »Mauryan Emperor Bindusara had friendly relations with the Greek King Antiochos-I of Syria. Deimachos was a Syrian Ambassador who came in the court of Bindusara. »Mauryan Emperor Ashoka the Great: In his rock edict 13th mentions the names of 5 Hellenic kings– Antiochus II of Syria, Ptolemy II of Egypt, Antigonus of Messedonia, Magas of Syrina, Alexander of Epirus .He sent missionaries to all of them. »This shows that Mauryan dynasty maintained the diplomatic relations with distant countries like Syria in the West. 147 | P a g e

shop.ssbcrack.com

MCQs

INDIAN HISTORY

MCQs

676.The rulers of which among the following dynasties adopted the title Devaputra ? [A] Maurya [B] Sunga [C] Kushana [D] Saka-Kshatrapa Correct Answer: C [ Kushana ] Explanation: Kushanas Empire: Kanishka (100 – 126 AD), a prominent ruler of the Kushan Dynasty who achieved remarkable milestones in the state of Jammu & Kashmir. His descendants called him as Devaputra which means son of gods. 677.The Mahasamghika School arose at which among the following places? [A] Bodha Gaya [B] Rajagriha [C] Sravasti [D] Vaisali Correct Answer: D [ Vaisali ] Explanation: Also known as Mahasamghika, Mahasanghika, or Mahasanghika. »Great monastic community. »One of first schools of Nikaya Buddhism »Result of (the first schism at) the Second Council at Vaishali where they were outvoted and seceded. »The teachings of this school concerning the nature of Buddhas and Bodhisattvas. 678.Which of the following pairs is not correctly matched ? [A] Sakiyas →Kapilvastu [B] Koliyas→Ramagrama [C] Kalamas →Allakappa [D] Mallas →Kusinagara Correct Answer: C [ Kalamas →Allakappa ] Explanation: Sakiyas of Kapilavastu : A republican clan with the capital at Kapilavattu, was a vassal of King Prasenjit of Kosala It was a great center of Buddhism. Koliyas, a republican clan had 2 chief settlements in Ramagama and Devadaha. Buddha was related to the Koliyans through his mother and wife. After the demise of the Buddha, the Koliyans obtained a share of his relics and erected a stupa over it. Mallas had 2 confederacies; one at Kusinara and the other at Pava Kusinara was famous as it was the site of Buddha’s Mahaparinirvana. Malla Roja initially opposed Buddhism, later accepted him after he heard the Buddha’s discourses. Bulis of Allakappa, Moriyas of Pipphalivana and Kalamas of Kesaputta. They were followers of the Buddha.All are related to Gautam Bhudha and Kautilya also mention many self governing clans in which these exist. 679.Who among the following laid down for punishment for a person becoming mendicant without making adequate provision for dependent wife and children? [A] Manu [B] Yajnavalkya [C] Kautilya [D] Narada Correct Answer: C [ Kautilya ]

148 | P a g e

shop.ssbcrack.com

MCQs

INDIAN HISTORY

MCQs

Explanation: Chanakya : He is also known by Vishnugupta, Kautilya,was born around 350 BC and is known for his being the chief architect of Mauryan Empire and writing the pioneering work in the Economics and Political Science that is Arthashstra. He is known as Indian Machiavelli in the western world. 680.With which among the following places, the twenty third Jain Tirthankara was associated? [A] Vaishali [B] Kausambi [C] Varanasi [D] Sravasti Correct Answer: C [ Varanasi ] Explanation: Tirthankar Parshvanath was the son of king Ashvasena and queen Vama of Varanasi. He achieved Nirvana on the Sammet Sikhar. He lived in Varanasi around 800 BC. Parsvanatha is the 23rd Jain Tirthankar. 681.Which of the following Upanishads is written in prose ? [A] Isa [B] Katha [C] Brihadaranyaka [D] Svetasvatara Correct Answer: C [ Brihadaranyaka ] Explanation: Brihadaranyaka will be correct answer of this question. 682. Who among the following were teachers of Gautama Buddha before his enlightenment ? 1. Alara Kalama 2. Udraka Ramputra 3. Makkhali Gosala 4. Nigantha Nataputta Indicate your answer from the codes given below. [A] 1 and 4 [B] 3 and 4 [C] 2 and 3 [D] 1 and 2 Correct Answer: D [ 1 and 2 ] Explanation: i.At ‘Vaishali’, Alara Kalama (Sankhya philosopher) taught the techniques of meditation and the teaching of Upanishads to Gautam Buddha. Ii.At ‘Rajgriha’, Udraka Ramputra taught him to achieve the highest level of meditation. Iii.At last, at the age of 35,Gautam Bhudha attained nirvana at ‘Bodh gaya ‘in Bihar. 683.Which among the following is an extant Shaka of the Rigveda Samhita? [A] Saunaka [B] Ashvalayan [C] Shakala [D] Sankhayana Correct Answer: C [ Shakala ] Explanation: Rigveda is the oldest Veda .In it,only two shakas of the Rig-Veda, Shakala sakha and Baskala shaka, remain alive out of the 21 which existed at one time.

149 | P a g e

shop.ssbcrack.com

MCQs

INDIAN HISTORY

MCQs

684.In the Junagarh inscription, which among the following Saka ruler achievements are highlighted? [A] Moga [B] Azes [C] Rudraraman [D] Nahapana Correct Answer: C [ Rudraraman ] Explanation: The military achievements, territories and many personal qualities of Rudraraman are highlighted in the famous Junagadh inscription, written in 150 AD. It is the first major inscription to be written in sanskrit. 685.During the reign of which Kushana king, the Fourth Buddhist Council was held? [A] Kanishka [B] Huvishka [C] Vasudeva [D] Vasishka Correct Answer: A [ Kanishka ] Explanation: The Fourth Buddhist Council was held at Kundalvana, Kashmir in 72 AD during the reign of Kushan king Kanishka. It was held under the Presidentship of Vasumitra to compose commentaries on the Tripitika. 686.The reign of which of the following reigns was known as golden age for Mathura school of art? [A] Indo-Greeks [B] Sakas [C] Kushanas [D] Gupta Correct Answer: C [ Kushanas ] Explanation: The period of Kushana King Kanishka, Huviska and Vasudeva is considered to be golden age for Mathura school of art. The theme of Mathura school of art may vary from Buddhist to Brahmanical to sometimes secular. The more stress is given to the inner beauty and facial emotions rather than bodily gesture. The sculptures were made on White-spotted red stones. 687.Who was the first Saka king in India? [A] Moga [B] Rudradaman [C] Azes [D] Ghatotkacha Correct Answer: A [ Moga ] Explanation: An Indo-Scythian king, Moga (or Maues) was the first Saka king in India who established Saka power in Gandhara and extended supremacy over north-western India. 688.The governors called Strategos were introduced by __? [A] Sakas [B] Indo-Greeks [C] Kushanas [D] Bactrians Correct Answer: B [ Indo-Greeks ] Explanation: The Indo-greeks introduced practice of military governorship. The governors were called Strategos. 150 | P a g e

shop.ssbcrack.com

MCQs

INDIAN HISTORY

MCQs

689.Which Greek King of Bactria invaded India about 190 B.C.? [A] Demetrius [B] Alexander [C] Seleucus Nicator [D] Menander Correct Answer: A [ Demetrius ] Explanation: Demetrius,the king of Bactria invaded India about 190 B.C. And conquered a considerable part of the Mauryan Empire in the north-west. 690.The conversation between Nagasena and Menander-I related to Buddhism is recorded in which book? [A] Milindapanho [B] Panhomenanda [C] Nagapanho [D] Menandapanho Correct Answer: A [ Milindapanho ] Explanation: Before becoming a Buddhist, Menander (an Indo-Greek king) asked Nagasena many questions relating to Buddhism.These questions and Nagasena’s answers are recorded in a form of book known as ‘Milindapanho (or The Questions of Milinda). 691.Which Buddhist monk converted Milinda (Indo-Greek king) to Buddhism? [A] Nagasena [B] Gautam Budha [C] Shakyamuni [D] Mahadharmaraksita Correct Answer: A [ Nagasena ] Explanation: The Indo-Greek king, Milinda (or Menander I ) was converted to Buddhism by Nagasena (or Nagarjuna), a Buddhist monk and philospher. Before becoming a Buddhist, Menander asked Nagasena many questions relating to Buddhism.These questions and Nagasena’s answers are recorded in a form of book known as ‘Milindapanho (or The Questions of Milinda). 692. Menander I, the famous Indo-Greek ruler set up his capital at which place? [A] Sialkot [B] Pathankot [C] Rajkot [D] Sialdah Correct Answer: A [ Sialkot] Explanation: The famous Indo-Greek, Menander I (165/155–130 BCE) set up his capital at Sakala or Sialkot. 693.Which Greek ambassador set up a pillar in honour of Vishnu? [A] Megasthenes [B] Heliodoros [C] Theodorus [D] Plato Correct Answer: B [ Heliodoros] Explanation: The Greek ambassador Heliodoros set up a pillar in honour of Vishnu (Vasudev) at Vidisha. He was sent to the court of Sunga King, Bhagbhadra by the Greek King of Taxila, Antialkidas. 151 | P a g e

shop.ssbcrack.com

MCQs

INDIAN HISTORY

MCQs

694.Who were the first rulers in India to issue gold coins attributed to the Kings? [A] Sungas [B] Kanvas [C] Indo Greeks [D] Sakas Correct Answer: C [ Indo Greeks ] Explanation: The Indo-Greeks were the first rulers in India to issue coins which can definitely be attributed to the kings. They were the first to issue gold coins in India. 695.The famous book ‘Brihat Katha’ was written by__? [A] Gunadhya [B] Sarva Varman [C] Panini [D] Radhagupt Correct Answer: A [ Gunadhya ] Explanation: The famous book ‘Brihat Katha’ was written by Gunadhya, who was the great scholar in the court of Satvahana King Hala. 696.Which among the following Satkarni ruler recovered Malwa from the Shaka rulers? [A] Gautamiputra Satkarni [B] Satkarni I [C] Hala [D] Yajnsari Satkarni Correct Answer: D [ Yajnsari Satkarni ] Explanation: Yajnsari Satkarni (165-194 A.D.) recovered Malwa from the Shaka rulers. 697.Which of the following inscription are related to Satavahana period? [A] Nanaghat [B] Nasik [C] Paithan [D] a & b Correct Answer: D [ a & b ] Explanation: The Nasik and Nanaghat inscriptions are the major sources that gives detailed information about the Satavahana empire. The Nasik inscription was made by Gautami Balasari and Nanaghat inscription was issued by Naganika. 698.Which of the following ruler of Satavahana Empire composed Gathasaptashati? [A] Simuka [B] Gautamiputra Satkarni [C] Pulumayi [D] Hala Correct Answer: D [ Hala ] Explanation: Gathasaptashati is composed by Savahana king Hala. It is a collection of poems in Maharashtri Prakrit. The theme of most of the poems is based on love.

152 | P a g e

shop.ssbcrack.com

MCQs

INDIAN HISTORY

MCQs

699.The rulers of which dynasty make land grants to Brahmanas? [A] Maurya [B] Sunga [C] Satvahana [D] Kanvas Correct Answer: C [ Satvahana ] Explanation: The rulers of Satvahanas dynasty were the first to make land grant to Brahamanas. 700. As per Asoka’s inscriptions, which among the following place was declared tax free and proclaimed only 1/8th part as taxable? [A] Kushinagar [B] Lumbini [C] Kathmandu [D] Sarnath Correct Answer: B [ Lumbini] Explanation: At the 20th anniversary of his enthronement, Asoka announced Lumbini as tax-free and proclaimed only 1/8th part as taxable. Description of this fact is found in the inscriptions of Nigliva and Rumindei.

153 | P a g e

shop.ssbcrack.com

MCQs

INDIAN HISTORY

MCQs

MEDIEVAL INDIAN HISTORY A Brief Note on Medieval Indian History After the death of Harsha the Rajputs came into prominence on the political horizons of North India. The Rajputs were known for their bravery and chivalry but family feuds and strong notions of personal pride often resulted into conflicts. The Rajputs weakened each other by constant wrangling. The disunity among Rajputs allowed the foreigners (Turks) to enter India. The defeat of Prithvi Raj Chauhan (the greatest Rajput warrior of the time) at the hands of Mohammad Ghori, in the battle of Tarain 1192, marked a new chapter in the history of India. After the death of Mohammad Ghori, Qutub-Uddin Aibak (Ghori's lieutenant in India) founded the Slave Dynasty. With this the Delhi Sultanate came into being. Aibak was followed by his slave, Iltutmism, who was succeeded by his daughter, Razia (1236 - 1239). Razia sat on the throne of Delhi for a short while. The Slave dynasty was followed by the Khalji, Tughlaq, Sayyids and Lodi dynasty. Some of the notable among the Sultanate rulers were Balban, Alauddin Khalji and Mohammad Bin Tughlaq. Alauddin Khalji (1296 - 1316 AD) was not only a distinguished commander but also an able administrator. He is remembered for his military campaigns in the south as well as market reforms and price control measures. Muhammad Bin Tughlaq (1324 - 1351 AD) was a visionary who but unfortunately all his projects failed. His most controversial project was the transfer of capital from Delhi to Daulatabad. With the death of Ibrahim Lodi in the battle of Panipat, (at the hands of Babur, the founder of Mughal Empire) the Delhi Sultanate came to an end. The Sultanate introduced, in the sub continent, the Islamic concepts of society and governance, and thus prepared the ground for a dazzling interaction between two world civilizations. Babar (1526-30 AD) founded the Mughal Empire in India. He was a descendant of Timur as well as Changez Khan. He was ousted by his own cousins from his small principality in Central Asia and sought fortune in India. Babar came to India and defeated Ibrahim, the last Lodi Sultan in 1526. Babar was succeeded by his son Humayun but he was ousted from Delhi by Sher Shah, an Afghan chieftain. Though Sher Shah (1540-55 AD) ruled only for a brief period of almost five years yet he showed great administrative skills. He is remembered as the builder of the Grand Trunk road and also for reforms in the revenue system. Though Humayun was successful in regaining Delhi but he was not destined to rule Delhi for long and died the same year. With this began the reign of one of the most glorious rulers of India, Akbar the great. Akbar (1556-1605 AD) consolidated political power and extended his empire over practically the whole of north India and parts of the south. Akbar was a great ruler and very well realized that if the empire was to attain stability, enough attention should be paid to all the subjects. Keeping this thing in mind he sought cooperation from the Rajputs. Jehangir (1605-27), the son of Akbar was a pleasure-loving man of refined taste. Contemporary historians have recorded that during his reign the Persian nobility related to his wife Nur Jahan had become very powerful at the royal court. Jehangir was followed by his son Shah Jahan (1628-58 AD). Shah Jahan was a great lover of buildings of whom the Taj Mahal is the most famous. Other notable buildings built by Shah Jahan are the Red Fort and the Jama Masjid at Delhi. Aurangzeb (1658-1707 AD) was a brave general and an able administrator but these virtues were overshadowed by his religious dogmatism and fanaticism. The Mughal Empire reached its zenith during the reign of Aurangzeb. But at the same time, he wasted his energy and resources in his long drawn out conflicts with the Marathas and other local rulers and principalities. After the death of Aurangzeb, the 154 | P a g e

shop.ssbcrack.com

MCQs

INDIAN HISTORY

MCQs

mighty Mughal Empire started to totter. His successors were weak and incapable of holding the far-flung empire together. The imperial authority was challenged from all corners and the provincial governors began to assert their independence. In western India, Shivaji (1637-80 AD) united the Marathas into an efficient military unit gave them a sense of national identity. They adopted guerrilla tactics to batter the Mughals and put a severe drain on their economic and psychological resources. The main contenders for political supremacy of India in the 17th and 18th Centuries were the Marathas, the Sikhs in Punjab and Hyder Ali (1721 - 1782 AD) in Mysore. MEDIEVAL HISTORY • Time period between 8th -18th century AD. • Medieval period divided into two parts: • Early medieval period – period from 8th to beginning of 13th century • Later Medieval period - period from 13th century upto the 18th century. THE RAJPUTS Origin • • • •

Started around 8th century. Belonged to Surya vamsh or Chandra Vamsh. Descendants of Agni – Fire God. Historians believed that the Rajputs were Kshatriyas of Vedic period.

Rajput Kingdom • • •

After the death of Harsha Vardhana, Rajputs came into power. Established number of independent kingdoms. Gurjara Pratiharas, the Chauhans, the Chalukyas and the Chandelas are well known Rajput kingdoms.

Period • • • •

Ruled Northern and Central India for many years. Many invaders attacked India during this period. Conducted war and conquered the kingdoms. Well known for honouring premises.

Painting and Literature • • • • •

Patrons of beautiful buildings. Also known as Rajasthani painting, a style of Indian paintings. Ramayana and Mahabharata are great ancient epics. Palaces and temples were decorated with paintings. In Sanskrit, they created literary works.

Important Battles • • • • •

Battle of Tarain: Prithviraj (Chauhan dynasty) defeated Muhammed Ghori near Thaneswar. Battle of Chandawar: Muhammed Ghori defeated Jai Chandra in 1194 AD. Battle of Khanwal: Took place between Rana of Mewar, Rana Sanga and Bebur of Farghana in 1527 AD. Battle of Chanderi: Fought between Medini Rai of Chanderi and Babur in 1528 AD. Babur won. Battle of Delhi: Main event in India’s history. Chandra Vikramaditya was defeated by Mughals.

155 | P a g e

shop.ssbcrack.com

MCQs

INDIAN HISTORY

MCQs

Fall of Rajputs • •

Captured by Arabs, Turks and Mughals. Around the 13th century, the Rajput’s rule came to an end.

The Delhi Sultanate • •

Five dynasties founded after the Turkish Invasion are known as Delhi sultanate. They are as follows:

Slave Dynasty (1206-1290) • • • • • • • •

Also called as Ilbari dynasty, Yamini dynasty and Mamluk dynasty. Qutub-ud-din Aibak founded the slave dynasty in 1206 AD, who was a slave of Mohammed Ghori. He was the first Muslim ruler in India. Lahore was the capital of Qutub-ud-din Aibak. He was also known as ‘Lakh Baksh’ or ‘giver of lakhs’. He started the construction of the Qutub Minar in 1199 in Delhi, in memory of Quaja Qutub - ud-din Bhaktiar Kaki, a saint. Qutub Minar is a five storied building and it was completed by Ithumish. In the year 1210, while playing polo, he fell from the horseback and died.

Khilji Dynasty (1290-1320) • • • • • • •

Founded by Malik Firoz in 1290 and took up the title Jalaluddin Khilji (1290-96) Alauddin Khilji – nephew of Jalaluddin Khilji. He killed him and captured the throne in 1296. First Turkish Sultan of Delhi. He indicated ‘Kingship knows no Kinship’. He abolished the Zamindari system. The Khilji dynasty came to an end when Mubarak Shah Khilji was killed by Khusrau Khan.

TUGHLAQ DYNASTY (1320 - 1412) Ghiyasuddin Tughlaq (1320-1325 AD) • • • •

Founded by Ghiazuddin Tughlaq. Real name was Ghazi Malik. After killing Khuzru Khan in 1320, he founded the dynasty. Built new city near Delhi named Tughlakabad. When he died, his son Jauna (Ulugh Khan) succeeded him under the title Mohammed-bin-Tughlaq.

Mohammed-bin-Tughlaq (1325 – 1351 AD) • • • • •

Captured the throne in 1325 AD. Introduced administrative reforms: Taxation in Doab, transfer of Capital to Daulatabad, introduced token currency etc. In 1327, he shifted his capital from Delhi to Devagiri. Succeeded by Firoz Shah Tughlaq his elderly cousin.

Firoz Shah Tughlaq (1351-1388 AD) • • •

First sultan of Delhi to abuse Jaziya. Author of Fatuhat-i-Firozshahi. Constructed canals for irrigation.

156 | P a g e

shop.ssbcrack.com

MCQs

• • •

INDIAN HISTORY

MCQs

Made Iqtadari system hereditary. After his rule, Mohammed Shah Tughlaq came to the throne. During his rule, Timur appointed Khizr Khan.

Sayyid Dynasty (1414-1451 AD) • • •

Founded by Khizr Khan in 1414. Alauddin Alamshah or Shah Alam was the last sultan. In 1451, he was killed by Bahalol Lodhi.

Lodhi Dynasty (1451-1526 AD) • • • • • • •

Founded by Bahlol Lodhi in 1451 and lasted up to 1526. Sikhandar Lodhi ruled from 1489 to 1517 and shifted the capital from Delhi to Agra. Considered as ‘Maker of Agra city’. Ibrahim Lodhi was the last Lodhi or Delhi Sultan. On 21st April 1526, In the First Battle of Panipat, Babar defeated Ibrahim Lodhi. Mohammed Ghazni, was the first to assume the title sultan. Persian - official language of Delhi Sultanate.

VIJAYANAGAR KINGDOM • •

Founded in 1336 Kingdom lasted for 230 years and was made up of four dynasties. ➢ Sangama – (1336 – 1485) ➢ Saluva – (1485 – 1505) ➢ Tuluva – (1505-1565) ➢ Aravidu – (1565 – 1672)

Sangama History (1336-1485) • • • • • • •

Founders – Harihara and Bukka Rai Next ruler was Deva Raya I, constructed dams on the Tungbhadra River with the help of saint Vidyaranya. Devaraya II (1423-1426) – Greatest Sangama ruler. He wrote ‘Mahanataka Sudhanidhi’. Abdur Razzaq, Persian Ambassador visited his court. Replaced by Saluva dynasty lasted for two decades. A new dynasty called Tuluva founded by Vira Narasimha.

Tuluva History • • • • • • •

Krishnadeva Raya (1509-1529) – greatest ruler. Nicolocont, the Italian traveler visited his court. Krishnadeva Raya known as ‘Andhra Bhoja’ Built a new city named ‘Nagalapuram’, decorated with Hazura and Vithalswamy temple. Sadasiva – last ruler of dynasty. In 1565, Battle of Talikota Sadashiv was defeated. A new dynasty called Aravidu dynasty found by Rama Raya’s brother, Thirumala.

THE MUGHAL EMPIRE •

They were originally Turks.

157 | P a g e

shop.ssbcrack.com

MCQs

• • • •

INDIAN HISTORY

MCQs

Belonged to Chaghtai branch Also known as Second Classic Age. Mughal Empire also called as Timurid Empire because of relation to Amir Timur. Twenty in number. Ruled from 1526-1857.

Mughal Emperors • • • • • • •

Babur – (1526-1530) Humayun – (1530-40 & 1555-1556) Akbar – (1556-1605) Jahangir – (1605-1627) ShahJahan – (1628-1658) Aurangzeb Alamgir Later Mughal or fall of Mughal

Babur (1526-1530) • • • • • • • • • •

Founder of Mughal Empire. Fifteenth descendant of Timur on Father’s side and 14th descendant of Chengizkhan on mother’s side. At age 11, he became the ruler of Samarkhand. Captured Kabul in 1504. Attacked India 5 times for wealth. The First attack of India in 1519 Bhera, first place captured by Babur. On 16th March, 1527, defeated Rana Sangha of Mewar, in the Battle of Khanwa. In 1528, Battle of Chanderi, defeated another ruler Medini Raj. In 1529, Battle of Ghaghra, he defeated Afghans under Muhammed Lodhi. Died in 1530. His biography was written in Turkish language as ‘Tuzuk-i-Baburi or Baburnamah’.

Humayun (1530-40 & 1555-56) • • • • • • • •

Born in 1507 in Kabul, son of Babur and Mahim Begum. On 29th December 1530, became the Mughal emperor at the age of 23. Divided the Empire among his three brothers – Askari, Hindal and Kamran. In 1539, Battle of Chausa, he was defeated for the first time by Shershah Suri. In 1540, Shershah defeated Humayun in the battle of Kanauj and founded Sur dynasty. The period from 1540 to 1555 – known as the period of temporary eclipse. His biography was written by his sister, Gulbadan Begum as ‘Humayun Namah’. Built Dinpanah in Delhi on 1533.

Akbar the Great (1556-1605) • • • • • •

Jalaluddin Muhammed Akbar, son of Humayun and Hamida Banu Begum. Born in Amarkot in 1542. At Kalanur, age of 14 he came to the throne on 14 th February, 1556. In the second battle of Panipat he killed Hemu on 2nd November 1556. After dismissing Bairamkhan, he became an independent ruler in 1560. Later he married Salima Begum.

158 | P a g e

shop.ssbcrack.com

MCQs

• • • • • • • • • • •

• • • • •

INDIAN HISTORY

MCQs

In 1561, defeated Baz Bahadur, Sultan of Malwa. In 1562, he married Joda Bhai, daughter of Raja Bharmal. In 1572, captured Gujarat and built new capital city Fathepur Sikri (city of victory) near Agra. In 1575, constructed prayer house Ibadatkhana, in Fathepur Sikri. In 1585, Ralph Fitch visited his court. In 1576 he defeated Maharana Pratap in the Battle of Haldighat. Died in 1605. Maintained scholastic assembly in his court. They are as follows: Abul Fazul: Historian who wrote Akbar’s biographical works Ain-i-Akbari and Akbar Namah Abul Faizi: Abul’s brother and Persian poet. Translated Mahabharata as ‘Razam Namah’ in Persian. Mian Tansen: Court musician, Ramtanu was his original name. Composed Raga, Rajdarbari in honour of Akbar. Birbal: Akbar’s court jesterwhose real name was Mahesh Das. Raja Todar Mal: Akbar’s finance minister. Translated Bhagavata Purna into Persian. Man Singh: Military commander of Akbar’s court. Badauni: Historian who translated Ramayana into Persian. Tulsidas: Hindi poet who wrote Ramacharitamanas.

Jahangir (1605 – 1627) • • • • • • • • • • •

Salim was his earlier name, Akbar called him Sheik Baba. In 1605, he came to the throne. He was the son of Akbar and Jodabai. In 1611, he married an Afghan widow named Mehr-un-nisa. Later he gave her the titles, Noor Mahal, Noor Jahan and Padshah Begum. In 1609, he received William Hawkins who came to India to obtain a trade concession. In 1615, Sir Thomas Roe, the first ambassador of James - I reached Jahangir’s court. His period is called the ‘Golden age of Mughal painting’. Built Shalimar and Nishant gardens in Srinagar. Anarkali was Jahangir’s lover. Jahangir wrote his autobiography as ‘Tuzukh-i-Jahangiri’ in Persian language. Died in 1627.

Shah Jahan (1628 – 1658) • • • • • • • •

Born on 5th January, 1592 in Lahore. Jagat Gosain, his mother and his childhood name was Khurram. Married Arjumand Banu Begum also known as Mumtaz Mahal. His period is considered the ‘Golden Age of Mughal Architecture’ and he is also known as ‘Prince of Builders’.

In 1631, he started the construction of Taj Mahal in memory of his wife and completed it in 1653. Situated on the banks of Yamuna River in Uttar Pradesh. In 1658, he was imprisoned by his son Aurangzeb. Died in 1666.

159 | P a g e

shop.ssbcrack.com

MCQs

INDIAN HISTORY

MCQs

Aurangzeb (1658 – 1707) • • • • •

He imprisoned his father and made himself Padshah in 1658. Also known as ‘Zinda Pir’. Banned music and dance. He was the last Mughal Emperor. In 1675, 9th Sikh Guru ‘Guru Tej Behadur’ was executed. He died on 20th February, 1707.

THE MARATHA AGE • • • • • • • • • •

• • • • • •

Chatrapathi Sivaji – Greatest leader of Marathas. Belonged to the Bhonsle clan of Marathas. Born on 19th February, 1627 at Shivener. Son of Shaji Bhonsle and Jiga Bai. Dadaji Kondadev was his tutor. In 1646, he captured his first place named Torna. In 1666, he visited Aurangazeb’s court at Agra. He crowned himself on 16th June, 1674 and became Chatrapathi and assumed title ‘Haidavadhasmodharak’. At the age of 53, he died in 1680. Shahu became Chatrapathi in 1708. There were seven Peshwas namely, ➢ Balaji Vishwanath (1713-20) ➢ Balaji Baji Rao I (1720 – 1740) ➢ Balaji Baji Rao (1740 – 1761) ➢ Madho Rao (1761 – 1772) ➢ Narayan Rao (1772 – 73) ➢ Madho Rao Narayan (1773 – 95) ➢ Baji Rao II (1795 – 1818) Of these, Baji Rao-I was ablest and Baji Rao II was weakest. Peshwa – Maratha Chief Minister. First Maratha War – 1775 – 82 Second Maratha War – 1803 – 05 Third Maratha War – 1816 – 19 Maratha script is known as Modi script.

160 | P a g e

shop.ssbcrack.com

MCQs

INDIAN HISTORY

MCQs

MEDIEVAL INDIAN HISTORY (MULTIPLE CHOICE QUESTIONS) 1. During whose reign Changez Khan invaded India? [A] Balban [B] Iltutmish [C] Razia [D] Alauddin Khilji Correct Answer: B [Iltutmish] 2. Which among the following is a incorrect statement regarding market regulations by Ala-ud-din Khilji? [A] The price controls were regulated on almost the entire market [B] They vanished immediately after the death of the sultan [C] The price controls were regulated on almost the entire market [D] All are correct Correct Answer: D [All are correct] 3. Which among the following was called as the Central Pillar of the administrative system of Delhi sultanate? [A] Izra system [B] Iqta System [C] devsirme system [D] Chahalgami Correct Answer: B [Iqta System] 4. Which among the following is incorrect regarding Buland darwaja? [A] It was built by Akbar in 1602 [B] It was built in Fatehpur Sikri [C] It was built to commemorate Akbar’s victory on Gujarat only [D] All are correct Correct Answer: D [All are correct] 5. Out of the battles fought between contemporary rulers and Babur, which among the following resulted in the firm establishment of Babur’s rule in North India? [A] Battle of Panipat [B] Battle of Khanva [C] Battle of Ghagra [D] Second Battle of Panipat Correct Answer: B [Battle of Khanva] 6. After the attack of Mahmud of Ghazanavi, who among the following reconstructed the Somnath Temple? [A] Bhimraja -I [B] Bhimdev [C] Mularaj I [D] Jay Singh Siddharaaj Correct Answer: B [Bhimdev] 161 | P a g e

shop.ssbcrack.com

MCQs

INDIAN HISTORY

MCQs

7. Which among the following represents incorrect founder of a dynasty? [A] Parmar Dynasty – Upendra Raja [B] Chandela Dynasty – Nannuka [C] Kalachuri Dynasty – Gangeyadev [D] Solanki Dynasty – Mularaja I Correct Answer: C [Kalachuri Dynasty – Gangeyadev] Explanation: Kalachuri Dynasty was founded by Kokalla and Gangeyadev was the most powerful king of this dynasty 8. Which among the following statement is INCORRECT? [A] Vijay sen was the most famous and powerful king of Sen Dynasty [B] Samanta Sen was the founder of Sen Dynasty [C] Samanta Sen was the son of Hemanta Sen [D] Adbhutta Sagara was a work of Ballal Sen Correct Answer: C [Samanta Sen was the son of Hemanta Sen] Explanation: Samanta Sen was the Father of Hemanta Sen 9. Which among the following is not correct ? [A] The capital of pandyas was Madurai [B] The capital of Cheras was Vanchi [C] Capital of the Videha Kingdom – Mithila [D] All are correct Correct Answer: D [All are correct] 10. The word coromandal cost has been derived from the kingdom of which of the following? [A] Cheras [B] Cholas [C] Chalukyas [D] Pandyas Correct Answer: B [Cholas] 11. The eighth-century tripartite power struggle was among which of the following? [A] Cholas, Rastrakutas and Yadavas, [B] Chalukyas, Pallavas and Pandyas [C] Cholas, Pandyas and Chalukyas [D] Cholas, Pandyas and Chalukyas Correct Answer: B [Chalukyas, Pallavas and Pandyas] 12. Tower of Victory ” or Vijay stambh in Rajasthan is located in [A] Ajmer [B] Chittorgarh [C] Jaipur [D] Udaipur Correct Answer: B [Chittorgarh]

162 | P a g e

shop.ssbcrack.com

MCQs

INDIAN HISTORY

MCQs

13. Who among the following changed the name of Tanna Mishra to Tansen? [A] Akbar [B] Raja Vikramjit of Gwalior [C] Raja Mansingh Tomar [D] Swami Haridas Correct Answer: B [Raja Vikramjit of Gwalior] 14. Which among the following is the modern location of ” Battle of Gogunda” ? [A] Gujarat [B] Madhya Pradesh [C] Rajasthan [D] Harayana Correct Answer: C [Rajasthan] Explanation: It was battle of Haldighati fought between the Mughal Empire and the forces of Mewar on June 21, 1576 at Haldighati near Gogunda town of Rajasthan 15. Karkota Dynasty was confined to which of the following areas? [A] Kannuj [B] Kashmir [C] Deccan [D] Gujarat Correct Answer: B [Kashmir] 16. Sakambhari near Ajmer is a place related to emergence of which of the following dynasties? [A] Gahadwals [B] Parmaras [C] Pratiharas [D] Chauhanas Correct Answer: D [Chauhanas] 17. Who among the following assumed the title of Islam Shah Suri, ? [A] Shershah Suri [B] Jalal Khan [C] Firuz Shah Suri [D] Ibrahim Khan Suri Correct Answer: B [Jalal Khan] Explanation: Jalal Khan assumed this title. He was son of Shershah and was his successor 18. Who invaded India in 1398 AD during the reign of Nasiruddin Mahmud of Tughlaq Dynasty? [A] Chegiz khan [B] Khijr Khan [C] Timur [D] Babur Correct Answer: C [Timur] 19. In the reign of Firuz Shah Tughalq which of the following tax was 1/6 of the booty captured during war? [A] Kharaz [B] Khams 163 | P a g e shop.ssbcrack.com

MCQs

INDIAN HISTORY

MCQs

[C] Jaqat [D] Zaziya Correct Answer: B [Khams] 20. That part of land granted by the sultan to military chiefs for maintenance of a given number of troopers called ? [A] Muahatsib [B] Iqta [C] Nazir [D] Usharaf Correct Answer: B [Iqta] 21. Which among the following is not a work of Amir Khusrau? [A] Khazyan-ul-Futuh [B] Tughluq Nama [C] Tarik-i-Alai [D] Tabqat-i-Naisiri Correct Answer: D [Tabqat-i-Naisiri] 22. Rihla” was a work of which of the following? [A] Mir Hasan Dehlvi [B] Ibn Battuta [C] Malik Murtaza [D] Muhammad al-Idrisi Correct Answer: B [Ibn Battuta] 23. Who among the following was considered as the “mixture of opposites”.? [A] Firuz Tughlaq [B] Mohammed bin Tughlaq [C] Ghiyasuddin Tughlaq [D] Kaiqubad Correct Answer: B [Mohammed bin Tughlaq] 24. Who among the following was known as Lakha Baksh? [A] Mohammed Ghori [B] Qutubuddin Aibak [C] Iltutmish [D] Balban Correct Answer: B [Qutubuddin Aibak] 25. What was the term used in the medieval India for the land directly ruled by the Central Government? [A] Muqtis [B] Khalisa [C] Inam [D] Inam Correct Answer: B [Khalisa] 164 | P a g e

shop.ssbcrack.com

MCQs

INDIAN HISTORY

MCQs

26. Which among the following was NOT a landmark of Alauddin Khilji’s Reign? [A] Introduction of Rationing system [B] Introduction of New Market regulation to provide essential commodities at reasonable rates [C] Introduction of Fixed price market by appointment of Diwan–I-Riyasat & Shahan-i-mandi [D] Creation of Agricultural department called Diwan-i-kohi Correct Answer: D [Creation of Agricultural department called Diwan-i-kohi] Explanation: Creation of Agricultural department called Diwan-i-kohi- landmark of Mohammed Bin Tughlaq reign 27. Which among the following was not a Landmark of the Iltutmish’s rule? [A] Introduction of silver tanka & Copper Jetal [B] introduction of iqtadari System [C] Organization of Chalisa [D] Putting forward the’ Divine Right Theory” Correct Answer: D [Putting forward the’ Divine Right Theory”] Explanation: Divine Right Theory was put forward by Balban 28. The Chandella’s Kingdom was annexed by which of the following rulers? [A] Mohammed Ghori [B] Qutubuddin Aibak [C] Iltutmish [D] Alauddin Khilji Correct Answer: D [Alauddin Khilji] 29. Who among the following founded the principality of Ghazani in AD 962? [A] Mahmood Ghazani [B] Subuktgeen [C] Alptigin [D] Shah Mahmoud Correct Answer: C [Alptigin] 30. Who among the following was defeated by Mohammed Ghori in battle of Chandawar? [A] Prthviraj Chauhan [B] Mularaja [C] Jaichandra [D] Jaypala Correct Answer: C [Jaichandra] 31. Chandradeva, was the founder of which of the following dynasty? [A] Chauhans [B] Gahadwalas [C] Chalukyas [D] Chandellas Correct Answer: B [Gahadwalas]

165 | P a g e

shop.ssbcrack.com

MCQs

INDIAN HISTORY

MCQs

32. Which among the following rulers belonged to Mamulak Dynasty? [A] Alauddin Khilji [B] Ibrahim Lodhi [C] Balban [D] Mohammed Ghori Correct Answer: C [Balban] 33. Who among the following wrote Astangasangraha.? [A] Banabhatta [B] Vagabhatta [C] Aryabhatta [D] Kalhana Correct Answer: B [Vagabhatta] 34. ”Loom ” was a central technological innovation in which of the following era? [A] Sultanate Era [B] Rajput Era [C] Mughal Era [D] None of the above Correct Answer: A [Sultanate Era] 35. Which of the following Mughal emperor is famous for his golden chain of justice? [A] Akbar [B] Shahjahan [C] Jahanagir [D] Babur Correct Answer: C [Jahanagir] 36. Which of the following Sultanate Rulers made Delhi as capital in place of Lahore? [A] Qutubuddin Aibak [B] Iltutmish [C] Razia Sultana [D] Akbar Correct Answer: B [Iltutmish] 37. Which of the following Sultanate Rulers made Delhi as capital in place of Lahore? [A] Qutubuddin Aibak [B] Iltutmish [C] Razia Sultana [D] Akbar Correct Answer: B [Iltutmish] 38. Who among the following built the mosque Quwal-ul-lslam at Delhi? [A] Alauddin Khilji [B] Iltutmish [C] Qutubuddin Aiabak [D] Babur Correct Answer: C [Qutubuddin Aiabak] 166 | P a g e

shop.ssbcrack.com

MCQs

INDIAN HISTORY

MCQs

39. Which among the following rulers is related to ” market regulatory measures ” in medieval India? [A] Alauddin Khilji [B] Balban [C] Babur [D] Humayun Correct Answer: A [Alauddin Khilji] 40. Who among the following conspired against Ibrahim Lodi and invited Babur to invade Delhi? [A] Bahlol Lodi [B] Daulat Khan Lodi [C] Sikandar Lodi [D] None of the above Correct Answer: B [Daulat Khan Lodi] 41. How many years Humayun spent in exile after he lost his kingdom in Hindustan? [A] 10 years [B] 12 years [C] 15 years [D] 20 years Correct Answer: C [15 years] 42. Who among the following was the last ruler of Slave Dynasty? [A] Aram Shah [B] Kaiqubad [C] Khusrau [D] None of the above Correct Answer: B [Kaiqubad] 43. Who among the following Mughal rulers took measures against the practice of “Sati”? [A] Jahanagir [B] Akbar [C] Shahjahan [D] Humayun Correct Answer: B [Akbar] 44. who among the following succeded Ruknuddin Feroz ? [A] Balban [B] Razia [C] Muizzudin Bahram Shah [D] Iltumish Correct Answer: B [Razia] Explanation: He was brother of Razia and Son of Iltutumish who was assassinated within 6 months of getting the throne 45. The glorious works in which among the following can be best credited for calling the Shahjahan’s reign as ” Glorious Epoch ” in the history of Mughal Empire ? 167 | P a g e

shop.ssbcrack.com

MCQs

INDIAN HISTORY

MCQs

[A] Trade & Commerce [B] Architecture [C] Music [D] Dance Correct Answer: B [Architecture] 46. For which of the following Shershah Suri is not known? [A] Organization of the system of Civil Administration [B] Issuing first Rupiya which was in use till 20th century [C] Introducing the system of irrigation by canals [D] All are intiatives of shershah suri Correct Answer: C [Introducing the system of irrigation by canals] 47. The city of Azimabad , which was abandoned since 7th century AD was revived by Shershah Suri and thus he founded the modern ________? [A] Shergarh (Pakistan ) [B] Patna [C] Sasaram [D] Lothal Correct Answer: B [Patna] 48. Which among the following sentence is INCORRECT? [A] Sikandar Lodhi founded Agra [B] Harihar I founded the Vijaynagar Dynasty of South [C] Balban completed the construction of Qutub Minar [D] All are correct Correct Answer: C [Balban completed the construction of Qutub Minar] Explanation: Iltutmish completed the construction of Qutub Minar 49. Which among the following is NOT correctly matched? [A] Ali Gurshap – Alauddin Khilji [B] Nur-ud-din – Jahangir [C] padshah gazi – Aurangazeb [D] All are correct Correct Answer: C [padshah gazi – Aurangazeb] Explanation: padhash gazi was a title assumed by Jahangir, Ali Gurshap was the real name of Alauddin Khilji 50. Pakistan Resolution or Lahore Resolution was a formal political statement adopted by the Muslim League in which year ? [A] 1940 [B] 1942 [C] 1945 [D] 1947 Correct Answer: A [1940]

168 | P a g e

shop.ssbcrack.com

MCQs

INDIAN HISTORY

MCQs

51. Which among the following was a gold coin issued in vijayanagar empire of south India ? [A] Tara [B] Varaha [C] Kakini [D] None of the above Correct Answer: B [Varaha] Explanation: Tara was a silver coin and Kakini was copper coin 52. Who among the following was the father of Harshavardhana? [A] Prabhakarvardhana [B] Rajyvardhana [C] Grahavarman [D] Ashoka Correct Answer: A [Prabhakarvardhana] 53. Kalachuri Empire is related to which of the following modern area? [A] Karnataka [B] Madhya Pradesh [C] Andhra Pradesh [D] Tamil Nadu Correct Answer: B [Madhya Pradesh] 54. Dantidurga also known as Dantivarman was the founder of which of the following dynasty ? [A] Rasthrakutas [B] Pallavas [C] Cholas [D] Cheras Correct Answer: A [Rasthrakutas] Explanation: In the mid of the 8th century, Dantidurga , who was one of the chieftain of ancient Rastrakuta family fought his way to the front and overthrew Kirtivarman II, thus bringing an end to the main branch of Chalukyas. This was the beginning of the Rastrakuta Empire in Deccan. 55. When Nader Shah invaded India, who among the following was emperor at Delhi ? [A] Muhammad Shah [B] Alamgir I [C] Alamgir II [D] Farrukhsiyar Correct Answer: A [Muhammad Shah] 56. The buildings in Fatehpur sikri are built by using mostly which of the following ? [A] Fired bricks [B] red stone [C] Marble [D] sand stone Correct Answer: B [red stone]

169 | P a g e

shop.ssbcrack.com

MCQs

INDIAN HISTORY

MCQs

57. Hindi poet Jagannath Pandit Raj was the most favorite poet of which of the following Emperors? [A] Jahangir [B] Shahjahan [C] Akbar [D] Humayun Correct Answer: B [Shahjahan] 58. Vasucharita is a work of which of the following ? [A] Ram raja [B] Krishndev rai [C] Vitthalantha [D] Kalidasa Correct Answer: A [Ram raja] Explanation: Ramraja or Ramarajabhushanudu who was a Telugu poet and a notable musician was one of the Astadiggajas (a collective title for Telugu poets in the court of Krishnadevaraya of Vijayanagara Empire). 59. The following Chalukya king performed an ashwamedha yajna? [A] Pulkesin I [B] Pulkesin II [C] Pulkesin III [D] Vikramaditya I Correct Answer: A [Pulkesin I] 60. Mir Bakshi was the officer in Akbar’s reign with which of the following responsibilities? [A] Revenue Collection [B] Payment of salaries [C] Head of the military administration [D] Head of Nobility Correct Answer: C [Head of the military administration] 61. Mokhadaji Gohil, a Gohil Rajput ruler of Ghogha, near Bhavnagar was a contemporary of which of the following Sultans? [A] Ghiyas ud din Tughluq Shah I [B] Muhammad bin Tughluq [C] Qutubudin Aibak [D] Illtumish Correct Answer: B [Muhammad bin Tughluq] 62. Who among the following is called Zinda Peer? [A] Babur [B] Akbar [C] Aurangzeb [D] Humayun Correct Answer: C [Aurangzeb]

170 | P a g e

shop.ssbcrack.com

MCQs

INDIAN HISTORY

MCQs

63. Who among the following scholars of Akbar’s Court translated Atharva veda into Persian? [A] Haji Ibrahim Sarhindi [B] Abdul Qadir Badayuni [C] Naqib khan [D] None of the above Correct Answer: A [Haji Ibrahim Sarhindi] 64. Who among the following written Manmaul Bahrain? [A] Aurangzeb [B] Akbar [C] Darashikoh [D] Abu Fazal Correct Answer: C [Darashikoh] 65. In which year , after defeating Jaypala, Mahmud of Ghazni proceeded to Gujarat where he destroyed the temple of Somnath? [A] 900 A.D [B] 935 A.D [C] 1025 A.D [D] 810 A.D Correct Answer: C [1025 A.D] 66. Ramanuja taught in which of the following temples? [A] Madurai [B] Mamallapuram [C] Srirangam [D] Tirumala Correct Answer: C [Srirangam] 67. Who among the following led a campaign against Warangal in 1309? [A] Ala-ud-din Khilji [B] Malik Kafur [C] Bakhtiar Khilji, [D] Mohmmad bin Tuglaq Correct Answer: B [Malik Kafur] 68. Deogiri was first of all subjugated by which Muslim invader? [A] Ala-ud-din Khilji [B] Jalal-ud-din Firoz [C] Malik Kafur [D] Qutubudin Aibak Correct Answer: A [Ala-ud-din Khilji] Explanation: The first attack on the south of the Narbada was made in 1294 by Ala-ud-din, who marched 700 miles into Berar and Khandesh, and compelled Raja Ramachandra Deva, the Yadava 69. In which year Akbar built the Buland Darwaja ? [A] 1590 [B] 1600 171 | P a g e

shop.ssbcrack.com

MCQs

INDIAN HISTORY

MCQs

[C] 1601 [D] 1605 Correct Answer: C [1601] Explanation: Buland Darwaja is located in Fatehpur Sikri built by Akbar in 1602 A.D to commemorate his conquest of Gujarat 70. During which of the following events Indians got the earliest contact with Islam? [A] Arab invasion of Sindh in 7th Century [B] Turkish Invasion in 11th century [C] Arab merchants at the Malabar Coast [D] None of the above Correct Answer: C [Arab merchants at the Malabar Coast] Explanation: Arab traders brought Islam to Malabar during the time of the Prophet Muhammad, this is the earliest evidence of Islam propagation in India 71. Who among the following translated Baburnama from chagtai to Persian [A] Abdul Rahim Khan-I-Khana [B] Abul Fazal [C] Khawaja Reazuddin Atash [D] Humayun Correct Answer: A [Abdul Rahim Khan-I-Khana] Explanation: Babur has written his biography i.e. Baburnama which is also known as Tuzk-e Babri. Babur and Jahangir are the only two emperors of Mughal Empire who wrote their own biographies. Baburnama is also known to be the first true autobiography in the Islamic literature. Baburnama was written in Chagatai Turkic, which was Babur’s mother tongue. It was translated into Persian by Abdul Rahim Khan-I-Khana. He was son of Bairam khan 72. “He can be attributed for the firm establishment of Mughal rule in India in the later part of 16th century”. Who was he? [A] Ali Quli Khan [B] Bairam Khan [C] Bairam Khan [D] Mir Jafar Correct Answer: C [Bairam Khan] Explanation: Bairam Khan, His most notable battle was at the Second Battle of Panipat. He was appointed as a guardian for Akbar. 73. In which of the following forts Jahanagir Mahal is located ? [A] Lahore Fort [B] Agra Fort [C] Red Fort [D] Golconda Fort Correct Answer: B [Agra Fort] Explanation: Jahangir Mahal is located In Agra fort , built by Akbar in Hindu Design and was used by the Rajput wives of Akba

172 | P a g e

shop.ssbcrack.com

MCQs

INDIAN HISTORY

MCQs

74. Dwarasamudra was the capital of which of the following dynasties? [A] Hoysala [B] Satavahana [C] Pandyas [D] Cholas Correct Answer: A [Hoysala] 75. 1526 AD is significant in India’s History because of __ [A] Foundation of Mughal empire in North India [B] Arrival of Ibn batuta [C] Arrival of Vasco Di Gama [D] Establishment of East India Company Correct Answer: A [Foundation of Mughal empire in North India] 76. Manyakheta (modern Malkhed) on the banks of Kagina River in Gulbarga district, Karnataka state was the capital of ______? [A] Satvahanas [B] Rastrakutas [C] Hoysalas [D] Kakatiyas Correct Answer: B [Rastrakutas] 77. Who among the following died in consequence of falling from his horse playing chaugan? [A] Humayun [B] Qutub Ud Din Aibak [C] Mohd Ghori [D] Alludin Khilji Correct Answer: B [Qutub Ud Din Aibak] 78. Razia Sultan acceded to the throne of Delhi in __________ ? [A] 1220 A.D [B] 1226 A.D [C] 1236 A.D [D] 1235 A.D Correct Answer: C [1236 A.D] 79. Maritime power was a distinct character of which of the following dynasties? [A] Chalukyas [B] Cholas [C] Pandyas [D] Rasthrakutas Correct Answer: D [Rasthrakutas] 80. Which among the following religion is related to Dilwada temples at Mount Abu ? [A] Vaishnav sect of Hindus [B] Shaiv Sect of Hindus [C] Jainism [D] Buddhism Correct Answer: C [Jainism] 173 | P a g e

shop.ssbcrack.com

MCQs

INDIAN HISTORY

MCQs

81. Vaikuntaperumal Temple at kanchipuram was built by __________? [A] Narsimhavarman II [B] Nandivarman II [C] Parmesvaravarman II [D] Parmesvaravarman I Correct Answer: B [Nandivarman II] Explanation: Vaikunta Perumal Temple was built by Pallava King Nandivarman-II in the 8th century, he was a worshipper of Lord Vishnu. 82. Who built the tomb of itmad ud daula? [A] Jahanagir [B] Nur Jahan [C] Shershah [D] Akbar Correct Answer: B [Nur Jahan] 83. Amoghavarsha I was a famous king of _________? [A] Rashtrakuta Dynasty [B] Pratihara Dynasty [C] Chalukya Dynasty [D] Solanki Dynasty Correct Answer: A [Rashtrakuta Dynasty] Explanation: The reign of Amoghavarsha is from 800 to 878 AD (though some scholars says that his reign was 62 years), one of the longest reigns. He was the greatest ruler of the Rastrakuta Empire. He indulged in constant wars with the Eastern Chalukyas of Vengi and remained successful. Amoghavarsha was a patron of the Jains and liberally patronized the Digambara sect of Jains. His contribution led to rapid progress made by the Digambara Jain sect in the 9th and 10 th century under Jinasena and Gunabhadra. (As evident from Mahapurana). Both of these Digambar Jain monks enjoyed favor of more than one monarchs and much responsible to eclipse Buddhism. 84. Krishna I , who was a king of Rashtrakuta dynasty is related to which of the following ? [A] Shiva Temples of Ellora [B] Ajanata Caves [C] Khajuraho Temples [D] Ellora caves Correct Answer: A [Shiva Temples of Ellora] Explanation: Krishna I am known for completing the establishment of Rastrakuta supremacy over the dominions of Chalukyas. Last Badami Chalukya Kirtivarman II though defeated by Dantidurga, but remained in power till 757 AD when he was destroyed by Krishna I. This is mentioned in the copper plate grant of Govinda III. Krishna I is best known for execution of the most marvelous architectural work in India i.e. Kailas Temple at Ellora. Kailas Temple is the most extensive and most opulent example of rock cut architecture in India. 85. Who has written the compilation of poems ,Prithviraj Raso ? [A] Prithvi Raj III [B] Chand Bardai [C] Purandara Dasa [D] Kabir Correct Answer: B [Chand Bardai] 174 | P a g e

shop.ssbcrack.com

MCQs

INDIAN HISTORY

MCQs

86. Which among the following place is the birth place of Humayun? [A] Agra [B] Kabul [C] Badakhashan [D] Sasaram Correct Answer: B [Kabul] 87. Out of Capital of Akbar’s Kingdom, Buland Darwaja , Panch Mahal and Akbar’s Mausoleum , which are located at the same place? [A] Panch Mahal and Akbar’s Mausoleum [B] Buland Darwaja , Panch Mahal and Akbar’s Mausoleum [C] Capital of Akbar’s Kingdom, Buland Darwaja , Panch Mahal [D] Capital of Akbar’s Kingdom and Akbar’s Mausoleum Correct Answer: C [Capital of Akbar’s Kingdom, Buland Darwaja , Panch Mahal] Explanation: Capital of Akbar Kingdom, Buland Darwaja , Panch Mahal (all fatehpur sikri ) mausoleum at Sikandara 88. Which among the following was a unit of Mughal Empire for the purpose of Administration and Revenue Collection? [A] Paraganas [B] Subahs [C] Parautis [D] Grama Correct Answer: B [Subahs] 89. Fort of Allahabad was built by ________? [A] Akbar [B] Shershah Suri [C] Humayun [D] Babur Correct Answer: A [Akbar] 90. The ryotwari system which was instituted in some parts of British India, was one of the two main systems used to collect revenues from the cultivators of agricultural land. It was originally introduced by________? [A] Akbar [B] Shershah Suri [C] Jahanagir [D] Humayun Correct Answer: B [Shershah Suri] 91. Shershah Suri introduced the coins made of _____? [A] Silver & Copper Only [B] Gold & Silver Only [C] Gold, Silver & Copper [D] Gold & Copper Correct Answer: C [Gold, Silver & Copper] 175 | P a g e

shop.ssbcrack.com

MCQs

INDIAN HISTORY

MCQs

92. Raja Todar Mal introduced standard weights and measures, a land survey and settlement system, revenue districts and officers during Akbar’s reign. In whose reign as a finance minister Raja Todarmal developed his expertise? [A] Humayun [B] Akbar [C] Shershah Suri [D] Babur Correct Answer: C [Shershah Suri] 93. Who was the first Indian Ruler who could bring Kabul and Kandhar under Indian Empire only after Kushanas? [A] Alauddin Khilji [B] Ibrahim Lodi [C] Babur [D] Akbar Correct Answer: C [Babur] 94. Which of the following battles helped Shershah to establish Suri Dynasty ? [A] Battle of Kanauj 1540 [B] Battle of Chausa 1539 [C] Battle of Duhariya 1532 [D] Battle of Chandewar Correct Answer: B [Battle of Chausa 1539] 95. Who among the following forbade the inscription of kalma on Coins? [A] Akbar [B] Shahjahan [C] Aurangzeb [D] Jahangir Correct Answer: C [Aurangzeb] 96. What was the main responsibility of Mir Bakshi during the Mughal Era? [A] All incomes and expenditures [B] Military department [C] Revenue department [D] Royal correspondence Correct Answer: B [Military department] 97. Ibaadat Khana which was constructed by Akbar at fatehpur Sikri was basically for the purpose of ______? [A] Discuss matters related to propagation of Islam [B] Discuss matters related to expansion of Din-i-Ilahi [C] Discuss matters related to all religions [D] Discuss matters related to Hinduism Correct Answer: C [Discuss matters related to all religions] Explanation: Abul Fazal encouraged Akbar for debates on doctrinal and philosophical enquiries. Akbar displayed a curiosity in these discussions. The debate took place in the Ibadat Khana or Hall of Worship. The Ibadat Khana is now recognized to be the Diwan-i-Khas, which was founded in 1574 at the City of 176 | P a g e shop.ssbcrack.com

MCQs

INDIAN HISTORY

MCQs

Fatehpur Sikri. It was opened for Sunni Muslims initially and was opened to all religions viz. Sufis, Shias, Christians, Zoroastrians, Hindus and Jains. 98. Moti Masjid was built in Red Fort by which of the following Mughal Emperor? [A] Shahjahan [B] Aurangazeb [C] Shah Aalam II [D] Akbar Correct Answer: B [Aurangazeb] 99. In the Indian History which period is called as “Golden Age of Architecture”? [A] Mughal Period [B] Sultanate Period [C] British period [D] Gupta period Correct Answer: D [Gupta period] 100. Hazi Ibrahim Sarhindi is known to be a Persian Writer during the reign of Akbar was translated the following Sanskrit work in Persian? [A] Ramayana [B] Mahabharata [C] Atharva Veda [D] Rigveda Correct Answer: C [Atharva Veda] Explanation: Akbar had a committee of scholars with cross lingual expertise. Some of them were Naqeeb Khan, Mulla Abdul Qadir Badayuni, Mulla Sheri and Sultan Haji Thanesari. • In that era Mahabharat was translated into Persian and was renamed as Razm Namah (Saga of the Battlefield). • Ramayana was translated by Mulla Abdul Qadir Badayuni. Badayuni also took up the task of Singhasan Batisi into Persian , which was later titled Khurd Afza in Persian. • Atharva Veda was translated by Haji Ibrahim Sirhindi. Rajataringini by Kalhana was translated by Maulana Shah Mohammad Shahabadi. Panchtantra were rendered into Persian by Mulla Hussain Waiz and it was called Anwar-i-Suhaili. • Panchatantra was also translated in Persian by Maulana Husain Faizi and its translated name was Yar-i-Danish. • Abul Fazal, Vazeer of Akbar and author of Akbarnama had translated Panchatantra in persian as Anvar-i-Saadat. 101. The Treaty of Purandar was signed on June 11, 1665 between Mughals and Maratha Chhatrapati Shivaji Maharaj. Who represented Mughals in this treaty ? [A] Jai Singh I [B] Man Singh I [C] Ram Singh II [D] Jai Singh II Correct Answer: A [Jai Singh I] 102. In the council of 8 Ministers of Shivaji which was called as Ashtapradhan , who was known as chief Minister and was responsible for General Administration? 177 | P a g e shop.ssbcrack.com

MCQs

INDIAN HISTORY

MCQs

[A] Peshwa [B] Amatya [C] Sachiv or Shuru Navis [D] Majumdar Correct Answer: A [Peshwa] 103. Humayun Nama was written by which of the following authors? [A] Humayun [B] Khwand Amir [C] Gulbadan Beghum [D] Abu Faza; Correct Answer: C [Gulbadan Beghum] 104. During the reign of which among the following Mughal rulers, maximum number of books on classical music were written? [A] Akbar [B] Jahanagir [C] Shahjahan [D] Aurangzeb Correct Answer: D [Aurangzeb] 105. During the reign of which among the following rulers, Malik Muhammad Jaisi completed his epic work ‘Padmavat’? [A] Shershah [B] Akbar [C] Jahangir [D] Shahjahan Correct Answer: A [Shershah] 106. Which among the following Mughal emperor had prohibited smoking use of tobacco in 1617 AD? [A] Akbar [B] Jahanagir [C] Shahjahan [D] Aurangzeb Correct Answer: B [Jahanagir] 107. Ibn Battuta, the famous Muslim explorer who came to India during regime of Mohammad Bin Tughlaq, belonged to which country? [A] Afghanistan [B] Morocco [C] Iraq [D] Iran orrect Answer: B [Morocco] Explanation: Ibn Battuta was a Muslim Moroccan explorer. He is known for his extensive travels, accounts of which were published in the Rihla.Battuta is considered one of the greatest travelers of all time. He came to India during the regime of Muhammad bin Tughlaq.

178 | P a g e

shop.ssbcrack.com

MCQs

INDIAN HISTORY

MCQs

108. Who among the following rulers had stamped the figure of Goddess Lakshmi on his coins and had his name inscribed in Nagari Characters? [A] Muhammad Ghazni [B] Muhammad Ghori [C] Muhammad Bin Tughlaq [D] Iltutmish Correct Answer: B [Muhammad Ghori] Explanation: Muhammad Ghori is known to have adopted the seated goddess Lakshmi type of the coins of Gahadavalas for circulation in the Gahadavala territories. He got stamped the figure of Goddess Lakshmi on his coins and had his name inscribed in Nagari Characters. 109. Who among the following defeated Muhammad Ghori near Mount Abu? [A] Bhimdev Solanki II [B] Vijaya Dev [C] Bhimdev Solanki I [D] None of these Correct Answer: A [Bhimdev Solanki II] Explanation: Muhammad of Ghor, Muhammad of Ghori was one of the rulers of the Ghurid dynasty from the famous house of Sur who were rulers of Ghor for five hundred years. He is cred with laying the foundation of Muslim domination in India that lasted for several centur 110. Which among the following rulers of the Chola Empire conquered the Maldives during his regime ? [A] Raja Raja Chola I [B] Rajendra Chola [C] Kulothunga Chola I [D] Vikrama Chola Correct Answer: A [Raja Raja Chola I] Explanation: Raja Raja Chola I, popularly known as Raja Raja the Great, is one of the greatest emperors of the Tamil Chola Empire of India who ruled between 985 and 1014 CE. By conquering several small kingdoms in South India, he expanded the Chola Empire as far as Sr 111. Faujdar and Amalguzar were the chief officials of the ‘Sarkars’ (administrative divisions) in Mughal Period. Amalguzar was related to which of the following operations? [A] Law & Order [B] Revenue [C] Defence [D] Audit and Accounts Correct Answer: B [Revenue] 112. Which among the following terms precisely defines the loans given to cultivators for seeds, farm implements etc. in the Sultanate and Mughal period? [A] Dams [B] Dahsala [C] Taqqavi [D] Nasaq Correct Answer: C [Taqqavi] 179 | P a g e

shop.ssbcrack.com

MCQs

INDIAN HISTORY

MCQs

113. Akbar had adopted the ‘Ray’ System of revenue assessment which he had abolished later on as part of his land revenue reforms. Who among the following had introduced the ray system? [A] Alauddin Khilji [B] Balban [C] Iltutmish [D] Sher Shah Suri Correct Answer: A [Alauddin Khilji] 114. In whose times the Mughal Empire reached its territorial climax? [A] Akbar [B] Shah Jahan [C] Aurangzeb [D] Bahadur Shah I Correct Answer: C [Aurangzeb] 115. Jujhar Singh, who was a Bundela Chief had revolted against: [A] Jahanagir [B] Shahjahan [C] Aurangzeb [D] Akbar Correct Answer: B [Shahjahan] 116. Babur, the founder of the mighty Mughal Empire traced his lineage from: [A] Timur from paternal line and Chengiz Khan from Maternal Line [B] Chengiz Khan from Paternal Line and Timur from Maternal line [C] Chagatai Turks from paternal and maternal side. [D] Mongoloids from paternal and Maternal side Correct Answer: A [Timur from paternal line and Chengiz Khan from Maternal Line] 117. “Sur Sagar “, a treatise on music was written during the reign of which of the following Mughal emperors? [A] Humayun [B] Akbar [C] Jahanagir [D] Shahjahan Correct Answer: B [Akbar] Explanation: Surdas wrote Sursagar, Tulsidas who wrote Ramcharitmanas was also a contemporary of Akbar. 118. Ustad Isa is related to the design and architecture of which of the following Buildings in Mughal Era? [A] Taj Mahal [B] Buland Darwaza [C] Red fort [D] Itimad-ud-daula’s tomb Correct Answer: A [Taj Mahal] Explanation: Ustad Isa was the chief architect of Taj Mahal. 180 | P a g e

shop.ssbcrack.com

MCQs

INDIAN HISTORY

MCQs

119. Which Mughal Emperor had put a ban on killing of animals in certain days? [A] Shahjahan [B] Akbar [C] Darashikoh [D] Jahangir Correct Answer: B [Akbar] Explanation: Akbar put a ban in 1583 on killing animals on certain days. 120. Hamim Humam who was the chief of Royal School (Pathshala) adorned the court of which Mughal Emperor? [A] Akbar [B] Jahanagir [C] Humayun [D] Shahjahan Correct Answer: A [Akbar] Explanation: Hamim Humam was one of the Nav Ratnas of Akbar. 121. ‘Padshah-nama’ (Chronicle of the Emperor) written by Abdul Hamid Lahori, is the official visual history giving a detailed account of the reign of which of the following Mughal Emperor? [A] Babur [B] Jahanagir [C] Shah Jahan [D] Aurangzeb Correct Answer: C [Shah Jahan] 122. Who among the following was the Sultan of Delhi, when the Mongols appeared for the first time in India’s soil? [A] Balban [B] Iltutmish [C] Jalal-ud Din Firoz [D] Razia Correct Answer: B [Iltutmish] 123. Which among the following dynasty is known to have constructed the temples at Khajuraho? [A] Chandelas [B] Chauhans [C] Pratiharas [D] Rastrakutas Correct Answer: A [Chandelas] Explanation: Chandela dynasty was well established and there was peace and prosperity in 10th and 11th century. Art and culture flourished there as the kings were great patrons of poetry and theatre. The culmination of their cultural achievements was at their capital city of Khajuraho (in present-day Madhya Pradesh), where, between the 10th and 12th centuries, one of the most splendid temple cities in the history of the world was created. There were originally 85 temples at Khajuraho, of which 25 remain today. The first king who started construction in Khajuraho was Harsha who built the 64 Yogini Temples. The most notable prince of this dynasty was King Dhangawhose time is known for building the most beautiful 181 | P a g e

shop.ssbcrack.com

MCQs

INDIAN HISTORY

MCQs

Khaujraho Temples of Parsvanath and Vishwavanath. His grandson Vidyadhara built the Kandariya Mahadev Temple. 124. Which among the following rulers of Delhi Sultanate declared himself as Sikandar-I-Sani? [A] Balban [B] Kaiqubad [C] Alauddin Khilji [D] Firoz Tughlaq Correct Answer: C [Alauddin Khilji] Explanation: Alauddin Khilji was a militarist and imperialist to the core. He was very ambitious. Alauddin, whose original name was Ali Gurshap, assumed the title Sikandar-i-Sani (Alexander the Second) and proclaimed Delhi as Dar-ul-Khilafa (Seat of the Caliphate). 125. Mahzarnama was declared by ____? [A] Babur [B] Humayun [C] Akbar [D] Shahjahan Correct Answer: C [Akbar] Explanation: In 1579, Mahzar Nama was declared by which the emperor declared that if there were conflictions in the views of the debaters, he was entitled to choose any of the interpretations. With the Mahzar Nama, Akbar pounced upon the dominance of the intolerant orthodox and allowed free development of a genuine religious spirit. Mahzar Nama was actually an idea of the father of Abul Fazal and Faizi , set that the authority of the King was higher than that of a Mujtahid (doctor of the faith) and if there is a variance, the emperor’s decision should be binding on the Muslims of India. With this edict, Akbar’s judgment was set above every legal and religious authority, so it was the promulgation of the doctrine of Imperial infallibility. 126. Who among the following are considered to be the first owners of Koh-i-noor diamond? [A] Kakatiyas [B] Khiljis [C] Tughlaqs [D] Mughals Correct Answer: A [Kakatiyas] Explanation: There are two versions of origin of this diamond. First version is that it was mined in 13th century in Kakatiya Territories {near Guntur in modern Andhra Pradesh}. It was under possessions of Kakatiya Rulers until Malik Kafur, the general of Alauddin Khilji raided Warangal and possibly acquired this gem. It remained with Khiljis and then passed on to several Delhi Sultanate dynasties until came into possession of Babur and successive Mughals. Another version says that it was discovered in 1650s in the diamond mines of Golkonda and was presented by Mir Jumla to Shah Jahan, who got it embedded into the peacock throne. 127. Who among the following was the writer of Tarikh-i-Firozshahi? [A] Ziauddin Barni [B] Ghiyasuddin Tughlaq [C] Abul Fazal [D] Firoz Tughlaq Correct Answer: A [Ziauddin Barni] 182 | P a g e

shop.ssbcrack.com

MCQs

INDIAN HISTORY

MCQs

Explanation: We know about the period of Muhammad Bin Tughlaq from the material of Ziauddin Barni. Barni’s main works are Twarikh-i-Firuzshahi and Fatwa-i-Jahandari. 128. In which year Arabs invaded Sindh? [A] 700 A.D [B] 710 A.D [C] 712 A.D [D] 715 A.D Correct Answer: C [712 A.D] Explanation: It was Raja Dahir who faced Arab invasion in 712 AD. He was attacked, defeated and killed by Mohammad Bin Qasim, a general of Umayyad Caliphate 129. Etti, Kavidi and Eradi are names of which among the following in sangam age? [A] taxes [B] Titles [C] Places [D] crops Correct Answer: B [Titles] 130. At which among the following places, Gokal Jat revolted against mughals in the 17th century? [A] Hisar [B] Mathura [C] Ujain [D] Bundelkhand Correct Answer: B [Mathura] Explanation: In 1669-70, the Jats of Mathura region rose under the leadership of a local Zamindar Gokul. Religion was main factor behind this revolt because Abdul Nabi, local officer of imperial Mughal government, had destroyed the temples of Hindus and disrespected their women. We note here that Jat uprising occurred at a time Mughal government was by no means weak or imbecile. The Mughal army pursued the Jats and the leader Gokul was captured and executed. However, the rebel did not subdue completely. Taking advantage of the civil war amongst the sons of Shahjahan, the Jats again rose in arms and this rebellion finally culminated in establishment of sovereign Jat Kingdom under Raja Surajmal with Bharatpur its capital 131. Rajasekhara, the eminent Sanskrit poet, dramatist and critic, who is best known for his work Kavyamimamsa was a court poet of which among the following kings of Gurjara Pratiharas? [A] Bhoj II [B] Mihirbhoja I [C] Mahendrapala I [D] Mahipala I Correct Answer: C [Mahendrapala I] 132. Sri Harsha was a court poet of which among the following Gahadwala Kings? [A] Govindchandra [B] Jayachandra [C] Chandradev [D] Virachandra Correct Answer: B [Jayachandra] 183 | P a g e

shop.ssbcrack.com

MCQs

INDIAN HISTORY

MCQs

133. In which among the following battles, Jaichandra was defeated and killed by Mohammad Ghori? [A] Battle of Kannauj [B] Battle of Chandawar [C] Battle of Tarain III [D] Battle of Talikota Correct Answer: B [Battle of Chandawar] 134. Who among the following was the founder of Gahadvala Dynasty? [A] Chandradev [B] Govindchandra [C] Madanpala [D] Jayachandra Correct Answer: A [Chandradev] Explanation: The Gahadvala, or Gaharwar, was a Hindu and Buddhist Indian dynasty that ruled the kingdom of Kannauj for approximately a hundred years, beginning in the late eleventh century. The founder of the Kannauj Gahadvala dynasty was Chandradeva, who took control of Kannauj at the end of the 11th century (1090A.D.). During the rule of his successor, Govindachandra, from 1114–1154, the state of Gahadvala reached the pinnacle of its power. His minister, Lakshmidhara, was the author of several legal works, including Kalpadruma. 135. Vikrampura and Vijaypuri were two capitals of which among the following Sen kings? [A] Ballal Sen [B] Vijay Sen [C] Samnta Sen [D] Hemanta Sen Correct Answer: B [Vijay Sen] 136. Who among the following was the first ruler of sen dynasty? [A] Ballal Sen [B] Hemant Sen [C] Vijay sen [D] Lakshaman Sen Correct Answer: B [Hemant Sen] Explanation: Around the time of Mahipala’s death in around 1043 AD, a vassal of the Palas named Hemanta Sen founded the Sena Dynasty. This Dynasty appeared in 11th century and disappeared in 12th century 137. In which among the following battles, artillery was used for the first time? [A] First Battle of Panipat [B] Third Battle of Tarain [C] First Battle of Khanwa [D] Second Battle of Panipat Correct Answer: A [First Battle of Panipat] 138. In which of the following years, Akbar had centralized the control and functioning of various mints? [A] 1570 [B] 1572 184 | P a g e

shop.ssbcrack.com

MCQs

INDIAN HISTORY

MCQs

[C] 1576 [D] 1578 Correct Answer: A [1570] 139. Ghazi Malik was the founder and first ruler of which among the following dynasties? [A] Khilji Dynasty [B] Mamluk Dynasty [C] Tughlaq Dynasty [D] Slave Dynasty Correct Answer: C [Tughlaq Dynasty] 140. In Sangam age, the term ” Pillai” was used for which among the following? [A] lenders [B] farmers [C] labours [D] students Correct Answer: D [students] 141. Who among the following was the founder of Rastrakuta dynasty ? [A] Dantidurga [B] Krishna -I [C] Indra – II [D] Govinda -I Correct Answer: A [Dantidurga] Explanation: In the mid of the 8th century, Dantidurga , who was one of the chieftain of ancient Rastrakuta family fought his way to the front and overthrew Kirtivarman II, thus bringing an end to the main branch of Chalukyas. This was the beginning of the Rastrakuta Empire in Deccan. 142. Which among the following Mughal painters is credited for painting Siberian Crane and the Dodo for the first time? [A] Abu al-Hasan [B] Bishandas [C] Farrukh Beg [D] Ustad Mansur Correct Answer: D [Ustad Mansur] Explanation: Ustad Mansur as a painter started his career during the last few years of Akbar`s reign as a minor painter. However, later he flourished during the time of Jahangir. Ustad Mansur was a leading painter in Emperor Jahangir’s court who was famous for depicting plants and animals. Most important paintings were of Siberian Crane and Dodo. The latter was an important source of reference for Zoologists till date. The Siberian Crane painted on paper is highly intricate showing the wrinkles on the bare skin, the legs and a small feather stuck to the claw. 143. Which among the following travelers has recorded his experiences in “Kitab-i-Rehla” ? [A] Amir Khusro [B] Ibn Battuta [C] Malik Murtaza [D] Al-Bayhaqi Correct Answer: B [Ibn Battuta] 185 | P a g e

shop.ssbcrack.com

MCQs

INDIAN HISTORY

MCQs

Explanation: Ibn batutta was an Arab traveler and adventure frm Morocoo. His book is a primary source of history of the region of Muhmmad bin Tughluq it throws ligh on the socio-political condition of his times. 144. which among the following Muslim writers has written the famous Hindi work “Padmavat”? [A] Malik Muhammad Jayasi [B] Abdul Rahim Khan-I-Khana [C] Amir Khusro [D] Bulleh Shah Correct Answer: A [Malik Muhammad Jayasi] Explanation: Padmavat of Malik Muhammad Jayasi is a poem about the beauty and Jauhar of Padmani. 145. During the reign of Krishnadev Raya which of the following famous temples were built in Vijaynagar empire? [A] Hampi [B] Hazura [C] Belgaum [D] None of them Correct Answer: B [Hazura] 146. Surdas was a disciple of which among the following? [A] Ramanujacharya [B] Ramananda [C] Vallabhacharya [D] Gyaneshwar Correct Answer: C [Vallabhacharya] 147. Who among the following composed Ananga? [A] Kabir [B] Gyaneshwar [C] Tukaram [D] Raidasa Correct Answer: B [Gyaneshwar] 148. Who among the following is considered to be the teacher of Amir Khusro? [A] Moinuddin Chishti [B] Baba Farid [C] Nizamuddin Auliya [D] Bakhtiyar Kaki Correct Answer: C [Nizamuddin Auliya] Explanation: Nizamuddin Auliya was a famous sufi saint. During rule of akbar, he had settled down in delhi. He was highly respected by the people. Beside his grave in Delhi, a big mosque has been built. 149. Who among the following laid the foundation of Shalimar & Nishaat Garden in Kashmir? [A] Akbar [B] Jahangir

186 | P a g e

shop.ssbcrack.com

MCQs

INDIAN HISTORY

MCQs

[C] Shahjahan [D] Nur Jahan Correct Answer: B [Jahangir] 150. Nadir Shah attacked India, at the time of which among the following later Mughals? [A] Jahandar Shah [B] Shah Alam [C] Akbar II [D] Mohammad Shah Correct Answer: D [Mohammad Shah] Explanation: Mohammad Shah Rangila was able to keep the throne for around 29 years partially because the first thing he did was to eliminate the Sayyid Brothers. During his time Nadir Shah attacked and looted Delhi and took the Peacock Throne with himself. The invasion of Nadir Shah fastened the disintegration of the Mughal Empire. During his time, the states of Hyderabad, Bengal, Awadh were established as independent Kingdoms. 151. Who among the following are known as King Makers in Indian History? [A] Sayyid Brothers [B] Hussain Brothers [C] Hassan Family [D] Tardi Begh Correct Answer: A [Sayyid Brothers] 152. After the death of Aurangzeb , which of the following Mughals abolished jazia tax for the first time? [A] Jahandar Shah [B] Rafi Ul Darjat [C] Mohammad Shah Rangila [D] Farrukhsiyar Correct Answer: A [Jahandar Shah] 153. “Muazzama” was the title of which of the following Mughal Emperors? [A] Shah Alam II [B] Mohammad Shah [C] Bahadur Shah Zafar [D] Jahandar Shah Correct Answer: C [Bahadur Shah Zafar] 154. Who among the following poets used to write the Urdu Ghazals with the pen name “Asad”? [A] Mir Taqi Mir [B] Dushyant [C] Mirza Ghalib [D] Amir Khusrow Correct Answer: C [Mirza Ghalib] Explanation: His original name was “Mirza Asadullah Baig Khan. Most of us know that “Ghalib” was his pen name. But before Ghalib, he wrote with a pen name “Asad“: which means a Lion. He was honored with title of Dabeer-ul-Mulk by Bahadur Shah Zafar II. When another title Mirza Nosha was given to him, he came to be known as Mirza Ghalib. 187 | P a g e

shop.ssbcrack.com

MCQs

INDIAN HISTORY

MCQs

155. Who among the following was the first to adopt the new religion propounded by Akbar called Din-iIlahi? [A] Abul Fazal [B] Faizi [C] Birbal [D] Raja Todarmal Correct Answer: C [Birbal] Explanation: Din-i-Illahi was an eclectic doctrine that contained elements from very diverse fields. It overthrew almost every ceremonial rule whether Islam or Muslim, but took the good ideas from the Brahmins as well as from the missionaries and adopted “Sun” as a symbol of the worship of the creator. He started a new Illahi era. The new cult was immediately professed by a small band of the courtiers of Akbar including Faizi, Abul Fazal, Birbal and a few others 156. Who among the following is said to be the pioneer of guerilla warfare in the Deccan region? [A] Shivaji [B] Shah ji Bhosle [C] Malik Ambar [D] Maloji Bhosale Correct Answer: C [Malik Ambar] 157. Which among the following empire was known as “Mulk-e-Khadim”? [A] Mysore Sultanate [B] Madurai Sultanate [C] Maratha Kingdom [D] Bijapur Kingdom Correct Answer: C [Maratha Kingdom] 158. Which among the following empire was known as “Mulk-e-Khadim”? [A] Mysore Sultanate [B] Madurai Sultanate [C] Maratha Kingdom [D] Bijapur Kingdom Correct Answer: C [Maratha Kingdom] 159. In which among the following territories, Marco Polo had travels in the last decade of 13th century in India? [A] Chola [B] Pandya [C] Madurai Sultanate [D] Cheras Correct Answer: B [Pandya] Explanation: Marco polo an Italian traveler landed in Pandya Empire (at Kayal) in 13th century and impressed by the wealth and magnificence of the King, Prince as well as people, tagged it as the richest kingdom in existence. 160. When was Guru Nanak Dev born? [A] 20 October 1469 [B] 20 October 1569 188 | P a g e

shop.ssbcrack.com

MCQs

INDIAN HISTORY

MCQs

[C] 20 October 11779 [D] 20 October 1669 Correct Answer: A [20 October 1469] Explanation: Guru Nanak Dev was born on 20th October 1469 and this day is celebrated as Prakash Diwas. 161. The Battle of Chandawar was fought in last decade of 12th century between which among the following rulers? [A] Mohammad Ghori & Prithviraj Chauhan [B] Mohammad Ghori and Jaichand [C] Jaichand and Prithviraj Chauhan [D] Prithviraj Chauhan and Bhimdev Solanki Correct Answer: B [Mohammad Ghori and Jaichand] 162. Which Mughal King sat on the throne with title of ” Padshah Gazi” ? [A] Jahangir [B] Shah Jahan [C] Aurangzeb [D] Mohammad Shah Correct Answer: A [Jahangir] 163. “Hasan Nizami ” and “Fakh-e-Mudabbir were the court poets of which ruler of Mamluk Sultanate? [A] Qutb-ud-din Aybak [B] Iltutmish [C] Balban [D] Razia Correct Answer: A [Qutb-ud-din Aybak] 164. In which year Mohd. Ghori attacked first in India on Multan? [A] 1172 A.D [B] 1175 A.D [C] 1185 A.D [D] 1190 A.D Correct Answer: B [1175 A.D] Explanation: Muhammad Ghori led a series of the campaigns in India. He began with Multan in 1175 against a Muslim ruler and got victorious. 165. On which of the following Ramanujacharya gave stress? [A] Moksha [B] Artha [C] Bhakti [D] Karma Correct Answer: C [Bhakti] Explanation: Rāmānuja presented the epistemic and soteriological importance of bhakti, or the devotion to a personal God (Vishnu in Rāmānuja’s case) as a means to spiritual liberation. His theories assert that there exists a plurality and distinction between Ātman (soul) and Brahman (metaphysical, ultimate reality), while he also affirmed that there is unity of all souls and that the individual soul has the potential to realize identity with the Brahman. 189 | P a g e

shop.ssbcrack.com

MCQs

INDIAN HISTORY

MCQs

166. During the reign of which among the following Mughal Emperors, Mughal Architecture reached its zenith? [A] Akbar [B] Jahangir [C] Shahjahan [D] Aurangazeb Correct Answer: C [Shahjahan] 167. In which year, Mohammad Bin Qasim invaded India? [A] 688 A.D [B] 712 A.D [C] 765 A.D [D] 812 A.D Correct Answer: B [712 A.D] 168. The famous Lingraj Temple is located in which among the following states? [A] West Bengal [B] Odisha [C] Gujarath [D] Assam Correct Answer: B [Odisha] 169. Bronze image of Nataraja Shiva is ascribed to which among the following dynasties? [A] Chola [B] Chalukya [C] Pallava [D] Chera Correct Answer: A [Chola] 170. “Amoghvarsha” belonged to which dynasty of kings? [A] Satavahana [B] Rastrakuta [C] Pallava [D] Hoyasala Correct Answer: B [Rastrakuta] 171. Elizabeth I was contemporary of which Mughal ruler? [A] Babur [B] Humayun [C] Akbar [D] Jahangir Correct Answer: C [Akbar] 172. Abdul Qadir Badauani”, Naqib Khan & Shaikh Sultan of Thaneshwar were the scholars in which among the following mughal emperors’ court ? [A] Akbar [B] Jahangir 190 | P a g e

shop.ssbcrack.com

MCQs

INDIAN HISTORY

MCQs

[C] Shahjahan [D] Aurangzeb Correct Answer: A [Akbar] 173. Who among the following has written “Kitab Ul Hind” ? [A] Al-Beruni [B] Hākim Nishapuri [C] Juwayni [D] Malik ibn Dinar Correct Answer: A [Al-Beruni] Explanation: Al Beruni was the First Muslim Scholar to study India and its Brahmanical tradition. He is called father of Indology and the first anthropologist. He is called one of the earliest and greatest polymath of the Islamic World. Alberuni in his book Kitab-ul-Hind appreciated a very high degree of proficiency of Indians in construction of Tanks and reservoirs at holy places. 174. Which Sultan had forbidden the Nobles to hold banquets & Festivities, and make marriage alliances without his permission? [A] Iltutmish [B] Alauddin Khilji [C] Balban [D] Firoz Shah Tughlaq Correct Answer: B [Alauddin Khilji] 175. “Alai Darwaja” is a gateway to which monument? [A] Qutub Minar [B] Jama Masjid [C] Humayun’s Tomb [D] Safdarjang’s Tomb Correct Answer: A [Qutub Minar] Explanation: Mamluk dynasty did not employ true Islamic architecture styles and used false domes and false arches. The first example of the true arch and true dome is in Alai Darwaja located in Qutub Complex. It was built by Ala-ud-din Khilji in 1311 AD. 176. In which among the following cities is located the famous monument Rumi Darwaja? [A] Bulandshahar [B] Bhopal [C] Lucknow [D] Murshidabad Correct Answer: C [Lucknow] Explanation: The Rumi Darwaza was built under the patronage of Nawab Asaf-Ud-daula in 1784 in Lucknow, Uttar Pradesh. It is an example of Awadhi architecture. 177. Which one of the following books is the official History of Revolt of 1857 published by publication division, Ministry of Information and broadcasting, Government of India? [A] Eighteen Fifty-Seven [B] Theories of Indian Mutiny 191 | P a g e

shop.ssbcrack.com

MCQs

INDIAN HISTORY

MCQs

[C] The Sepoy Mutiny and the Revolt of 1857 [D] None of the above Correct Answer: A [ Eighteen Fifty-Seven] Explanation: Eighteen Fifty Seven was written by well known Indian Historian Dr.Surendra Nath Sen ,with a foreward by Maulana Azad. Published on May 1957 by the publication division ,Ministry of Information and broadcasting ,Government of India. 178. Which of the following Mughal Emperors was known as Firdaus Manzil? [A] Mohammad Shah Rangila [B] Shah Alam II [C] Farrukhsiyar [D] Rafi ul Darjat Correct Answer: B [Shah Alam II] 179. What was the common name of Mehrunissa? [A] Jahan Ara [B] Nur Jahan [C] Hamida Bano [D] Gulbadan Bano Correct Answer: B [Nur Jahan] 180. The famous “Treaty of Purandhar” was signed between Shivaji and which among the following in 1665? [A] Jaswant Singh [B] Jai Singh [C] Shaista Khan [D] Aurangzeb Correct Answer: B [Jai Singh] Explanation: The treaty of Purandhar was signed on Jun 11th 1665 between Rajput ruler Jai Singh-I who was the commander of Mughal army and Maratha ruler Shivaji. 181. A manuscript titled “Sarab Gutaka” is ascribed to which among the following mystics? [A] Bhagat Pipa [B] Surdas [C] Bhagat Trilochan [D] Bhagat Namdev Correct Answer: A [Bhagat Pipa] 182. Kabir was a disciple of which among the following saints? [A] Ravidas [B] Ramananda [C] Jaydeva [D] Sukhanada Correct Answer: B [Ramananda] Explanation: Ramananda was first Bhakti saint and founder of Bhakti Movement of northern India. He preached in Hindi, the language of the masses. Kabir was a disciple of Ramananda. Ramananda 12 disciples included Anantananda, Sursurananda, Sukhanand, Naraharidāsa, Bhavanand, Bhagat Pipa, Kabir, 192 | P a g e

shop.ssbcrack.com

MCQs

INDIAN HISTORY

MCQs

Sen, Dhanna, Ravidas and two women disciples viz. Sursuri and Padyawati. Among them, Kabir was most radical and adopted non-conformist stand later on. 183. Akbarnama was written originally in which among the following languages? [A] Chagtai Turkic [B] Persian [C] Arabic [D] Urdu Correct Answer: B [Persian] Explanation: Abul Fazal was the author of Akbarnama which is written in Persian. It is the official history of Akbar’s reign in 3 volumes. This book gives the history of Akbar’s forefathers from Timur to Humayun and Akbar’s reign till 1602. 184. Who among the following were the last belligerents against whom Mohammad Ghori had led a campaign? [A] Khokhars [B] Gohils [C] Baghelas [D] Bhattis Correct Answer: A [Khokhars] 185. Which among the following Muslim Scholar came to India during the time of Mahmud of Ghazni? [A] Al-beruni [B] Amir Khusrau [C] Abu Zayd Balkhi [D] Muhammad al-Idrisi Correct Answer: A [Al-beruni] Explanation: Abu Rayhan Beruni or Alberonius (Latin) was a Persian Scholar and polymath of the 11th century. He accompanied Mahmud Ghazani in his conquests to South Asia. Al Beruni was the First Muslim Scholar to study India and its Brahmanical tradition. He is called father of Indology and the first anthropologist. He is called one of the earliest and greatest polymath of the Islamic World. 186. Aurangazeb was a good player of which among the following instruments? [A] Tabla [B] Veena [C] Mridang [D] Pakhawaj Correct Answer: B [Veena] 187. The 175 gms coin issued by Iltutmish was in which among the following metals? [A] Gold [B] Silver [C] Copper [D] Bronze Correct Answer: B [Silver] Explanation: Iltutmish introduced Silver Tanka and Copper Jital, the two coins of the Delhi Sultanate. The coins prior to Iltutmish were introduced by the invaders which bear the Sanskrit characters and even Bull and Shivalinga. Iltutmish was the first to introduce a “Pure Arabic Coin” in India. The Coins were 193 | P a g e shop.ssbcrack.com

MCQs

INDIAN HISTORY

MCQs

engraved with “The Mighty Sultan, Sun of the Empire and the Faith, Conquest-laden, Il-tutmish,” after he received an investiture of Sovereign Sultan of Delhi from the Caliph of Baghdad. The Silver Tanka issued by Iltutmish was weighing 175 grains. Gold Tanka of the same weight was later issued by Balban. 188. The temples of Halebid and Belur were built by which among the following dynasties? [A] Kalabhras [B] Hoyasalas [C] Rastrakutas [D] Kadambas Correct Answer: B [Hoyasalas] 189. Which among the following is a correct order of the dynasties of Delhi Sultanate? [A] Tuglaq →Khilji →Slave→Lodhi [B] Slave→ Lodhi →Khilji →Tuglaq [C] Slave→ Khilji →Tuglaq →Lodhi [D] Slave→ Tuglaq →Khilji →Lodhi Correct Answer: C [Slave→ Khilji →Tuglaq →Lodhi] Explanation: Slave dynasty: 1206-90 Khalji dynasty: 1290-1320 Tuglaq dynasty: 1320 – 1414 Sayyid dynasty: 1414-1451 Lodi dynasty: 1451-1526 190. The year 1576, is famous for which among the following battles? [A] Battle of Talikota [B] Battle of Haldighati [C] Battle of Gagron [D] Battle of Khargaon Correct Answer: B [Battle of Haldighati] Explanation: Battle of Haldighati took place in 1576 between Rana of Mewar Maharana Pratap and Akbar forces led by Man Singh. It was a decisive victory for Akbar’s chieftain Man Singh. Abul Fazal called this war as “Battle of Khamnaur” 191. The infamous Third Battle of Panipat was fought between which among the following belligerents? [A] British and Marathas [B] Mughals and Marathas [C] Afghans and British [D] Marathas and Afghans Correct Answer: D [Marathas and Afghans] Explanation: Third Battle of Panipat was fought in 1761. This was the largest war of the 18th century. The Marathas were led by Sadashivrao Bhau. The battle followed a two months seige and skirmishes. The Marathas were badly defeated in the war suffering a life of around 60-70 thousand warriors. They lost their ablest commanders. Balaji Baji Rao could not absorb this shock and died soon afterwards. 192. Which among the following building built by Shah Jahan at Agra is similar in pattern of the Saint Basil’s Cathedral in Moscow? 194 | P a g e

shop.ssbcrack.com

MCQs

INDIAN HISTORY

MCQs

[A] Taj Mahal [B] Agra fort [C] Pearl Mosque [D] Shah Jahani Mahal Correct Answer: C [Pearl Mosque] Explanation: Pearl Mosque also known as Moti Masjid 193. Which among the following was the Capital of Bahamani Sultanate? [A] Halebidu [B] Gulbarga [C] Humpi [D] Badami Correct Answer: B [Gulbarga] Explanation: From 1347 to 1425 Gulbarga was the capital of Bahamani Sultanate In 1425, Ahmad Shah shifted capital from Gulbarga to Bidar 194. Which of the following Mughal Emperor’s is also known to be a very good player of Veena? [A] Akbar [B] Jahangir [C] Shah Jahan [D] Aurangzeb Correct Answer: D [Aurangzeb] 195. Which among the following was located on the banks of Tungabhadra River? [A] Vijayanagar Empire [B] Chalukyas of Vatapi [C] Chalukyas of Badami [D] Bahmani Kingdom Correct Answer: A [Vijayanagar Empire] Explanation: Vijayanagar Empire was established by Hari Hara-I and Bukka-I in the year 1336 on the banks of Tungabhadra River 196. Which among the following two cities were joined by “Sadak-i-Azam” or the precursor of the Grand Trunk Road built by Shershah Suri? [A] Agra & Calcutta [B] Delhi & Patna [C] Delhi & Sasaram [D] Agra & Sasaram Correct Answer: D [Agra & Sasaram] 197. Who among the following Kings of Rajasthan was a victim of Patricide? [A] Rana Sanga [B] Rana Kumbha [C] Rana Udai Singh [D] Rawal Ratan Singh Correct Answer: B [Rana Kumbha] Explanation: Patricide is the killing of one’s of father Rana Kumbha was killed by his son Udaysimha (Udai Singh I) 195 | P a g e shop.ssbcrack.com

MCQs

INDIAN HISTORY

MCQs

198. The Kirti Stambh of Chittorgarh was built to commemorate the victory of Rana Kumbha in which among the following battles? [A] Battle of Chittor [B] Battle of Sarangpur [C] Battle of Gagron [D] Battle of Gogunda Correct Answer: B [Battle of Sarangpur] 199. The temple city of Osian, near Jodhpur was built by which among the following Rajput Kings? [A] Pratiharas [B] Solankis [C] Chauhanas [D] Parmaras Correct Answer: A [Pratiharas] 200.Which among the following Sikh Gurus had been given the “Palace of Amritsar” by Mughal Emperor Akbar? [A] Guru Ram Das [B] Guru Arjun Das [C] Guru Angad [D] Guru Hargovind Correct Answer: A [Guru Ram Das] 201.Areas of which of the following current states did not come under control of Vijayanagar emperor Krishnadeva Raya? [A] Kerala [B] Tamil Nadu [C] Odisha [D] Maharashtra Correct Answer: D [ Maharashtra] Explanation: Vijayanagar emperor Krishnadeva Raya (1509-1529) was a statesman, administrator and patron of arts. He united almost south India by bringing areas of modern Andhra Pradesh, Karnataka, Tamil Nadu, parts of Kerala and Odisha under his control. 202. Which of the following rulers died in the Kalinjar Fort situated in the Baghelkhand region of central India? [A]Mahmud of Ghazni [B] Sher Shah Suri [C] Humayun [D] Jahangir Correct Answer: B [Sher Shah Suri] Explanation: Kalinjar Fort is situated in the Baghelkhand region of central India. It is the place where Sher Shah Suri met his death in 1545. 203. Which of the following rulers earned the title of Purvapaschima Samudradhishavara? [A] Samudragupta [B] Karikala Chola 196 | P a g e

shop.ssbcrack.com

MCQs

INDIAN HISTORY

MCQs

[C] Harihara I [D] Parantaka-I Correct Answer: C [Harihara I] Explanation: Harihara-I of Vijayanagara Kingdom gained control over most of the area south of the Tungabhadra River and earned the title of Purvapaschima Samudradhishavara (“master of the eastern and western seas”). 204. Kamal-ud-Din Abd-ur-Razzaq ibn Ishaq Samarqandi, a Persian Islamic Scholar, visited which Indian city during early 1440s? [A] Surat [B] Calicut [C] Chennai [D] Chinsurah Correct Answer: B [Calicut] Explanation: Calicut is a metropolitan city in the state of Kerala in southern India on the Malabar Coast. Kamal-ud-Din Abd-ur-Razzaq ibn Ishaq Samarqandi visited Calicut in early 1440s. He wrote a narrative of what he saw in Calicut which is valuable as information on Calicut’s society and culture. 205. Ruqaiya Sultan Begum was the wife of which Mughal Ruler? [A] Humayun [B] Jahangir [C] Akbar [D] Shah Jahan Correct Answer: C [Akbar] Explanation: Ruqaiya Sultan Begum was the first wife of Mughal Emperor Akbar. She was Empress consort of the Mughal Empire from 1557 to 1605. Ruqaiya was a first cousin of her husband, and was a Mughal princess by birth. Her father, Hindal Mirza, was the youngest brother of Akbar’s father Humayun. 206. Which Mughal Ruler had higher number of Hindus as Mansabdars than Muslims under his administration? [A] Akbar [B] Aurangzeb [C] Jahangir [D] Shah Jahan Correct Answer: B [Aurangzeb] Explanation: Aurangzeb had higher number of Hindus as Mansabdars than Muslims under his administration 207. Who among the following is associated with translation of Rajatarangini in Persian? [A] Dara Shikoh [B] Zain-ul-Abidin [C] Abdul Razzaq Jilani [D] Bande Nawaz Correct Answer: B [Zain-ul-Abidin] Explanation: Rajatarangini is a metrical legendary and historical chronicle of the north-western Indian subcontinent, particularly the kings of Kashmir. It was written in Sanskrit by Kashmiri historian Kalhana in the 12th century CE. It was translated into Persian by the orders of the later Muslim ruler Zain-ul-Abidin. 197 | P a g e shop.ssbcrack.com

MCQs

INDIAN HISTORY

MCQs

208. Who among the following introduced the Zabti system and Dahshala system? [A] Raja Todar Mal [B] Firuz Shah Tughlaq [C] Raja Man Singh I [D] Ibrahim Lodi Correct Answer: A [Raja Todar Mal] Explanation: Raja Todar Mal, a finance minister of Akbar, brought new system of revenue collection known as zabti system and dahshala system. 209. Which ruler sent two of his brothers Ulugh Khan and Nusrat Khan to invade Gujarat? [A] Alauddin Khilji [B] Sikandar Lodi [C] Muhammad Shah [D] Firuz Shah Tughlaq Correct Answer: A [Alauddin Khilji ] Explanation: Alauddin Khilji sent two of his general brothers Ulugh Khan and Nusrat Khan to invade Gujarat. Gujarat was invaded and the temples of Somnath and Rudra Mahalaya were sacked. 210. The famous Meenakshi Amman Temple in Madurai was built during which dynasty? [A] Pandyas [B] Chola [C] Chera [D] Pallava Correct Answer: A [Pandyas] Explanation: Meenakshi Amman Temple in Madurai was built during the reign of the Pandyas. Rock cut and structural temples are significant part of pandyan architecture. Vimana, mandapa and shikhara are some of the features of the early Pandyan temples. 211. In 1293 A.D., Marco Polo, an Italian traveller, visited which Indian Kingdom? [A] Kakatiya [B] Satavahana [C] Pandyan [D] Chera Correct Answer: A [Kakatiya ] Explanation: In 1293 A.D., Marco Polo, an Italian traveller visited Motupalli, a famous sea port during Kakatiya Dynasty. He wrote about the prosperity and power of the kingdom under Rudramadevi. 212. Which of the following statements is incorrect about the Lingayatism? [A] It was founded by the 12th-century philosopher and statesman Basava [B] It is a distinct Shaivite religious tradition in India [C] Lingayat thinkers rejected the custodial hold of Brahmins over the Vedas and the shastras [D] Contemporary Lingayatism is influential in North India, especially in the state of Jammu & Kashmir Correct Answer: D [Contemporary Lingayatism is influential in North India, especially in the state of Jammu & Kashmir] Explanation: Fourth statement is incorrect because Contemporary Lingayatism is influential in South India, especially in the state of Karnataka.

198 | P a g e

shop.ssbcrack.com

MCQs

INDIAN HISTORY

MCQs

213. “Silver Tanka” and “Copper Jital” coins were introduced by: [A] Iltutmish [B] Abu Bakr Shah [C] Firuz Shah Tughlaq [D] Sikander Lodi Correct Answer: A [Iltutmish] Explanation: Iltutmish introduced Silver Tanka and Copper Jital , the two coins of the Delhi Sultanate. The coins prior to Iltutmish were introduced by the invaders which bear the Sanskrit characters and even Bull and Shivalinga. Iltutmish was the first to introduce a “Pure Arabic Coin” in India. The Coins were engraved with “The Mighty Sultan, Sun of the Empire and the Faith, Conquest-laden, Il-tutmish,” after he received an investiture of Sovereign Sultan of Delhi from the Caliph of Baghdad. The Silver Tanka issued by Iltutmish was weighing 175 grains. Gold Tanka of the same weight was later issued by Balban. 214. In 1398, when Timur invaded India, the ruler of Delhi Sultanate was: [A]Mahmud Shah Tughluq [B] Jalal ud din Firuz Khilji [C] Aram Shah [D] Ghiyas-ud-din Balban Correct Answer: A [Mahmud Shah Tughluq] Explanation: Mahmud Shah Tughlaq was the last sultan of the Tughlaq dynasty to rule the Islamic Delhi Sultanate. During his reign in 1398, Amir Timur the Chagtai ruler invaded India. He carried away with him a large booty from Delhi and the surrounding area. Soon after the invasion, the Tughlaq dynasty came to an end. 215.Who among the following was appointed as the supreme authority in justice during the era of Mughal Emperors? [A]Qazi-ul-Quzat [B] Qazi-ul-Hazat [C] Qazi-Faiz-ul-Islam [D] None of these Correct Answer: A [Qazi-ul-Quzat] Explanation: Besides Mughal rulers, Qazi-ul-Quzat was the supreme authority in justice. Since it was difficult for the emperor to do-away with justice in all the cases, he appointed Qazi-ul-Quzat to do the justice in accordance with Muslim Law. 216.Which of the following rulers of India issued Mahzarnama to take all the religious matters into his own hands ? [A] Jahangir [B] Akbar [C] Aurangzeb [D] Shah Alam Correct Answer: B [Akbar] Explanation: Akbar declared or issued Mahzarnama to take all the religious matters into his own hands. This made him supreme in the religious matters. He issued Mahzarnama to curb the dominance of Ulema. It was written by Faizi in 1579.

199 | P a g e

shop.ssbcrack.com

MCQs

INDIAN HISTORY

MCQs

217.The ambassador of Emperor James I, who reached in the court of Jahangir in 1615 was __: [A] Sir James Hay [B] Sir Thomas Roe [C] Sir Thomas Howard [D] Sir John Digby Correct Answer: B [Sir Thomas Roe] Explanation: As an ambassador of Emperor James l, Sir Thomas Roe reached in the court of Mughal Emperor Jehandri at Agra in 1615. Jehangir presented him the Mansab of 400. 218.Who among the following was the contemporary leader of Chengiz Khan ? [A] Genghis Khan [B] Mirza Muhammad Haidar Dughlat Beg [C] Jalaluddin Surkh-Posh Bukhari [D] Iitutmish Correct Answer: D [Iitutmish] Explanation: Iitutmish , the Sultan of Delhi, was contemporary of Mongol leader Chengiz Khan. In 1221 A.D., there was a danger of expected attack of Chengiz khan on Delhi. 219.The real name of Mughal Emperor Shah Alam II was__: [A] Alamgir II [B] Ahmed Shah Durrani [C] Mirza Najaf Khan [D] Ali Gauhar Correct Answer: D [Ali Gauhar] Explanation: The real name of the Mughal emperor Shah Alam II (1759-1806) was Ali Gauhar. 220.Which among the following was the real name of Mughal Emperor Muhammad Shah ? [A] Khujasta Akhtar [B] Shahriyar Shah Bahadur [C] Nadir Shah [D] Raushan Akhtar Correct Answer: D [Raushan Akhtar] Explanation: Raushan Akhtar was the real name of Muhammad Shah. 221.Which among the following terms was used for the Royal cavalry of the Maratha Army System? [A] The Shiledars [B] The Bargirs [C] The Hazari [D] The Subedars Correct Answer: B [The Bargirs] Explanation: Bargir was the Royal cavalry of the Maratha army system. There were two kinds of cavalry viz. Bargirs and the Shiledars. Bargirs were provided horses from the state and thus, the horses were property of the royal household and were looked after by state officers. Shiledars used to keep their own horses.

200 | P a g e

shop.ssbcrack.com

MCQs

INDIAN HISTORY

MCQs

222.Who among the following built the Alai Darwaza? [A] Akbar [B] Jahangir [C] Allauddin Khilji [D] Humayun Correct Answer: C [Allauddin Khilji] Explanation: Alai Darwaza was built by Allauddin Khilji. 223. Who among the following was the founder of Bijapur state ? [A] Yusuf Adil Shah [B] Murad II [C] Mehmood II [D] Ibrahim Zubayri Correct Answer: A [Yusuf Adil Shah] Explanation: Yusuf Adil Shah was the founder of Bijapur state. 224.Which of the following Mughal rulers banned engraving Kalma on coins? [A] Jahangir [B] Aurangzeb [C] Shahjahan [D] Muhammad Shah Correct Answer: B [Aurangzeb] Explanation: Aurangzeb stopped engraving Kalma on coins. Forbade the Parsis to celebrate their festival Navaroz. Released an order to ban the music everywhere and arrest those who listen to the music. His drink was plain water and he used to sleep on ground, something that made him a Zinda Fakir. He ended the Mughal pomp of Jharokha Darshan, use of almanacs, the Mughal custom of weighing the emperor in gems -distributing the wealth to the poors, on coronation anniversaries. He reintroduced Jazia. In summary he did all that was never done by his great grandfather, grandfather and father. 225.Baburnama was written in which language? [A] Para-Mongolic [B] Chagatai Turkic [C] Hijazi Arabic [D] Bahrani Arabic Correct Answer: B [Chagatai Turkic] Explanation: Babur has written his biography i.e. Baburnama which is also known as Tuzk-e Babri. Babur and Jahangir are the only two emperors of Mughal Empire who wrote their own biographies. Baburnama 226.Which of the following rulers of Delhi Sultanate faced maximum number of Mongol Attacks? [A] Alauddin Khilji [B] Muhammad Tughlaq [C] Firozshah Tughlaq [D] Bahlol Lodi Correct Answer: A [Alauddin Khilji] Explanation: The maximum number of Mongol attacks were faced by Alauddin Khilji. He built Siri his capital mainly to deter the Mongols. 201 | P a g e

shop.ssbcrack.com

MCQs

INDIAN HISTORY

MCQs

227.Who was the first ruler to issue Pure Arabic coin in India? [A] Mohammad Ghori [B] Qutubuddin Aibak [C] Iltutmish [D] Razia Sultan Correct Answer: C [Iltutmish] Explanation: Iltutmish introduced Silver Tanka and Copper Jital, the two coins of the Delhi Sultanate. The coins prior to Iltutmish were introduced by the invaders, which bear the Sanskrit characters and even Hindu Gods, Bull and Shivalinga. For example, Muhammad Ghor 228.Which of the following rulers of Delhi Sultanate abolished the Iqtas? [A] Alauddin Khilji [B] Muhammad Tughlaq [C] Firozshah Tughlaq [D] Balban Correct Answer: A [Alauddin Khilji] Explanation: Alauddin Khilji suddenly abolished the system of small Iqtas with a stroke of pen and brought them under the central Government, it is regarded as one of the most important agrarian reform of Alauddin Khilji. 229.In medieval India, Solanki queen Naikidevi defeated which of the following invaders in the Battle of Kayadara? [A] Mahmud Ghaznavi [B] Mohammad Ghori [C] Sabuktigin [D] Mohammad Bin Qasim Correct Answer: B [Mohammad Ghori] Explanation: The worst defeat of Mohammad Ghori in India was inflicted by the Solankis in the Battle of Kayadara near Mount Abu. Raja Bhimdev II was a young men and real regent was his mother Naikidevi. Naikidevi inflicted such a major defeat to Muhammad Ghori that this invasion became Muhammad’s first and last attack on India from the Gujarat side. He never turned to Gujarat later on. 230.Which of the following Sena rulers introduced Kulinism in Bengal? [A] Hemant Sen [B] Ballala Sen [C] Vijay Sen [D] Lakshman Sen Correct Answer: B [Ballala Sen] Explanation: One of the vassals of Pala called Hemant Sen founded the Sena Dynasty. The third king of this dynasty Ballala Sena (1160-1178) introduced the social reforms in Bengal known as Kulinism. 231.Which of the following term is used for writes of Charyapadas? [A] Yogachari [B] Mahasiddha [C] Sakyapa [D] Charyapurusha Correct Answer: B [Mahasiddha] 202 | P a g e

shop.ssbcrack.com

MCQs

INDIAN HISTORY

MCQs

Explanation: Charyapada is a collection of Buddhist poems composed most probably during the Pala Dynasty (However, there are various controversies on the origin). These mystic poems are from the tantric tradition. 232.Which of the following Pala Kings established Vikramshila university? [A] Gopala [B] Dharmapala [C] Devapala [D] Devapala 2 Correct Answer: B [Dharmapala] Explanation: Dharamapala, the second ruler of Pala dynasty was a pious Buddhist King and is best known for establishing the Vikramshila University. The Vikramshila University is located at Kahalgaon near Bhagalpur in Bihar. Dharampala had also built a Vihara at Somapu 233.Which of the following was capital of the Solanki dynasty in Gujarat? [A] Veraval [B] Anhilwada [C] Bharuch [D] Visnagar Correct Answer: B [Anhilwada] Explanation: The Solankis, whose headquarters were at Anhilwada were patrons of Somnath. Notable King was Mularaja, whose period is known as beginning of the Gujarati Culture, language and script. Raja Bhoj of Dhar overshadowed the successors of Mularaja and they were reduced to vassals of the Malwa Kingdom. 234.Which of the following battles is subject matter of Alha Khand, a 12th Century epic poetic work in Hindi? [A] Battle of Mandsaur [B] Battle of Chittor [C] Battle of Mahoba [D] Battles of Tarain Correct Answer: C [Battle of Mahoba] Explanation: Paradidev (Parmal) who fought Prithviraj Chauhan-III in 1182 AD in the Battle of Mahoba. This battle is the subject matter of the Alha Khand (c. 12th Century) is an early poetic work in Hindi which consists of a number of ballads describing the brave acts of two Rajput heroes, Alha and Udal. Alha and Udal were great fighters, still sung in the folk songs of Bundelkhand. 235.Which of the following regions was known as Jejakabhukti in early Medieval India? [A] Rohelkhand [B] Bundelkhand [C] Deccan [D] Saurastra Correct Answer: B [Bundelkhand] Explanation: Bundelkhand was known as Jejakabhukti, which roughly corresponds to the old Mahajanapadas of Chedi. The notable Kings among notable kings among the Chandels of Jejakabhukti were Dhanga, Ganda and Parmal. The legacy of the Chandelas of Jejakabhukti lies in the building of famous Khajuraho temples 203 | P a g e

shop.ssbcrack.com

MCQs

INDIAN HISTORY

MCQs

236.Who rebuilt the Somnath temple in 815 AD after it was destroyed by Arab Invaders in 725 AD? [A] Vikramaditya II [B] Nagabhatta-I [C] Nagabhatta-II [D] Mihirbhoja Correct Answer: C [Nagabhatta-II] Explanation: Nagabhatta II is best known for rebuilding the Somnath Temple in 815 AD, which was destroyed by Arab invaders in 725AD. This was a large structure of Red Sandstone; again destroyed in 1024AD by Mahamud of Ghazni. 237.The Rajput confederacy had checked invasion of which of the following Arab invaders in Battle of Rajasthan? [A] Mahmud Ghaznavi [B] Al Junayd [C] Muhammad bin Qasim [D] Al-Hakam Correct Answer: B [Al Junayd] Explanation: A Rajput confederacy of Gurjara Pratihara King Nagabhatta-I and his feudatories such as Chauhans and Guhilots was able to check and revert back Arab Invader Al Junayd, in the Battle of Rajasthan. 238.The temple city of Osean in Rajasthan was established by which of the following Rajputs? [A] Gurjar Pratiharas [B] Chauhans [C] Solankis [D] Sisodias Correct Answer: A [Gurjar Pratiharas] Explanation: Gurjar Pratiharas established Marwar in Rajasthan and built the Temple city of Osean, near Jodhpur. Their origin is from Mandsaur and notable Kings are Nagabhatta-I, Nagabhatta-II and Mihirbhoja. 239.Which of the following Rajput clan does not belong to the Agnikula? [A] Parmara [B] Pratihara [C] Chalukya [D] Kachwaha Correct Answer: D [Kachwaha] Explanation: The four Rajput clans from Agnikunda are Chauhans, Chalukyas, Parmaras and Pratiharas. 240.Chachnama is the oldest chronicle of which of the following regions? [A] Kashmir [B] Gandhar [C] Sindh [D] Punjab Correct Answer: C [Sindh] Explanation: At the time of Arab Invasions, Sindh was under Raja Dahir who reigned till 712 AD from his capital Brahamanabad. He was the last Hindu Ruler of Sindh and parts of Modern Punjab (Pakistan). The

204 | P a g e

shop.ssbcrack.com

MCQs

INDIAN HISTORY

MCQs

Arab conquest of Sindh is mentioned in the oldest Chronicle of Sindh called “Chach Nama”. He was attacked, defeated and killed by Mohammad Bin Qasim, a general of the Umayyad Caliphate. 241.Who among the following was appointed as the supreme authority in justice during the era of Mughal Emperors? [A] Qazi-ul-Quzat [B] Qazi-ul-Hazat [C] Qazi-Faiz-ul-Islam [D] None of these Correct Answer: A [Qazi-ul-Quzat] Explanation: Besides Mughal rulers, Qazi-ul-Quzat was the supreme authority in justice. Since it was difficult for the emperor to do-away with justice in all the cases, he appointed Qazi-ul-Quzat to do the justice in accordance with Muslim Law. 242.Which of the following rulers of India issued Mahzarnama to take all the religious matters into his own hands ? [A] Jahangir [B] Akbar [C] Aurangzeb [D] Shah Alam Correct Answer: B [Akbar] Explanation: Akbar declared or issued Mahzarnama to take all the religious matters into his own hands. This made him supreme in the religious matters. He issued Mahzarnama to curb the dominance of Ulema. It was written by Faizi in 1579. 243.The ambassador of Emperor James I, who reached in the court of Jahangir in 1615 was __: [A] Sir James Hay [B] Sir Thomas Roe [C] Sir Thomas Howard [D] Sir John Digby Correct Answer: B [Sir Thomas Roe] Explanation: As an ambassador of Emperor James l, Sir Thomas Roe reached in the court of Mughal Emperor Jehandri at Agra in 1615. Jehangir presented him the Mansab of 400. 244.Who among the following was the contemporary leader of Chengiz Khan ? [A] Genghis Khan [B] Mirza Muhammad Haidar Dughlat Beg [C] Jalaluddin Surkh-Posh Bukhari [D] Iitutmish Correct Answer: D [Iitutmish] Explanation: Iitutmish, the Sultan of Delhi, was contemporary of Mongol leader Chengiz Khan. In 1221 A.D., there was a danger of expected attack of Chengiz khan on Delhi. 245.The real name of Mughal Emperor Shah Alam II was__: [A] Alamgir II [B] Ahmed Shah Durrani 205 | P a g e

shop.ssbcrack.com

MCQs

INDIAN HISTORY

MCQs

[C] Mirza Najaf Khan [D] Ali Gauhar Correct Answer: D [Ali Gauhar] Explanation: The real name of the Mughal emperor Shah Alam II (1759-1806) was Ali Gauhar. 246.Which among the following was the real name of Mughal Emperor Muhammad Shah ? [A] Khujasta Akhtar [B] Shahriyar Shah Bahadur [C] Nadir Shah [D] Raushan Akhtar Correct Answer: D [Raushan Akhtar] Explanation: Raushan Akhtar was the real name of Muhammad Shah. 247.Which among the following terms was used for the Royal cavalry of the Maratha Army System? [A] The Shiledars [B] The Bargirs [C] The Hazari [D] The Subedars Correct Answer: B [The Bargirs] Explanation: Bargir was the Royal cavalry of the Maratha army system. There were two kinds of cavalry viz. Bargirs and the Shiledars. Bargirs were provided horses from the state and thus, the horses were property of the royal household and were looked after by state officers. Shiledars used to keep their own horses. 248.Who among the following built the Alai Darwaza? [A] Akbar [B] Jahangir [C] Allauddin Khilji [D] Humayun Correct Answer: C [Allauddin Khilji] Explanation: Alai Darwaza was built by Allauddin Khilji 249. Last Updated: May 11, 2017No comments? Who among the following was the founder of Bijapur state ? [A] Yusuf Adil Shah [B] Murad II [C] Mehmood II [D] Ibrahim Zubayri Correct Answer: A [Yusuf Adil Shah] Explanation: Yusuf Adil Shah was the founder of Bijapur state. 250.Which of the following Mughal rulers banned engraving Kalma on coins? [A] Jahangir [B] Aurangzeb [C] Shahjahan [D] Muhammad Shah Correct Answer: B [Aurangzeb] 206 | P a g e

shop.ssbcrack.com

MCQs

INDIAN HISTORY

MCQs

Explanation: Aurangzeb stopped engraving Kalma on coins. Forbade the Parsis to celebrate their festival Navaroz. Released an order to ban the music everywhere and arrest those who listen to the music. His drink was plain water and he used to sleep on ground, something that made him a Zinda Fakir. He ended the Mughal pomp of Jharokha Darshan, use of almanacs, the Mughal custom of weighing the emperor in gems -distributing the wealth to the poors, on coronation anniversaries. He reintroduced Jazia. In summary he did all that was never done by his great grandfather, grandfather and father. 251.Baburnama was written in which language? [A] Para-Mongolic [B] Chagatai Turkic [C] Hijazi Arabic [D] Bahrani Arabic Correct Answer: B [Chagatai Turkic] Explanation: Babur has written his biography i.e. Baburnama which is also known as Tuzk-e Babri. Babur and Jahangir are the only two emperors of Mughal Empire who wrote their own biographies. Baburnama is also known to be the first true autobiography in the Islamic literature. 252.Which of the following rulers of Delhi Sultanate faced maximum number of Mongol Attacks? [A] Alauddin Khilji [B] Muhammad Tughlaq [C] Firozshah Tughlaq [D] Bahlol Lodi Correct Answer: A [Alauddin Khilji] Explanation: The maximum number of Mongol attacks were faced by Alauddin Khilji. He built Siri his capital mainly to deter the Mongols. 253.Who was the first ruler to issue Pure Arabic coin in India? [A] Mohammad Ghori [B] Qutubuddin Aibak [C] Iltutmish [D] Razia Sultan Correct Answer: C [Iltutmish] Explanation: Iltutmish introduced Silver Tanka and Copper Jital, the two coins of the Delhi Sultanate. The coins prior to Iltutmish were introduced by the invaders, which bear the Sanskrit characters and even Hindu Gods, Bull and Shivalinga. For example, Muhammad Ghor. 254.Which of the following rulers of Delhi Sultanate abolished the Iqtas? [A] Alauddin Khilji [B] Muhammad Tughlaq [C] Firozshah Tughlaq [D] Balban Correct Answer: A [Alauddin Khilji] Explanation: Alauddin Khilji suddenly abolished the system of small Iqtas with a stroke of pen and brought them under the central Government, it is regarded as one of the most important agrarian reform of Alauddin Khilji. 255.In medieval India, Solanki queen Naikidevi defeated which of the following invaders in the Battle of Kayadara? 207 | P a g e shop.ssbcrack.com

MCQs

INDIAN HISTORY

MCQs

[A] Mahmud Ghaznavi [B] Mohammad Ghori [C] Sabuktigin [D] Mohammad Bin Qasim Correct Answer: B [Mohammad Ghori] Explanation: The worst defeat of Mohammad Ghori in India was inflicted by the Solankis in the Battle of Kayadara near Mount Abu. Raja Bhimdev II was a young men and real regent was his mother Naikidevi. Naikidevi inflicted such a major defeat to Muhammad Ghori that that this invasion became Muhammad’s first and last attack on India from the Gujarat side. He never turned to Gujarat later on. 256.Which of the following Sena rulers introduced Kulinism in Bengal? [A] Hemant Sen [B] Ballala Sen [C] Vijay Sen [D] Lakshman Sen Correct Answer: B [Ballala Sen] Explanation: One of the vassals of Pala called Hemant Sen founded the Sena Dynasty. The third king of this dynasty Ballala Sena (1160-1178) introduced the social reforms in Bengal known as Kulinism. 257.Which of the following term is used for writes of Charyapadas? [A] Yogachari [B] Mahasiddha [C] Sakyapa [D] Charyapurusha Correct Answer: B [Mahasiddha] Explanation: Charyapada is a collection of Buddhist poems composed most probably during the Pala Dynasty (However, there are various controversies on the origin). These mystic poems are from the tantric tradition. The writers of Charyapada are calledMahasiddhas and they were from Bengal, Bihar, Orissa and Assam. 258.Which of the following Pala Kings established Vikramshila university? [A] Gopala [B] Dharmapala [C] Devapala [D] Devapala 2 Correct Answer: B [Dharmapala] Explanation: Dharamapala, the second ruler of Pala dynasty was a pious Buddhist King and is best known for establishing the Vikramshila University. The Vikramshila University is located at Kahalgaon near Bhagalpur in Bihar. Dharampala had also built a Vihara at Somapuri, another at Paharpur and yet another Vihara at Odantapuri. The five places viz. Nalanda, Vikramshila, Somapuri, Paharpur and Odantapuri are called Five Mahaviharas. Bakhtiyar Khilji destroyed the Vikramshila University in 1200 AD. Somapura Mahavihara is now located in Bangladesh. It is one of the best known Mahaviharas of Buddhism in Indian subcontinent and is a UNESCO World Heritage Site. 259.Which of the following was capital of the Solanki dynasty in Gujarat? [A] Veraval [B] Anhilwada 208 | P a g e

shop.ssbcrack.com

MCQs

INDIAN HISTORY

MCQs

[C] Bharuch [D] Visnagar Correct Answer: B [Anhilwada] Explanation: The Solankis, whose headquarters were at Anhilwada were patrons of Somnath. Notable King was Mularaja, whose period is known as beginning of the Gujarati Culture, language and script. Raja Bhoj of Dhar overshadowed the successors of Mularaja and they were reduced to vassals of the Malwa Kingdom. 260.Which of the following battles is subject matter of Alha Khand, a 12th Century epic poetic work in Hindi? [A] Battle of Mandsaur [B] Battle of Chittor [C] Battle of Mahoba [D] Battles of Tarain Correct Answer: C [Battle of Mahoba] Explanation: Paradidev (Parmal) who fought Prithviraj Chauhan-III in 1182 AD in the Battle of Mahoba. This battle is the subject matter of the Alha Khand (c. 12th Century) is an early poetic work in Hindi which consists of a number of ballads describing the brave acts of two Rajput heroes, Alha and Udal. Alha and Udal were great fighters, still sung in the folk songs of Bundelkhand. 261.Which of the following regions was known as Jejakabhukti in early Medieval India? [A] Rohelkhand [B] Bundelkhand [C] Deccan [D] Saurastra Correct Answer: B [Bundelkhand] Explanation: Bundelkhand was known as Jejakabhukti, which roughly corresponds to the old Mahajanapadas of Chedi. The notable Kings among notable kings among the Chandels of Jejakabhukti were Dhanga, Ganda and Parmal. The legacy of the Chandelas of Jejakabhukti lies in the building of famous Khajuraho temples 262.Who rebuilt the Somnath temple in 815 AD after it was destroyed by Arab Invaders in 725 AD? [A] Vikramaditya II [B] Nagabhatta-I [C] Nagabhatta-II [D] Mihirbhoja Correct Answer: C [Nagabhatta-II] Explanation: Nagabhatta II is best known for rebuilding the Somnath Temple in 815 AD, which was destroyed by Arab invaders in 725AD. This was a large structure of Red Sandstone; again destroyed in 1024AD by Mahamud of Ghazni. 263.The Rajput confederacy had checked invasion of which of the following Arab invaders in Battle of Rajasthan? [A] Mahmud Ghaznavi [B] Al Junayd [C] Muhammad bin Qasim [D] Al-Hakam Correct Answer: B [Al Junayd] 209 | P a g e shop.ssbcrack.com

MCQs

INDIAN HISTORY

MCQs

Explanation: A Rajput confederacy of Gurjara Pratihara King Nagabhatta-I and his feudatories such as Chauhans and Guhilots was able to check and revert back Arab Invader Al Junayd, in the Battle of Rajasthan. 264.The temple city of Osean in Rajasthan was established by which of the following Rajputs? [A] Gurjar Pratiharas [B] Chauhans [C] Solankis [D] Sisodias Correct Answer: A [Gurjar Pratiharas] Explanation: Gurjar Pratiharas established Marwar in Rajasthan and built the Temple city of Osean, near Jodhpur. Their origin is from Mandsaur and notable Kings are Nagabhatta-I, Nagabhatta-II and Mihirbhoja. 265.Which of the following Rajput clan does not belong to the Agnikula? [A] Parmara [B] Pratihara [C] Chalukya [D] Kachwaha Correct Answer: D [Kachwaha] Explanation: The four Rajput clans from Agnikunda are Chauhans, Chalukyas, Parmaras and Pratiharas. 266.Chachnama is the oldest chronicle of which of the following regions? [A] Kashmir [B] Gandhar [C] Sindh [D] Punjab Correct Answer: C [Sindh] Explanation At the time of Arab Invasions, Sindh was under Raja Dahir who reigned till 712 AD from his capital Brahamanabad. He was the last Hindu Ruler of Sindh and parts of Modern Punjab (Pakistan). The Arab conquest of Sindh is mentioned in the oldest Chronicle of Sindh called “Chach Nama”. He was attacked, defeated and killed by Mohammad Bin Qasim, a general of the Umayyad Caliphate. 267.The Mansabdari system was borrowed from ? [A] Afghanistan [B] Turkey [C] Mongolia [D] Persia Correct Answer: C [Mongolia] Explanation: Mansabdar implies the generic term for the military-kind grading of all royal functionaries of the Mughal Empire. The Mansabdari system introduced by Akbar was borrowed from the system followed in Mongolia. 268.Which of the following ladies wrote an historical account during the Mughal period? [A] Gulbadan Begum [B] Noorjahan Begum [C] Jahanara Begum [D] Zebun-nissah Begum Correct Answer: A [Gulbadan Begum] 210 | P a g e shop.ssbcrack.com

MCQs

INDIAN HISTORY

MCQs

Explanation: Gulbadan Begum (1523-1603) was a Perso-Turkic Princess, the daughter of Emperor Babur. She is most known as the author of Humayun Nama, the account of the life of her brother, Humayun. 269.Who was the first Indian ruler to organize Haj pilgrimage at the expense of the state? [A] Alauddin Khilji [B] Feroz Tughlaq [C] Akbar [D] Aurangzeb Correct Answer: C [Akbar] Explanation: Akbar was the first Indian ruler to organize Haj pilgrimage at the expense of the state. 270.Which among the following was the single biggest item of import to the Vijayanagar empire? [A] Precious stones [B] Horses [C] Luxury goods [D] Raw Silk Correct Answer: B [Horses] Explanation: Kings of Vijayanagar, imported Arabian horses on a large scale, in order to improve the breed of cavalry horses in their own districts. Thus, the single biggest item of import to the Vijayanagar empire was Horses. 271.During the Delhi Sultanate, who among the following were called the Barids? [A] Craftsmen [B] Bodyguards of the Sultan [C] Officer-in-charge of state exchequer [D] The spy / news reporters Correct Answer: D [The spy / news reporters] Explanation: Barid-i-mumalik was the head of the information and intelligence department. Only a nobleman who enjoyed the fullest confidence of Sultan was appointed the chief barid. The Barid-iMumalik had to keep information of all that was happening in the Sultanate. At local level there were barids who used to send regular news concerning the matters of the state to the central office. Apart from barids, another set of reporters also existed who were known as Munihiyan. 272.The Sayyid dynasty of the Delhi Sultanate is called so because__? [A] Its founder and his successors adopted the title Sayyid [B] Its founder and his successors belonged to the Sayyid tribe of eastern Turkistan [C] Its founder and his successors were descendant of the prophet Muhammad [D] Its founder was a scholar of Islamic theology Correct Answer: C [Its founder and his successors were descendant of the prophet Muhammad] Explanation: Khizr Khan was the descendant of the prophet Muhammad .The Sayyid dynasty was the fourth dynasty of the Delhi Sultanate from 1414 to 1451 A.D. Khizr Khan was the founder ruler of Sayyid dynasty and he is said to be a descendant of Prophet Muhammad. 273.Who among the following Sultans of Delhi assumed the title Sikandar-i-Sani? [A] Balban [B] Alauddin Khalji [C] Muhammad-bin-Tughlaq [D] Sikandar Lodi Correct Answer: B [Alauddin Khalji] 211 | P a g e shop.ssbcrack.com

MCQs

INDIAN HISTORY

MCQs

Explanation: Alauddin Khilji was a militarist and imperialist to the core. He was very ambitious. Alauddin, whose original name was Ali Gurshap, assumed the title Sikandar-i-Sani (Alexander the Second) and proclaimed Delhi as Dar-ul-Khilafa (Seat of the Caliphate). 274.Which among the following was the work of Sultan Feroz Shah Tughlaq? [A] Fautuhat-i-Ferozshahi [B] Fatawa-i-Jahandari [C] Tarikh-i-Ferozshahi [D] Tughlaqnama Correct Answer: A [Fautuhat-i-Ferozshahi] Explanation: Fautuhat-i-Ferozshahi was authored by Sultan Feroz Shah Tughlaq himself. He was a Turkic Muslim ruler of the Tughlaq Dynasty, who reigned over the Sultanate of Delhi from 1351 to 1388 A.D. 275.The irrigation tax was charged on the farmers for the first time by which among the following Sultans? [A] Alauddin Khilji [B] Ghiyasuddin Tughlaq [C] Mohammad Tughlaq [D] Firoz Tughlaq Correct Answer: D [Firoz Tughlaq] ExplanationFiroz Shah Tughlaq was the cousin brother of Muhammad Bin Tughlaq, and became the ruler of Delhi on 23rd march 1351 A.D. He imposed only four taxes sanctioned by Islamic viz., kharaj (land tax), khams (1/5 of the looted property during wars), Jizya (religious tax on the Hindus), and Zakat (2½per cent of the income of the Muslims which was spent for the welfare of Muslim subjects and their religion). He imposed irrigation tax after getting it sanctioned from the Ulema. 276.Ibn Batuta was a traveler from which among the following current nations? [A] Algeria [B] Morocco [C] Tunisia [D] Libya Correct Answer: B [Morocco] Explanation: Ibn Batuta was a Moroccan and Berber explorer. He is known for his extensive travels, accounts of which were published in the Rihla. 277.During the invasion of Mohammed Bin Kasim, many Hindus in Sindh were forcibly converted to Islam. They were recoverted to Hinduism by the authority of __? [A] Devala Smriti [B] Narada Smriti [C] Visnu Smriti [D] Yajnavalkya Smriti Correct Answer: A [Devala Smriti] Explanation: During the invasion of Mohammed Bin Kasim in ad 712 many Hindus in Sindh were converted to Islam through questionable means. All of them were reconverted to Hinduism on the authority of Deval Smriti written by Deval Rishi. Deval smriti writes down rules for reconversion into Hindu fold of the Hindus forcibly converted into Islam if someone showed his or her desire to be reconverted to Hinduism with due atonement.

212 | P a g e

shop.ssbcrack.com

MCQs

INDIAN HISTORY

MCQs

278.Mohammad Ghori was first defeated by which of the following rulers? [A] Prithviraja Chauhan [B] Jayachandra [C] Vidyadhara Chandella [D] Bhima II Correct Answer: D [Bhima II] Explanation: He was defeated by Bhima II, the Solanki ruler of Gujarat in 1178. 279.Which among the following is a correct meaning of office of Qazi-ul-Quzat under the Mughals? [A] Head of the Finance department [B] Censor of Public morals [C] Supreme Authority in Justice [D] Incharge of charities Correct Answer: C [Supreme Authority in Justice] Explanation: Chief judge of the empire Qazi-ul-Quzat was the supreme authority in justice. Since it was difficult for the emperor to do-away with justice in all the cases, he appointed Qazi-ul-Quzat to do the justice in accordance with Muslim Law. 280.The Mahzarnama was promulgated by Emperor Akbar in__? [A] 1560 A.D. [B] 1572 A.D. [C] 1576 A.D. [D] 1579 A.D. Correct Answer: D [1579 A.D.] Explanation: Akbar issued Mahzarnama (or Infallibility Decree) to take all the religious matters into his own hands. This made him supreme in the religious matters. He issued Mahzarnama to curb the dominance of Ulema. It was written by Faizi in 1579 A.D. 281.The real name of Afzal Khan, the commander of Adil Shahi dynasty of Bijapur was __? [A] Abdullah Bhatari [B] Samsuddin [C] Sabar [D] Muhammad Khan Correct Answer: A [Abdullah Bhatari] Explanation: Afzal Khan was a medieval Indian commander who served the Adil Shahi dynasty of Bijapur, and fought against Shivaji. He was killed at a meeting with Shivaji and his army was defeated in the Battle of Pratapgad 1656. 282.The Portuguese built their first fortress in India at __? [A] Cochin [B] Goa [C] Anjidiv [D] Cannanore Correct Answer: A [Cochin] Explanation: The Portuguese build their first fortress in Cochin. 283.Which among the following Mughal Emperor was also known as Roshan Akhtar? 213 | P a g e

shop.ssbcrack.com

MCQs

INDIAN HISTORY

MCQs

[A] Ahmed Shah [B] Muhammad Shah [C] Jahandar Shah [D] Shah Alam Correct Answer: B [ Muhammad Shah] Explanation: Muhammad Shah Rangile also known as Roshan Akhtar, was the Mughal emperor between 1719 and 1748. During his time, Nadir Shah attacked and looted Delhi and took the Peacock Throne with himself. 284.The lowest in rank in the Maratha infantry was of__? [A] Nayak [B] Hawaldar [C] Zumladar [D] Hazari Correct Answer: B [Hawaldar] Explanation: Hawaldar was the lowest in rank in the Maratha Infantry. 285.Which among the following office was held by the superintendent of port under the Mughals ? [A] Mutsaddi [B] Mir-i-Bahar [C] Tahvildar [D] Mushrif Correct Answer: B [Mir-i-Bahar] Explanation: Administration under Akbar: The Mir-i-Bahar was incharge of customs and boats and ferry taxes, and port duties in coastal towns. 286.Who among the following was the Sultan at Delhi when the Vijayanagar empire was founded? [A] Firoz Tughlaq [B] Sikandar Lodi [C] Ghiyasuddin Tughlaq [D] Muhammad-bin-Tughlaq Correct Answer: D [Muhammad-bin-Tughlaq] Explanation: Vijayanagar was a South Indian Hindu empire, founded in 1336 by Harihara and Bukka during the region of Sultan of Delhi Muhammad-bin-Tughlaq. 287.Which of the following Rajput rulers is known to have donated for the reconstruction of a mosque? [A] Mihir Bhoja [B] Bhoja Parmar [C] Prithviraj III [D] Jai Singh Siddharaj Correct Answer: D [Jai Singh Siddharaj] Explanation: Jaya Singh Siddharaja , Chalukya King of Gujarat: He was the son of Kama and ascended the throne in 1094 A.D He gave shelter to many scholars of different religion and castes. He was a Rajput ruler who gave donation of one lac Baltoras (coins) for rebuilding a mosque in Cambay. 288.Which one of the following pairs is not correctly matched? [A] Baz Bahadur- Malwa [B] Sultan Muzaffar Shah-Gujarat 214 | P a g e

shop.ssbcrack.com

MCQs

INDIAN HISTORY

MCQs

[C] Yusuf Adil Shah-Ahmednagar [D] Qutub Shah -Golkunda Correct Answer: C [Yusuf Adil Shah-Ahmednagar] Explanation: Yusuf Adil Shah was the founder of the Adil Shahi dynasty that ruled the Sultanate of Bijapur. 289.From the times of which among the following Chola rulers, the Gangaikondacholapuram became the capital of the Chola empire? [A] Parantaka Chola I [B] Rajendra Chola I [C] Kulottunga Chola I [D] Vikrama Chola Correct Answer: B [Rajendra Chola I] Explanation: Gangaikondacholapuram was constructed by Rajendra Chola I to commemorate his conquest over the chalukyas and other feudatories Kalonga, Gangas, Palas etc .These victories led him to assume the title Gangaikonda. 290.Which of the following parts were situated on the west coast of South India ? 1. Kaveripattanam 2. Korkai 3. Musiri 4. Tondi Choose the correct answer from the codes given below. [A] 1, 2 [B] 2, 3 [C] 3, 4 [D] 2, 3, 4 Correct Answer: C [3, 4] Explanation: Musiri and Tondi were situated on the west coast of South India. 291.Ibn Battuta, the famous Muslim explorer who came to India during regime of Mohammad Bin Tughlaq, belonged to which country ? [A] Afghanistan [B] Morocco [C] Iraq [D] Iran Correct Answer: B [Morocco] Explanation: Ibn Battuta was a Muslim Moroccan explorer. He is known for his extensive travels, accounts of which were published in the Rihla.Battuta is considered one of the greatest travelers of all time. He came to India during the regime of Muhammad bin Tughlaq. 292.Who among the following rulers had stamped the figure of Goddess Lakshmi on his coins and had his name inscribed in Nagari Characters ? [A] Muhammad Ghazni [B] Muhammad Ghori [C] Muhammad Bin Tughlaq [D] Iltutmish Correct Answer: B [Muhammad Ghori] 215 | P a g e

shop.ssbcrack.com

MCQs

INDIAN HISTORY

MCQs

Explanation: Muhammad Ghori is known to have adopted the seated goddess Lakshmi type of the coins of Gahadavalas for circulation in the Gahadavala territories. He got stamped the figure of Goddess Lakshmi on his coins and had his name inscribed in Nagari Characters. 293.Who among the following defeated Muhammad Ghori near Mount Abu? [A] Bhimdev Solanki II [B] Vijaya Dev [C] Bhimdev Solanki I [D] None of these Correct Answer: A [Bhimdev Solanki II] Explanation: Muhammad of Ghor, Muhammad of Ghori was one of the rulers of the Ghurid dynasty from the famous house of Sur who were rulers of Ghor for five hundred years. He is cred with laying the foundation of Muslim domination in India that lasted for several centur 294.Which among the following rulers of the Chola Empire conquered the Maldives during his regime ? [A] Raja Raja Chola I [B] Rajendra Chola [C] Kulothunga Chola I [D] Vikrama Chola Correct Answer: A [Raja Raja Chola I] Explanation: Raja Raja Chola I, popularly known as Raja Raja the Great, is one of the greatest emperors of the Tamil Chola Empire of India who ruled between 985 and 1014 CE. 295.The death of Mughal Emperor Aurangzeb occurred at which among the following places? [A] Agra [B] Bijapur [C] Ahmednagar [D] Pune Correct Answer: C [ Ahmednagar ] Explanation: Aurangzeb had died in Ahmednagar on 20 February 1707 at the age of 88. Unlike other earlier Mughals, he was buried in a modest open-air grave in Khuldabad, which shows his deep devotion to his Islamic beliefs. 296. Ghazni was a small principality in [A] Mongolia [B] Turkey [C] Persia [D] Afghanistan Correct Answer: [D] Afghanistan 297. Who was the author of Kitab-ul-Hind? [A] Abu Said [B] Abul Fazl [C] Firdausi [D] AI-Beruni Correct Answer: [D] AI-Beruni 298. Who is known as the "slave of a slave"? [A] Muhammad bin Oasim 216 | P a g e

shop.ssbcrack.com

MCQs

INDIAN HISTORY

MCQs

[B] Mahmud of Ghazni [C] Ilitutmish [D] Outbuddin Aibak Correct Answer: [C] Ilitutmish 299. Who was the first Sultan of Delhi to issue regular currency and to declare Delhi as the capital of his empire? [A] Balban [B] Aram Shah [C] Nasiruddin Mahmud [D] IIitutmish Correct Answer: [D] IIitutmish 300. Who among the following came to India at the instance of Sultan Mahmud of Ghazni? [A] AI-Masudi [B] AI-Beruni [C] Sulaiman [D] Abdul Haq Correct Answer: [B] AI-Beruni

217 | P a g e

shop.ssbcrack.com

MCQs

INDIAN HISTORY

MCQs

MODERN INDIAN HISTORY A Brief Note on Modern Indian History During the late 16th and the 17th Centuries, the European trading companies in India competed with each other ferociously. By the last quarter of the 18th Century the English had outdone all others and established themselves as the dominant power in India. The British administered India for a period of about two centuries and brought about revolutionary changes in the social, political and the economic life of the country. Once the British set their foot solidly on Indian soil, they began the commercial exploitation of the natural resources of India. By the middle of the 19th Century arrogant exploitation of the people had tried the patience of the Indians to the limit. The British imperialism reached its zenith between the middle of the nineteenth century and the First World War. The exploitative policies of the British in India saw the birth of nationalist agitation against it. With increasing intrusion of aliens in their lives, a group of middle class Indians formed the Indian National Congress (1885). The anti-British struggle became truly a mass movement with the arrival of Mahatma Gandhi (1869 1948). It was followed by numerous movements against the British rule. With the passage of time and stubbornness of the Indians the British had come to realize that the day was not far off when they will have to quit India. Successive campaigns had the effect of driving the British out of India in 1947, but with independence came the independence of the country into Pakistan. Freedom Fighters India His brave deeds earned Vallabhbhai Patel the title of the iron man of India. For his role in the Bardoli Satyagraha, Patel came to be called the Sardar. Sardar Patel was a famous lawyer but gave up his practice in order to fight for the freedom of the country. After independence he became the deputy PM of India and played an important role the integration of India by merging numerous princely states with the Indian Union. Indian Independence The feeling of nationalism had started growing in the minds of Indians as early as the middle of the nineteenth century but it grew more with the formation of the Indian national Congress in 1885. Though the Congress started on a moderate platform but with the passage of time and apathetic attitude of the British government, the national movement began to shape well. Mahatma Gandhi Mahatma Gandhi was born as Mohandas Karamchand Gandhi on 2nd October 1869. He was the most popular as well as the most influential political and spiritual leaders of India. His contribution to the freedom struggle of India is priceless and the country owes its independence, partly, to this great man. The Satyagraha movement, which led to India's independence, was founded by Mahatma Gandhi only. European Advents • A Sea route to India was discovered by Vasco da Gama in 1497-1499 AD. • First arrived at Kappad near Kozhikode in Kerala. • The ship name was Saint Gabriel, Vasco-da-Gama landed at Kappad. 218 | P a g e

shop.ssbcrack.com

MCQs

INDIAN HISTORY

MCQs

• • •

He arrived for the second time in 1502 and a third time in 1504. Died at Fort Cochin. Dom Francisco de Almedia – the first Portuguese governor in the East. The Policy was called “The blue water policy”. • In 1507, Portuguese arrived at Madras. • Albuquerque, greatest Portuguese governor of the East. • He tried to abolish Sati. • The Portuguese introduced Agricultural Products. • In 1604, they introduced Tobacco in Akbar’s court. • In 1556, they started first Press in India at Goa. • In 1510, they captured Goa from Sultan of Bijapur. The Dutch • In 1602, Dutch East India Company was formed. • First factory at Masulipattanam in 1605. • Captured Cochin in 1663. • In 1741 Battle of Kolachal, Marthandavarma defeated the Dutch. • Finally defeated by the English in the Battle of Bedara in 1759. The English and the French • The English East India Company was founded in 1600 AD by a group of merchants. • John Company was the earlier name. • Hector, the first ship reached Surat on 24th August 1606. • In 1612, the company became a joint stock company. • Sir Thomas Roe, the first ambassador of James - I landed in Surat. • Met Jahangir in 1613. • The French East India Company was founded in 1664 AD. • Factories were established in Surat and Masulipatnam. • First Carnatic War between the French and English in 1742. Ended in 1748. • Second Carnatic War – 1748 to 1754. • Robert Clive – The Governor of English during the war. • Third Carnatic War – 1758-1763. • Battle of Plassey in the year 13th June 1757.

REVOLT OF 1857 Political reasons • “Doctrine of Lapse – Lord Dalhousie, Governor General of India till 1848 – 1856. • “Subsidiary Alliance” – Lord Wellesley. • Takeover of Avadha on ground of ‘Misgovernance’. • Lord Canning, first viceroy of India in 1858 – 1862 AD gave an announcement to Mughals. • A large section among army had irregular soldiers and Pindaris.

219 | P a g e

shop.ssbcrack.com

MCQs

INDIAN HISTORY

MCQs

Economic Reasons • Corruption uncontrolled and inefficient administrative machinery of the company. • Racial Discrimination in both military and civil administrations. • The Revenue system that was introduced took land from cultivation and gave money to the lender. • De-industrialization in the country. Military Reasons • Alien rule – Dispensar of Indian Sepoys. • In 1854, Post office act as withdrawal of postage facility for Sepoys. • Foreign Service allowance for Sepoys serving both in Sindh and Punjab. • Racial discrimination of British against Indian soldiers. Social and Religious Reasons • Social discrimination of the British against Indians. • Through missionaries, spreading Christianity. • The Government took the decision to tax on temple and mosque lands. • In 1850, Enactment of Religious Disabilities Act, enabled to convert to ancestral property. • Opposition to the Indian traditional laws such as Sati, Child Marriage and female infanticide. Immediate Reasons • A new Enfield rifle army, introduced by government. • Indians believed that grease used in cartridges was composed of beef and pig fat. • Due to usage of cartridges, both Hindu and Muslim sepoys were enraged as it was against their religion. Impacts of the Revolt • British Government control in India was moved to the British Crown. • Minister of British Government was called the Secretary of State and responsible for India’s Government. • The Doctrine of Lapse was withdrawn. • End of Mughal and Peshwa rule. • In the army, English soldiers were increased and changes were made in the administration .

Revolt Of 1857 (At A Glance) Centre

Beginning date

Ending date

Indian Leader

British Suppressor

Delhi 1857

11th May, 1857

20th Sep

John Nicholson

Kanpur 1857

4th June 1857

6th Dec

Lucknow

4th June 1857

1858

Jhansi

4th June 1857

Allahabad Jagadishpur (Bihar)

5th June 1857 August 1857

18th June 1858 March 1858 Dec 1858

Bahadur Shah II ‘Zafar’ Nana Sahib, Tantiya Tope Begum Hazrath Mahal Rani LaxmiBai Liyaqat Ali Kunwar and Amir singh

Colonel Neil William Taylor and Vineet Eyre.

220 | P a g e

Colin Campbell Colin Campbell Huge Rose

shop.ssbcrack.com

MCQs

INDIAN HISTORY

MCQs

Governor Generals Warren Hastings (1772 – 1785 AD): • • • • •

The Revenue’s collection was taken over by a company. Judicial powers were given to Zamindars. In each district, establishment of criminal and civil courts. During his period, First Anglo Maratha War (1776-82) ended with the Treaty of Salbai. Second Anglo Mysore war (1780 – 84) ended with the Treaty of Mangalore war (1784).

Lord Cornwallis (1786 – 1793 AD) • •

Permanent settlement of Bengal (Zamindari System) Revenue and criminal courts were re-organized.

Lord Wellesley (1798 – 1805 AD) • • • •

The Subsidiary Alliance system was introduced. To train the company’s servants, opened colleges in Calcutta. Known as ‘Father of Civil services in India’. In 1799, Fourth Anglo – Mysore War, defeated Mysore force under Tipu Sultan.

Lord Minto I (1807 – 1813 AD) • •

Treaty of Amritsar was concluded by Maharaja Ranjit Singh (1809). In 1813, Charter Act was passed.

Lord William Bentinck (1828 – 1833 AD) • • • • • • •

In 1829, Sati was abolished. Increased power of the Magistrate. Appointed Indians as Judges. Banned Female infanticide and human sacrifice. Hindu Law of inheritance reformed. In 1833, Charter Act was passed. First Governor General of India.

Lord Dalhousie (1848 – 1856 AD) • • • • • • •

Introduced the Doctrine of Lapse. In 1853, introduced the first Railway line from Bombay to Thane. Setup universities in Calcutta, Bombay and Madras. Started competitive examination for Indian civil services. In 1854, a Post Office Act was passed. In 1856, Hindu Marriage Act was passed. The title of ‘Nawab of Carnatic’ was abolished.

Growth of Modern Education in India First Phase (1758 – 1812) • •

During this period, British East India Company was interested in the education of subjects. The two minor expectations were

221 | P a g e

shop.ssbcrack.com

MCQs

INDIAN HISTORY

MCQs

➢ In 1781, set up Calcutta Madrsah by Warren Hastings for the study & teaching of Muslim law and subjects. ➢ In 1792, Sanskrit college in Varanasi by Jonathan Duncan for study of Hindu law and philosophy. • Both were designed to help the administration laws in the courts. Second phase (1813 – 1853) • Christian Missionaries and humanitarians, to encourage modern education in India. • In 1813 Charter Act, spent one lakh annually to encourage the learning Indians in India. • During this phase, two controversies about the nature of education arose. • In 1844, Lord Hardinge decided to give jobs to Indians who were educated in English schools. • ‘Downward Filtration theory’ –Education and ideas from the upper classes. • The policy was continued till the end of British rule. • In 1854, it was officially abandoned. Third Phase (1854 – 1900) • • • •

Educational dispatch of 1854 was also called as Wood’s Dispatch. Considered as Magna Carta of English education in India. Rejected ‘Filtration theory’. Established University of Calcutta (Jan 1857) Bombay (Jul 1857), Madras (Sep 1857), Punjab (1882) and Allahabad (1887). • Hunter Commission was appointed by Lord Ripon. Fourth Phase (1901 – 1920) • • • • • •

In 1902, Lord Curzon appointed Universities Commission under Thomas Raleigh. In 1904, Indian Universities Act was passed. Criticized by nationalists In 1910, department of education was established. Lord Chelmsford appointed Saddler Commission, to review the working of Calcutta University. Seven new universities were opened. They were Banaras, Mysore, Patna, Aligarh, Dhaka, Lucknow and Osmania. • Established Kashi Vidyapeeth and Jamia Milia Islamia. Fifth Phase (1921 – 1947) • Officially came under Indian control. • The number of universities was increased. • The quality of higher education was improved. • Established Inter-university board in 1924. • Achievement in women’s education and education of backward classes. Hartog Committee (1929) • Primary education was improved. • Recommended the policy of consolidation. • Universities should be improved. • In 1937, Wardha scheme of Basic Education worked under Zakir Hussain committee. Sargeant plan of Education (1944) • • • •

Established elementary and high schools. Compulsory education for all children under the ages of 6 to 11. Academic and Technical, Vocational are two types in high schools. Abolished Intermediate courses.

222 | P a g e

shop.ssbcrack.com

MCQs

INDIAN HISTORY

MCQs

SOCIAL AND CULTURAL UPRISINGS Brahmo Samaj Movement • • • • • • • • • • •

Founder – Raja Ram Mohan Roy in 1828. Established Atmiya Sabha in 1814. Sati Pratha, Casteism and advocated widow marriage were criticized. Established Vedanta college both in western and Indian and physical sciences also offered. He was a Linguist. Lots of languages known, including Sanskrit, Persian, Arabic, English, French, Latin, Greek and Hebrew. Opposed Sanskrit system of education. Important leaders- Devendranath Tagore and Keshab Chandra Sen. In 1865, Tagore dismissed Keshab. Keshab started Sangat Sabha, Prarthana Samaj and Brahmo Samaj of India. Founder of Prarthana Sabha – Justice M.G. Ranade. Precepts of Jesus – Book written by Raja Ram Mohan Roy.

Arya Samaj • • • • • • • •

Founder – Swami Dayanand Saraswati in 1875. Sayings – ‘Go back to Vedas’ and ‘India for the Indians’. His book ‘Satyarth Prakash’ is a commentary on Veda and he also wrote Veda Bhashya Bhumika and Veda Bhashya. A large number of educational institutions were established. E.g. Gurukulas, DAV school etc. Started Suddhi Movement – to convert non-Hindus to Hinduism. He is known as Luther of Hinduism. In 1866, started Dayanand Anglo Vedic College. Started newspaper, ‘Aryaprakash’.

Ramakrishna Mission • • • • • • • • • •

Shri Ramakrishna Paramahamsa (1834 – 1886), born in village Kumarpukur in the Hoogly village of Bengal. His early name ‘Shuddirama Gadhadhar Chatterjee’. Vivekananda (1861 – 1903) was the most famous disciple of Ramakrishna. Founder of Ramakrishna Mission – Swami Vivekananda (Narendra Dutta as his earlier name) in 1897. Attended Parliament of Religions in Chicago on 11 th September, 1893. In 1900, he was invited to the Congress. In 1899, the mission was shifted to Belur. Started two papers as - Prabudha Barat in English and Udbodhana. Also called as ‘Patriot Saint of India’ and ‘Cyclonic Hindu’. In 1898, Sister Nivedita, an Irish lady was initiated to Brahmacharya by Vivekananda.

Theosophical Society • •

Founded by Madame Blavatsky and Col. H.S. Olcott in New York in 1875. Shifted his headquarters to Adayar near Madras in 1882.

223 | P a g e

shop.ssbcrack.com

MCQs

• • • • • •

INDIAN HISTORY

MCQs

His philosophy was inspired by the Upanishads. Dr. Annie Besant, came to India in 1893. In 1898, she started Central Hindu School at Benaras. Later it became Benaras Hindu University under Madan Mohan Malavya in 1916. First woman President of INC in 1917. Started Home Rule League in 1916 with Dadabhai Naoroji as its President.

Young Bengal Movement • • • •

Founder – Henry Vivian Derozio, teacher in Calcutta Hindu College. Derozians are his followers. They attacked old traditions and decadent customs. In 1828, Academic Association was initiated. Supported women’s education and their rights.

Deva Samaj • •

Founder - Shiv Narayan Agnihotri at Lahore in 1887. Religion - Deva Shastra, Teaching – Devadharma.

INDIAN NATIONAL MOVEMENT • On 28th December 1885, Indian National Congress founded on premises of Gokuldas Tejpal School in Mumbai. • Presided by W.C.Banerjee and attended by 72 delegates. • In INC, A.O.Hume played an instrumental role. • The phases of INC are as follows: • Moderate Phase (1885 – 1905) ➢ The position of INC was not clear during this phase. ➢ The movement was confined to the educated Indians. ➢ Dadabhai Naoroji was the most important among the leaders. • Extremist Phase (1905 – 1919) ➢ Lala Lajpat Rai, Bala Gangadhar Tilak and Bipin Chandra Pal were the leaders during this phase. ➢ Swaraj government made the call of INC. • Gandhian Phase (1919 – 1947) ➢ Dominated by the objective of Purna Swaraj. ➢ Dynamic leadership of Gandhi with unique method of non-violence. Important Movements of Gandhian phase Khilafat Movement • • • • • •

The main cause was the defeat of Turkey in the First World War. The whole movement was based on Muslims. Muslims are India had been upset over British against Turkey. Maulana Abul Kalam Azad, M.A. Ansari and Ali brothers were important leaders. The Khilafat Committee had been formed on 19 th October 1919. Mahatma Gandhi was interested in bringing Hindus and Muslims together.

Non-Cooperation movement 224 | P a g e

shop.ssbcrack.com

MCQs

INDIAN HISTORY

MCQs



Under the leadership of Gandhi, INC launched its first innovative protest, Non-Cooperation movement on 1st August, 1920. • British courts, offices and all kinds of government run organizations were boycotted. Quit India Movement • Also called as August Movement. • Launched on 8th August, 1942. • Result of Gandhi’s protest against the return of Sir Stafford Cripps. • He gave the slogan “Do or Die”. Significance of Indian National Movement Rowlett Act (March 1919) • As per this act, on the basis of suspicion any person could be arrested. • No petition could be filed against such arrests. • This act was called as ‘Black Act’. Jallianwala Bagh Massacre (13th April, 1919) • During the harvest festival, a public meeting was organized in the garden to support Satyagraha. • General Dyer marched in and opened fire on the crowd without any warning. • 379 people were killed and 1137 injured in the incident. Swaraj Party (Jan 1923) • Suspension of Non-cooperation movement split within Congress in the Gaya session in December 1922. • Nehru and Chittranjan Das formed a group on 1st January known as Swaraj Party. Simon Commission (Nov, 1927) • Sir John Simon was appointed as the Chairman by Government of India Act of 1919. • All seven members were Englishmen, there was no Indian. • Lala Lajpat Rai was seriously injured during police lathi charge on 30th October, 1928. • After one month he passed away. Poona Act (1932) • Agreement upon a joint electorate between untouchables and the Hindus. • Took place at Yerawada jail in Pune on 24th September, 1932. Cripps Mission (1942) • Indian Cooperation sent Sir Stafford Cripps to India on 23rd March, 1942. • Major political parties of the country rejected Cripps proposals. • “Postdated cheque” as Cripp’s proposals. The Cabinet Mission (1946) • Pethick Lawrence, Sir Stafford Cripps and A.V. Alexander were three members of the British cabinet sent to India on 15th March, 1946. • Under historic announcement, self-determination and framing of a constitution for India were conceded. • It is known as ‘Cabinet Mission’. Mount Batten Plan (1947) • • •

Prime Minister Atlee, announced the House of Commons on 20th February, 1947. British government transferred power to Indians in June 1948. Atlee sent Lord Mountbatten as Viceroy to India.

225 | P a g e

shop.ssbcrack.com

MCQs

• •

INDIAN HISTORY

MCQs

Congress and Muslim League approved of him. Partition of the country into India and Pakistan came into effect from 15th August, 1947.

Important National leaders • Shaheed Bhagat Singh (1907 – 1931) • Mahatma Gandhi (1869 – 1948) • Pandit Jawaharlal Nehru (1889 – 1964) • Chandrasekhar Azad (1906 – 1931) • Subhas Chandra Bose (1897 – 1945) • Dr. Rajendra Prasad (1884 – 1963) • Sardar Valla Bhai Patel (1875 – 1950) • Lal Bahadur Shastri (1904 – 1966) • Bal Gangadhar Tilak (1856 – 1920) • Gopal Krishna Gokhale (1886 – 1915) 1.4. CULTURE AND HERITAGE Religion • Indus Valley civilization is the oldest and the best civilization of the world. • Vedas are the earliest documents of the human mind. • Four Vedas – Rig, Sama, Yajur and Atharva Veda. • Ramayana and Mahabharata are two popular ancient epics. • Bhagavad Gita – Full of philosophical principles. • Lord Buddha, addressed that happiness could be gained. • Hinduism – large number of gods and goddesses besides the ten incarnations of Lord Vishnu. • Guru Nanak Dev – Founder of Sikhism, preached simplicity, purity of life, and religious tolerance. • Astrology and Astronomy were popular. • Aryabhatta – first major mathematician Astronomer. • He calculated the time of solar eclipse more than 2000 years ago. • C.V. Raman, Subramanian Chandrasekhar and Venkatraman Ramakrishnan are the Indian Scientists who won Nobel prizes in the field of science. Literature • • • • • • • •

India has a rich literary heritage. Rich Regional literature. India- Linguistically diverse community. Sanskrit – Prakrit and Ardha Agahi were spoken by the masses. During the Mughal rule, Sanskrit was the court language. Urdu and Hindustani are common languages in North India. The division of society on the basis of caste system. Raja Rammohan Roy, Dayanand Saraswati, Paramahamsa Dev, Swami Vivekananda, Rabindranath Tagore, Mahatma Gandhi and Jawaharlal Nehru have contributed to Indian culture. Music • Expression of human thoughts and emotions. • Based on the concept of ragas and tales. • Two classical Music styles – Carnatic and Hindustani. • Bhimsen Joshi, M.S. Subhalakshmi, Kishore Amonkar, Ustad Bismillah Khan, Ustad Zakir Hussains are the Indian musicians who have gained popularity both in India and abroad. 226 | P a g e

shop.ssbcrack.com

MCQs

INDIAN HISTORY

MCQs

Dance • Unbroken tradition over 2000 years. • Themes are derived from mythology, legends and classical literature. • Two forms of dances – Folk and Classical dance. • Based on Rasa, Bhava and Abhinaya. • Five main Classical dances – Bharatanatyam, Kathakali, Manipuri, Kathak and Odissi. IMPORTANT FACTS ➢ Pre-historic period is called as Stone Age. ➢ Period of Paleolithic age – 5, 00,000 BC – 8000 BC. ➢ Period of Mesolithic age – 8000 BC – 6000 BC. ➢ Period of Neolithic age – 6000 BC – 1000 BC. ➢ Indus Valley Civilization belongs to Bronze Age during 2500 – 1750 BC. ➢ Mohenjo-Daro is the largest building in this civilization. ➢ Period of Aryan and Vedic Age is 1500 BC – 1000 BC. ➢ Four Vedas: Rig Veda, Sama Veda, Yajur Veda and Sama Veda. ➢ Haryanka dynasty was founded by Bimbisara in Magadha. ➢ Bhanananda was the last ruler of Nanda Dynasty. ➢ Rishabadeva was the founder of Jainism. ➢ There were 24 Tirthankaras. ➢ Mahavira was the last Tirthankara. ➢ Guatama Buddha was the founder of Buddhism. ➢ Three types of Buddhism – Mahayana, Hinayana and Vajrayana. ➢ Chandragupta Maurya founded Mauryan Empire during 322 – 185 BC. ➢ Chanakya was the author of Arthashastra. ➢ Sri Gupta was the founder of Gupta period. ➢ The last emperor of North India was Harshavardhana. ➢ Sangam Age is called as ‘Golden Age’. ➢ Three Kingdoms – Chera, Chola and Pandiya. ➢ Vijayanagar Kingdom founded in 1336. Chatrapathi Shivaji was the greatest leader of Marathas. ➢ Vasco da Gama discovered the sea route to India in 1948 AD. ➢ Battle of Plassey - 13th June 1757. ➢ Lord Dalhousie - introduced Doctrine of Lapse policy. ➢ Lord Wellesley – introduced Subsidiary Alliance Policy. ➢ Sati was abolished by Lord William Bentick in 1829. ➢ Raja Ram Mohan Roy was the founder of Brahmo Samaj in 1828. ➢ Arya Samaj founded by Swami Dyayanand Saraswathi. ➢ Dr. Annie Besant came to India in 1893. ➢ Home Rule League started in 1916. ➢ Indian National Congress founded in 1885. ➢ Under the leadership of Gandhiji, Non-cooperation movement launched by INC on 1st August 1920. ➢ Quit India Movement launched on 8th August 1942. ➢ In Jallian Walla Bagh tragedy 379 people were killed and 1137 were injured. ➢ Simon Commission – 1927 by Sir John Simon. ➢ India Got Independence – 15th August 1947. ➢ Ramayana and Mahabharata are the two great ancient epics. ➢ Arya Bhatta was the first astronomer in Independent India. 227 | P a g e

shop.ssbcrack.com

MCQs

INDIAN HISTORY

MCQs

MODERN INDIAN HISTORY (MULTIPLE CHOICE QUESTIONS) 1. Satyashodhak Samaj whose main aim was to liberate the social Shudra and Untouchables castes from exploitation and oppression was established by _____? [A] Rajaram Mohan Rai [B] Narayan Guru [C] Mahatma Jyotirao Phule [D] Lahuji Raghoji Salve Correct Answer: C [Mahatma Jyotirao Phule] 2. Which of the following date signified attack on Pearl harbor by Japan? [A] January 7, 1941 [B] December 20, 1941 [C] March 7, 1941 [D] December 7, 1941 Correct Answer: D [December 7, 1941] 3. In which year Nawab of Bengal Sirajuddaula, attacked British Factory in Calcutta? [A] 1753 [B] 1756 [C] 1759 [D] 1760 Correct Answer: B [1756] 4. Which Governor of Madras presidency made treaty of Mangalore with Tipu ? [A] Eyercoote [B] Stephenson [C] Wellesely [D] Lord Macartany Correct Answer: D [Lord Macartany] 5. Which among the following terms referred to Free Pass or Duty Free Trade? [A] Parwana [B] Nazarana [C] Dustak [D] Rahadari Correct Answer: C [Dustak] 6. In which of the following wars, Rober Clive was one among the prisoners of war? [A] Carnatic War I [B] Carnatic War II [C] Carnatic War III [D] Seven Years War Correct Answer: A [Carnatic War I]

228 | P a g e

shop.ssbcrack.com

MCQs

INDIAN HISTORY

MCQs

7. Which among the following was capital of Independent state of Bengal? [A] Dacca [B] Murshidabad [C] Alinagar [D] Midnapore Correct Answer: B [Murshidabad] 8. Who among the following is known to purchase Kashmir from the East India Company for a payment of 7,500,000 rupees (75 lakh) and was granted the title Maharaja of Jammu and Kashmir (by Treaty of Amritsar) ? [A] Sher Singh [B] Duleep singh [C] Gulab Singh [D] Lal Singh Correct Answer: C [Gulab Singh] 9. In which of the following years the custom duties were completely waived for the trade for East India Company? [A] 1711 [B] 1715 [C] 1717 [D] 1720 Correct Answer: C [1717] 10. During British Era, East India Company had established botanical gardens at 4 locations as centers for improving botanical knowledge and experimentation under the local Governments. Which among the following is not among them ? [A] Lucknow [B] Poona [C] Sibpur [D] Madras Correct Answer: A [Lucknow] 11. In which of the following years the custom duties were completely waived for the trade for East India Company? [A] 1711 [B] 1715 [C] 1717 [D] 1720 Correct Answer: C [1717] 12. During British Era, East India Company had established botanical gardens at 4 locations as centers for improving botanical knowledge and experimentation under the local Governments. Which among the following is not among them ? [A] Lucknow [B] Poona 229 | P a g e

shop.ssbcrack.com

MCQs

INDIAN HISTORY

MCQs

[C] Sibpur [D] Madras Correct Answer: A [Lucknow] 13. Bhoodan Yojna which was a measure of gifting land by the rich landlords to the poor laborers was initiated by whom among the following? [A] Acharya Narendra Dev [B] Acharya Vinoba Bhave [C] Raj Naraina [D] Medha patkar Correct Answer: B [Acharya Vinoba Bhave] 14. Who among the following was the editor of Bengal Gazette or the Calcutta General Advertiser , which was the first English language newspaper, and first printed newspaper, to be published in the Indian subcontinent? [A] Daniel Defoe [B] Oliver Goldsmith [C] James Augustus Hickey [D] David Hume Correct Answer: C [James Augustus Hickey] 15. Which of the following country was officially declared to be the world’s first atheist state after world war II? [A] Algeria [B] Albania [C] Croatia [D] Slovenia Correct Answer: B [Albania] 16. Founding Partner of which of the following companies of India became first Finance Minister of Pakistan? [A] Tata Motors [B] Mahindra & Mahindra [C] Larsen & Toubro [D] Nicholas Piramal Correct Answer: B [Mahindra & Mahindra] Explanation: The company was established in Ludhiana in 1945 as Mahindra & Mohammed. Ghulam Mohammed went to Pakistan and became first Finance Minister of that country. 17. Who among the following viceroys of India wrote a book ” Persia” ? [A] Lord Elgin [B] Lord Curzon [C] Lord Minto [D] Lord Irwin Correct Answer: B [Lord Curzon]

230 | P a g e

shop.ssbcrack.com

MCQs

INDIAN HISTORY

MCQs

18. At the time of Independence of India in 1947, what was women literacy ? [A] 6% [B] 8% [C] 10% [D] 15% Correct Answer: B [8%] 19. Who was the first English President of Indian National Congress? [A] William Wedderburn [B] George Yule [C] A O Hume [D] Henery Cotton Correct Answer: B [George Yule] 20. Which among the following is not correctly matched? [A] First Sikh War – Lord Hardinge [B] Second Sikh War – Lord Dalhousie [C] Fourth Mysore war – Lord Wellesley [D] Permanent settlement of Benga-Lord curzon Correct Answer: D [Permanent settlement of Benga-Lord curzon] Explanation: Permanent settlement of Bengal – (Lord Cornwallis) 21. Who among the following is known to organize a military force of Sikhs called Khalsa to avenge the murder of his father thus establishing Khalsa Panth ? [A] Guru Teg Bahadur [B] Guru Gobind Singh [C] Banda Bahadur [D] Guru Arjun Dev Correct Answer: B [Guru Gobind Singh] Explanation: The 9th Guru of Sikhs and father of Guru Gobind Singh , Guru Teg Bahadur was captured and executed by Aurangazeb in 1675 because he refused to embrace Islam. This made Sikhs resent to the religious intolerance of Mughal . His son Guru Gobind Singh had raised this military force organized his followers into the force called Khalsa. 22. In which year first fleet of Dutch reached in India? [A] 1498 [B] 1510 [C] 1550 [D] 1595 Correct Answer: D [1595] 23. In which of the following sessions of Indian National Congress , our national anthem was first sung 27 December 1911 ? [A] Lahore [B] Calcutta [C] Delhi [D] Surat Correct Answer: B [Calcutta] 231 | P a g e

shop.ssbcrack.com

MCQs

INDIAN HISTORY

MCQs

24. In which of the following months do India and Pakistan exchange their nuke installation lists every year? [A] January [B] February [C] March [D] April Correct Answer: A [January] 25. Which of the following country’s government is known to impose Policy of Apartheid between 1948 and 1994 which curtailed rights of the black inhabitants and maintained rights of whites? [A] Australia [B] New Zea Land [C] South Africa [D] United States Correct Answer: C [South Africa] 26. Which among the following acts of United states is known to be prohibiting discrimination based on race, color, religion, sex, and national origin by federal and state governments as well as some public places? [A] Civil Rights Act of 1871 [B] Civil Rights Act of 1960 [C] Civil Rights Act of 1964 [D] Civil Rights Act of 1968 Correct Answer: C [Civil Rights Act of 1964] 27. Which among the following is the highest civilian bravery award? [A] Bharat Ratna [B] Paramvir Chakra [C] Sarvottam Jeevan Raksha Padak [D] Jeevan Raksha Padak Correct Answer: C [Sarvottam Jeevan Raksha Padak] 28. The first Industrial Revolution took place in? [A] England [B] USA [C] France [D] Germany Correct Answer: A [England] 29. Who was India’s first I.C.S. Officer? [A] Satyendra Nath Tagore [B] Sukumar Sen [C] Uday Singh [D] Annada Shankar Ray Correct Answer: A [Satyendra Nath Tagore]

232 | P a g e

shop.ssbcrack.com

MCQs

INDIAN HISTORY

MCQs

30. In which year unity of Germany became a reality? [A] 1870 [B] 1871 [C] 1875 [D] 1880 Correct Answer: B [1871] Explanation: On 18 Jan 1871, the King of Prussia was crowned the German emperor of Versailles 31. Who among the following is the first Indian to win a Hoover Prize? [A] Atal Bihari Vajpayee [B] APJ Abdul Kalam [C] Man Mohan Singh [D] Sonia Gandhi Correct Answer: B [APJ Abdul Kalam] 32. Which among the following date in world history is known as D-day, a date also known as of Operation Neptune and Operation Overlord? [A] 3 March 1943 [B] 6 March 1943 [C] 6 June 1944 [D] 6 June 1945 Correct Answer: C [6 June 1944] 33. Who among the following are known to colonize an island first of all? [A] Britishers [B] French [C] Portuguese [D] Dutch Correct Answer: C [Portuguese] 34. Till which year Calcutta was capital of India? [A] 1912 [B] 1913 [C] 1914 [D] 1915 Correct Answer: A [1912] Explanation: in this year the capital of British India was transferred to Delhi 35. In which century maximum investment was made by the British in Core & Infrastructure sector in India? [A] 17th century [B] 18th century [C] 19th century [D] 20th century Correct Answer: C [19th century]

233 | P a g e

shop.ssbcrack.com

MCQs

INDIAN HISTORY

MCQs

36. The main objective of British developing India Railway system was to facilitate the traffic between? [A] Source of raw material and export centers [B] Source of raw material and ports [C] Source of raw material and production centers [D] Source of raw material and markets Correct Answer: C [Source of raw material and production centers] 37. Who among the following Governor General of India is famous for Sati Reforms & suppression of human sacrifice? [A] Lord William Bentinck [B] Lord Curzon [C] Lord Rippon [D] Lord Ellenborough Correct Answer: A [Lord William Bentinck] 38. Who among the following established Ramakrishna Mission? [A] Swami Vivekananda [B] Swami Ashokananda [C] Swami Nityaswarupananda [D] Swami Kirtidananda Correct Answer: A [Swami Vivekananda] 39. Kemal Ataturk is known as founder and builder of Modern_______? [A] Turkey [B] Georgia [C] Syria [D] Kuwait Correct Answer: A [Turkey] 40. Poverty and Un-British Rule in India by Dadabhai Naoroji, was the original classic work on poverty in India. In which year this work was published? [A] 1900 [B] 1901 [C] 1903 [D] 1905 Correct Answer: B [1901] 41. In which year Treaty of Amritsar was signed between Ranjit Singh and East India Company? [A] 1800 [B] 1803 [C] 1806 [D] 1809 Correct Answer: D [1809] 42. During the end of which of the Mughal rulers the Mughal empire faced the problem of jagirdari crisis? [A] Humayun [B] Akbar 234 | P a g e

shop.ssbcrack.com

MCQs

INDIAN HISTORY

MCQs

[C] Jahanagir [D] Aurangzeb Hide Answer Correct Answer: D [Aurangzeb] 43. In which year split in All India Trade Union Congress took place? [A] 1920 [B] 1925 [C] 1929 [D] 1935 Correct Answer: C [1929] 44. Who among the following is known to oppose the Age of consent bill? [A] Behramji Malabari [B] Bal Gangadhar Tilak [C] Mahatma Gandhi [D] Bipin Chandra Pal Correct Answer: B [Bal Gangadhar Tilak] 45. For which of the following commodities, the Dutch wanted to barter cotton piece goods in the Indonesian Archipelago? [A] Tobacco [B] Rice [C] Pepper and Spices [D] Silk Correct Answer: C [Pepper and Spices] 46. The dual government of Bengal was result of a farman in 1765 by which of the following? [A] British [B] Shah Alam [C] Nawab Sirajuddaula [D] Raja Khan Correct Answer: B [Shah Alam] 47. Who among the following introduced the modern concept of rule in India? [A] Mughal [B] British [C] Portuguese [D] French Correct Answer: B [British] 48. During which of the following period the Bharat Ratna awards were discontinued? [A] 1958-61 [B] 1963-66 [C] 1977-80 [D] 1997-2005 Correct Answer: C [1977-80] 235 | P a g e

shop.ssbcrack.com

MCQs

INDIAN HISTORY

MCQs

49. Which among the following is oldest? [A] Border security force [B] Central Reserve Police Force [C] Assam Rifles [D] National Security Guards Correct Answer: C [Assam Rifles] 50. Which among the following does not correctly denotes the headquarters of Indian Army’s Operational Command? [A] Eastern Command – Kolkata [B] Western Command – Chandimandir [C] Northern Command – Udhampur [D] Southern Command – Chennai Correct Answer: D [Southern Command – Chennai] Explanation: Southern Command has its headquarters in Pune. 51. Kemal Ataturk is known as founder and builder of Modern_______? [A] Turkey [B] Georgia [C] Syria [D] Kuwait Correct Answer: A [Turkey] 52. Poverty and Un-British Rule in India by Dadabhai Naoroji, was the original classic work on poverty in India. In which year this work was published? [A] 1900 [B] 1901 [C] 1903 [D] 1905 Correct Answer: B [1901] 53. In which year Treaty of Amritsar was signed between Ranjit Singh and East India Company? [A] 1800 [B] 1803 [C] 1806 [D] 1809 Correct Answer: D [1809] 54. During the end of which of the Mughal rulers the Mughal empire faced the problem of jagirdari crisis? [A] Humayun [B] Akbar [C] Jahanagir [D] Aurangzeb Correct Answer: D [Aurangzeb] 55. In which year split in All India Trade Union Congress took place? [A] 1920 [B] 1925 236 | P a g e

shop.ssbcrack.com

MCQs

INDIAN HISTORY

MCQs

[C] 1929 [D] 1935 Correct Answer: C [1929] 56. Who among the following is known to oppose the Age of consent bill? [A] Behramji Malabari [B] Bal Gangadhar Tilak [C] Mahatma Gandhi [D] Bipin Chandra Pal Correct Answer: B [Bal Gangadhar Tilak] 57. For which of the following commodities, the Dutch wanted to barter cotton piece goods in the Indonesian Archipelago? [A] Tobacco [B] Rice [C] Pepper and Spices [D] Silk Correct Answer: C [Pepper and Spices] 58. The dual government of Bengal was result of a farman in 1765 by which of the following? [A] British [B] Shah Alam [C] Nawab Sirajuddaula [D] Raja Khan Correct Answer: B [Shah Alam] 59. Who among the following introduced the modern concept of rule in India? [A] Mughal [B] British [C] Portuguese [D] French Correct Answer: B [British] 60. During which of the following period the Bharat Ratna awards were discontinued? [A] 1958-61 [B] 1963-66 [C] 1977-80 [D] 1997-2005 Correct Answer: C [1977-80] 61. Which among the following is oldest? [A] Border security force [B] Central Reserve Police Force [C] Assam Rifles [D] National Security Guards Correct Answer: C [Assam Rifles]

237 | P a g e

shop.ssbcrack.com

MCQs

INDIAN HISTORY

MCQs

62. Which among the following does not correctly denotes the headquarters of Indian Army’s Operational Command? [A] Eastern Command – Kolkata [B] Western Command – Chandimandir [C] Northern Command – Udhampur [D] Southern Command – Chennai Correct Answer: D [Southern Command – Chennai] Explanation: Southern Command has its headquarters in Pune 63. The MIG series of Super fighters have come to India from which of the following ________ ? [A] Russia [B] USA [C] UK [D] France Correct Answer: A [Russia] 64. During the world war II which among the following was not a major Axis Power? [A] Germany [B] Japan [C] Hungary [D] France Correct Answer: D [France] 65. In which year Calcutta Flag designed by Sachindra Prasad Bose unfurled in Calcutta? [A] 1904 [B] 1905 [C] 1906 [D] 1907 Correct Answer: C [1906] 66. Which among the following in Goa houses the mortal remains of St. Francis Xavier? [A] Se Cathedral [B] Basilica of Bom Jesus [C] Church of St. Anne [D] S. Caetano Correct Answer: B [Basilica of Bom Jesus] 67. In which of the following states of India, the earliest settlements of Jews are found [A] Kerala [B] Tamil nadu [C] Punjab [D] Haryana Correct Answer: A [Kerala] 68. Which year is known for India’s Smiling Buddha? [A] 1970 [B] 1972 238 | P a g e

shop.ssbcrack.com

MCQs

INDIAN HISTORY

MCQs

[C] 1974 [D] 1975 Correct Answer: C [1974] 69. Seventeen Point Agreement” was a turning point in the history of which of the following neighbors of India? [A] Bhutan [B] Bangaldesh [C] Nepal [D] Tibet Correct Answer: D [Tibet] 70. During whose era the Agra Canal was finished? [A] Akbar [B] Jahangir [C] Bahadurshah Jafar [D] Birtish period Correct Answer: D [Birtish period] 71. Who among the following is given credit of starting agitation for political reforms ? [A] Tantia Tope [B] Kunwar singh [C] Nan Sahib [D] Raja Rammohan Roy Correct Answer: D [Raja Rammohan Roy] 72. Which of the following Union Territory of India was occupied by Japan during World War II? [A] Lakshadweep [B] Goa, Daman & Diu [C] Pondicherry Union [D] Andaman & Nocobar Islands Correct Answer: D [Andaman & Nocobar Islands] 73. Raja Rammohun Roy Library Foundation (RRRLF) was established in May, 1972 by the Department of Culture, Govt. of India to spread library services all over the country in cooperation with State Governments, Union Territory Administrations and Organization working in the field. Where are the headquarters of Raja Rammohun Roy Library Foundation? [A] Kolkata [B] Mumbai [C] Madras [D] Hyderabad Correct Answer: A [Kolkata] 74. Tilak Swarajya Fund was started to finance which of the following movements? [A] Civil Disobedience Movement [B] Non-Cooperation Movement [C] Champaran Satyagraha [D] Rowlatt Satyagraha Correct Answer: B [Non-Cooperation Movement] 239 | P a g e

shop.ssbcrack.com

MCQs

INDIAN HISTORY

MCQs

75. In which of the following sessions Mahatma Gandhi became president of INC? [A] Delhi [B] Gaya [C] Belgaum [D] Lucknow Correct Answer: C [Belgaum] 76. In which year a British Committee of INC was founded? [A] 1885 [B] 1887 [C] 1889 [D] 1891 Correct Answer: C [1889] 77. Which of the following is related to Gokuldas Tejpal Sanskrit pathshala? [A] First Meeting of Indian National Congress [B] Second Session of Indian National Congress [C] First fast of Gandhi Ji [D] First Ashram of Gandhi Ji Correct Answer: A [First Meeting of Indian National Congress] Explanation: The First meeting of the INC was organized by A 0 Hume at Gokaldas Tejpal Sanskrit College on 28thDecember 1885 (Bombay). Its first President was W C Bonnerjee. It was the first organized expression of the Indian National movement on an all-India scale. 78. Which among the following formulated a Dominion Status Constitution for India? [A] Fourteen Points of Jinnah [B] Nehru Report [C] Butler Report [D] Simon commission Correct Answer: B [Nehru Report] 79. Who among the following were the most vocal opponents of Ilbert Bill? [A] Zamindars of Bengal & Bihar [B] Advocates and legal practitioners of Kolkata [C] British tea and indigo plantation owners in Bengal [D] British Army Officials Correct Answer: C [British tea and indigo plantation owners in Bengal] 80. In which of the following sessions Indian National Congress was split between two groups moderates and extremists? [A] Surat [B] Lucknow [C] Delhi [D] Bombay Correct Answer: A [Surat] Explanation: Surat Split 1907. please note this session was originally planned for Nagpur

240 | P a g e

shop.ssbcrack.com

MCQs

INDIAN HISTORY

MCQs

81. Who among the following chaired a committee of the All Parties Conference, which presented Nehru Report in 1928? [A] Jawahar Lal Nehru [B] Moti Lal Nehru [C] Bhim Rao Ambedkar [D] Muhammad Ali Jinnah Correct Answer: B [Moti Lal Nehru] 82. Who among the following gave communal award? [A] Ramsay Macdonald [B] S Baldwin [C] Llyod George [D] Winston Churchill Correct Answer: A [Ramsay Macdonald] Explanation: The Communal Award was announced by the British Prime Minister Ramsay Macdonald in August 1932 to grant separate electorates to minority communities, including Muslims, Sikhs, and Dalit 83. Collection of land revenue through Zamindars on behalf of the east India Company was called as __? [A] Land Reforms [B] Jageerdari [C] Permanent settlement [D] Ryotwari Correct Answer: C [Permanent settlement] 84. May 11, 1857 is known for which of the following incidence in Indian History? [A] The 3rd Light Cavalry of the British East India Company’s army rebels against its British officers, thus beginning the rebellion [B] Indian combatants capture Delhi from the East India Company [C] Three universities are established in British India. [D] Execution of Mangal pandey Correct Answer: B [Indian combatants capture Delhi from the East India Company] 85. Headquarters of “Anusheelan Samiti” is located at: [A] Mumbai [B] Pune [C] Kolkata [D] Varnasi Correct Answer: C [Kolkata] 86. Consider the following incidences: 1. Foundation of Muslim league 2. Jallianwalla Bagh Massacre 3. Khilafat Movement 4. Civil Disobedience Movement Arrange the above in Correct Chronological order: [A] 1,2,3,4 [B] 1,3,2,4 241 | P a g e

shop.ssbcrack.com

MCQs

INDIAN HISTORY

MCQs

[C] 2,1,3,4 [D] 2,4,1,3 Correct Answer: A [1,2,3,4] Explanation: Muslim league 1906, Jallianwallla Bagh 13.4.1919, Khilafat Movement 1919-1924, Civil Disobedience 1930 87. Which among the following universities were not set up under Sadler Commission of 1917? 1. Patna University 2. Osmania University 3. Aligarh Muslim University [A] 1 only [B] 2 only [C] 1 & 2 only [D] 1, 2, 3 all were set up under Sadler Commission of 1917 Correct Answer: D [1, 2, 3 all were set up under Sadler Commission of 1917] 88. In which year General committee of Public Instruction was replaced by a Council of education? [A] 1836 [B] 1841 [C] 1856 [D] 1878 Correct Answer: B [1841] 89. Who among the following made English the court language replacing the Persian Language? [A] Lord Macaulay [B] Lord Bentick [C] Charles Wood [D] Lord Curzon Correct Answer: B [Lord Bentick] 90. Consider the following statements: 1. The charter act of 1813 allowed the Christian missionaries to spread their religious ideas in India. 2. The East India Company , for the first time acknowledged the state responsibility for promotion of Education India Which among the above statements are correct ? [A] 1 only [B] 2 only [C] Both 1 & 2 [D] None of the above Correct Answer: C [Both 1 & 2] 91. Which among the following suspended the Non cooperation movement? [A] Jallianwala Bagh Incident [B] Chauri Chaura Incident 242 | P a g e

shop.ssbcrack.com

MCQs

INDIAN HISTORY

MCQs

[C] Peterloo massacre of 1819, [D] None of the above Correct Answer: B [Chauri Chaura Incident] 92. During tenure of which of the following all high Indian officials were dismissed and all posts worth more than £500 a year were reserved for Europeans? [A] Lord Dalhousie [B] Lord Curzon [C] Lord Wellesley [D] Lord Cornwallis Correct Answer: D [Lord Cornwallis] Explanation: He is known for beginning of Indian Civil Service and Europeanization of Higher postings 93. In which year First Roundtable conference on Kashmir was held in New Delhi ? [A] 2002 [B] 2004 [C] 2005 [D] 2006 Correct Answer: D [2006] Explanation: The first-round table conference on Kashmir was held on 25th May 2006 under prime minister man Mohan Singh. 94. Which of the following administers Jawaharlal Nehru Award for International Understanding, instituted by Government of India? [A] Indian Council of Social Science Research [B] Indian Council for Cultural Relations [C] Jawahar Lal Nehru University [D] Council of Scientific and Industrial Research Correct Answer: B [Indian Council for Cultural Relations] 95. In which of the following gives International Gandhi Award? [A] Government of India [B] Gandhi Memorial Leprosy Foundation [C] United nations [D] Gandhi Foundation Correct Answer: B [Gandhi Memorial Leprosy Foundation] Explanation: To commemorate Mahatma Gandhi’s service and scientific approach to leprosy, in 1986 the Gandhi Memorial Leprosy Foundation instituted the International Gandhi Award 96. The battle of Aliwal was fought between which of the following? [A] English and Marathas [B] English and Nawab of Awadh [C] English and Sikhs [D] English and Tipu Sultan Correct Answer: C [English and Sikhs]

243 | P a g e

shop.ssbcrack.com

MCQs

INDIAN HISTORY

MCQs

97. My Country My Life” is an autobiography of which of the following leaders? [A] Dr. R venkatramanan [B] Lal Krishna Advani [C] AB Vajpayee [D] APJ Abdul Kalam Correct Answer: B [Lal Krishna Advani] 98. In which year Government of India launched the International Gandhi Peace Prize ? [A] 1990 [B] 1992 [C] 1995 [D] 1996 Correct Answer: C [1995] Explanation: 1995 on the occasion of the 125th birth anniversary of Mahatma Gandhi. 99. Pingali Venkaya is related to which of the following? [A] Indian Army [B] India’s National Flag [C] India’s National Anthem [D] India’s Constitution Correct Answer: B [India’s National Flag] Explanation: Tricolor flag was first accepted by Indian National Congress in 1931. It had charkha in the center of the white band which was later replaced by Wheel taken from Sarnath Asoka pillar. The design of the national flag was adopted by the Constituent Assembly of India on 22 July 1947. In the constituent assembly, Dr. Rajendra Prasad was the head of the Ad hoc committee on National Flag. The flag was designed by Pingali Venkayya. 100. Which among the following is not a Book By Jawahar Lal Nehru? [A] The Question of Language [B] Where are we? [C] The Unity of India [D] Freedom in Exile Correct Answer: D [Freedom in Exile] Explanation: Freedom in Exile is an autobiography of Dalai Lama 101. Which of the following is NOT a correct statement with respect to Surendranath Banerjee? [A] He was co-founder of Indian National Association [B] He was founder of Ripon College in Kolkata [C] He is known to have evolved National Consciousness as a formal concept [D] He co-founded Sadharan Brahmo Samaj with Anandmohan Bose Correct Answer: D [ He co-founded Sadharan Brahmo Samaj with Anandmohan Bose] Explanation: The fourth statement is not correct. Banarejee had co-founded Indian National Association with Anandmohan Bose but was not related to Sadharan Brahmo Samaj which was created by Bose along with Shibnath Shastri, Sib Chandra Deb, Umesh Chandra Dutta etc. 102.The Cartaz system with reference to naval trade was used by which of the following? [A] Dutch [B] French 244 | P a g e shop.ssbcrack.com

MCQs

INDIAN HISTORY

MCQs

[C] English [D] Portuguese Correct Answer: D [ Portuguese] Explanation: Cartaz system referred to a naval trade license or pass issued by the Portuguese in Indian ocean during the sixteenth century. A similar system used by British was navicert system in 20th century. 103. In 1835 the Cacher Levy with about 750 men was raised to counter the tribal raids. It is known with which of the following names today? [A] Central Reserve Police Force [B] Rajputana Rifles [C] Assam Rifles [D] National Security Guards Correct Answer: C [Assam Rifles] 104. In the wake of the Chinese aggression in 1962, which of the following squadron was formed to strengthen our supply lines to the Army in the Northern Sector encompassing J&K; and Ladakh regions? [A] 25 Squadron (Himalayan Eagles) [B] No 44 Squadron (Mighty Jets) [C] No. 43 Squadron (Nabhasa Jivan Dhara) [D] 41 Squadron (Otters) Correct Answer: A [25 Squadron (Himalayan Eagles)] 105. Demand for Pakistan was raised by Mohammed Ali Jinnah in which of the following years? [A] 1938 [B] 1940 [C] 1942 [D] 1944 Correct Answer: B [1940] Explanation: In 1940 at Lahore Session of Muslim League Demand for Pakistan was raised by Mohammed Ali Jinnah 106. At which of the following places held was held the All India Khilafat Conference in 1919? [A] Lahore [B] Mumbai [C] Delhi [D] Dhaka Correct Answer: C [Delhi] 107. Which among the following event is related to Attack on Lord Hardinge? [A] Ghadar Conspiracy [B] Hindu German Conspiracy [C] Delhi Conspiracy [D] Silk Letter Conspiracy Correct Answer: C [Delhi Conspiracy] 108. Which of the following revolutionaries has written Bandi Jeevan? [A] Chandra Shekhar Azad [B] Sachindra Nath Sanyal 245 | P a g e

shop.ssbcrack.com

MCQs

INDIAN HISTORY

MCQs

[C] Ras Behari Bose [D] Bhagat Singh Correct Answer: B [Sachindra Nath Sanyal] 109. Which among the following is not correctly matched? [A] Khond Uprising : Chakra Bisoi [B] Khonda Dora Campaign : Korra Mallaya [C] Bhuyan & Juang Rebellion : ratna patnayak [D] All are correctly matched Correct Answer: D [All are correctly matched] 110. Which among the following state is related to Moplah Rebellion? [A] Kashmir [B] Kerala [C] Orissa [D] Assam Correct Answer: B [Kerala] Explanation: The Moplah Rebellion or the Malabar Rebellion was an extended version of the Khilafat Movement in Kerala in 1921. The Government had declared the Congress and Khilafat meetings illegal. So, a reaction in Kerala began against the crackdown of the British in Eranad and Valluvanad taluks of Malabar. But the Khilafat meeting incited so much communal feelings among the Muslims peasants , known as Moplahs, that it turned out to become an antihindu movement from July 1921 onwards. The violence began and the Moplahs attacked the police stations and took control of them. They also seized the courts, and the government treasuries. It became a communal riot when the kudiyaan or tenant Moplahs attacked their Hindu jenmis or landlords and killed many of them. Thus the Hindu Landlords became the victims of the atrocities of the Moplahs. 112. Who among the following was the founder of Bihar Vidyapeeth? [A] Abdul kalam Azad [B] Dr. Rajendra Prasad [C] Lok Manya Tilak [D] JB Kriplani Correct Answer: B [Dr. Rajendra Prasad] Explanation: Dr. Rajendra Prasad Established Bihar Vidyapeeth with Mahatma Gandhi and Maulana Majharul Haq in 1921 113. Boycott of British Goods is related to which of the following? [A] Swadeshi Movement [B] Quit India Movement [C] Non Cooperative Movement [D] Home Rule Movement Correct Answer: A [Swadeshi Movement] 114. Which among Mountbatten plan, CR Formula, Wavell Plan and Cabinet Mission is chronologically earliest? [A] CR Formula [B] Wavell Plan 246 | P a g e

shop.ssbcrack.com

MCQs

INDIAN HISTORY

MCQs

[C] Cabinet mission [D] Mountbatten Plan Correct Answer: A [CR Formula] Explanation: CR Formula-1944 Wavell Plan-1945 Cabinet mission-1946 Mountbatten Plan-1947 115. In which year the first nationwide non cooperation movement was launched in India? [A] 1910 [B] 1915 [C] 1920 [D] 1925 Correct Answer: C [1920] 116. Who among the following founded “National Herald”? [A] Bal Gangadhar Tilak [B] Mahatma Gandhi [C] Jawahar Lal Nehru [D] Indira Gandhi Correct Answer: C [Jawahar Lal Nehru] 117. Great Game’ is a term used for the strategic rivalry and conflict between which of the following ? [A] British Empire and Russian Empire [B] British Empire and French Empire [C] British Empire and Portuguese Empire [D] British Empire and Chinese Empire Correct Answer: A [British Empire and Russian Empire] 118. Which among the following is not correct about C. R. Formula given by C Rajgopalachari? [A] It favored talks with Jinnah and the Muslim League [B] It plebiscite among Muslims weather they should form a separate state [C] It formed the basis of the 1944 Gandhi-Jinnah talks [D] All are correct Correct Answer: D [All are correct] Explanation: Objective of the C R Formula was to solve the political deadlock between the All India Muslim League and Indian National Congress. League’s position was that the Muslims and Hindus of British India were of two separate nations and hence the Muslims had the right to their own nation. The Congress was opposed to the idea of partitioning India. The Core principle of the CR Formula was a proposal for the Congress to offer the League the Muslim Pakistan based on plebiscite of all the peoples in the regions where Muslims made a majority. 119. Which of the following movement was started to support Muslim resentment against ottoman Empire Policy of British? [A] Moplah Riots [B] Khilafat Movement [C] Reshmi Rumal Movement [D] Ghadar Conspiracy Correct Answer: B [Khilafat Movement] 247 | P a g e shop.ssbcrack.com

MCQs

INDIAN HISTORY

MCQs

Explanation: The Khilafat Movement (1919-20 ) was essentially a movement to express Muslim support for the Caliph of Turkey against the allied powers particularly Britain. 120. Which among the following is not correct about VD savarkar? [A] He is considered a central figure in the Hindu nationalist Movement [B] He supported and took part in the Quit India struggle in 1942 [C] He founded a revolutionary organization called Mitra Mela [D] He was one of those accused in the assassination of Mahatma Gandhi Correct Answer: B [He supported and took part in the Quit India struggle in 1942] Explanation: Serving as the president of the Hindu Mahasabha, Savarkar endorsed the ideal of India as a Hindu Rashtra and controversially opposed the Quit India struggle in 1942 121. Which among the following statements is not correct? [A] Phoenix Ashram was founded by Mahatma Gandhi [B] Life Divine is a work of Aurobindo Ghosh [C] in 1765 British acquired the Diwani of Bengal, Bihar and Orissa [D] All are correct Correct Answer: D [All are correct] 122. Bring out the wrong statements? [A] Leo Tolstoy was one of the people who influenced Gandhi Ji very Much [B] Religious and socials reforms contributed a lot to develop nationalism in India [C] The main purpose of Crips Mission was to form constituent assembly [D] All are correct Correct Answer: D [All are correct] 123. Bring out the Incorrect Statements? [A] Purna Swarajya Resolution was adopted by INC in Lahore Session [B] Gopal Krishna Gokhle was founder of ‘Servants of India Society” [C] The Chittagong Armory raid and temporary capture of port town were organized under Surya Sen [D] All are correct Correct Answer: D [All are correct] 124. ‘Yudh Abhyas is a millitary exercise between: [A] India – UK [B] India – USA [C] India – Russia [D] India – Israel Correct Answer: B [India – USA] 125. Which among the following is the leaders of China’s socio-political programs ‘Great Leap Forward’ and the ‘Cultural Revolution’? [A] Zeng Qinghong [B] Wang Zuo [C] Mao Zemin [D] Mao Zedong Correct Answer: D [Mao Zedong] 248 | P a g e

shop.ssbcrack.com

MCQs

INDIAN HISTORY

MCQs

126. Which among the following sentences is not correct? [A] British army was defeated in the first Anglo-Maratha War [B] The Second Anglo-Mysore War was concluded by Treaty of Wadgaon [C] Hyder Ali Died in the Second Anglo-Mysore war [D] Third Anglo Mysore war led to the Treaty of Seringpatnam Correct Answer: B [The Second Anglo-Mysore War was concluded by Treaty of Wadgaon] Explanation: The Second Anglo-Mysore War was concluded by Treaty of Mangalore 127. For how many times Mahatma Gandhi was nominated for Nobel Peace Prize ? [A] 3 [B] 4 [C] 5 [D] He never nominated Correct Answer: C [5] Explanation: Mahatma Gandhi was nominated for the Nobel Peace Prize five times between 1937 and 1948 but never received the prize, being assassinated on 30 January 1948 two days before the closing date for the 1948 Peace Prize nominations. 128. Who was the first recipient of Dada Saheb Phalke Award? [A] Prithviraj kapoor [B] Raj kapoor [C] Devika Rani [D] Guru Dutt Correct Answer: C [Devika Rani] 129. Which among the following is not a work of Rabindranath Tagore? [A] The Gardner [B] Gitanjali [C] King of Dark Chamber [D] All of the above are works of Rabindranath Tagore Correct Answer: D [All of the above are works of Rabindranath Tagore] 130. Lord Clement Atlee was prime minister of England at which of the following events? [A] Simon Commission [B] Government of India Act 1935 [C] Jallianwalla Bagh Massacre [D] India’s Independence Correct Answer: D [India’s Independence] 131.” Hindus and Muslims form one nation in India. They are two eyes of India; Injure one and you injure the other” whose thoughts were these? [A] Mahatma Gandhi [B] Mohammed Iqbal [C] Sir Sayed Ahmad Khan [D] Jinnah Correct Answer: C [Sir Sayed Ahmad Khan]

249 | P a g e

shop.ssbcrack.com

MCQs

INDIAN HISTORY

MCQs

132. In which of the following states the influence of the ” Ahrar movement (1931)” was mostly confined to ? [A] Punjab [B] Sindh [C] Bengal [D] Madras Correct Answer: A [Punjab] 133. Who among the following imposed Press censorship? [A] Lord William Bentinck [B] Lord Curzon [C] Lord Wellesley [D] Lord Rippon Correct Answer: C [Lord Wellesley] Explanation: In 1799 Lord Wellesley promulgated the Press Regulations, which had the effect of imposing pre-censorship on an infant newspaper publishing industry. 134. Sarda Act which was sponsored by Harbilas Sarda to the British India Legislature in India was passed on 28 September 1929. This act was to prevent which of the following malice in Indian Society ? [A] Sati System [B] Child marriage [C] Dowry System [D] Widow remarriage Correct Answer: B [Child marriage] 135. Year 1674 AD marks which of the following important events of life of Shivaji ? [A] Shivaji’s Coronation [B] Treaty of Purander [C] Battle of surat [D] None of the above Correct Answer: A [Shivaji’s Coronation] Explanation: Shivaji was formally crowned Chhatrapati (meaning lord of the umbrella, the umbrella being an important symbol of royalty), on June 6, 1674 at Raigad fort, and given the title Kshatriya Kulavantas Sinhasanadheeshwar Chhatrapati Shivaji Maharaj 136. Who was prime Minister of England when Simon Commission came to India to review the working of Government of India Act 1919? [A] Stanley Baldwin [B] David Lloyd Georg [C] Ramsay MacDonald [D] Minto Correct Answer: A [Stanley Baldwin] 137. Who among the following was a member of Simon Commission who subsequently became the British Prime Minister and later was to oversee the granting of independence to India & Pakistan in 1947? [A] Winston Churchill [B] Neville Chamberlain 250 | P a g e

shop.ssbcrack.com

MCQs

INDIAN HISTORY

MCQs

[C] Clement Attlee [D] Lord Mountbatten Correct Answer: C [Clement Attlee] 138. Adi Granth” was compiled by which of the following Sikh Guru’s in India? [A] Guru Amardas [B] Guru Arjun Dev [C] Guru Ram das [D] None of the above Correct Answer: B [Guru Arjun Dev] 139. Who among the following was Commander-in-Chief, India, between 1902 and 1909 noted for largescale reforms, the greatest of which was the merger of the three armies of the Presidencies into a unified force? [A] Sir Charles Monro [B] Lord Kitchener [C] Beauchamp Duff [D] Canning Correct Answer: B [Lord Kitchener] Explanation: Lord Kitchener, also known for Reconstruction of the disorganized Indian Army against the wishes of the viceroy Lord Curzon. 140. Indian National Congress was founded during whose term among the following in 1885? [A] Lord Canning [B] Lord Dufferin [C] Lord Curzon [D] Lord Bentick Correct Answer: B [Lord Dufferin] 141. Which among the following is Not true about C. Rajagopalachari? [A] He served as Last Governor-General of India [B] He served as the Premier of the Bombay Presidency [C] He founded swatantra party [D] All are correct Correct Answer: B [He served as the Premier of the Bombay Presidency] Explanation: He served as Premier of Madras presidency and Governor of West Bengal. 142. Which among the following is NOT correctly matched? [A] 1929 : Bhagat Singh Threw Bomb at Central Assembly House [B] 1930: Simon Commission came to India [C] 1931: Second Round table Conference [D] All are correct Correct Answer: B [1930: Simon Commission came to India] Explanation: Simon commission came to India in 1928 143. Who among the following is related to” Servants of India Society”? [A] Gopal Krishna Gokhle [B] Dadabhai Nauroji 251 | P a g e

shop.ssbcrack.com

MCQs

INDIAN HISTORY

MCQs

[C] Raja Ram Mohan roy [D] Mahatma Gandhi Correct Answer: A [Gopal Krishna Gokhle] Explanation: Servants of India Society was founded in 1915 by Gopal Krishna Gokhale to build a dedicated group of people for social service and reforms. In the field of famine relief, union organisation, cooperatives and uplift of tribals and depressed, the Society did commendable work. 144. Which of the following policy was launched in 1992 just after the end of the cold war, following the collapse of the Soviet Union? [A] Economic Liberalization [B] Look East Policy [C] National Population Policy [D] National Policy on Education Correct Answer: B [Look East Policy] Explanation: India’s Look East Policy was for the first time pursued by Narsimharao Government in 1992. Under this policy, India focused its attention toward South-East Asia, which was long neglected during the Cold war period. This policy was further cemented by successive prime ministers and now converted into “Act East Policy” by the current government. 145. Where did Netaji Subhash Chandra Bose established provisional Government of Free India ? [A] Singapore [B] Rangoon [C] Malaysia [D] Germany Correct Answer: A [Singapore] Explanation: On 21st October 1943, Subhash set up the Provisional Government of Free India in Singapore. 146. Vinoba Bhave was the first person to offer satyagraha. In which year he offered individual satyagraha? [A] 1939 [B] 1940 [C] 1941 [D] 1942 Correct Answer: B [1940] Explanation: The Congress was in a confused state again after the August Offer. The radicals and leftists wanted to launch a mass Civil Disobedience Movement, but here Gandhi insisted on Individual Satyagraha. The Individual Satyagraha was not to seek independence but to affirm the right of speech. • The other reason of this Satyagraha was that a mass movement may turn violent and he would not like to see the Great Britain embarrassed by such a situation. This view was conveyed to Lord Linlithgow by Gandhi when he met him on September 27, 1940. The non-violence was set as the centerpiece of Individual Satyagraha. This was done by carefully selecting the Satyagrahis. • The first Satyagrahi selected was Acharya Vinoba Bhave, who was sent to Jail when he spoke against the war. • Second Satyagrahi was Jawahar Lal Nehru. • Third was Brahma Datt, one of the inmates of the Gandhi’s Ashram. 252 | P a g e

shop.ssbcrack.com

MCQs

INDIAN HISTORY

MCQs

They all were sent to jails for violating the Defense of India Act. This was followed by a lot of other people. But since it was not a mass movement, it attracted little enthusiasm and in December 1940, Gandhi suspended the movement. The campaign started again in January 1941, this time, thousands of people joined and around 20 thousand people were arrested. 147. Who among the following is / are credited for founding of the Indian National Congress? [A] A.O. Hume [B] A.O. Hume and Dinshaw Edulji Wacha [C] Dadabhai Nauroji, A.O. Hume and Dinshaw Edulji Wacha [D] None of the above Correct Answer: C [Dadabhai Nauroji, A.O. Hume and Dinshaw Edulji Wacha] 148. At which of the following sessions of Congress the resolution of Poorna Swaraj was passed? [A] Bombay [B] Luckonw [C] Lahore [D] Karachi Correct Answer: C [Lahore] Explanation: In the Lahore session of December 1929, Congress passed the Poorna Swaraj resolution. It was the same session in which Jawaharlal Nehru was elected as president of the Congress 149. Under Government of India Act 1935, Congress contested elections in which year? [A] 1936 [B] 1937 [C] 1938 [D] 1939 Correct Answer: B [1937] 150. Which among the following place related to salt agitation? [A] Palampur [B] Dharsana [C] Mehsana [D] None of the above Correct Answer: B [Dharsana] 151. Violence in Burma leads to the rebellion of Saya San was a result of which of the following? [A] Government of India Act 1919 [B] Simon commission [C] Government of India Act 1909 [D] Government of India Act 1935 Correct Answer: B [Simon commission] 152. Which among the following was the most important reason of objection to Simon Commission by Indian nationalists? [A] There was No Indian member in Simon Commission [B] Inexperienced people were sent to look into the state of Indian constitutional affairs [C] Both 1 & 2 [D] None of the above Correct Answer: A [There was No Indian member in Simon Commission] 253 | P a g e

shop.ssbcrack.com

MCQs

INDIAN HISTORY

MCQs

153. Who among the following was greeted with Hartals when he visited India in November 1921? [A] Prince of wales [B] Elizabeth II [C] Edward VIII [D] Simon commission Correct Answer: A [Prince of wales] 154. Champaran Satyagraha was the first to be started but the word Satyagraha was used for the first time in which of the following? [A] Anti Rowlatt agitation [B] agitation against Simon Commission [C] Amritsar Massacre of 1919 [D] Non-cooperation movement Correct Answer: A [Anti Rowlatt agitation] 155. Which of the following dispute made Gandhi ji to undertake a fast for the first time? [A] Minto-Morley Reforms [B] Ahmedabad Mill Strike [C] Punjab Unrest [D] Poona pact Correct Answer: B [Ahmedabad Mill Strike] 156. The impact of the first world war drew two classes of Indian society into the national movement. Who were they? [A] Hindu & Muslims [B] Peasants and Business groups [C] Middle Class and educated groups [D] Sepoys Correct Answer: B [Peasants and Business groups] 157. Which among the following was a reason of eruption of Farazi Movement? [A] attacks of Christian Missionaries on Islam [B] Oppression caused by Hindu Money lenders [C] Oppressive Policy of the Company towards Muslim Tenants [D] None of the above Correct Answer: C [Oppressive Policy of the Company towards Muslim Tenants] 158. Which among the following may not be called a consequence of 1857 revolts? [A] European Troops were increased in India [B] Different Classes of Sepoys were mixed [C] Doctrine of lapse [D] Hindu’s & Muslims began to be appeased equally Correct Answer: D [Hindu’s & Muslims began to be appeased equally] 159. The play Neeldarpan is associated with? [A] Santhal Hool [B] Indigo revolt 254 | P a g e

shop.ssbcrack.com

MCQs

INDIAN HISTORY

MCQs

[C] Pabna Riots [D] Deccan riots Correct Answer: B [Indigo revolt] Explanation: Neel Darpan was a Bengali play written by Dinabandhu Mitra in 1858–1859. The play was essential to Nilbidraha, or Indigo revolt of February–March 1859 in Bengal, when farmers refused to sow indigo in their fields as a protest against exploitative farming under the British Raj. 160. Who among the following attained martyrdom in jail while on hunger strike? [A] Jatin Mukherjee [B] Jatin Das [C] Sachin Sanyal [D] chandra shekar Azad 161. In which year Hindu mahasabha was established? [A] 1910 [B] 1915 [C] 1916 [D] 1918 Correct Answer: B [1915] Explanation: Hindu Mahasabha was founded in 1915 by Madan Mohan Malviya.It worked with Arya Samaj and other Hindu organizations. It was directly link with Rashtriya Swam Sevak Sangh founded in 1925 at Nagpur by K.B.Hegewar.The first All India Hindu Mahasabha Conference was organized at Hardwar in 1915. The Sabha ecame more aggressive after 1929 and started propagating Hindu Rashtra which was totally differ from Gandhiji’s Ram Rajya. 162. Who among the following has written Gandhi’s Emissary ? [A] Arun Shourie [B] Ajoy Bose [C] Sudhir Gosh [D] Kunwar singh Correct Answer: C [Sudhir Gosh] 163. Treaty of yabandu is related to which of the following states ? [A] Punjab [B] Assam [C] Karnataka [D] Odisha Correct Answer: B [Assam] 164. Raja Wodeyar founded which of the following Kingdoms? [A] Mysore [B] Vellore [C] Vijaynagar [D] Bahamani Correct Answer: A [Mysore]

255 | P a g e

shop.ssbcrack.com

MCQs

INDIAN HISTORY

MCQs

165. The East India Company was given a permission to set up a factory in Masulipattanam in which yea [A] 1610 [B] 1613 [C] 1615 [D] 1620 Correct Answer: B [1613] 166. During the revolt of 1857, who among the following was betrayed by a zamindar friend and captured while asleep by Britishers and put to death? [A] Nana Saheb [B] Tantia Tope [C] Khan Bahadur Khan [D] Kunwar singh Correct Answer: B [Tantia Tope] 167. The Government of India recognizes 7-9 January as the Pravasi Bharatiya Divas since 2003 . Which of the following events is known to occur on 9 January? [A] Arrival of Mahatma Gandhi in India from South Africa. [B] Launch of Person of Indian Origin Card (PIO Card) [C] First Global Convention of People of Indian Origin in New York [D] None of the above Correct Answer: A [Arrival of Mahatma Gandhi in India from South Africa.] 168. The second world war commenced in 1939 on the following dates? [A] August 1 [B] September 1 [C] October 1 [D] November 1 Correct Answer: B [September 1] 169. Paunar Ashram is related to which of the following social activist? [A] Vinoba Bhave [B] Baba Amte [C] Swami Sahajanand Saraswati [D] Ambedkar Correct Answer: A [Vinoba Bhave] Explanation: Paunar Ashram was founded by the great social reformer and activist Acharya Vinoba Bhave for the treatment of leprosy patients. 170. Montagu-Chelmsford Reforms which formed the base of Government of India Act 1919, introduced which of the following in India ? [A] Self governance [B] Dyarchy [C] Indian Administrative Services [D] Indian police Services Correct Answer: B [Dyarchy]

256 | P a g e

shop.ssbcrack.com

MCQs

INDIAN HISTORY

MCQs

171. Which of the following Charter acts abolished the monopoly of the East India Company to trade with China? [A] Charter Act of 1793 [B] Charter Act of 1813 [C] Charter Act of 1833 [D] Charter Act of 1853 Correct Answer: C [Charter Act of 1833] 172. Which among the following are called Dharamchakra? [A] The chakra which is a weapon of lord Vishnu in Hindu Mythology [B] The first sermon of Mahavira Jain [C] The chakra which has 24 spokes in the central white band in Indian Flag [D] None of the above Correct Answer: C [The chakra which has 24 spokes in the central white band in Indian Flag] 173. Who among the following referred Mahatma Gandhi as father of Nation? [A] Subhash Chandra Bose [B] Sardar Vallabh Bhai Patel [C] Jawahar lal Nehru [D] Rabindranath Tagore Correct Answer: A [Subhash Chandra Bose] 174. On which date Jallianawala Massacre occurred? [A] August 13, 1919 [B] April 13, 1919 [C] January 13, 1919 [D] June 13, 1919 Correct Answer: B [April 13, 1919] 175. First NAM summit was held in which country? [A] Belgrade 1961 [B] Cairo 1964 [C] Delhi 1961 [D] Algiers1961 Correct Answer: A [Belgrade 1961] Explanation: NAM was founded in Belgrade in 1961, and was largely conceived by India’s first Prime Minister, Jawaharlal Nehru; Indonesia’s first president, Sukarno; Egypt’s second president, Gamal Abdel Nasser; Ghana’s first president Kwame Nkrumah; and Yugoslavia’s president, Josip Broz Tito. 176. The Third battle of Panipat was fought between ? [A] Babur & Ibrahim Lodi [B] Ahmed Shah Abdali & Marathas [C] Akbar & Shershah Suri [D] Shershah Suri & Humayun Correct Answer: B [Ahmed Shah Abdali & Marathas] Explanation: Third battle of Panipat took place on January 14, 1761between Ahmed Shah Abdali & Marathas 257 | P a g e

shop.ssbcrack.com

MCQs

INDIAN HISTORY

MCQs

177. In which year National Liberal Federation was formed in India? [A] 1915 [B] 1918 [C] 1919 [D] 1921 Correct Answer: C [1919] 178. Indian Muslims were allotted reserved seats in the Municipal and District Boards, in the Provincial Councils and in the Imperial Legislature, in which of the following acts? [A] Government of India Act 1909 [B] Government of India Act 1919 [C] Indian Councils Act 1861 [D] Indian Councils Act 1891 Correct Answer: A [Government of India Act 1909] 179. Indian Muslims were allotted reserved seats in the Municipal and District Boards, in the Provincial Councils and in the Imperial Legislature, in which of the following acts? [A] Government of India Act 1909 [B] Government of India Act 1919 [C] Indian Councils Act 1861 [D] Indian Councils Act 1891 Correct Answer: A [Government of India Act 1909] 180. William Digby is known to write on which of the aspects of India ? [A] Indian education [B] Indian society [C] Famines in India [D] Poverty in India Correct Answer: C [Famines in India] Explanation: Digby was based in India during British raj and witnessed to several famines that devastated India, including a disastrous famine in Madras in the mid-1870s. He blamed British policies for fueling and exacerbating the famines, agitating for humanitarian aid from Europe to India. 181. Who has initiated the slogan” Inqlab Zindabad”? [A] Bhagat Singh [B] Chandra shekahar Azad [C] Iqbal [D] Hasrat Mohani Correct Answer: D [Hasrat Mohani] Explanation: “Inquilab Zindabad phrase which translates to “Long Live the Revolution!” was a revolutionary chant during the British rule over India. It was coined by Maulana Hasrat Mohani. 182. Which date is associated with the incident of Chauri Chaura? [A] 4th February 1922 [B] 6th February 1922 [C] 4th August 1922 [D] 6th August 1922 Correct Answer: A [4th February 1922] Explanation: Due to Chauri Chaura incident Gandhiji withdraws NonCooperation movement. 258 | P a g e shop.ssbcrack.com

MCQs

INDIAN HISTORY

MCQs

183. All India Kisan Sabha which was first peasants organization of India was constituted in which year ? [A] 1920 [B] 1936 [C] 1942 [D] 1945 Correct Answer: B [1936] 184. Lord William Bentinck is credited for which of the following in the Indian History ? [A] Education reforms [B] Laws [C] Abolition of sati [D] Local government Correct Answer: C [Abolition of sati] 185. During India’s First War of independence (1857-58), Begum Hazrat Mahal led the revolt from which of the following places? [A] Benaras [B] Lucknow [C] Kanpur [D] Delhi Correct Answer: B [Lucknow] 186. Who among the following is related to Repeal of Vernacular Press Act of 1878? [A] Lord Rippon [B] Lord Lytton [C] Lord Curzon [D] Lord Irwin Correct Answer: A [Lord Rippon] Explanation: The Vernacular press act: 1. It was passed during the times of Lord Lytton 2. It empowered the magistrate to deposit a security, which could be confiscated if the printer violated the Bond 3. It was repealed during times of Lord Ripon 187. Who wrote the book neel darpan? [A] Bankim Chandra Chaterjee [B] Rabindra Nath Tagore [C] Dinbandhu Mitra [D] None of the above Correct Answer: C [Dinbandhu Mitra] 188. Pindari War (1817-1818) is also known as ________? [A] First Anglo-Maratha War [B] Second Anglo-Maratha War [C] Third Anglo-Maratha War [D]First Carnatic war Correct Answer: C [Third Anglo-Maratha War] 259 | P a g e

shop.ssbcrack.com

MCQs

INDIAN HISTORY

MCQs

189.Treaty of Seringapatnam was signed after which of the Anglo-Mysore wars? [A] 1st [B] 2nd [C] 3rd [D] 4th Correct Answer: C [3rd] 190. Who among the following English officer was related to First Anglo-Mysore war? [A] Colonel Smith [B] Warren Hastings [C] Lord Wellesly [D] Jhon simon Correct Answer: A [Colonel Smith] 191. Which of the following Governor Generals was impeached for charges of corruption? [A] Earl Cornwallis [B] Warren Hastings [C] Lord Mayo [D] Lord Minto Correct Answer: B [Warren Hastings] 192. Which of the following Sikh Guru collected all the work of the previous Gurus and dictated it in the form of verses ? [A] Guru Har Govind [B] Guru Har Kishan [C] Guru Arjun Dev [D] Guru Gobind Correct Answer: C [Guru Arjun Dev] 193. The pioneers of Ocean trade with India were______? [A] Dutch [B] Portuguse [C] British [D] French Correct Answer: B [Portuguse] 194. In 1818 Brahmo Samaj founder Rammohan Roy wrote a dissertation on meaning and importance of ________? [A] Sam Veda [B] Yajur Veda [C] Rig Veda [D] Gayatri Mantra Correct Answer: D [Gayatri Mantra] 195. Who had became a member of the Brahmo Samaj in 1856 and founded his own breakaway “Brahmo Samaj of India” in 1866? [A] Dwijendranath Tagore [B] Kesab Chander Sen 260 | P a g e shop.ssbcrack.com

MCQs

INDIAN HISTORY

MCQs

[C] Ishwar Chandra Vidyasagar [D] Raja Ram Mohan Roy Correct Answer: B [Kesab Chander Sen] 196. Satyagraha was started by Gandhi ji in 1919 to protest against the _____________? [A] Salt Law [B] Rowlatt Act [C] Act of 1909 [D] Act of 1919 Correct Answer: B [Rowlatt Act] 197. First Round Table Conference was held during November 1930 January 1931. Who was the Viceroy of India at that time? [A] Lord Reading [B] Lord Irwin [C] Lord Linlithgow [D] Lord wavel Correct Answer: B [Lord Irwin] 198. Lord Wavell was the Viceroy of India at which of the following events? [A] First session of Roundtable conference [B] Cabinet Mission Plan [C] Formation of Congress Ministries in the Provinces [D] Quit India movement Correct Answer: B [Cabinet Mission Plan] 199. By which charter act, financial allocation for education was made for the first time? [A] Charter Act 1773 [B] Charter Act 1793 [C] Charter Act 1813 [D] Charter Act 1833 Correct Answer: C [Charter Act 1813] 200. On February 20, 1947, The British Government declared its intention to quit India by __________? [A] August, 1947 [B] December, 1947 [C] June 1947 [D]June 1948 Correct Answer: D [June 1948] 201. On February 20, 1947, The British Government declared its intention to quit India by __________? [A] August , 1947 [B] December, 1947 [C] June 1947 [D]June 1948 Correct Answer: D [June 1948]

261 | P a g e

shop.ssbcrack.com

MCQs

INDIAN HISTORY

MCQs

202. Nadir Shah ,the ruler of Persia who looted India’s Koh-i-Noor diamond and Peacock Throne, defeated the Mughal Army at Karnal in which year ? [A] 1720 A.D [B] 1739 A.D [C] 1749 A.D [D] 1761 A.D Correct Answer: B [1739 A.D] 203. Who among the following has written Bidrohi, the one of the most famous Bengali poem? [A] Rabindranath Tagore [B] Bankim Chandra Chattopadhhyaya [C] Kazi Nazrul Islam [D] Sambhunath Pandit Correct Answer: C [Kazi Nazrul Islam] Explanation: Bidrohi is one of the most famous Bengali poems, written by Kazi Nazrul Islam in December, 1921. It was first collected in 1922 in a volume titled Agnibeena: the first anthology of Nazrul’s poems. 204. Who among the following established the “Sadar Diwani Adalat”? [A] Warren Hastings [B] Cornwallis [C] John Macpherson [D] Wellesly Correct Answer: B [Cornwallis] 205. Which among the following acts provided for direct control of Indian Affairs by the British Government? [A] Government of India Act 1858 [B] Pitts India Act of 1784 [C] Government of India Act 1909 [D] Indian Councils Act 1892 Correct Answer: A [Government of India Act 1858] 206. An organization formed in Germany in 1914 during World War I by Indian students and political activists residing in the country was [A] Ghadar Party [B] Indian Independence Committee [C] Indian National Army [D] Communist Party of India Correct Answer: B [Indian Independence Committee] 207. Who among the following also launched a Home rule Movement in India, apart from Annie Besant? [A] Arubindo Ghosh [B] Bal Gangadhar Tilak [C] Gopal Krishna Gokhale [D] Madan Mohan Malviya Correct Answer: B [Bal Gangadhar Tilak]

262 | P a g e

shop.ssbcrack.com

MCQs

INDIAN HISTORY

MCQs

208. Who among the following prescribed the separate electorates for India on the basis of the Communal Award in August 1932? [A] Lord Irwin [B] Ramsay MacDonald [C] Winston Churchill [D] None of the above Correct Answer: B [Ramsay MacDonald] Explanation: On August 16, 1932, the British Prime Minister McDonald announced the Communal Award. Thus it is also known as McDonald Award. The Communal Award was basically a proposal on minority representation. 209. During Freedom Struggle of India, which among the following movements started with Dandi ? [A] Swadeshi Movement [B] Noncooperative Movement [C] Civil Disobedience Movement [D] Quit India Movement Correct Answer: C [Civil Disobedience Movement] Explanation: Salt Satyagraha began with the Dandi March on March 12, 1930 and was the part of the first phase of the Civil Disobedience Movement. Gandhi led the Dandi march from Sabarmati Ashram to the sea coast near the village of Dandi. In this journey of 24 days and covering a distance of 390 kilometer, thousands of people joined him. He reached Dandi on April 6, 1930, and broke the salt law. This triggered the Civil Disobedience Movement and millions of Indians jumped in the tumult. 210. Who among the following is the author of “Mahatma Gandhi and His Myths”? [A] Dominique Lepierre [B] Mark Shepard [C] Minoo Masani [D] None of the above Correct Answer: B [Mark Shepard] 211. Who among the following was the first recipient of Bharat Ratna ? [A] CV Raman [B] C. Rajgopalachari [C] Dr. Radhakrishnan [D] Jawahar Lal Nehru Correct Answer: A [CV Raman] Explanation: Bharat Ratna was first awarded in the year 1954. There were 3 recipients in the year 19541. C. Rajagopalachari→ He was the first Governor General of free India. 2. Dr. Sarvepalli Radhakrishnan (1888-1975).→He was the second President of India & the first VicePresident of India. 3. C.V. Raman→He was a noble laureate of India who was awarded the Noble Prize for Physics in the year 1930. However, first ever Indian to receive this award was the famous scientist, Chandrasekhara Venkata Raman. 212. Who among the following was the first Indian to get selected in ICS (Indian Civil Services)? [A] Ras Bihari Bose [B] Satyendra Nath Tagore 263 | P a g e shop.ssbcrack.com

MCQs

INDIAN HISTORY

MCQs

[C] Devendranath Tagore [D] Surendranath Banerjee Correct Answer: B [Satyendra Nath Tagore] Explanation: Satyendranath Tagore was selected for the Indian Civil Service in June, 1863. He completed his probationary training and returned to India in November 1864. He was posted as Judge, Satara after this examination. 213. Librahan Commission was established to probe into which among the following? [A] Anti Sikh Riots of 1984 [B] Babri Mosque Demolition [C] Post Godhara Riots [D] None of the above Correct Answer: B [Babri Mosque Demolition] 214. Who among the following was the author Mahatma-Life of Mohandas Karamchand Gandhi ? [A] D. G. Tendulkar [B] Colonel G. B. Singh [C] Susheela Naiyyar [D] None of the above Correct Answer: A [D. G. Tendulkar] 215. Balwantrai Mehta, a pioneer of Panchayati Raj and a distinguished freedom fighter, was Chief minister of which state? [A] Maharastra [B] Gujarat [C] Andhra Pradesh [D] Madya Pradesh Correct Answer: B [Gujarat] Explanation: BBalwantrai Mehta was one of the legendry freedom fighters of the country who participated in the Bardoli Satyagraha. He is best known as second Chief Minister of Gujarat.He is credited for pioneering the concept the Panchayati Raj in India and also known as Father of Panchayati Raj in India. 216. On which of the following charges, Bal Gangadhar Tilak was arrested in 1908? [A] Murder of Douglas [B] Sedition [C] Opposing the age of Consent Bill [D] Inciting Chapekar Brothers to commit violence Correct Answer: B [Sedition] 217. In which year, States Reorganization Act went into effect? [A] 1949 [B] 1950 [C] 1956 [D] 1958 Correct Answer: C [1956] 218. In 1956 during the Reorganization of the Indian States, the state of Hyderabad was split up between? 264 | P a g e

shop.ssbcrack.com

MCQs

INDIAN HISTORY

MCQs

[A] Andhra Pradesh & Maharastra [B] Andhra Pradesh & Karnataka [C] Andhra Pradesh, Maharastra & Karnataka [D] Andhra Pradesh, Maharastra, Goa & Karnataka Correct Answer: C [Andhra Pradesh, Maharastra & Karnataka] 219. Swaraj party was out come of which among the following incidents? [A] Chauri Chaura [B] Quit India Movement [C] Arrival of Simon Commission [D] Bardoli Satyaghara Correct Answer: A [Chauri Chaura] 220. Which among the following treaty was signed after Battle of Buxar? [A] Treaty of Allahabad [B] Treaty of Carnatic [C] Treaty of Alinagar [D] Treaty of Kanpur Correct Answer: A [Treaty of Allahabad] Explanation: The important outcome of the Battle of Buxar was the Treaty of Allahabad signed on 16 August 1765 between Lord Clive and Mughal Emperor Shah Alam II, who had submitted to the British in the battle. As per this treaty: • Mughal Emperor granted Fiscal Rights (Diwani) or right to administer the territory and collect taxes to the East India Company at Bengal, Bihar and Orissa. Thus the British became the masters of fate of the people of Bihar, Bengal and Orissa and now they would collect the revenue. • In lieu of this Right, the Company gave an annual tribute of 26 Lakh Rupees to the Mughals • The districts of Kora and Allahabad were returned to Mughal Emperor. • Awadh was returned to Shuja-ud-Daulah but Allahabad and Kora was taken from him. • The Nawab of Awadh paid 53 Lakhs rupees of war indemnity to the British. Thus Clive, in person settled the fate of almost half of the Northern India. The fiscal administration of Bengal, Behar, and Orissa and the territorial jurisdiction of the Northern Circars is called the Dual System of Government 221. Which among the following incidents established the supremacy of British over French in India? [A] Battle of Buxar [B] Battle of Wandiwash [C] Carnatic Wars [D] Battle of Chingleput Correct Answer: B [Battle of Wandiwash] Explanation: The British forces under Sir Eyre Coote, defeated the French in the Battle of Wandiwash in 1760 and besieged Pondicherry. After Wandiwash, the French capital of Pondicherry fell to the British in 1761. 222. 1853 is a landmark in India’s Transport because of ? [A] First train [B] First cargo plane [C] First ship [D] None of them Correct Answer: A [First train] 265 | P a g e

shop.ssbcrack.com

MCQs

INDIAN HISTORY

MCQs

223. Le Corbusier was the architect of which city among the following? [A] New Delhi [B] Chandigarh [C] Mumbai [D] Jaipur Correct Answer: B [Chandigarh] Explanation: The best-known architect recruited to the efforts of creating a new India was one of the founders of modernism in architecture, the Swiss Le Corbusier. The showpiece of his genius is Chandigarh, built as the capital of Punjab and Haryana. Outside Chandigarh, Le Corbusier’s most influential structures include: • Sarabhai House, Ahamadabad • Millowner’s Association Building, Ahmedabad. • Interstate Bus Terminal, New Delhi Permanent Exhibition Hall of Prāgati Maidan, New Delhi 224. Baba Amte is known for which of the following social work? [A] Work for Poor & homeless people [B] Work To Save Ganga [C] Work for Leprosy Patients [D] Work for Landless laborers Correct Answer: C [Work for Leprosy Patients] 225. Who was the first Indian to get elected in House of commons? [A] Bhikaji Kama [B] JRD Tata [C] Dadabhai Naoroji [D] Dinshaw Edulji Wacha Correct Answer: C [Dadabhai Naoroji] Explanation: Dadabhai Naoroji is known as mentor of both Gopal Krishna Gokhale and Mahatma Gandhi. His magnum opus ‘‘Poverty and UnBritish Rule in India’’ propounded the ‘‘drain theory’’ He was the first Indian to become a member of the House of Commons on the Liberal Party ticket. He served as Member in the House of Commons from 1892 to 1895. He became the president of INC thrice, in 1886, 1893 and 1906. 226. Under which among the following Governor Generals of India, the Public works department was started in 1848? [A] Lord Dalhousie [B] Lord William Bentinck [C] Lord Wellesley [D] Lord Viscount Canning Correct Answer: A [Lord Dalhousie] 227. Which among the following newspaper established by Mahatma Gandhi which was an important tool for the political movement led by Gandhi and the Natal Indian Congress to fight racial discrimination and win civil rights for the Indian immigrant community in South Africa? [A] Satyagraha [B] Indian Voice 266 | P a g e

shop.ssbcrack.com

MCQs

INDIAN HISTORY

MCQs

[C] Indian opinion [D] Harijan Correct Answer: C [Indian opinion] 228. In which year, India launched it Look East Policy? [A] 1986 [B] 1992 [C] 1994 [D] 1996 Correct Answer: B [1992] Explanation: India’s Look East Policy was for the first time pursued by Narsimharao Government in 1992. Under this policy, India focused its attention toward South-East Asia, which was long neglected during the Cold war period. This policy was further cemented by successive prime ministers and now converted into “Act East Policy” by the current government. 229. Which among the following was the first conglomeration of the scattered princely states , which later became modern Rajasthan after several more mergers? [A] Greater Rajputana [B] Matsya Union [C] United State of Greater Rajasthan [D] Rajputana Union Correct Answer: B [Matsya Union] 230. ‘Rajmala’ is an important source of History of which among the following states of India? [A] Nagaland [B] Assam [C] Tripura [D] Jammu & Kashmir Correct Answer: C [Tripura] Explanation: Rajmala is a chronicle of the kings of Tripura 231. Before its name was shortened by the British into United Provinces in 1935, by which among the following names Modern Uttar Pradesh was known? [A] United Provinces of Northern India [B] Presidency of Agra and Allahabad [C] United Provinces of Agra and Oudh [D] United Provinces of Agra, Allahabad and Oudh Correct Answer: C [United Provinces of Agra and Oudh] 232. Atal Bihari Vajpayee was sworn in as India’s Prime Minister for how many times? [A] 1 [B] 2 [C] 3 [D] 4 Correct Answer: C [3] Explanation: 6 May 1996-01 June 1996, 19 March 1998 -13 October 1999 and 13 October 1999 – 22 May 2004 267 | P a g e shop.ssbcrack.com

MCQs

INDIAN HISTORY

MCQs

233. Who among the following was the first chairman of UPSC? [A] Sir Ross Barker [B] Sir David Petrie [C] Sir Eyre Gorden [D] Sir F.W. Robertson Correct Answer: A [Sir Ross Barker] Explanation: First Public Service Commission was established on October 1, 1926 under the Chairmanship of Sir Ross Barker. UPSC is constitutional body which conducts the prestigious civil services examination to select IAS, IFS and IPS officers among others. It has been established under Article 315 of the Constitution and consists of a Chairman and ten Members; appointed and removed by President. The chairman and members of the Commission hold office for a term if six years or until they attain the age of 65 years, whichever is earlier 234. How many Prime Ministers of India have been awarded Bharat Ratna till date? [A] 3 [B] 4 [C] 5 [D] 7 Correct Answer: D [7] Explanation: So far Seven Prime Ministers of India have been awarded the Bharat Ratna. Atal Bihari Vajapayee is the seventh Prime Minister to receive the award, after (1) Jawaharlal Nehru, (2) Indira Gandhi, (3) Rajiv Gandhi, (4) Morarji Desai, (5) Lal Bahadur Shastri and (6) Gulzarilal Nanda 235. At which among the following places, the Muslim League was founded in 1906? [A] Kolkata [B] Murshidabad [C] Dhaka [D] Lucknow Correct Answer: C [Dhaka] Explanation: In Dec. 1906 “All India Muslim League” was set up under the leadership of Aga Khan, Nawab Salimullah Khan of Dacca and Nawab Mohsin-ul Mulk at Dacca. The League supported the Partition of Bengal and opposed the Swadeshi Movement and demanded the special safeguards for its community and separate electorate of Muslims. 236. Who among the following was Viceroy of India at the time of partition of Bengal (1905)? [A] Lord minto [B] Lord curzon [C] Lord Elgin [D] Lord Chelmsford Correct Answer: B [Lord curzon] 237. Fraser Commission was established for reforms in which among the following in 1902? [A] Army [B] eduction [C] Police [D] Civil service Correct Answer: C [Police] 268 | P a g e shop.ssbcrack.com

MCQs

INDIAN HISTORY

MCQs

Explanation: In 1902-03, a Police Commission was established for the Police reforms under Sir Andrew Frazer and Lord Curzon got much success in the Police intelligence. 238. For reforms in which among the following, Hartog Commission was established? [A] Society [B] Education [C] Civil services [D] Economic reforms Correct Answer: B [Education] 239. Which among the following is the year of famous Queen Victoria’s Proclamation assuming the Government of India? [A] 1857 [B] 1858 [C] 1859 [D] 1860 Correct Answer: B [1858] Explanation: On November 1, 1858, Lord Canning sent forth the royal proclamation in a grand Darbar at Allahabad via which Queen Victoria intimated that she had assumed the direct government of India. This proclamation also set forth the principles on the basis of which India was to be governed in future and the British policy towards people of British-Indian territories and princes 240. Which among the following had an objective of introducing the self-governing institutions to India? [A] Indian Council Act 1909 [B] Mont-Ford Reforms [C] Cabinet Mission [D] Government of India Act 1935 Correct Answer: B [Mont-Ford Reforms] 241. Sadakat Ashram memorial in Patna is dedicated to which former President of India ? [A] Dr. Rajendra Prasad [B] V V Giri [C] Muhammad Hidayatullah [D] Basappa Danappa Jatti Correct Answer: A [Dr. Rajendra Prasad] 242. Mahatama Gandhi had launched his first Satyagraha in India from which among the following places? [A] Kheda [B] Bardoli [C] Champaran [D] Sabarmati Correct Answer: C [Champaran] Explanation: The Champaran Satyagraha of 1917 was Mahatma Gandhi’s first Satyagraha. 243. In which year, Vijay Laxmi Pandit became the first woman President of the United Nations General Assembly (UNGA) ? [A] 1949 [B] 1951 269 | P a g e

shop.ssbcrack.com

MCQs

INDIAN HISTORY

MCQs

[C] 1953 [D] 1959 Correct Answer: C [1953] 244. In which city is the dargah (tomb) of sufi saint Moinuddin Chishti located ? [A] Delhi [B] Ghaziabad [C] Ajmer [D] Gwalior Correct Answer: C [Ajmer] 245. In 1612, at which among the following places, the British established their first factory (trading post) in India ? [A] Cochin [B] Surat [C] Kassim Bazar [D] Hughli Correct Answer: B [Surat] 246. Who among the following has authored the famous book ‘India Wins Freedom’ ? [A] Jawahar Lal Nehru [B] Maulana Abul Kalam Azad [C] Sardar Patel [D] Dr. Rajendra Prasad Correct Answer: B [Maulana Abul Kalam Azad] 247. Gol Ghar, a beehive shaped structure built in 1786 to store grains for the British Army, is located in which city ? [A] Bhopal [B] Patna [C] Varnasi [D] Lucknow Correct Answer: B [Patna] 248. What was the original name of Mirabehn, an associate and disciple of Mahatma Gandhi? [A] Oliver Schriener [B] Millie Graham Polock [C] Madeline Slade [D] Margarate Cousins Correct Answer: C [Madeline Slade] Explanation: Madeleine Slade was in England and was named as Meera Behn by Mahatma Gandhi , whose disciple and associate she was. She was awarded Padma Vibhushan in 1982. 249. Who presided over the first session of the All India Trade Union Congress in 1920 ? [A] Purshottam Das Tandon [B] Lala Lajpat Rai [C] Kasturbhai Lalbhai [D] Govind Vallabh Pant 270 | P a g e shop.ssbcrack.com

MCQs

INDIAN HISTORY

MCQs

Correct Answer: B [Lala Lajpat Rai] Explanation: On October 31, 1920, the first session of the All India Trade Union Congress was held at Bombay under Lala Lajpat Roy, thus marking the beginning of AITUC. 250. The spiritual guru Gadadhar Chattopadhyay is best known with which among the following names ? [A] Upendranath Brahmachari [B] Ramakrishna Paramahamsa [C] Sri Arubindo [D] Swami Prabhupad Correct Answer: B [Ramakrishna Paramahamsa] 251. In 1906, at the founding session of all India Muslim League in Dhaka, who became the first president of the party ? [A] Nawab Salimullah Khan [B] Khwaja Abdul Ghani [C] M. Shamsher Ali [D] Muhammad Ali Bogra Correct Answer: A [Nawab Salimullah Khan] 252. Treaty of Yandabu was signed after which among the following? [A] First Anglo-Burmese War [B] Second Anglo-Burmese War [C] Anglo-Bhutanese War [D] Third Anglo Afghan war Correct Answer: A [First Anglo-Burmese War] 253. In which year, Raja Ram Mohan Roy established the “Atmiya Sabha” a precursor in the socio-religious reforms in Bengal? [A] 1812 [B] 1815 [C] 1823 [D] 1826 Correct Answer: B [1815] 254. Who among the following was the immediate successor of Guru Nanak? [A] Guru Angad [B] Guru Arjun [C] Guru Amardas [D] Guru Ramdas Correct Answer: A [Guru Angad] 255. Who among the following was the immediate successor of Guru Nanak? [A] Guru Angad [B] Guru Arjun [C] Guru Amardas [D] Guru Ramdas Correct Answer: A [Guru Angad] 271 | P a g e

shop.ssbcrack.com

MCQs

INDIAN HISTORY

MCQs

256. At which among the following places, the modern armory was established by Hyder Ali? [A] Mysore [B] Dindigul [C] Dharmapuri [D] Hosur Correct Answer: B [Dindigul] 257. Who started the process of decentralization of finances in India? [A] Lord Rippon [B] Lord Mayo [C] Lord Curzon [D] Lord Minto Correct Answer: B [Lord Mayo] Explanation: Lord Mayo or Lord Naas served as 4th Viceroy of India from 12 January 1869 to 8 February 1872. He started the process of decentralization of finance. In foreign affairs, he followed the policy of non-intervention. He opened up Mayo College in Ajmer for educating children of the aristocratic families. India’s census began during his tenure. He was the first Governor General to be murdered in office by a Pathan Sher Ali in Port Blair. 258. Who founded the Banaras Hindu University ,the largest residential Central University in Asia located in Varanasi [A] Vivekananda [B] Annie Besant [C] Pandit Madan Mohan Malviya [D] Jawahar lal Nehru Correct Answer: C [Pandit Madan Mohan Malviya] Explanation: In 1915, under Lord Hardinge, BHU act was passed by which Pandit Madan Mohan Malviya established the Banaras Hindu University. The idea of this university was mooted as early as 1904. 259. Which act was introduced by British govt to address the short comings of the East India Company? [A] Pitts India Act [B] British Union Act [C] 1707 Act of Union [D] India Act 1784 Correct Answer: A [Pitts India Act] Explanation: Pitt’s India Act 1784 or the East India Company Act 1784 was passed in the British Parliament to rectify the defects of the Regulating Act 1773. It resulted in dual control or joint government in India by Crown in Great Britain and the British East India Company, with crown having ultimate authority. With this act, East India Company’s political functions were differentiated from its commercial activities for the first time. The relationship between company and crown established by this act kept changing with time until the Government of India Act 1858 provided for liquidation of the British East India Company. 260. In which province Muslim League formed their first government after 1946 elections? [A] Bengal [B] Madras [C] Bombay [D] United province Correct Answer: A [Bengal] 272 | P a g e shop.ssbcrack.com

MCQs

INDIAN HISTORY

MCQs

261. Kohinoor Diamond was found probably from which among the following mines? [A] Golconda [B] Kalahandi [C] Panna [D] Bijapur Correct Answer: A [Golconda ] 262. Who among the following established the Dal Khalsa? [A] Guru Gobind Singh [B] Guru Arjun Dev [C] Kapur Singh [D] Bhai Mani Singh Correct Answer: C [Kapur Singh] 263. Who among the following established Tattvabodhini Sabha? [A] Raja Rammohan Roy [B] Debendranath Tagore [C] Keshub Chandra Sen [D] Dwarkanath Tagore Correct Answer: B [Debendranath Tagore] Explanation: Before Debendranath joined the Brahmo Samaj, he had organized a Tattvaranjini Sabhaat Calcutta. The same Sabha was later known as Tattvabodhini Sabha. The objective of Tattvabodhini Sabha was to encourage religious inquiries and disseminate the essence of Upanishads. 264. The name of Baba Ram Singh is associated with which among the following movements? [A] Sanyasi Rebellion [B] Kuka Movement [C] Khaksar Movement [D] Santhal rebellion Correct Answer: B [Kuka Movement] 265. Diwan Mani Ram Datta was a leader of revolutionaries of 1857 in which among the following regions? [A] Gujarat [B] Rajputana [C] Punjab [D] Assam Correct Answer: D [Assam] 266. Nana Saheb lead the revolutionaries of 1857 from which among the following placed? [A] Gwalior [B] Meerut [C] Kanpur [D] Lucknow Correct Answer: C [Kanpur] 267. In which among the following provinces of India, the first signs of unrest during the mutiny of 1857 appeared? 273 | P a g e

shop.ssbcrack.com

MCQs

INDIAN HISTORY

MCQs

[A] Madras [B] Bombay [C] Punjab [D] Bengal, Bihar and Odisha Correct Answer: D [Bengal, Bihar and Odisha] 268. Every year in January, India and Pakistan exchange their list of nuclear installations through their diplomatic missions under an agreement on prohibition of attacks against nuclear installation and facility. In which year, such agreement was signed? [A] 1986 [B] 1988 [C] 1990 [D] 1992 Correct Answer: B [1988] Explanation: The agreement, which was signed on December 31, 1988 and came into force on January 27, 1991, says the two countries inform each other of nuclear installations and facilities to be covered under the agreement on January 1 of every calendar year. 269. What was civil disobedience Movement? [A] Active refusal to obey certain laws, demands and commands of a government [B] Revolution [C] Freedom from slavery [D] All the above Correct Answer: A [Active refusal to obey certain laws, demands and commands of a government] 270. Who represented India in the 2nd round table conference? [A] Mahatma Gandhi [B] B. R. Ambedkar [C] Jawaharlal Nehru [D] Muhammad Ali Jinnah Correct Answer: A [Mahatma Gandhi] Explanation: The Second Round Conference opened on September 7, 1931. Gandhi represented Indian National Congress and Sarojini Naidu represented Indian women. Madan Mohan Malaviya, Ghanshyam Das Birla, Muhammad Iqbal, Sir Mirza Ismail Diwan of Mysore, S K Dutta and Sir Syed Ali Imam were other people that attended the conference. This conference saw an overwhelming number of Indian delegates. These included loyalists, communalists, careerists, big landlords, representatives of the princes etc. 271. Who said that Gandhi may die, but Gandhism shall remain forever ? [A] Mahatma Gandhi [B] Jawaharlal Nehru [C] Subhash Chandra Bose [D] Rabindranath Tagore Correct Answer: A [Mahatma Gandhi] Explanation: Gandhiji said Gandhi may die, but Gandhism shall remain forever in 1931 Karachi session. 272. Who among the following has written the “Civil Disobedience” that had a profound impact on Gandhi? [A] Leo Tolstoy [B] John Ruskin 274 | P a g e shop.ssbcrack.com

MCQs

INDIAN HISTORY

MCQs

[C] Karl Marx [D] Henry David Thoreau Correct Answer: D [Henry David Thoreau] Explanation: Henry David Thoreau was an American author, poet, and philosopher, who is best known for his book Walden, a reflection upon simple living in natural surroundings, and his essay Civil Disobedience, an argument for individual resistance to civil government in moral opposition to an unjust state. Thoreau’s philosophy of civil disobedience influenced the political thoughts and actions of such later figures as Leo Tolstoy, Mahatma Gandhi, and Martin Luther King, Jr. 273. Which among the following personality is known for publishing the Mooknayak (Leader of the Silent) in Mumbai? [A] GG Agarkar [B] K. M. Munshi [C] Dr. Bhimrao Ambedkar [D] Shyamji Krishna Varma Correct Answer: C [Dr. Bhimrao Ambedkar] 274. Who among the following established the Swadhyay Movement and the Swadhyay Parivar organization? [A] Achyutananda [B] Satsvarupa dasa Goswami [C] Swami Sivananda [D] Pandurang Shastri Athavale Correct Answer: D [Pandurang Shastri Athavale] 275. Who among the following is known as the “Saint of Dakhineshwar”? [A] Ramkrishna Paramahansa [B] Swami Vivekananda [C] Swami Dayanand Saraswati [D] Saint Tikaram Correct Answer: A [Ramkrishna Paramahansa] 276. The first factory by the Dutch was established at which among the following places? [A] Surat [B] Machilipatnam [C] Negapatnam [D] Pulicat Correct Answer: B [Machilipatnam] Explanation: The Dutch East India Company was created in 1602 as “United East India Company” and its first permanent trading post was in Indonesia. In India, they established the first factory in Masulipattanam in 1605, followed by Pulicat in 1610, Surat in 1616, Bimilipatam in 1641 and Chinsura in 1653. 277. Who among the following were the first to establish ” Printing Press” in India? [A] British [B] Portuguese [C] Dutch [D] French Correct Answer: B [Portuguese] 275 | P a g e shop.ssbcrack.com

MCQs

INDIAN HISTORY

MCQs

278. Which Portuguese Viceroy followed the “Blue Water” policy in India? [A] Afonso de Albuquerque [B] Francisco de Almeida [C] Diogo Lopes de Sequeira [D] None of them Correct Answer: B [Francisco de Almeida] Explanation: In 1503 Francisco de Almeida was appointed the First Governor and Viceroy of the Portuguese India. 279. who among the following has written the book” Communist Manifesto”? [A] Friedrich Engels [B] Karl Marx [C] Leonid Gatovsky [D] Vladimir Lenin Correct Answer: B [Karl Marx] 280. First President of Indian National Congress? [A] George Yule [B] Dadabhai Naoroji [C] womesh Chandra Banarjee [D] Romesh Chunder Dutt Correct Answer: C [womesh Chandra Banarjee] Explanation: Under the Presidentship of Woomesh Chandra Banerjee, the first meeting of Indian National Congress was held in Bombay in 1885. The meeting was attended by 72 English-educated people that included advocates, traders and zamindars. 281. Who was the Last Viceroy of India? [A] Richard Wellesley [B] Warren Hastings [C] Lord Mountbatten [D] Lord Northrook Correct Answer: C [Lord Mountbatten] Explanation: Lord Mountbatten served as last Viceroy of India from 12 February 1947 – 15 August 1947; and then first Governor General of Free India from 15 August 1947 – 21 June 1948. During his tenure, India got independence in the form of two dominions of India Pakistan. Two Boundary Commissions were set, Pakistan raided Kashmir and Mahatma Gandhi was shot dead. 282. Which among the following was Last Hindu Empire of India? [A] Maratha Empire [B] Durrani Empire [C] Kingdom of Mysore [D] Sikh Empire Correct Answer: A [Maratha Empire] 283. What the foundations of Gandhi an principles ? [A] Capitalist [B] Socialist 276 | P a g e

shop.ssbcrack.com

MCQs

INDIAN HISTORY

MCQs

[C] Ethical [D] Religious Correct Answer: C [Ethical] 284. In which year, former PM Rajiv Gandhi was assassinated? [A] 1989 [B] 1990 [C] 1991 [D] 1992 Correct Answer: C [1991] 285. Who among the following has written the book Bahubivah? [A] Ishwar Chand Vidyasagar [B] Sarat Chandra Chattopadhyay [C] Bankim Chandra Chattopadhyay [D] Sri Aurobindo Correct Answer: A [Ishwar Chand Vidyasagar] 286. In which year Calcutta became Capital of British India? [A] 1768 [B] 1770 [C] 1772 [D] 1776 Correct Answer: C [1772] 287. Year 1757 is remembered to which among the following battles? [A] Battle of Plassey [B] Battle of Chinsura [C] Battle of Buxar [D] Battle of Poona Correct Answer: A [Battle of Plassey] 288. In which year, IAS (Not ICS) exam was conducted in India for the first time? [A] 1949 [B] 1950 [C] 1951 [D] 1953 Correct Answer: B [1950] 289. In which year, Sabarmati Ashram was established? [A] 1914 [B] 1915 [C] 1916 [D] 1917 Correct Answer: B [1915]

277 | P a g e

shop.ssbcrack.com

MCQs

INDIAN HISTORY

MCQs

290. In which year, Partition of Bengal was annulled? [A] 1909 [B] 1910 [C] 1911 [D] 1912 Correct Answer: C [1911] 291. The Namgyal Monarchy used to rule which among the following current states of India, once upon a time? [A] Arunachal Pradesh [B] Megahalaya [C] Nagaland [D] Sikkim Correct Answer: D [Sikkim] 292. The name of “Operation Trojan Horse” has been linked to martyrdom of which among the following freedom fighters? [A] Bhagat Singh, Rajguru and Sukhdev, [B] Chandra Shekhar Azad [C] Khudi Ram Bose [D] Madan Lal Dhingra Correct Answer: A [Bhagat Singh, Rajguru and Sukhdev,] 293. During which of the following years, Port Blair was the headquarters of the Azad Hind government under Subhash Chandra Bose? [A] 1941-42 [B] 1942-43 [C] 1943-44 [D] 1944-45 Correct Answer: C [1943-44] 294. Takht Harmandir Sahib, the birthplace of Guru Gobind Singh is located in which among the following cities? [A] Amritsar [B] Bhatianda [C] Patna [D] Ropar Correct Answer: C [Patna] 295. Which among the following freedom fighter is known to have written “Gita Rahasya ” ? [A] Gopal Krishna Gokhle [B] Bal Gangadhar Tilak [C] Bidhan Chandra Roy [D] KM Munshi Correct Answer: B [Bal Gangadhar Tilak]

278 | P a g e

shop.ssbcrack.com

MCQs

INDIAN HISTORY

MCQs

296. In which year, Netaji Subhash Chandra Bose was conferred Bharat Ratna? [A] 1988 [B] 1991 [C] 1992 [D] 1996 Correct Answer: C [1992] 297. In which among the following states of India, the parallel Government was running between 1942 to 1944 during the Quit India Movement? [A] Odisha [B] Bihar [C] West Bengal [D] Bombay Correct Answer: C [West Bengal] 298. By which among the following acts, the ban on missionaries in India was lifted? [A] Charter Act 1793 [B] Charter Act 1813 [C] Charter Act 1853 [D] Charter Act 1773 Correct Answer: B [Charter Act 1813] 299. On what date Indian Rebellion of 1857 started? [A] 10th May [B] 16th April [C] 26th June [D] 11th July Correct Answer: A [10th May] Explanation: The Indian Rebellion of 1857 began as a mutiny of sepoys of the East India Company’s army on 10th May 1857, in the town of Meerut. It soon escalated into other mutinies and civil rebellions largely in the upper Gangetic plain and central India, with major hostilities confined to present-day Uttar Pradesh, Bihar, northern Madhya Pradesh, and the Delhi region 300. Which among the following organizations was started by Behramji Malabari in Bombay? [A] Bombay Asiatic Society [B] Seva Sadan [C] East India Association [D] None of the above Correct Answer: B [Seva Sadan] Explanation: The Parsi social reformer Behramji M. Malabari campaigned all his life against child marriage and “enforced widowhood”. He founded the Seva Sadan as a social reform and humanitarian organization in 1885. The Seva Sadan specialized in the care of socially discarded and exploited women of all castes, providing education, welfare and medical services. 301. In which year East India Company was brought under British parliamentary control? [A] 1773 [B] 1793 279 | P a g e

shop.ssbcrack.com

MCQs

INDIAN HISTORY

MCQs

[C] 1813 [D] 1833 Correct Answer: A [1773] 302. Which governor of Madras had signed a treaty with Tipu? [A] George MaCartney [B] Sir Archibald Campbell [C] William Medows [D] Sir Charles Oakeley Correct Answer: A [George MaCartney] 303. Who among the following were the first to come to India to establish trade relationships? [A] Portuguese [B] Dutch [C] French [D] British Correct Answer: A [Portuguese ] 304. Goa was captured by Albuqurque from ruler of which among the following states? [A] Bijapur [B] Golconda [C] Mysore [D] Vijaynagar Correct Answer: A [Bijapur] Explanation: Afonso de Albuquerque was the second governor of the Portuguese India and is known as founder of Portuguese colonial empire in India. In 1510 Albuqurque acquired Goa from Bijapur 305. Which of the following organizations provided medical help to the Turkish troops in the Balkan War? [A] International Council of Nurses [B] Pax Romana [C] Global Humanitarian Forum [D] Red Crescent Society Correct Answer: D [Red Crescent Society] Explanation: Red Crescent Society: Worldwide humanitarian organization providing assistance without discrimination as to nationality, race, religious beliefs, class or political opinions. It provided medical help to the Turkish troops in the Balkan War. 306. The first Individual Satyagrahi, Acharya Vinoba Bhave offered Satyagraha in which among the following way? [A] By not paying taxes [B] By burning British Flag [C] By making an antiwar speech [D] By making a speech against the Viceroy of India Correct Answer: C [By making an antiwar speech] 307. Who was the first Muslim President of Indian National Congress? [A] Hakim Azmal Khan [B] Abul Kalam Azad 280 | P a g e

shop.ssbcrack.com

MCQs

INDIAN HISTORY

MCQs

[C] Rafi Ahmad Kidwai [D] Badruddin Taiyabji Correct Answer: D [Badruddin Taiyabji] Explanation: Badruddin Taiyabji became the Ist Indian Barrister in Bombay; became the 2nd Indian Chief Justice; was the founding member of Bombay presidency association and INC and also presided over the 3rd congress session in Madras in 1887. 308. Who was elected the President of Indian National Congress in the Surat Session 1907 famous for Surat Split? [A] Dr. Rash Bihari Ghosh [B] Lala Lajpat Rai [C] Dadabhai Naoroji [D] Pherozeshah Mehta Correct Answer: A [Dr. Rash Bihari Ghosh] Explanation: The growing differences between the Moderates and the Extremists came at Surat Session 1907, when against the wishes of Extremists who preferred Lala Lajpat Rai to be the President; Dr. Rash Bihari Ghosh was elected as the Congress President. The Extremists left the Congress. Result was the Congress remained under the control of the Moderates. 309. Who was elected the President of Indian National Congress in the Surat Session 1907 famous for Surat Split? [A] Dr. Rash Bihari Ghosh [B] Lala Lajpat Rai [C] Dadabhai Naoroji [D] Pherozeshah Mehta Correct Answer: A [Dr. Rash Bihari Ghosh] Explanation: The growing differences between the Moderates and the Extremists came at Surat Session 1907, when against the wishes of Extremists who preferred Lala Lajpat Rai to be the President ; Dr. Rash Bihari Ghosh was elected as the Congress President. The Extremists left the Congress. Result was the Congress remained under the control of the Moderates. 310. During British Era, the Duke Memorandum became the basis of which among the following? [A] Nehru Report [B] Mont-Ford Reforms [C] Indian Councils Act 1909 [D] Government of India Act 1935 Correct Answer: B [Mont-Ford Reforms] Explanation: Duke Memorandum is associated with Sir William Duke, a member of the English Round Table Group and he had formulated a scheme which eventually became the basis of Joint Report of Montague and Chelmsford. 311. The Battle of Chillianwalla was a part of? [A] Anglo-Maratha Wars [B] Anglo-Carnatic Wars [C] Anglo-French Wars [D] Anglo-Sikh Wars Correct Answer: D [Anglo-Sikh Wars] 281 | P a g e

shop.ssbcrack.com

MCQs

INDIAN HISTORY

MCQs

Explanation: Battle of Chillianwalla was fought on 13 January 1849 during the Second Anglo-Sikh War. Sikhs fought British in this battle under Shersingh. 312. In which year Jahangir issued the charter to East India Company for establishing the trade establishments in India? [A] 1615 [B] 1616 [C] 1617 [D] 1618 Correct Answer: C [1617] 313. August 9 1925 is known in the Indian History for which of the following incidences? [A] Chauri Chaura incidence [B] Kakori Train Robbery [C] Jallianwallah Bagh Incidence [D] The seize of Cawnpore Correct Answer: B [Kakori Train Robbery ] 314. In which year, first nationwide elections were held in Afghanistan? [A] 1952 [B] 1958 [C] 1965 [D] 1968 Correct Answer: C [1965] 315. Which among the following rulers of Afghanistan was a Reformer Ruler and known to have established diplomatic relations of Modern Afghanistan with many countries? [A] Abdur Rahman [B] Amanullah Khan [C] Habibullah Kalakani [D] None of them Correct Answer: B [Amanullah Khan ] 316. Who among the following was Amir of Afghanistan, when Durand line was demarcated? [A] Dost Mohammad [B] Akbar Khan [C] Abdur Rahman [D] Habibulah Correct Answer: C [Abdur Rahman ] 317. At which of the following places Dost Mohammad Khan was deported as a prisoner after the first Anglo Afghan War? [A] Surat [B] Dehradun [C] Bombay [D] Calcutta Correct Answer: D [Calcutta] 282 | P a g e

shop.ssbcrack.com

MCQs

INDIAN HISTORY

MCQs

318. Who among the following was the first president of Constituent Assembly? [A] Dr. Rajendra Prasad [B] Sachidanand Sinha [C] Jawahar Lal Nehru [D] B R Ambedkar Correct Answer: B [Sachidanand Sinha ] 319. The Cabinet assembly which was elected for an undivided India met for the first time on______? [A] June 3, 1946 [B] June 6 1946 [C] June 9, 1946 [D] June 12, 1946 Correct Answer: C [June 9, 1946 ] 320. In which year constitution for India was drafted by Nehru Committee? [A] 1917 [B] 1926 [C] 1928 [D] 1929 Correct Answer: C [1928] 321. On which date Indian Independence Act 1947 got Royal Assent? [A] 18 June 1947 [B] 18 July 1947 [C] 18 March 1947 [D] 18 April 1947 Correct Answer: B [18 July 1947 ] 322. By which of the following for the first time, British government recognized the “Right of Dominion” for India? [A] August Declaration [B] August Offer [C] Cripps Mission [D] Montague Declaration Correct Answer: C [Cripps Mission ] 323. In the proposed federal polity of the Government of India act 1935, how many members had to be elected on communal basis for the Upper House of the bicameral legislature at the center? [A] 156 [B] 150 [C] 104 [D] 125 Correct Answer: B [150] Explanation: The upper house was called Council of States and it consisted of 260 members. Out of these 260 members 156 were to represent the provinces and 104 to the native states. Out of the 156 which were trepresent the provinces, 150 were to be elected on communal basis. Seats reserved for Hindus, Muslim Sikhs, were to be filled by direct elections and Seats reserved for Indian Christians, Anglo Indians and Europeans was to be filled by indirect method of a electoral college consisting of their representativmembers. 283 | P a g e shop.ssbcrack.com

MCQs

INDIAN HISTORY

MCQs

324. Consider the following features of Government of India act 1935: 1. provision for Federation of India 2. Compulsory accession to Federation of India 3. Partial Reorganization of provinces Which among the above features hold correct ? [A] 1 only [B] 1 & 2 [C] 2 & 3 [D] 1 & 3 Correct Answer: D [1 & 3 ] Explanation: accession to this federation was voluntary 325. Which among the following act provided for the Public Service Commission in India for the first time? [A] Government of India Act 1919 [B] Indian Councils act 1909 [C] Government of India Act 1935 [D] Government of India act 1858 Correct Answer: A [Government of India Act 1919 ] 326. Which among the following acts had a separate preamble? [A] Government of India Act 1919 [B] Indian Councils Act 1909 [C] Indian Councils Act 1904 [D] Indian Councils Act 1892 Correct Answer: A [Government of India Act 1919 ] 327. “Increasing association of Indians in every branch of the administration and the gradual development of self-governing institutions with a view to the progressive realization of responsible government in India as an integral part of the British Empire”. The above statement is known as _____? [A] Montague declaration [B] August Offer [C] Queen Victoria’s Proclamation [D] None of them Correct Answer: A [Montague declaration ] Explanation: Please note that it was also known as August Declaration but NOT August Offer. 328. Which among the following led to Jallianwalla Bagh massacre? [A] The Arms Act [B] The Rowlatt Act [C] The Public Safety Act [D] Vernacular Press Act Correct Answer: B [The Rowlatt Act ] 329. The government of India act 1935 provided for: 1. Provincial autonomy 2. Establishment of federal court 284 | P a g e

shop.ssbcrack.com

MCQs

INDIAN HISTORY

MCQs

3. all India federation at the center Which among the above hold correct ? [A] 1 only [B] 1 & 2 only [C] 2 & 3 only [D] all of them Correct Answer: D [all of them ] 330. The Montague-Chelmsford Report formed the basis of _______? [A] Indian Councils act 1909 [B] The Government of India Act 1919 [C] The Government of India act 1935 [D] The Indian Independence Act 1947 Correct Answer: B [The Government of India Act 1919 ] 331. Which among the following was NOT a provision of Charter act of 1833? [A] Trading activities of the East India Company were to be abolished [B] The designation of the supreme authority was to be changed as the Governor General of India in Council [C] All law making powers to be conferred on Governor General in council [D] An Indian to be appointed as law member in the Governor General in Council Correct Answer: D [An Indian to be appointed as law member in the Governor General in Council ] 332. Which of the following acts of the British India strengthened the Viceroy’s authority over his executive councils by substituting “portfolio” or departmental system for Corporate Functioning? [A] Indian Councils Act 1861 [B] Government of India Act 1858 [C] Indian Councils Act 1892 [D] Indian councils Act 1909 Correct Answer: A [Indian Councils Act 1861 ] 333. Which of the following acts of the British India strengthened the Viceroy’s authority over his executive councils by substituting “portfolio” or departmental system for Corporate Functioning? [A] Indian Councils Act 1861 [B] Government of India Act 1858 [C] Indian Councils Act 1892 [D] Indian councils Act 1909 Correct Answer: A [Indian Councils Act 1861 ] 334. After returning from South Africa, which among the following was the first successful satyagraha of Mahatma Gandhi? [A] Chauri-Chaura [B] Dandi [C] Champaran [D] Bardoli Correct Answer: C [Champaran ]

285 | P a g e

shop.ssbcrack.com

MCQs

INDIAN HISTORY

MCQs

335. Who among the following was the governor general who followed a spirited “Forward” Policy towards Afghanistan? [A] Lord Dufferin [B] Lord Mayo [C] Lord Elgin [D] Lord Ellenborough Correct Answer: D [Lord Ellenborough ] 336. Where is located the “Hazur Sahib” Gurudwara, which is one of the five takhts of Sikhism? [A] Amritsar [B] Patna [C] Nanded [D] Bhatinda Correct Answer: C [Nanded ] Explanation: The location of the five Takhts is as follows: Akal Takht, at Amritsar, Keshgar Sahib, at Anandpur Sahib, Punjab, Patna Sahib, at Patna, Hazur Sahib, at Nanded and Damdama Sahib, at Talwandi Sabo, Bhatinda or Bathinda. 337. What was the number of districts in Bombay Presidency? [A] 23 [B] 24 [C] 25 [D] 26 Correct Answer: D [26] 338. Which among the following was also known as Western Presidency in early times of East India Company? [A] Bombay [B] Surat [C] Panaji [D] Pune Correct Answer: B [Surat] 339. Which among the following act was known as the Anarchical and Revolutionary Crime act 1919? [A] Indian Arms Act [B] Pitts India Act [C] Ilbert Bill [D] Rowlatt Act Correct Answer: D [Rowlatt Act ] 340. Who among the following was the Vice president of the Interim Government formed in 1946? [A] Dr. S Radha Krishnan [B] Jawahar Lal Nehru [C] C Rajgopalachari [D] Dr. Rajendra Prasad Correct Answer: B [Jawahar Lal Nehru ]

286 | P a g e

shop.ssbcrack.com

MCQs

INDIAN HISTORY

MCQs

341. Which among the following acts is also known to be a beginning of the parliamentary System in India? [A] Indian Councils Act 1892 [B] Indian Councils Act 1904 [C] Indian Councils Act 1909 [D] Government of India Act 1919 Correct Answer: A [Indian Councils Act 1892 ] Explanation: The act was 1892 can be said to be a First step towards the beginning of thparliamentary system in India, where the members are authorized to ask questions. Indian Councils a 1892 can also be said to introduce the principle of representation 342. Who among the following is known to have started portfolio system in India? [A] Lord Dalhousie [B] Lord Canning [C] Lord Elgin [D] Sir John Lawrence Correct Answer: B [Lord Canning] 343. In which year “Poorna Swarajya” resolution was adopted by the Indian National Congress? [A] 1927 [B] 1928 [C] 1929 [D] 1930 Correct Answer: C [1929] 344. Father of renaissance of Western India was__? [A] B.M. Malabari [B] M.G. Ranade [C] R. G. Bhandarkar [D] K.T. Telang Correct Answer: B [M.G. Ranade ] Explanation: Justice Mahadev Govind Ranade (1842-1901) was a distinguished Indian scholar, social reformer and author, sometimes called a Father of renaissance of Western India. He was one of the founding members of the Indian National Congress . He established the “Widow Marriage Association” in 1861 to encourage and popularize it Ranade founded the Poona Sarvajanik Sabha in 1870, to represent the Government, the aspiration of the people. Known to be the mentor and political guru of famous freedom fighters Gopal Krishna Gokhale, and Bal Gangadhar Tilak. He was against caste system, untouchability and was a strong supporter of widow-remarriage. 345. Who is often called as Hindu Luther of Northern India ? [A] Dayanand Saraswati [B] Ishwar Chandra Vidyasagar [C] Radhakant Dev [D] Keshav Chandra Sen Correct Answer: B [Ishwar Chandra Vidyasagar] Explanation: Ishwar Chandra Vidyasagar was a great 19th century Bengali scholar, reformer, writer and philanthropist, whose ideas remain relevant even in modern India. He had devoted his life to improving 287 | P a g e

shop.ssbcrack.com

MCQs

INDIAN HISTORY

MCQs

the status of Hindu widows and encouraging remarriage. The outcome of these efforts was the Hindu Widow Remarriage Act of 1856. That’s why he has come to be called the Hindu Luther of Northern India. 346. Who inspired the young Bengal Movement in the 19th century ? [A] Ram Tanu Lahiri [B] Henry Vivian Derozio [C] Rasik Kumar Malik [D] Piyare Chand Mitra Correct Answer: B [Henry Vivian Derozio ] Explanation: Henry Louis Vivian Derozio (1809 –1831) was assistant headmaster of Hindu College, Kolkata, a radical thinker and one of the first Indian educators to disseminate Western learning and science among the young men of Bengal. He constantly encouraged them to think freely, to question and not to accept anything blindly. His teachings inspired the development of the spirit of liberty, equality and freedom. His activities brought about intellectual revolution in Bengal. It was called the Young Bengal Movement and his students, also known as Derozians, were fiery patriots. 347. Who is regarded as “Maker of Modern India”? [A] M. G. Ranade [B] Mahatma Gandhi [C] Keshav Chandra Sen [D] Ram Mohan Roy Correct Answer: D [Ram Mohan Roy ] Explanation: Raja Rammohan Roy has come to be called the ‘Maker of Modern India’. He was the main force behind introduction of the western education and English language in India. He advocated the study of English, Science, Western Medicine and Technology. He spent his money on a college to promote these studies. He was the founder of the Brahmo Samaj and a great leader of social reform. It was as a result of his persistent campaign that the custom of Sati was declared illegal in Bengal in 1829 A.D. By Lord William Bentick. He was the chief advocate of the modern process of education and the scientific learning. 348. Which of the following cities was capital of Maharaja Ranjit Singh? [A] Amritsar [B] Patiala [C] Lahore [D] Kapurthala Correct Answer: C [Lahore ] Explanation: Maharaja Ranjit Singh (called “The Lion of the Punjab”) (1780-1839) was a Sikh ruler of the Punjab. His tomb is located in Lahore, Pakistan. He is remembered for uniting the Punjab as a strong state and his possession of the Koh-i-noor diamond. He took the title of Maharaja on April 12, 1801 (to coincide with Baisakhi day), with Lahore having served as his capital from 1799. 349. As per Warren Hastings reforms, which among the following court was set up as appeal court in criminal cases? [A] Circuit court [B] Provincial Court [C] Sadar Diwani Adalat [D] Sadar Nizamat Adalat Correct Answer: D [Sadar Nizamat Adalat ] 288 | P a g e

shop.ssbcrack.com

MCQs

INDIAN HISTORY

MCQs

Explanation: The old system of justice was very simple as zamindar decided all petty cases. This was open to abuse and gave enough scope for the rich to oppress the poor. Hastings set upon himself to reform the judicial system. He established two courts in each districts, the Diwani Adalat to decide civil cases and the Faujdari Adalat to try criminal cases. In the Diwani Adalat, the Collector was to preside assisted by this ‘native dewan’. The Faujdari Adalat was to be presided over by the Qazi or Mufti of the district and two maulavis subject to the supervision of the Collector. In addition to these, two superior courts were established at Calcutta – Sadar Diwani Adalat, as a Court of Appeal in civil cases and Sadar Nizamat Adalat to hear criminal appeals. 350. “The most unqualified blunder committed in the whole history of the British India” This comment by JJ McLeod points to which among the following wars? [A] First Anglo Afghan War [B] Second Anglo-Afghan War [C] Third Anglo-Afghan War [D] First Anglo Sikh War Correct Answer: A [First Anglo Afghan War ] Explanation: First Anglo-Afghan War (1839 to 1842) is also known as Auckland’s Folly. It was a stupe project of Lord Auckland which resulted in the death of thousands of British (Indian) soldiers and waste of Crores of Rupees. 351. Which of the following act formally made the Governor General of Bengal as Governor General of India? [A] Regulating act 1773 [B] Pitts India act 1784 [C] Charter Act 1813 [D] Charter act 1833 Correct Answer: D [Charter act 1833 ] Explanation: The Charter Act of 1833 made the Governor General of Bengal the Governor General of British India and all financial and administrative powers were centralized in the hands of Governor General-in-Council. 352. Which among the following included the administrative /Judicial reforms of Warren Hastings? 1. Establishment of a Board of Revenue at Calcutta 2. Abolishing the judicial functions of the Zamindars 3. Appointment of Indian Judges in Criminal Courts Select the correct option from the codes given below: [A] Only 1 & 2 [B] Only 2 & 3 [C] Only 1 & 3 [D] 1, 2 & 3 Correct Answer: D [1, 2 & 3 ] 353. Under which among the following treaties, Clive had secured the grant of the diwani of Bengal, Bihar and Orissa from Mughal Emperor Shah Alam? [A] Treaty of Allahabad [B] Treaty of Buxar [C] Treaty of Bassein [D] Treaty of Sugauli 289 | P a g e shop.ssbcrack.com

MCQs

INDIAN HISTORY

MCQs

Correct Answer: A [Treaty of Allahabad] Explanation: Clive had secured the grant of the diwani of Bengal, Bihar and Orissa from Mughal Emperor Shah Alam under Treaty of Allahabad 1765. 354. Clive’s dual administration: 1. gave rise to immediate conflict between British and Nawab 2. led to Anglo Maratha clash 3. separated power from responsibility Which among the above is / are correct statements? [A] Only 1 & 2 [B] Only 2 & 3 [C] Only 3 [D] 1, 2 & 3 Correct Answer: C [Only 3 ] Explanation: The first two statements should be reversed to understand what happened exactly.Since there was a dual administration in place, it clearly defined the respective position of the Nawab and the East India Company. This left no chance of near future conflicts between the two.Further, the direct assumption of responsibility of administration of English Company might have to come in conflict with the rising power of the Marathas. Third statement is correct.The system separated power from responsibility. The English got the right of collecting the revenue but they had no responsibility for the internal administration. The condition of Bengal worsened on account of the double government. The famine of 1770 and the epidemic that followed accounted for one third of population of Bengal. The English Company did nothing for the relief of the people and remained a silent spectator to the appalling distress of the people. The dual system which deserved all possible condemnation was finally abolished by Warren Hastings in 1772. 355. Which among the following is / are correct statements with respect to the French East India Company? 1. It was a private concern 2. It was chartered by King Louis XIV 3. It was abolished immediately after French Revolution Select the correct option from the codes given below: [A] Only 1 & 2 [B] Only 2 & 3 [C] Only 1 & 3 [D] Only 2 Correct Answer: D [Only 2 ] Explanation: The first statement is not correct because the French East India Company was government backed concern while the English East India Company was a private concern. The second statement is correct. Third statement is not correct because French East India Company was abolished in 1769, two decades earlier than the French Revolution. 356. With reference to the relative position of French and the Britishers on the eve of Carnatic wars, consider the following observations: 1. The position of the English East India Company was superior to that of the French East India Company 2. The Naval Power of Frech was superior to the British Which among the above is / are correct statements? 290 | P a g e

shop.ssbcrack.com

MCQs

INDIAN HISTORY

MCQs

[A] Only 1 [B] Only 2 [C] Both 1 & 2 [D] Neither 1 nor 2 Correct Answer: A [Only 1] Explanation: The second statement is not correct because the sea power of the English was superior to that of the French. The merchant fleets of the English were bigger and maderegular voyages. 357. Which among the following was / were the consequences of the Third Battle of Panipat? 1. Decentralization of Maratha’s Power 2. Consolidation of East India Company in north-western India 3. Attack of Nadir Shah Select the correct option from the codes given below: [A] Only 1 [B] Only 1 & 2 [C] Only 1 & 3 [D] 1, 2 & 3 Correct Answer: A [Only 1 ] Explanation: First statement is correct. The third battle of Panipat shattered the dream of Marathas to establish their supremacy in whole India. Second statement is not correct because the third battle of Panipat gave opportunity to East India Company to establish and consolidate their rule in Bihar and Bengal (not north-west India as mentioned in question) and to challenge Marathas and other Indian powers and finally to establish their rule in India. The third statement is also incorrect because attack of Nadir Shah pre-dates third battle of Panipat. 358. Who were the first to start a joint stock company to trade with India? [A] Portuguese [B] Dutch [C] French [D] Danish Correct Answer: B [Dutch ] Explanation: To maximize profits, the Dutch East India Company established the world’s first stock market in Amsterdam on March 20, 1602, in which investors could speculate on commodity futures and buy stock in the trading company. It was the Dutch East India Company to start a joint stock company to trade with India 359. Who was the first Indian ruler to join the subsidiary Alliance? [A] The Nawab of Oudh [B] The Nizam of Hyderabad [C] Sultan of Madurai [D] The king of Travancore Correct Answer: B [The Nizam of Hyderabad ] Explanation: The Subsidiary Alliance System was used by Lord Wellesley (Governor General of India) to bring the Indian states within the boundary of the British political power. Under this doctrine , The Nizam of Hyderabad , the feeblest of all rulers was the first Indian ruler to accept the Subsidiary Alliance in 1798 A.D and came under the British Protection.

291 | P a g e

shop.ssbcrack.com

MCQs

INDIAN HISTORY

MCQs

360. The only licensed flag production unit in India in located at which among the following places? [A] Mysuru [B] Hubli [C] Dharwad [D] Nagpur Correct Answer: B [Hubli ] Explanation: Karnataka Khadi Gramodyoga Samyukta Sangha in Hubli is the only licensed flag production unit in India. 361. Which among the following organizations were founded by Pandit Madan Mohan Malviya? 1. Banaras Hindu University 2. Pragya Hindu Samaj 3. Hindu Mahasabha 4. Bharat Dharma Mahamandal Chose the correct answer from the codes given below: [A] Only 1 & 2 [B] Only 2 & 4 [C] Only 1 & 3 [D] 1, 2, 3 & 4 Correct Answer: C [Only 1 & 3 ] Explanation: The Hindu Mahasabha was founded in 1914 in Amritsar and established its headquarters in Haridwar. Amongst its early leaders was the prominent nationalist and educationalist Pandit Madan Mohan Malaviya, who founded the Benaras Hindu University, and the Punjabi populist Lala Lajpat Rai. Banaras Hindu University was established in 1915. Malviya was instrumental in the coming together of Hindu Mahasabha and Arya Samaj in 1923 to make common cause on issues like cow slaughter and reconversion. He was also closely associated with bodies like Pragya Hindu Samaj, Bharat Dharma Mahamandal and Sanatana Dharma Mahasabha. 362. With reference to Atal Bihari Vajpayee, which among the following is NOT a correct statement? [A] He was one of the founders of Bhartiya Jan Sangh [B] He served as political secretary to Dr. Syama Prasad Mookerjee [C] He is the first person to deliver a speech to the United Nations General Assembly in Hindi. [D] All the above are correct statements Correct Answer: A [He was one of the founders of Bhartiya Jan Sangh ] Explanation: The first statement is not correct. Bhartiya Jan Sangh was founded by Dr. Syama Prasad Mookerjee. 363. Who among the following had founded the Central Hindu College at Varanasi, which was later converted into Banaras Hindu University? [A] Madan Mohan Malviya [B] Annie Besant [C] Bal Gangadhar Tilak [D] Jamna Lal Bajaj Correct Answer: A [Madan Mohan Malviya ] Explanation: Madan Mohan Malviya had established BHU in 1915 with the support of some leaders, mainly Annie Besant. She had founded the Central Hindu College in 1898 in Varanasi and this college was later gifted to Pandit Madan Mohan Malviya for the later to get it converted into Banaras Hindu 292 | P a g e

shop.ssbcrack.com

MCQs

INDIAN HISTORY

MCQs

University. Also notable points is that foundation for the main campus of BHU was laid by Lord Hardinge on 4 February 1916, on occasion of Vasant Panchami. 364. The revolt of 1857 was described as a ‘National Rising’ by which of the following political leaders of Britain? [A] Canning [B] Gladstone [C] Palmerstone [D] Disraeli Correct Answer: D [Disraeli] Explanation: Benjamin Disraeli was leader of Conservative Party and opposition leader in the House of Commons. He described the revolt of 1857 as ‘a national rising’. 365. Which among the following tribal leaders was regarded as an incarnation of God and father of the world ? [A] Nanak Bhil [B] Jagia Bhagat [C] Birsa Munda [D] Siddhu Santha Correct Answer: C [Birsa Munda] Explanation: Birsa Munda was an Indian tribal freedom fighter and a folk hero, who belonged to the Munda tribe, and was behind the best known Millenarian movement of India that rose in the tribal belt of modern day Bihar, and Jharkhand .Birsa and his disciples set themselves the task of serving the faminestricken and sick people. He became a legend during his lifetime and was called “Dharati Aba” (father of the earth) and was worshipped as “Birsa Bhagwan”. 366. The play ‘Neeldarpan’ is associated with which among the following revolts? [A] Santhal Revolt [B] Pabna Riots [C] Indigo Revolts [D] Champaran Satyagraha Correct Answer: C [Indigo Revolts] Explanation: Neel Darpan was a Bengali play written by Dinabandhu Mitra in 1858–1859. The play was essential to Nilbidraha, or Indigo revolt of February–March 1859 in Bengal, when farmers refused to sow indigo in their fields as a protest against exploitative farming under the British Raj. 367. Kunwar Singh was a leader of the revolt of 1857 in which among the following modern states? [A] Bihar [B] Jharkhand [C] Rajasthan [D] Uttarakhand Correct Answer: A [Bihar] Explanation: Babu Veer Kunwar Singh, one of the leaders of the Indian Rebellion of 1857 belonged to a royal Kshatriya house of Jagdispur, currently a part of Bhojpur district, Bihar. 368. Which among the following leaders was instrumental in bringing the Extremists and the Moderates together at the Lucknow session of 1916 ? 293 | P a g e

shop.ssbcrack.com

MCQs

INDIAN HISTORY

MCQs

[A] Gopal Krishna Gokhale [B] Bal Gangadhar Tilak [C] Annie Besant [D] Mahatma Gandhi Correct Answer: C [Annie Besant] Explanation: Moderate and Extremist split in Surat Session 1907 and reunited in 1916 Session due to efforts of Annie Besant. Annie Besant (1 October 1847 – 1933) was a prominent British socialist, Theosophist, women’s rights activist, writer and orator and supporter of Irish and Indian self-rule. 369. On the suggestion of Rabindra Nath Tagore, the date of partition of Bengal (October 16, 1905) was celebrated as__? [A] Rakhi Bandhan Day [B] Brotherhood Day [C] Solidarity Day [D] Black Day Correct Answer: A [Rakhi Bandhan Day] 370. The Governor General of India at the time of formation of the Indian National Congress was __? [A] Lord Chelmsford [B] Lord Dalhousie [C] Lord Dufferin [D] None of these Correct Answer: C [Lord Dufferin] Explanation: Lord Dufferin (1826-1902) was the Governor General and Viceroy of India from 1884 to 1888). He had succeeded Lord Ripon in December 1884 and was known as one of the most successful diplomats of his time. Foundation of Indian National Congress by A O Hume in 1885 and Third AngloBurmese War of 1885 are two important events of his period. The Third Anglo-Burmese War resulted in the annexation of upper Burma. As a result, his tenure saw the final extinction of Burma as an independent power. 371. Raja Yoga, Karma Yoga, Bhakti Yoga and Jnana Yoga are the works of which among the following social reformers of India? [A] M. G. Ranade [B] Swami Vivekanand [C] Ramkrishna Paramhansa [D] Raja Ram Mohan Roy Correct Answer: B [Swami Vivekanand] 372. Which among the following was the most important reason for social and religious reforms in nineteenth century? [A] Scientific inventions [B] Industrial revolution [C] Western education and awakening [D] Influence of Newspapers Correct Answer: C [Western education and awakening] 373. With reference to the freedom struggle of India, who among the following was not a part of the “Bombay Triumvirate”? 294 | P a g e shop.ssbcrack.com

MCQs

INDIAN HISTORY

MCQs

[A] Dadabhai Naoroji [B] K.T. Telang [C] Pherozeshah Mehta [D] Badruddin Tyabji Correct Answer: A [Dadabhai Naoroji] Explanation: Bombay Triumvirate or the Three Stars of Bombay’s public life included Badruddin Tyabji, Pherozeshah Mehta and K.T. Telang . All these three veterans had started the Bombay Presidency Association in 1885. 374. Who wrote ‘Indian War of Independence, 1857′? [A] R.C Majumdar [B] V.D. Savarkar [C] S.B. Chaudhary [D] S.N. Sen Correct Answer: B [V.D. Savarkar] Explanation: V.D. Savarkar : He was the proponent of liberty as the ultimate ideal. Savarkar was a poet, writer and playwright. He was associated with the India House and founded student societies including Abhinav Bharat Society and the Free India Society. Savarkar published ‘The Indian War of Independence’ about the Indian rebellion of 1857 that was banned by British authorities. 375. Dalhousie’s worst political blunder was ___? [A] Annexation of Punjab [B] Occupation of lower Burma [C] Abolition of the ‘Doctrine of Lapse’ [D] Annexation of Oudh Correct Answer: D [Annexation of Oudh] Explanation: Annexation of Oudh in February 1856 was a very important decision taken by Lord Dalhousie. But the annexation of Oudh was surely the company’s and Dalhousie’s worst blunder. It proved disadvantageous for the English as the people of Oudh strongly participated in the Revolt of 1857. The English used their giant`s strength and had injured their reputation in the eyes of the Indians. During the Revolt of 1857, the sepoys of Oudh had brought about havoc and had added to the difficulties of the British. It is said that the forced abduction of Wajid Ali Shah and annexation of Oudh were offences against good faith and public conscience. 376. The Congress is in reality a civil war without arms. The above statement was made by ___? [A] W. Wedderburn [B] Sir Saiyid Ahmad Khan [C] D.W. Bethune [D] Lord Dufferin Correct Answer: B [Sir Saiyid Ahmad Khan] Explanation: The above statement was made by Sir Saiyid Ahmad Khan, who denounced congress in blunt terms. “The Congress is in reality,” he said, “a civil war without arms. The object of a civil war is to determine in whose hands the rule of the country shall rest.” 377. Who among the following founded Theosophical Society in USA? [A] Dr. Annie Besant [B] A. O. Hume 295 | P a g e

shop.ssbcrack.com

MCQs

INDIAN HISTORY

MCQs

[C] Tilak and Gokhale [D] Madam Blavatsky and Olcott Correct Answer: D [Madam Blavatsky and Olcott] Explanation: Madam HP Blavatsky, a Russian woman & Col. H.S Olcott, an American, founded the Theosophical Society in Newyork in 1875. They advocated the revival & strengthening of ancient religions of Hinduism. 378. The first President of the Indian Home Rule League (1916) was__? [A] Joseph Bapista [B] Annie Besant [C] N.C. Kelkar [D] B.G. Tilak Correct Answer: A [Joseph Bapista] Explanation: Indian Home Rule League of Bal Gangadhar Tilak was launched in April 1916 and Home Rule League in Sept 1916 by Annie Besant. Joseph Bapista was the first President of the Indian Home Rule League established in April 1916. 379. The founder of Satya Shodhak Samaj was ___? [A] Atmaram Pandurang [B] Gopal Hari Deshmukh [C] M. G. Ranade [D] Jyotiba Phule Correct Answer: D [Jyotiba Phule] Explanation: Jyotiba Phule was an activist, thinker, social reformer, writer, philosopher, theologist, scholar, editor and revolutionary from Maharashtra founded the Satya Shodhak Samaj in 1873 in Maharshtra. 380. Who among the following is known as Father of Muslim Renaissance in Bengal? [A] Syed Ahmad Khan [B] Ameer Ali [C] Nawab Abdul Latif Khan [D] Nawab Samiullah Khan Correct Answer: C [Nawab Abdul Latif Khan] Explanation: Nawab Abdul Latif Khan: (1828-1893)a prominent personality of mid 19th century Bengal, the pioneer of Muslim modernization and the architect of the Muslim Renaissance, was one of those great men who appeared as saviours of their frustrated, humiliated, demoralized and disorganised fellow countrymen under colonial rule. His chief contribution was in the field of education. He was among the first to understand that young Bengali Muslims should receive modern education. He understood that the Muslims of Bengal had fallen behind in everything because of their prejudices against modern education. He devoted his whole life to removing this self-destructive prejudice from their minds. 381. Who among the following was not one of the founding fathers of the All India Muslim League? [A] Nawab Moshin-ul-Mulk [B] Maulana Abul KalamAzad [C] Aga Khan [D] Nawab Salimullah of Dacca Correct Answer: B [Maulana Abul KalamAzad] 296 | P a g e

shop.ssbcrack.com

MCQs

INDIAN HISTORY

MCQs

Explanation: In Dec. 1906 “All India Muslim League” was set up under the leadership of Aga Khan, Nawab Salimullah Khan of Dacca and Nawab Mohsin-ul Mulk at Dacca. The League supported the Partition of Bengal and opposed the Swadeshi Movement and demanded the special safeguards for its community and separate electorate of Muslims. 382. Which was the only session of Indian National Congress, presided by Mahatma Gandhi? [A] Allahabad [B] Guwahati [C] Belgaum [D] Kakinada Correct Answer: C [Belgaum] Explanation: Gandhiji became the president of Indian National Congress in 1924 at the Belgaum session of Indian National Congress. 383. The resolution of Swadeshi was adopted in which session of Congress? [A] Madras session of 1903 [B] Bombay session of 1904 [C] Benaras session of 1905 [D] Calcutta session of 1906 Correct Answer: D [Calcutta session of 1906] Explanation: The resolution of Swadeshi was adopted in 1906 Calcutta session of the Indian National Congress. This session was headed by Dada Bhai Naoroji. Prior to this, in Banaras 1905 session, a resolution of boycott of British goods was put forward. However, it was 1906 session at Calcutta in which four resolutions on self-government, boycott movement, Swadeshi and national education were passed by the Congress. Dadabhai Naoroji said in his presidential speech: “we do not ask for favours we want only justice. Instead of going into any further divisions or details of our rights as British citizens the whole matter can be comprised in one word – self-government or Swaraj like that of the United Kingdom or colonies.” 384. President of Indian National Congress at the time of independence was___? [A] J. B. Kripalani [B] Maulana Abul Kalam Azad [C] Dr. Rajendra Prasad [D] Jawahar Lal Nehru Correct Answer: A [J. B. Kripalani] Explanation: J.B. Kripalani was the President of Indian National Congress at the time of independence. Acharya (scholar) Jiwantram Bhagwandas Kripalani was a Gandhian Socialist, environmentalist, mystic and freedom fighter, noted for his incorruptibility and determination. 385. Who is known as Father of Indian Unrest ? [A] Dadabhai Nauroji [B] G. K. Gokhale [C] Mahatma Gandhi [D] Bal Gangadhar Tilak Correct Answer: D [Bal Gangadhar Tilak] Explanation: Bal Gangadhar Tilak joined congress in 1890. Valentine Chirol called him “Father of Indian Unrest”, who first of all demanded complete “Swarajya”. 297 | P a g e

shop.ssbcrack.com

MCQs

INDIAN HISTORY

MCQs

386. The first Englishmen to preside over a Congress session was __? [A] George Yule [B] Dufferin [C] W. Wedderburn [D] None of the above Correct Answer: A [George Yule] Explanation: George Yule was a Scottish businessman who became the fourth President of INC in 1888,the first non-indian to hold that office. 387. Which among the following organizations merged with Congress in 1886? [A] East Indian Association [B] Indian National Association [C] British Indian Association [D] Indian League Correct Answer: B [Indian National Association] Explanation: The Second session of the Indian National Congress was held in 1886 in Calcutta. The President of the session was Dadabhai Naoroji. In this session the Indian Association merged with the Indian National Congress. (The Indian National Association was the founded by Surendranath Banerjee and Anand Mohan Bose in 1876.). 388. The British empire is rotten to the core, corrupt in every direction and tyrannical and mean. The above statement was made by __? [A] Mrs. Annie Besant [B] W. Digby [C] Sister Nivedita [D] William Wedderburn Correct Answer: C [Sister Nivedita] Explanation: Sister Nivedita : Born as Margaret Elizabeth Noble, she was more popularly known as sister Nivedita. She was a Scots-Irish social worker, author, teacher and a disciple of Swami Vivekananda. She was described as a real lioness by Vivekananda, ‘Lokmata’ (the mother of the people) by Rabindra Nath Tagore and Agnishikha (the flame of fire) by Aurobindo Ghosh. In England she was known as ‘The Champion for India’. Many of her letters written between 1900-1905 to her friend Miss Macleod reveal her hatred for British rule. One such letter writes “The British empire is rotten to the core – corrupt in every direction, and tyrannical and mean. 389. Which among the following was the venue of the INA trial? [A] Calcutta High Court [B] Supreme Court [C] Federal Court [D] Red Fort Correct Answer: D [Red Fort] Explanation: I.N.A. (Indian National Army) trial : P.K.Sehgal,Shah Nawaj Khan and Gurubaksh Singh Dhillon were put on trial at the Red Fort in Nov 1945. The chief defence advocate during INA trial was Bhulabhai Desai. 390. Which among the following was the most revolutionary secret organization of Bengal? [A] Abhinava Bharat [B] Mitra Mela 298 | P a g e shop.ssbcrack.com

MCQs

INDIAN HISTORY

MCQs

[C] Ghadar Party [D] Anushilan Samiti Correct Answer: D [Anushilan Samiti] Explanation: Anushilan Samiti: The most revolutionary secret organization of Bengal was founded by Pramath Nath Mitra in 1902. 391. The Grand Old Man of Indian politics was__? [A] Bipin Chandra Pal [B] Dadabhai Naoroji [C] Surendra Nath Bannerjee [D] Rasbehari Bose Correct Answer: B [Dadabhai Naoroji] Explanation: Dadabhai Naoroji is known as mentor of both Gopal Krishna Gokhale and Mahatma Gandhi. His magnum opus ‘‘Poverty and UnBritish Rule in India’’ propounded the ‘‘drain theory’’ He was the first Indian to become a member of the House of Commons on the Liberal Party ticket. He became the president of INC thrice, in 1886, 1893 and 1906. 392. Moderates and the Extremists in the Indian National Congress split in 1907 at __? [A] Surat [B] Calcutta [C] Nagpur [D] Madras Correct Answer: A [Surat] Explanation: The growing differences between the Moderates and the Extremists came at Surat Session 1907,when against the wishes of Extremists who preferred Lala Laj Pat rai to be the President ; Dr. Raj Bhirai Ghosh was elected as the Congress President.The Extremists left the Congress. After the surat Split, most of the extremist leaders including Lal,Bal,Pal were arrested and were given long term imprisonment. 393. Who among the following defended Aurbindo Ghosh in the Alipore conspiracy case? [A] Tej Bahadur Sapru [B] Motilal Nehru [C] Jawahar Lal Nehru [D] Chitranjan Das Correct Answer: D [Chitranjan Das] Explanation: Chitta Ranjan Das (Deshbandhu) (1870-1925). A great nationalist and famous jurist, he defended Aurobindo Ghose in the Alipore Conspiracy Case (1908) and was the defense counsel in the Dacca Conspiracy Case. He was the elected President of the Congress session held in Ahmedabad in 1921. He was a founder of the Swarajya Party. He presided over the All-India Trade Union Congress at Lahore in 1923 and at Ahmedabad in 1924. 394. Which city is known for the “Jhanda Satyagraha or Flag Satyagraha ” and observance of the All India Flag Day on June 18, 1923? [A] Kanpur [B] Nagpur [C] Nainital [D] Bhopal Correct Answer: B [Nagpur] 299 | P a g e

shop.ssbcrack.com

MCQs

INDIAN HISTORY

MCQs

Explanation: With the observance of the All India Flag Day on June 18, 1923, the Jhanda Satyagraha assumed national complexion with Nagpur as its headquarters and “Nagpur Chalo” as its slogan. 395. Which among the following was the venue of All India Khilafat Conference, 1919? [A] Lucknow [B] Delhi [C] Aligarh [D] Porbandar Correct Answer: B [Delhi] Explanation: All India Khilafat Conference: In November 1919, a joint conference of the Muslims and Hindus was called at Delhi in pursuance of the Muslim League President Fazl-ul-Haq. Gandhi ji suggested to start the non –cooperation movement which was opposed by Jinnah. In December 1919, the Khilafat Conference held its second session. The third Khilafat Conference was held in February 1920 at Bombay. 396. Who among the following were the main leaders of Khilafat Movement? [A] Syed Ahmad Khan and Agha khan [B] Mohammad Ali and Shaukat Ali [C] Muhammad Iqbal and Salimullah Khan [D] Muhammad Ali Jinnah and Sikandar Hayat Khan Correct Answer: B [Mohammad Ali and Shaukat Ali] Explanation: The main object of the Khilafat Movement was to force the British Government to change its attitude towards Turkey and restore the Turkish Sultan (Khalifa) to his former position. A Khilafat committee was formed under the leadership of Ali brothers, Maulana Azad, Hakim Ajmal Khan, and Hasrat Mohani. 397. Which among the following marks Mahatma Gandhi’s first fast unto death? [A] Kheda Satyagraha [B] Champaran Satyagraha [C] Ahmedabad Mill Strike [D] Jallianwala Bagh tragedy Correct Answer: C [Ahmedabad Mill Strike] Explanation: In March 1918, under the leadership of Gandhi, there was a strike in the cotton mills in Ahmedabad. It was at this juncture that Gandhi began the ‘first’ of his seventeen ‘fasts unto death’ on 15 March, 1918 398. Who among the following is said to have hatched the Delhi Conspiracy 1912? [A] Rasbehari Bose [B] Bhai Paramanand [C] Sachindranath Sanyal [D] Sohan Lal Pathak Correct Answer: A [Rasbehari Bose] Explanation: Delhi conspiracy refers to a conspiracy in 1912 to assassinate the then Viceroy of India, Lord Hardinge, on the occasion of transferring the capital of British India from Calcutta to New Delhi. On December 23rd 1912, when the possession of Lord Hardinge reached Chandni Chowk (Delhi), a bomb aimed at Hardinge ended up killing a man to his right and 20 other spectators. Basanta Kumar Bisbas, who threw the bomb disguised as a lady was arrested and hanged in Ambala jail. It is said that the Delhi Conspiracy was hatched by Ras Bihari Bose, but was never proved. 300 | P a g e

shop.ssbcrack.com

MCQs

INDIAN HISTORY

MCQs

399. Who among the following established the Bhil Seva Mandal in 1922? [A] Narain Malhar Joshi [B] Amritlal Vitthaldas Thakkar [C] Jyotiba Phule [D] Baba Amte Correct Answer: B [Amritlal Vitthaldas Thakkar] Explanation: Amritlal Vithaldas Thakkar, popularly known as Thakkar Bapa He was an Indian social worker who worked for upliftment of tribal people in Gujarat state in India. In 1922, he founded the Bhil Seva Mandal. 400. Who among the following established the Ved Samaj in Madras in 1864? [A] Keshabchandra Sen [B] Lokhitwadi [C] Shibnath Shastri [D] Debendranath Tagore Correct Answer: A [Keshabchandra Sen] Explanation: Reform in Southern India Brahmo Samaj under the leadership of Keshabchandra Sen made its appearance in 1864 in Madras with the name Ved Samaj. 401. The magna carta of western education system in India is___? [A] The report of the Committee of Public Instruction, 1823 [B] The Charter Act of 1833 [C] Report of the Hunter Commission, 1862 [D] Wood’s Despatch, 1854 Correct Answer: D [Wood’s Despatch, 1854] Explanation: Wood’s Despatch on Education in 1854 laid the foundation of Indian educational system and the establishments of Universities in Kolkata, Mumbai and Chennai. The despatch came to be considered as the Magna Carta of English education in India. The Scheme of education, as proposed by Wood`s Despatch, envisaged a co-ordinated system of education through out the country. 402. Who among the following preached the doctrine of “One religion, one caste and one God for mankind“? [A] Jyotiba Phule [B] Vivekananda [C] Sri Narayan Guru [D] Dr. B. R. Ambedkar Correct Answer: C [Sri Narayan Guru] Explanation: Sri Narayana Guru preached the doctrine of ‘One caste, One religion, One God.’ Its worth note that one of his athiest disciples, Sahadaran Ayyapan, changed into ‘no religion, no caste and no God for mankind. 403. The leader of revolt of 1857 in Lucknow was __? [A] Tatya Tope [B] Maulvi Ahmadullah Shah [C] Birjis Qadir [D] Begum Hazrat Mahal Correct Answer: D [Begum Hazrat Mahal] 301 | P a g e

shop.ssbcrack.com

MCQs

INDIAN HISTORY

MCQs

Explanation: Begum Hazrat Mahal: The wife of Nawab Wazid Ali Shah of Awadh. She ruled on behalf of her 11-year-old son Birjis Qadar .and led the revolt of 1857 in Lucknow. She refused to accept the pension offered to her by the British and choose to die unmourned in Nepal. 404. Which among the following was NOT annexed into the East India territories via the Doctrine of Lapse? [A] Satara [B] Nagpur [C] Jhansi [D] Mysore Correct Answer: D [Mysore] Explanation: The company took over the princely states of Satara (1848), Jaitpur and Sambalpur (1849), Nagpur and Jhansi (1854), Tanjore and Arcot (1855) and Awadh (Oudh, 1856, with the reason that the ruler was not ruling properly) and Udaipur using this doctrine. Mysore was not among them. 405. Who among the following entered into a triple alliance against Haider immediately before the first Anglo-Mysore war of 1767-69? 1. The English 2. Nizam of Hyderabad 3. The Marathas 4. Raja of Travancore Choose the correct option from the codes given below: [A] 1,2,3 [B] 1,3,4 [C] 1,2,4 [D] 2,3,4 Correct Answer: A [1,2,3] Explanation: In 1766 the British, the Marathas, and the Nizam of Hyderabad entered into a triple alliance against Haider. However, Haider soon bought off the Marathas. The Nizam abandoned the war in 1768, leaving the British to face Haider Ali alone. The latter attacked Arcot and reached the outskirts of Madras. He dictated peace on the basis of the status quo. The English also agreed to help Haider Ali against any third party invasion in future. 406. Which among the following was the first Presidency of British East India Company in India ? [A] Madras [B] Masulipattam [C] Surat [D] Hungli Correct Answer: C [Surat] Explanation: Surat was the first Presidency of British East India Company in India. The other Presidency of the Company at that time in the east was at Bantam in Java where Captain Lancaster had, earlier, during 1601-1603, established a factory of the Company. 407. Which among the following apparently impressed Jahangir to issue a farman in 1613 A.D. to the English to establish a factory at Surat? [A] Reconciliation between the English and Portuguese [B] A secret offer of naval help to the Mughal emperor to oust the Portuguese [C] A heavy dose of bribe to Nur Jahan [D] The defeat of Portuguese naval squadrons by the English 302 | P a g e shop.ssbcrack.com

MCQs

INDIAN HISTORY

MCQs

Correct Answer: D [The defeat of Portuguese naval squadrons by the English] Explanation: In 1613 AD, Jahangir issued a farman permitting English to establish a factory permanently at Surat. In 1612, the newly formed company had sent four ships under Captain Thomas Best to trade with India. These were mercantile ships but were capable of defending themselves. The defeated the Portuguse, who were alread there in Surat for at least 100 years. This defeat apparently so impressed the Emperor Jehangir that he granted the British squadron a firman to trade. 408. On which among the following occasions, Bombay was handed over to Britishers by the Portuguese? [A] Freedom of Portuguese from the control of Spain [B] Marriage of Charles II with the Portuguese princess Catherine of Braganza [C] Crushing of Spanish Armada by British in 1588 [D] The Treaty of Madrid in 1630 Correct Answer: B [Marriage of Charles II with the Portuguese princess Catherine of Braganza] Explanation: Marriage of Charles II with the Portuguese princess Catherine of Braganza was the occasion of handing over of Bombay to Britishers by the Portuguese (as part of dowry ). On 21 September 1668, the Royal Charter of 27 March 1668, led to the transfer of Bombay from Charles II to the British East India Company for an annual rent of £10. 409. Who among the following was the French explorer who visited court of Emperor Shah Jahan and left a detailed account of Takht-i-Taus (Peacock throne) ? [A] Jean-Baptiste Tavernier [B] Geronimo Verroneo [C] Pierre-Jean Grosley [D] Jean-Paul Dubois Correct Answer: A [Jean-Baptiste Tavernier] Explanation: Jean-Baptiste Tavernier,a French jeweller and traveller of the Mughal period has left a detailed account of Takht-i-Taus (Peacock throne). Peacock throne was a dazzling and spectacular display of Mughal architecture. Its construction was undertaken by the Emperor Shah Jahan and he personally spent a great deal of time and energy in the designing of the Throne. 410. Which among the following was the venue of the Second Round Table Conference at London? [A] St. James Palace [B] Kingsley Palace [C] Buckingham Palace [D] 10, Downing Street Correct Answer: A [St. James Palace] Explanation: Second Round Table Conference 1931 : It was held in London during the viceroyalty of Lord Willingdon during Sept- Dec. 1931 and Gandhi ji attended as the soul representative of Congress. But the conference was failed because Gandhiji didn’t’ agree on the policy of communal representation. Gandhi at the Second Round Conference, St. James Palace London. 411. “Springing Tiger: A Study of a Revolutionary” is a biographical work on __? [A] Bhagat Singh [B] Chandrashekhar Azad [C] Subhas Chandra Bose [D] Shyamji Krishna Verma Correct Answer: C [Subhas Chandra Bose] 303 | P a g e

shop.ssbcrack.com

MCQs

INDIAN HISTORY

MCQs

Explanation: Springing Tiger is the work of Hugh Toye about Subhash Chandra Bose. This book presents the little known facts about the World War II and the Anglo-Indian relations during those periods. The author vividly describes the life, philosophy, idealism, nationalism and political astuteness of Subhash Chandra Bose. 412. Consider the following events of Indian National Movement. 1. Gandhi Irwin Pact 2. Poona Pact 3. Karachi Session of Indian National Congress. 4. Individual Satyagraha Select the correct chronological order of the events from the codes given below. Codes: [A] 1, 3, 2, 4 [B] 2, 3, 4, 1 [C] 3, 4, 2, 1 [D] 4, 3, 2, 1 Correct Answer: A [1, 3, 2, 4] Explanation: Gandhi Irwin Pact signed on March 5,1931. Karachi Session of Indian National Congress held on March 26-31,1931. Poona Pact: agreement between Gandhi And BR Ambedkar on September 25,1932. Individual Satyagraha was started on 1940-41. 413. The first Indian woman to preside a session of Indian National Congress was__? [A] Rajkumari Amrit Kaur [B] Vijaya Lakshmi Pandit [C] Aruna Asaf Ali [D] Sarojini Naidu Correct Answer: D [Sarojini Naidu] Explanation: Sarojini Naidu, popularly known as ‘Nightingle of India’ was the second woman (After Annie Besant) and first Indian woman to become congress president. The Presided the Kanpur session in 1925. She was also the first woman governor of an Indian state ( United Provinces of Agra and Oudh). She was also a great poetess whose romanticism charmed people in India and Europe. Some of her literary works include the Golden Threshold (1905), The Broken Wing (1917), The Bird of Time and The Magic Lute. 414. The Congress and Muslim League had most cordial relationships in between which among the following years? [A] 1906 to 1916 [B] 1916 to 1922 [C] 1922 to 1928 [D] 1928 to 1934 Correct Answer: B [1916 to 1922] 415. Kumaran Asan is associated with the social renaissance in which among the following current states? [A] Kerala [B] Tamil Nadu [C] Andhra Pradesh [D] None of the above Correct Answer: A [Kerala] 304 | P a g e shop.ssbcrack.com

MCQs

INDIAN HISTORY

MCQs

Explanation: Kumaran Asan (1873–1924), also known as Mahakavi Kumaran Asan, was one of the triumvirate poets of Kerala, South India. He was also a philosopher, a social reformer and a disciple of Sree Narayana Guru. 416. The annulment of Partition of Bengal was done by __? [A] Lord Curzon [B] Lord Minto [C] Lord Hardinge [D] Lord Chelmsford Correct Answer: C [Lord Hardinge] 417. Which among the following regions was most affected by the Revolution of 1857? [A] Punjab [B] Maharastra [C] Avadh [D] Madras Correct Answer: C [Avadh] Explanation: The Revolt of 1857: Awadh, one of the main centers of the Revolt, was annexed by Lord Dalhousie, Governor General of India, in 1856. Begum Hazrat Mahal also known as Begum of Awadh, during the national liberation uprising of 1857-59 in India headed the rebels 418. Mahalwari System of Revenue Settlement was introduced in which of the following? [A] Bengal [B] Bombay [C] Madras [D] North west provinces Correct Answer: D [North west provinces] Explanation: Land revenue system Permanent settlement introduced in Bengal, Bihar, Orissa and districts of Benaras and Northern districts of Madras by Lorrd Cornwallis in 1793. Ryotwari System introduced in Bombay and Madras. Mahalwari system was introduced in the North-West Provinces, the Punjab, Delhi, Parts of Central India and Uttar Pradesh In this system, the land was not owned by an individual be it zamindar or any cultivator but by a group of estates or villages called Mahal. The Mahal was collectively known as the landlord and revenue was collected from the head of the Mahal, also known as Talukdar 419. What is the correct chronological order of setting up of the following Commissions? 1. Macdonell Commission 2. First Industrial Commission 3. First Fiscal commission [A] 1, 2, 3 [B] 2, 3, 1 [C] 2, 1, 3 [D] 1, 3, 2 Correct Answer: A [1, 2, 3] Explanation: Macdonell Commission 1901 was the famine commission appointed by Lord Curzon. First Industrial Commission: In 1916 an Industrial Commission was appointed to find out means for giving encouragement for the growth of Indian industries. The report of the commission was submitted in 1918.

305 | P a g e

shop.ssbcrack.com

MCQs

INDIAN HISTORY

MCQs

First Fiscal Commission: Indian Fiscal Commission was established in 1919 and it recommended tariff protection for certain industries. As a follow up to the recommendations to this commission, a tariff board was established in 1923 and in 1924, the Steel Industry of India was given protection. 420. The British East India Company ceased to be a trading Company via which among the following legislation? [A] Pitts India Act of 1784 [B] Charter Act of 1833 [C] Charter Act of 1813 [D] Government of India Act 1858 Correct Answer: B [Charter Act of 1833] Explanation: The charter act 1833: It is considered to be an attempt to codify all Indian Laws. The Governor General of Bengal now became the The Governor General of India. One of the Provision of this act – “the East India Company now lost its trading privilege i.e., tea and monopoly in China, henceforth it became a purely administrative body under the crown.” 421. Which one of the following native states was NOT annexed by the British on the basis of the Doctrine of Lapse? [A] Satara [B] Punjab [C] Jhansi [D] Karauli Correct Answer: B [Punjab] Explanation: Doctrine of Lapse: formula devised by Lord Dalhousie, governor-general of India (1848–56), to deal with questions of succession to Hindu Indian states. • Satara was annexed in 1848. • Sambalpur was annexed in 1849. • Karauli was annexed in 1852. • Jhansi was annexed in 1854. 422. Avadh was annexed into the British East India territories on which ground? [A] Doctrine of Lapse [B] Alleged misgovernment [C] Failure to pay subsidy [D] Maintenance of relations with foreign powers Correct Answer: B [Alleged misgovernment] Explanation: The kingdom of Oudh was the only great Indian state whose ruler Nawab Wajid Ali Shah was dispossessed on the ground of “intolerable misgovernment”. Awadh was annexed in February 1856 via a proclamation. 423. Who said that British established a robber state in Bengal between 1765 and 1772 ? [A] G. W. Forrest [B] Lord Macaulay [C] K. M. Panikkar [D] Nand Lal Chatterji Correct Answer: C [K. M. Panikkar]

306 | P a g e

shop.ssbcrack.com

MCQs

INDIAN HISTORY

MCQs

Explanation: K.M. Panikkar was an Indian scholar, journalist, historian, administrator and diplomat. He wrote that between 1765-1772, British had “established a robber state where, without reference to the rights of the others, they freely plundered and looted under the cover of thier rights”. 424. The first victim of the British policy of Subsidiary Alliance was___? [A] Avadh [B] Mewar [C] Mysore [D] Hyderabad Correct Answer: D [Hyderabad] Explanation: A subsidiary alliance is an alliance between a dominant nation and a nation that it dominates.The 1st victim of the policy of subsidiary alliance of Lord Wellesley was the Nizam of Hyderabad. 425. The Peshwa accepted the Subsidiary Alliance with the British via which among the following treaties? [A] Treaty of Purandhar [B] Treaty of Bassein [C] Treaty of Salbai [D] Treaty of Surji Arjungaon Correct Answer: B [Treaty of Bassein] 426. Under which among the following treaties, the British East India Company secured the Diwani right of Bengal, Bihar and Orissa? [A] Alinagar [B] Faizabad [C] Allahabad [D] Benaras Correct Answer: C [Allahabad] Explanation: Treaty of Allahabad ( 16 August 1765), it was the outcome of the Battle of Buxar (1764). Robert Clive (Governor General of Bengal) made a separate treaty with Mughal Emperor Shah Alam II and Nawab of Awadh Shuja-ud-daullah. A/c to this treaty, Mughal Emperor granted Fiscal Rights (Diwani) to the East India Company at Bengal, Bihar and Orissa. 427. Which among the following was the first establishment of Dutch in current territories of Odisha? [A] Calcutta [B] Chinsura [C] Pippli [D] Balasore Correct Answer: C [Pippli] Explanation: Pippli was the first place where the Dutch had established themselves in current territories of Odisha. It was soon abandoned for Balasore. 428. Which among the following British men tried to obtain a Firman for trade in Gujarat from Emperor Akbar? [A] Ralph Fitch [B] John Mildenhall [C] Sir Thomas Roe [D] Thomas Stephens 307 | P a g e shop.ssbcrack.com

MCQs

INDIAN HISTORY

MCQs

Correct Answer: B [John Mildenhall] Explanation: John Mildenhall was a British explorer and adventurer and one of the first to make an overland journey to India. He was the self-styled ambassador of the British East India Company in India. His is the first recorded burial of an Englishman in India. He bluffed in Akbar’s court that he was Her Britannic Majesty’s ambassador. Though he was admitted in the Durbar of Akbar, so that he became the first Englishman privileged to be received by the Great Mughal; yet he could never win the confidence of the Mughal Emperor. Portuguese Jesuits played to keep him or English away from the Mughals. 429. In which among the following years, Bombay transferred to the East India Company by Charles II ? [A] 1662 A.D. [B] 1664 A.D. [C] 1666 A.D. [D] 1668 A.D. Correct Answer: D [1668 A.D.] Explanation: On 21 September 1668, the Royal Charter of 27 March 1668, led to the transfer of Bombay from Charles II to the British East India Company for an annual rent of £10. 430. The most important reason for collection of “Chauth” and “Sardeshmukhi” by Marathas was __? [A] To increase the source of income [B] To expand his territories [C] To consolidate his political influence [D] Because of the opposition of Muslim rulers Correct Answer: A [To increase the source of income] Explanation: Shivaji collected the chauth and sardeshmukhi from the territory which was either under his enemies or under his own influence. The chauth was one fourth part of the income of a particular territory while the sardeshmukhi was one tenth. Shivaji collected these taxes simply by force of his arms. These constituted of the primary sources of income of Shivaji and helped in the extension of the power and territory of the Marathas. 431. Who among the following was not included in the “Asta Pradhan” of Shivaji ? [A] Sumant [B] Majmudar [C] Vakiyanavis [D] Vakeel Correct Answer: D [Vakeel] Explanation: Ashta Pradhan Pantpradhan or Peshwa – Prime Minister, general administration of the Empire. Amatya – Finance Minister, managing accounts of the Empire. Sacheev – Secretary, preparing royal edicts. Mantri – Interior Minister, managing internal affairs especially intelligence and espionage. Senapati – Commander-in-Chief, managing the forces and defense of the Empire. Sumant – Foreign Minister, to manage relationships with other sovereigns. Nyayadhish – Chief Justice, dispensing justice on civil and criminal matters. Panditrao – High Priest, managing internal religious matters. 432. Hind Swaraj, a book by Mahatma Gandhi in 1909 was originally written in __? [A] Hindi [B] Urdu [C] Gujarati [D] English Correct Answer: C [Gujarati] 308 | P a g e

shop.ssbcrack.com

MCQs

INDIAN HISTORY

MCQs

433. Fatehchand (one of the traitors of Plassey) was bestowed with the title of Jagath Seth by __? [A] Alivardi Khan [B] Sirajuddaula [C] Mir Zafar [D] Muhammad Shah Correct Answer: D [Muhammad Shah] Explanation: Fatehchand was adopted son of Manik Chand. He obtained the title of “Seth” from the Emperor Farrakhsiyar. Muhammad Shah bestowed him with the title of “Jagat Seth”. 434. With reference to Bahadur Shah Zafar, which among the following statements is not true? [A] He was an emperor without empire [B] He was a warrior without any war experience [C] Hassan Askari was his spiritual guide [D] He succeeded to the throne in 1845 A.D. Correct Answer: D [He succeeded to the throne in 1845 A.D.] 435. Who of the following wrote the first Indian poetic work in English? [A] Kashi Prasad Ghosh [B] Ramchandra Vidyavagish [C] Krishna Mohan Banerjee [D] Hariharanand Correct Answer: A [Kashi Prasad Ghosh] Explanation: First Indian poetic work in English, The Shair and Other Poems by Kashi Prasad Ghosh was published in 1830. 436. Who was the last Maratha chief to accept the Subsidiary Alliance of the British? [A] Gaikwad [B] Scindia [C] Holkar [D] Bhonsle Correct Answer: C [Holkar] Explanation: The doctrine of subsidiary alliance was introduced by Marquess (or Lord) Wellesley, British Governor-General of India from 1798 to 1805 . Under this doctrine, Indian rulers under British protection surrendered control of their foreign affairs to the British. Hyderabad first signed it. The five Maratha chiefs entered into subsidiary alliance as follows: Peshava →1802 Scindia →1804 Gaekwa →1805 Bhonsle →1806 Holkar →1818. 437. Which was the first State to be created on linguistic basis ? [A] Madras [B] Andhra Pradesh [C] Andhra State [D] Gujarath Correct Answer: C [Andhra State] Explanation: The congress in its Jaipur session appointed a three member committee also popularly known as JVP committee after the name of its leaders – Jawaharlal Nehru,Vallabh Bhai Patel and Pattabhai Sitaramaya.The committee rejected language as the bass of reorganization of states.. Potti Sreeramulu, one of the activists demanding the formation of a Telugu-majority state, died on 16 December 1952 after 309 | P a g e

shop.ssbcrack.com

MCQs

INDIAN HISTORY

MCQs

undertaking a fast-unto-death. This resulted in the creation of the first state on linguistic basis for Telugu speaking people called Andhra State on October 1, 1953. It was later renamed Andhra Pradesh. 438. The Bijauliya Movement is related to the agrarian struggle in the current state of __? [A] Kerala [B] Assam [C] Odisha [D] Rajasthan Correct Answer: D [Rajasthan] Explanation: Bijoliya Kisaan Andolan- It was a pioneer agrarian movement in the Mewar State in present Udaipur District.The Jagirdar of Bijolia was a Parmar Rajput having 96 villages in his jagir. There were 86 different taxes on peasants against which peasants revolted in 1905. The initial leadership was provided by Sitaram Das. The movement got linked to national movement. Vijay (Bijoy) Singh Pathik and Manik Lal Verma (future Chief Minister of Rajasthan) led a no tax movement in 1916. It was called Bijolia movement. The peasants refused to do begar and held back the taxes. The movement continued through 1920s and spread over to other States of Rajputana. 439. Who was the President of the “Flag Committee”? [A] B. R. Ambedkar [B] J. B. Kriplani [C] Hakim Ajmal Khan [D] Dr. Rajendra Prasad Correct Answer: D [Dr. Rajendra Prasad] Explanation: Dr. Rajendra Prasad was the head of the Ad hoc committee on National Flag in the constituent assembly. The flag of the congress party was accepted as the National Flag with few changes on July 22, 1947. The new flag code of India gives freedom to individual to hoist the flag on all days, but with due respect to the flag. 440. Which one of the following leaders was not a part of Non-cooperation movement ? [A] M. A. Ansari [B] M. A. Zinnah [C] Abul Kalam Azad [D] Hakim Ajmal Khan Correct Answer: B [M. A. Zinnah] Explanation: Non cooperation movement (1920-22) was led by Mahatma Gandhi Veterans like Bal Gangadhar Tilak, Bipin Chandra Pal, Mohammad Ali Jinnah, Annie Besant opposed the idea outright. But the younger generation of Indian nationalists were thrilled, and backed Gandhiji. The Congress Party adopted his plans, and he received extensive support from Muslim leaders like Abul Kalam Azad, Mukhtar Ahmed Ansari, Hakim Ajmal Khan, Abbas Tyabji, Maulana Mohammad Ali and Maulana Shaukat Ali. 441. Who is considered the “Father of Muslim Renaissance in Bengal” in Bengal? [A] Abdul Latif [B] Mirza Ghulam Ahmed [C] Muhammad Qasim [D] Rashid Ahmed Gangohi Correct Answer: A [Abdul Latif] Explanation: Nawab Abdul Latif Khan: (1828-1893) a prominent personality of mid 19th century Bengal, the pioneer of Muslim modernization and the architect of the Muslim Renaissance, was one of those great 310 | P a g e shop.ssbcrack.com

MCQs

INDIAN HISTORY

MCQs

men who appeared as saviours of their frustrated, humiliated, demoralized and disorganised fellow countrymen under colonial rule His chief contribution was in the field of education. He was among the first to understand that young Bengali Muslims should receive modern education. He understood that the Muslims of Bengal had fallen behind in everything because of their prejudices against modern education. He devoted his whole life to removing this self-destructive prejudice from their minds. 442. In which year English was made the medium of instruction in India ? [A] 1844 A.D [B] 1835 A.D. [C] 1833 A.D. [D] 1813 A.D. Correct Answer: B [1835 A.D.] Explanation: The English Education Act was a legislative Act of the Council of India in 1835 giving effect to a decision in 1835 by William Bentinck, the Governor-General of British India to reallocate funds the East India Company was required by the British Parliament to spend on education and literature in India. In the same year, British Government designated English as the medium of education for schools and universities. 443. The most fervent supporter of Gandhi’s proposal for an all-out campaign of civil disobedience during Quit India Movement was ? [A] Ram Manohar Lohia [B] Sardar Vallabhbhai Patel [C] Subhash Chandra Bose [D] Jai Prakash Narayan Correct Answer: B [Sardar Vallabhbhai Patel] Explanation: Sardar Vallabhbhai Patel- known as the “Iron Man of India” or “Bismarck of India”. Patel was most fervent supporter of Gandhi’s proposal for an all-out campaign of civil disobedience to force the British to Quit India. He participated in Gandhi’s call for individual disobedience, and was arrested in 1940 and imprisoned for nine months. He also opposed the proposals of the Cripps’ mission in 1942. 444. The biographical memoir of A. O. Hume was written by __? [A] W. Wederbirn [B] Lord Dufferin [C] J. Charles [D] None of the above Correct Answer: A [W. Wederbirn] Explanation: Sir William Wedderburn, or W. Wederbirn was a Scottish civil servant in India and a politician. He wrote a biographical memoir of A. O. Hume who died in 1912. (A. O. Hume was the founder of the Indian National Congress). 445. The Two Nation Theory was propounded in which session of the Muslim League? [A] Lahore Session, 1940 [B] Bombay Session, 1915 [C] Delhi Session, 1918 [D] Calcutta Session, 1917 Correct Answer: A [Lahore Session, 1940] Explanation: Fazlul Haq, the premier of Bengal, who along with Muslim League had formed the government of Bengal Province ,moved a resolution ,which was passed by Muslim League. In this session 311 | P a g e shop.ssbcrack.com

MCQs

INDIAN HISTORY

MCQs

,Jinnah in his presidential address gave the famous Two.Nation Theory as fellows: “India cannot be assumed today to be Unitarian and homogeneous nation,but on the contrary ,there are two nations in the main- the Hindus and Muslims”. The term Pakistan was not used in this session. Gandhiji rejected the two nation theory. 446. The Treaty of Alinagar was signed in ___? [A] February 1756 [B] September 1756 [C] February 1757 [D] April 1757 Correct Answer: C [February 1757] Explanation: February 9, 1757 the treaty of Alinagar (changed name of Calcutta) was signed between Robert Clive of the British East India Company and the Nawab of Bengal, Mirza Muhammad Siraj Ud Daula. According to this treaty a)All preivileges granted by Farrukshiyar were confirmed. b)All goods under the british dastak went to be duty free. c)The british were given right of making coin in Bengal. The signing of the treaty was one of the events leading up to the famous Battle of Plassey. 447. During the company rule, the Hindu Widows Remarriage Act was drafted by__ ? [A] Lord Canning [B] Lord Dalhousie [C] Lord Hardinge [D] None of the above Correct Answer: B [Lord Dalhousie] Explanation: The Hindu Widows’ Remarriage Act, 1856, also Act XV, 1856, enacted on 25 July 1856, legalized the remarriage of Hindu widows in all jurisdictions of India under East India Company rule was drafted by Lord Dalhousie. 448. The statement “Like summer gale revolt of Meerut was unprecedented and short-lived” was made by __? [A] S. N. Sen [B] R. C. Majumdar [C] S. B. Chaudhuri [D] V. D. Savarkar Correct Answer: A [S. N. Sen] Explanation: The Merrut 1857 Uprising was described by Dr.Surendra Nath Sen-“ Like summer gale revolt of Meerut was unprecedented and short-lived”. He was the author of a number of major works, mostly on the history of the Marathas.In 1956, the Indian government commissioned him to write a history of the Indian Popular Uprising of 1857–59; the work, entitled Eighteen Fifty-seven, was published in 1957. 449. The First President of Muslim League was __? [A] Nawab Viqar-ul-Mulk Mustaq Hussain [B] Mian Abdul Aziz [C] Hidayat Hussain Khan [D] Mohammad Ali Jinnah Correct Answer: A [Nawab Viqar-ul-Mulk Mustaq Hussain] Explanation: Muslim League, a political organization of India and Pakistan, founded in 1906 as the AllIndia Muslim League by Aga Khan III. Its original purpose was to safeguard the political rights of Muslims in India. Nawab Mushtaq Husain Viqar-ul-Mulk also known as Mushtaq Hussain, was a Muslim politician 312 | P a g e shop.ssbcrack.com

MCQs

INDIAN HISTORY

MCQs

and one of the founders of All India Muslim League. He is also known for his involvement in the Aligarh Movement. 450. Which party observed the “Black Day” on July 3, 1947 against Partition of India? [A] Indian National Congress [B] Forward Bloc [C] Hindu Mahasabha [D] Communist Party of India Correct Answer: C [Hindu Mahasabha] 451. Which party observed the “Black Day” on July 3, 1947 against Partition of India? [A] Indian National Congress [B] Forward Bloc [C] Hindu Mahasabha [D] Communist Party of India Correct Answer: C [Hindu Mahasabha] 452. Who was elected the President of Indian National Congress in the famous Tripuri Session of 1939 ? [A] Acharya Narendra Deo [B] Sarat Chandra Bose [C] Subhash Chandra Bose [D] Maulana Abul Kalam Azad Correct Answer: C [Subhash Chandra Bose] Explanation: 52nd Session : Subhash Chandra Bose was re-elected the President of INC at the Tripuri Session in 1939 by defeating the Gandhi ji’s Candidate Pattabhai Sitaramaiyya. He demanded that the Congress should deliver a six-months’ ultimatum to Britain and in the event of its rejection a country-wide struggle for ‘Poorna Swaraj’ should be launched. His warning and advice went unheeded, his powers as President were sought to be curtailed. He, therefore, resigned in April 1939, and announced, in May 1939, the formation of the Forward Bloc within the Congress. 453. The Hindu Mahasabha was organized for the first time in 1914 at ___: [A] Haridwar [B] Allahabad [C] Varanasi [D] None of the above Correct Answer: A [Haridwar] Explanation: Hindu Mahasabha was founded in 1914 by Madan Mohan Malviya.It worked with Arya Samaj and other Hindu organizations. It was directly link with Rashtriya Swam Sevak Sangh founded in 1925 at Nagpur by K.B.Hegewar.The first All India Hindu Mahasabha Conference was organized at Hardwar in 1915. The Sabha ecame more aggressive after 1929 and started propagating Hindu Rashtra which was totally differ from Gandhiji’s Ram Rajya. 454. Who among the following is known as Mother of Indian Revolutionaries ? [A] Annie Besant [B] Sarojini Naidu [C] Madam Cama [D] Usha Mehta Correct Answer: C [Madam Cama] 313 | P a g e shop.ssbcrack.com

MCQs

INDIAN HISTORY

MCQs

Explanation: Bhikaiji Rustom Cama,or Madam Cama was born on 24 September 1861 in Bombay. She was an outstanding lady of great courage, fearlessness, integrity, perseverance and passion for freedom.and is considered as the mother of Indian revolution because of her contributions to Indian freedom struggle. She was credited with designing India’s first tricolour flag with green, saffron and red stripes bearing the immortal words – Vande Matram. After fighting tirelessly for India’s freedom struggle on foreign land for several years, she came back to India and died on August 13, 1936. 455. Which Bengali writer suggested the adoption of Hindi as India’s National Language? [A] Bhudeva Mukherjee [B] Dinbandhu Mitra [C] Madhusudan Datta [D] Kali Prasanna Sinha Correct Answer: A [Bhudeva Mukherjee] Explanation: The factors that enhanced the Status of Hindi: The contribution of Bhudeva Mukherjee ,wellknown educationist and writer,played a key role in establishing the Nagari script in Bihar schools and law courts before 1892. Nagari Pracharni Sabha was founded in 1893 to promote the cause of Hindi and Nagari script.The Language policy of Indian National congress in the end of 1920’s paved the way of Hindi as the Mother tongue of Hindustan. 456. Who among the following had drafted the fundamental rights resolution at the Karachi Session, 1931? [A] Jawaharlal Nehru [B] Acharya Narendra Deo [C] Subhash Chandra Bose [D] Maulana Abul Kalam Azad Correct Answer: A [Jawaharlal Nehru] Explanation: The Congress met at Karachi in March 1931 to endorse the Gandhi-Irwin or the Delhi Pact.It was presided over by Sardar Patel. This session became memorable for its resolution on the Fundamental Rights and the National Economic Programs.In short , It set the parameters of the Swaraj was reflecting the then dominant leftwing ting of the national movements.It was drafted by the Pandit Jawaharlal Nehru. 457. The name of the operation started by the British Government to arrest the leaders of Quit India Movement was__? [A] Operation Reander Paste [B] Operation Zero Hour [C] Operation Thunderbolt [D] Operation Blue Star Correct Answer: C [Operation Thunderbolt] Explanation: The Quit India Movement , or the August Movement (August Kranti) was a civil disobedience movement launched in India in August 1942 in response to Mohandas Gandhi’s call for immediate independence. 458. During which among the following movements, Mahatma Gandhi remarked: “on bended knees I asked for bread and received stone instead” ? [A] Khilafat Movement [B] Non-Cooperation Movement [C] Dandi March [D] Quit India Movement 314 | P a g e shop.ssbcrack.com

MCQs

INDIAN HISTORY

MCQs

Correct Answer: C [Dandi March] Explanation: Dandi March(12th March 1930) also known as the Salt Satyagraha, was undertaken by Gandhiji . He led the Dandi march from his base, Sabarmati Ashram near Ahmedabad, to the sea coast near the village of Dandi. The triggering factor for this movement was the British monopoly of salt trade in India and the imposition of a salt tax. It was a direct action campaign of tax resistance and nonviolent protest against the British salt monopoly in colonial India, and triggered the wider Civil Disobedience Movement. “On bended knees I asked for bread and I have received stone instead”, was a remark made by Gandhi ji in the beginning of the march. 459. Which one of the following Bengali drama was directed against Polygamy ? [A] Bhanumati Chittavikas [B] Kulin Kulasarvasva [C] Vidhva Vivaha [D] Nava Natak Correct Answer: B [Kulin Kulasarvasva] Explanation:It was Kulin Kulasarvasva (“All about a Kulin Clan”) by Pandit Ramnarayan Tarkaratnan. A social satire against the practice of polygamy. It was the first social drama in a regional language(Bengali) attacking Brahmin polygamy, was presented by an aristocratic family in 1857. 460. d100 years ago, the Hindustan Gadar Party was launched in which among the following cities in United States? [A] Chicago [B] San Francisco [C] Oregon City [D] New York Correct Answer: B [San Francisco] Explanation: To mark the centenary of the Gadar movement, the Indian Government is planning to fund the conversion of the Gadar Memorial in San Francisco into a museum and library. The Hindustan Gadar Party, when founded in 1913, begun its operation from 436 Hill Street of the northern Californian city. It came to be known as ‘Yugantar Ashram’ and it was from here that the freedom fighters were active from 1913 to 1917. From this place they published a weekly magazine called Gadar to propagate the cause of Indian independence. 461. The Special Calcutta Session, 1920, in which Mahatma Gandhi moved the Non-cooperation resolution was presided by __? [A] C. R. Das [B] B. C. Pal [C] Lala Lajpat Rai [D] Motilal Nehru Correct Answer: C [Lala Lajpat Rai] Explanation: Special Calcutta Session, 1920, in which Mahatma Gandhi moved the Non-cooperation resolution was presided by Lala Lajpat Rai and it was supported by others. Non cooperation movement was launched for two major issues: 1. The British Government’s attitude towards the Khilafat Movement. 2. Its’ failure to protect the innocent people of the Punjab and punish the officers guilty of barbarous behavious towards them. Reference: Non-cooperation Movement. 315 | P a g e

shop.ssbcrack.com

MCQs

INDIAN HISTORY

MCQs

462. The famous book “Ghulamgiri” was written by__: [A] B. R. Ambedkar [B] Narayan Guru [C] Jyotiba Phule [D] M. P. Pillai Correct Answer: C [Jyotiba Phule] Explanation: Jyotirao Govindrao Phule (Mahatma) was born in Pune,was a great social reformer. He dedicated his book Gulamgiri to the American movement to free slaves, he linked the conditions of the black slaves in America with those of the lower castes in India. This comparison contains an expression of hope that one day, like the end of slavery in America, there would be an end to all sorts of caste discriminations in Indian society 463. Who among the following made the following statement? “The only lesson required in India at present is to learn how to die and the only way to teach is by dying ourselves. Therefore, I die and glory in my martyrdom” [A] Aurbindo Ghosh [B] Khudiram Bose [C] Chandra Shekhar Azad [D] Madan Lal Dhingra Correct Answer: D [Madan Lal Dhingra] Explanation: Madan Lal Dhingra was born in the holy city of Amritsar, He was an Indian political activist studying in England and there he came in contact with the Indian revolutionaty leaders like Shyamji Krishna Verma and Veer Damodar Savarkar.He was perhaps the first Indian freedom fighter to be executed on British soil. He assassinated Sir Curzan Wyllie, political Aide-de-Camp to the Secretary of State for India, Lord Morley in London in 1909. He was hanged at Pentonville Prison, London, on 17 August, 1909 464. Basumati, the oldest Bengali Daily paper was edited by ___: [A] Anand Mohan Bose [B] Surya Sen [C] Barinder Kumar Ghosh [D] V. D. Savarkar Correct Answer: C [Barinder Kumar Ghosh] Explanation: Barinder Kumar Ghosh was born at Norwood near London on 5th January in 1880.He was a younger brother of Aurobindo Ghosh. In 1906, to spread the revolutionary ideas and to provide fervor to Swadeshi movement ,he started a Bengali weekly-“the jugantar”. In 1907, he started the Maniktala group with Bagha Jatin and a few young revolutionary activists for the collection of arms and ammunitions and manufacturing explosives.He was sentenced to death by the trial known as Alipore Bomb Case. Later the sentence was commuted to life imprisonment and was sent to the Cellular Jail in Andaman in 1909. In 1920, Barindra Kumar Ghose was released. In 1933 Barindra Kumar Ghose launched English weekly, The Dawn of India. He was also associated with The Statesman newspaper and earned the title as a columnist. In 1950, he became the editor of the Bengali daily Dainik Basumati. 465. Curzon Wyllie, who was murdered by Madan Lal Dhingra in London, was___? [A] Secretary of State for India [B] Adviser to the Secretary of State of India [C] Law Member 316 | P a g e

shop.ssbcrack.com

MCQs

INDIAN HISTORY

MCQs

[D] [Window Title] Correct Answer: B [Adviser to the Secretary of State of India] Explanation: Madan Lal Dhingra was a great revolutionary from Punjab,associated with the Indian Home Rule Society, The Abhinav Bharat Society and the Indian House in London. On July 1, 1909 he shot dead Curzon Wyllie an adviser to the secretary of state of India, and Cowas Lolcaca at the meeting of the Indian National Association in London to avenge the atrocities committed by the British in India. 466. Who was the only Indian prince who actively participated in the Revolutionary Movement within and outside India? [A] Raja Aridaman Singh [B] Raja Hari Singh [C] Raja Kumar Singh [D] Raja Mahendra Pratap Correct Answer: D [Raja Mahendra Pratap] Explanation: Raja Mahendra Pratap was an Indian freedom fighter, journalist, writer, and Marxist revolutionary social reformist of India. He believed in the religious unity and racial equality. He started two paper-“Prem”in Hindi and “Nirbal Sewak” in Hindi and Urdu. In 1915, he formed the first government outside India in Afghanistan. He returned to India after 32 years of exile and became the member of the second Lok Sabha in 1957-1962. He was elected as an independent candidate from Mathura constituency. He was president of Indian Freedom Fighters’ Association and also the president of All India Jat Mahasabha .He died on 29 April 1979. 467.During which among the following events, Bal Gangadhar Tilak was given the epithet of ‘Lokmanya’? [A] Home Rule Movement [B] Surat Split [C] Swadeshi Movement [D] His imprisonment in 1908 Correct Answer: A [Home Rule Movement] Explanation: The name of Tilak became a household name during Homerule Movement and this let him earn the epithet Lokmanya. Home rule movement has been taken from Ireland.The two Home Rule League were set up in April 1916 by Bal Gangadhar Tilak and in Sept 1916 by Annie Besant.The main objective of Home Rule League were: - Self Government in the British Empire. Work for National education, social and political reforms. Abolition of untouchability. 468. The national anthem of India ‘Jana Gana Mana’ was first sung at __: [A] Calcutta, 1911 [B] Calcutta, 1912 [C] Delhi, 1911 [D] Mumbai, 1912 Correct Answer: A [Calcutta, 1911] Explanation: “Jana Gana Mana ” is the national anthem of India. Written in highly Sanskritised (Tatsama) Bengali, it is the first of five stanzas of a Brahmo hymn composed and scored by Nobel laureate Rabindranath Tagore. It was first sung in Calcutta Session of the Indian National Congress on 27 December 1911. “Jana Gana Mana” was officially adopted by the Constituent Assembly as the Indian national anthem on 24 January 1950. 27 December 2011 marked the completion of 100 years of Jana Gana Mana since it was sung for the first time.The original poem written by Rabindranath Tagore was translated into Hindi by Abid Ali.

317 | P a g e

shop.ssbcrack.com

MCQs

INDIAN HISTORY

MCQs

469. Who among the following poet and dramatist pioneered the Bengali drama ? [A] Bankim Chandra Chatterjee [B] Michael Madhusudan Dutta [C] Rajnarayan Dutt [D] Aurobindo Ghosh Correct Answer: B [Michael Madhusudan Dutta] Explanation: Michael Madhusudan Dutta was a popular 19th-century Bengali poet and dramatist. He was a pioneer of Bengali drama. His famous work Meghnad Bodh Kavya is a tragic epic. 470. With reference to the relations of the Princely states with British Crown, which among the following report said that “Paramountcy must remain paramount”? [A] Hunter Commission Report [B] Strachey Commission Report [C] Butler Commission Report [D] Campbell Commission Report Correct Answer: C [Butler Commission Report] Explanation: A three -member committee headed by Harcourt Butler, appointed on December 16, 1927 to examine the relations between the native states and the paramount power, declined to define what paramountcy was but clearly and fortrightly stated that “paramountcy must remain paramount”. The committee fully endorsed that the viceroy, not the governor general should remain the Crown agent in dealing with the native states. In fact the right to protect includes right to internal intervention. Paramountcy is the supreme sovereign power which was kept above the reach of law and interpretation, to be exercised at the appropriate time subject to resiraints of morality and constitutional propriety against an erring ruler of a native statewhen other correctional means had proved to be futile. It is a concept developed into a deterrent in the political relation between the British and the Indian rulers. 471. The policy announcement of “progressive realization of responsible Government in India as an integral part of the British Empire” was embedded in __: [A] Montagu-Chelmsford Report [B] Motilal Nehru Report [C] Hartog Committee Report [D] Cabinet Mission Report Correct Answer: A [Montagu-Chelmsford Report] 472. The objective of the Cripps Mission, headed by Sir Stafford Cripps was to __: [A] To prevent the launch of Quit India Movement [B] To persuade Indian Leaders to support the British War Efforts [C] To Convince the Congress Ministries to withdraw their resignations [D] To set up a Constitution making body Correct Answer: B [To persuade Indian Leaders to support the British War Efforts] Explanation: The Cripps mission was an attempt in late March 1942 by the British government to secure Indian cooperation and support for their efforts in World War II. The mission was headed by Sir Stafford Cripps, a senior left-wing politician and government minister in the War Cabinet of Prime Minister Winston Churchill. The mission proved a failure. 473. In context with the Constituent Assembly, which among the following observations is / are correct? 1. The members of Constituent Assembly were elected directly 2. The basis of seats allocation was population 3. Composition of the Constituent Assembly was roughly in line with the suggestions of the Cabinet 318 | P a g e shop.ssbcrack.com

MCQs

INDIAN HISTORY

MCQs

Mission plan Choose the correct option from the codes given below: [A] Only 1 & 2 are correct [B] Only 2 & 3 are correct [C] Only 1 & 3 are correct [D] 1, 2 & 3 are correct Correct Answer: B [Only 2 & 3 are correct] Explanation: Election of the members of the Constituent Assembly had been elected firstly, for undivided India. Its first sitting was held on December 9, 1946 and re-assembled as Constituent Assembly for divided India on 14 August 1947. oIts members were elected by indirect election by the members of the Provisional Legislative Assemblies that had been established in 1935. oThe Constituent Assembly was composed roughly along the lines suggested by the plan proposed by the committee of the British cabinet, known as the Cabinet Mission. As per the Cabinet Mission plan, each Province and each Princely State or group of States were allotted seats proportional to their respective population roughly in the ratio of 1:10 Lakh. As a result the Provinces (that were under direct British rule) were to elect 292 members while the Princely States were allotted a minimum of 93 seats. The seats in each Province were distributed among the three main communities, Muslims, Sikhs and general, in proportion to their respective populations. oMembers of each community in the Provisional Legislative Assembly elected their own representatives by the method of proportional representation with single transferable vote. oThe method of selection in the case of representatives of Princely States was to be determined by consultation. oAs a consequence of the Partition under the plan of 3 June 1947 those members who were elected from territories which fell under Pakistan ceased to be members of the Constituent Assembly. oThe numbers in the Assembly were reduced to 299 of which 284 were actually present on 26 November 1949 and appended their signature to the Constitution as finally passed. 474. Who among the following had drafted the “Declaration of Independence” pledge in 1930? [A] Motilal Nehru [B] Jawahar Lal Nehru [C] Mahatma Gandhi [D] C R Das Correct Answer: C [Mahatma Gandhi] Explanation: Declaration of Independence Pledge Though the congress passed the Poorna Swaraj Resolution in December 1929, it was a month later on January 26, 1930, when a Pledge of Indian Independence also known as Declaration of Independence was taken. While the Poorna Swaraj Resolution was drafted by Jawaharlal Lal Nehru, the “Declaration of Independence” pledge was drafted by Mahatma Gandhi in 1930 and it echoed the essence of American Declaration of Independence. After this pledge January 26, 1930 was declared as Independence Day by Indian National Congress. 475. Which among the following is India’s first Oil refinery ? [A] Guwahati Refinery [B] Vishakhapatnam Refinery [C] Digboi Refinery [D] Barauni Refinery Correct Answer: C [Digboi Refinery] Explanation: Digboi is a town and a town area committee in Tinsukia district in the north-eastern part of the state of Assam, India. Crude oil was discovered here in late 19th century. Digboi is known as the Oil City of Assam where the first oil well in Asia was drilled. The first refinery was started here as early as 1901. Digboi has the oldest oil well in operation. The Digboi Refinery modernization project was taken up 319 | P a g e shop.ssbcrack.com

MCQs

INDIAN HISTORY

MCQs

in large scale in order to overcome the technological obsolescence of the old refinery. Subsequently a number of other major projects were undertaken by Assam Oil Division to further revamp and modernize Digboi Refinery.Digboi refinery has been awarded the ISO-14001 and OHSMC certificate. 476. In which among the following countries, Battaglione Azad Hindoustan, a legion unit of Indian National Army (Azad Hind Fauj) was formed during the World War II ? [A] Japan [B] Germany [C] Italy [D] Singapore Correct Answer: C [Italy] Explanation: The Indian National Army ( Azad Hind Fauj) was an armed force formed by Indian nationalists in 1942 in Southeast Asia during World War II. The aim of the army was to secure Indian independence with Japanese assistance. Initially formed in 1942 immediately after the fall of Singapore under Mohan Singh, the first INA collapsed in December that year before it was revived under the leadership of Subhas Chandra Bose in 1943 and proclaimed the army of Bose’s Arzi Hukumat-e-Azad Hind . The Battaglione Azad Hindoustan (Italian: “Free India Battalion”) was a foreign legion unit formed in Italy in July 1942. It was headed by Mohammad Iqbal Shedai. 477. “The Bengalee” Newspaper was started by which among the following activists in 1879? [A] Surendranath Banerjee [B] Anandmohan Bose [C] Nabagopal Mitra [D] Rajnarayan Basu Correct Answer: A [Surendranath Banerjee] Explanation: Surendranath Banerjee was one of the earliest political leaders during the British Rule. He founded the newspaper “The Bengalee” in 1879 and founded the Indian National Association with Anandmohan Bose, the first Indian political organization of its kind on July 26, 1876. He is renowned today as pioneer leader of Indian politics. He published an important work, A Nation in Making, which was widely acclaimed. The British respected him and referred to him during his later years as “Surrender Not Banerjee”. 478. At which among the following places, Hindu Mela was started in 1867 by Nabagopal Mitra ? [A] Calcutta [B] Delhi [C] Allahabad [D] Varanasi Correct Answer: A [Calcutta] Explanation: Hindu Mela was founded in 1867 in Calcutta by Nabagopal Mitra with the active support from the Tagore Family. The Hindu Mela was also known as the Chaitra Mela.It was largely the product of the combined efforts of Rajnarayan Basu, Dwijendranath Tagore, and Nabagopal Mitra. 479. Who among the following played a dominant role in the famous Vaikom Satyagraha of 1924–25? [A] T. K. Madhavan [B] Muloor S.Padmanabha Panicker [C] Balarama Varma [D] K. Kelappan Correct Answer: D [K. Kelappan] 320 | P a g e shop.ssbcrack.com

MCQs

INDIAN HISTORY

MCQs

Explanation: Vaikom Satyagraha (1924–25) was a satyagraha (movement) in Travancore, India (now part of Kerala) against untouchability in Hindu society. The movement was centered at the Shiva temple at Vaikom, near Kottayam.The Satyagraha aimed at securing freedom of movement for all sections of society through the public roads leading to the Sri Mahadevar Temple at Vaikom. K. Kelappan played a dominant role in the famous Vaikom Satyagraha and was the leader of the Guruvayur Satyagraha in 1932. It was at Gandhiji’s request that he ended his fast at Guruvayur. 480. At which among the following places, All India Muslim League was established in 1906? [A] Dhaka [B] Naypyidaw [C] Islamabad [D] Muscat Correct Answer: A [Dhaka] Explanation: The All India Muslim League was rooted in 1906 at Dhaka. It emerged from the Aligarh Movement, formed originally to promote a modern education for Muslims. Muslim League, political organization of India and Pakistan, founded 1906 as the All-India Muslim League by Aga Khan III. Its original purpose was to safeguard the political rights of Muslims in India. 481. During the independence movement, newspaper ‘Kesari’ was published by_: [A] Subhash Chandra Bose [B] Bal Gangadhar Tilak [C] Mohammad Ali Jinnah [D] Lala Lajpat Rai Correct Answer: B [Bal Gangadhar Tilak] Explanation: Kesari is a newspaper founded in 1881 by Lokmanya Bal Gangadhar Tilak, a prominent leader of the Indian Independence movement. Bal Gangadhar Tilak used to run his two newspapers, the Kesari, in Marathi and Maratha in English from Kesari Wada. The Wada still has the offices of Kesari, and mementos of Tilak, including his writing desk original letters and documents. 482. On which among the following dates, the Second Round Table Conference was held? [A] November 17, 1932 [B] November 12, 1930 [C] September 7, 1931 [D] September 7, 1932 Correct Answer: C [September 7, 1931] Explanation: The second session opened on September 7, 1931. There were three major differences between the first and second Round Table Conferences. By the second namely Congress Representation, National Government and Financial Crisis. 483. Who among the following played a dominant role in establishing the All India Trade Union Congress? [A] Basawon Singh (Sinha) [B] Subodh Banerjee [C] Lala Lajpat Rai [D] Ashok Mehta Correct Answer: C [Lala Lajpat Rai] Explanation: The All India Trade Union Congress is the oldest trade union federations in India and one of the five largest. It was founded on 31 October 1920 in Bombay by Lala Lajpat Rai and Joseph Baptista. It 321 | P a g e

shop.ssbcrack.com

MCQs

INDIAN HISTORY

MCQs

was the primary trade union organization in India. Since then it has been associated with the Communist Party of India. 484. Who among the following was the first Indian woman president to chair the Indian National Congress at Kanpur session of 1925? [A] Sarojini Naidu [B] Annie Beasant [C] Nellie Sengupta [D] Indira Gandhi Correct Answer: A [Sarojini Naidu] Explanation: Sarojini Naidu presided the Indian National Congress at Kanpur session in 1925. 485. Which among the following Charter Acts ended the commercial monopoly of East India Company in India? [A] Charter Act of 1793 [B] Charter Act of 1813 [C] Charter Act of 1833 [D] Charter Act of 1853 Correct Answer: B [Charter Act of 1813] Explanation: Charter Acts of 1813 was an Act of the Parliament of the United Kingdom which renewed the charter issued to the British East India Company, and continued the Company’s rule in India. However, the Company’s commercial monopoly was ended, except for the tea trade and the trade with China. Reflecting the growth of British power in India. 486. Who among the following introduced the subsidiary alliance system in India? [A] Lord Clive [B] Lord Wellesley [C] Lord Cunningham [D] Lord Canning Correct Answer: B [Lord Wellesley] Explanation: The doctrine of subsidiary alliance was introduced by Lord Wellesley, British GovernorGeneral of India from 1798 to 1805. The Nizam of Hyderabad was the first to enter into such an alliance in 1798.Tipu Sultan of Mysore refused to do so, but after the British victory in the Fourth Anglo-Mysore War, Mysore was forced to become a subsidiary state in 1799. The Nawab of Awadh was the next to accept the Subsidiary Alliance, in 1801. 487. Who among the following signed the Treaty of Bassein in 1802 with the British East India Company? [A] Baji Rao II [B] Baji Rao I [C] Sultan Bahadur [D] None of the above Correct Answer: A [Baji Rao II] Explanation: The Treaty of Bassein (Now called Vasai) was a pact signed on December 31, 1802 between the British East India Company and Baji Rao II, the Maratha peshwa of Pune (Poona) in India after the Battle of Poona. The treaty was a decisive step in the dissolution of the Maratha Confederacy, which led to the East India Company’s usurpation of the peshwa’s territories in western India in 1818. 322 | P a g e

shop.ssbcrack.com

MCQs

INDIAN HISTORY

MCQs

488. Shivaji had formed a council of eight ministers that administered the Maratha empire. The council was named as: [A] Nyaya Pradhan [B] Ashta Sena [C] Ashta Siddhi [D] Ashta Pradhan Correct Answer: D [Ashta Pradhan] Explanation: Ashta Pradhan was a council of eight ministers that administered the Maratha empire. The council was formed in 1674 by founding Emperor Chhatrapati Shivaji.The term Ashta Pradhan literally translates to “the Prime Eight”, from the Sanskrit ashta (“eight”) and pradhan (“prime”). The body discharged the functions of a modern council of ministers; this is regarded as one of the first successful instances of ministerial delegation in India. The council is credited with having implemented good governance practices in the Maratha heartland, as well as for the success of the military campaigns against the Mughal Empire. 489. Which among the following events was a major set back to Khilafat Movement? [A] Announcements of concession to Muslims by British Government [B] Pact signed between Indian National Congress and Muslim League [C] Revolution in Turkey [D] Violence in Chauri Chaura Correct Answer: C [Revolution in Turkey] Explanation: In 1922, there was a revolution in Turkey under the leadership of Mustafa Kamal Pasha, under whom a secular government was established over there. This government buried the concept of Khilafat within two years and the result was that Khilafat Conference had lost its raison d ‘etre and was dying thereafter. 490. Which among the following events took place immediately before the massacre at Jallianwalla Bagh? [A] Partition of Bengal [B] llbert Bill [C] Passage of the Rowlatt Act [D] Minto-Morley Reforms Correct Answer: C [Passage of the Rowlatt Act] 491. Which among the following books was authored by Mahatma Gandhi? [A] Hindu View of Life [B] Hind Swaraj [C] Discovery of India [D] My Truth Correct Answer: B [Hind Swaraj] Explanation: Hind Swaraj or Indian Home Rule is a book written by Mohandas K. Gandhi in 1909. It is a book in which he expresses his views on Swaraj, Modern Civilization, Mechanisation etc. 492. Who among the following founded the Indian Society of Oriental Art to revive the ancient art traditions of India? [A] Abanindranath Tagore [B] Ravindranath Tagore [C] Surendranath Ganguly [D] Mukul Dey 323 | P a g e shop.ssbcrack.com

MCQs

INDIAN HISTORY

MCQs

Correct Answer: A [Abanindranath Tagore] Explanation: Abanidranath Tagore had founded Indian Society of Oriental Art in Kolkata to revive the ancient art traditions of India. He was the principal of government school of art and a great artist of modern India. 493. In which year, first census was conducted in India? [A] 1884 [B] 1872 [C] 1881 [D] 1891 Correct Answer: B [1872] Explanation: The first census was conducted in India in 1872, but regular census started in 1881. 494. Martyrdom of Chandrashekhar Azad took place on which among the following dates? [A] February 27, 1931 [B] February 11, 1931 [C] April 13, 1931 [D] August 14, 1931 Correct Answer: A [February 27, 1931] 495. In which of the following languages was the Ghadar Journal was first published? [A] Urdu [B] English [C] Punjabi [D] Pushtu Correct Answer: A [Urdu] Explanation: Ghadar journal was published in Urdu in 1913. After this, it was published in English, Punjabi, Marathi and Pushtu also. 496. On which among the following dates, Jallianwala Bagh Massacre took place? [A] April 16,1919 [B] April 13,1919 [C] April 3,1919 [D] April 14,1919 Correct Answer: B [April 13,1919] 497. Who among the following resigned from the membership of the Viceroy’s Executive Council as a protest against Jallianwala Bagh Massacre? [A] C. P. Ramaswami Iyer [B] Muhammad Ali Jinnah [C] B. N. Sarma [D] Shankaran Nair Correct Answer: D [Shankaran Nair] Explanation: Shankaran Nair resigned from the membership of the Viceroy’s Executive Council as a protect against Jallianwala Bagh Massacre on April 13, 1919.

324 | P a g e

shop.ssbcrack.com

MCQs

INDIAN HISTORY

MCQs

498. Which among the following was the first registered Trade Union of India? [A] Hindu Mazdoor Sabha [B] Bhartiya Mazdoor Sangh [C] Madras Labour Union [D] United Trade Union Congress Correct Answer: C [Madras Labour Union] Explanation: The first registered Trade Union of India was Madras Labour Union which was founded in 1918 by B.P. Wadia along with V. Kalyanasundaram Mudaliar. 499. Who among the following founded the Madras Labour Union? [A] Subodh Banerjee [B] B.P. Wadia. [C] Lala Lajpat Rai [D] Bharat Bhushan Pandey Correct Answer: B [B.P. Wadia.] Explanation: The first Trade Union of India was Madras Labour Union which was founded in 1918 by B.P. Wadia along with V. Kalyanasundaram Mudaliar 500. Who among the following presided over the Surat Session of Indian National Congress in 1907? [A] Dadabhai Naoroji [B] Gopal krishna Gokhale [C] Pandit Madan Mohan Malaviya [D] Ras Bihari Ghosh Correct Answer: D [Ras Bihari Ghosh] Explanation: Ras Bihari Ghosh presided over the Surat session of Indian National Congress in 1907. In this session, the Congress was divided into two wings viz. Moderates and Extremists. 501. Who among the following introduced the Vernacular Press Act? [A] Sir Ashley Eden [B] Alexander John Arbuthnot [C] Lord Lytton [D] Lord Stanley Correct Answer: C [Lord Lytton] Explanation: In 1878, Lord Lytton introduced the Vernacular Press Act to ban the vernacular press in India. The first victim was nationalist Newspaper ‘Soma Prakash’. 502. Who among the following started Ganapati Festival in 1893 and thereby gave it national character? [A] Ganesh Damodar Savarkar [B] Nana Patil [C] Lok manya Tilak [D] Vinoba Bhave Correct Answer: C [Lok manya Tilak] Explanation: Bal Gangadhar Tilak started Ganapati festival in 1893. By doing so, he transformed the traditional Ganapati festival of Maharashtra into a National festival. 503. Which among the following observed the Direct-Action Day on August 16, 1946? [A] Muslim League [B] Christian League 325 | P a g e

shop.ssbcrack.com

MCQs

INDIAN HISTORY

MCQs

[C] Sikh League [D] Hindu League Correct Answer: A [Muslim League] Explanation: Muslim League observed the Direct-Action Day on August 16, 1946. The objective of Muslim league was to create an environment of communal riots and terrorism to exhibit that Hindus and Muslims could not reside together. 504. Who among the following was nominated as first Satyagrahi by Mahatma Gandhi for the Individual Satyagarha of 1940? [A] Vinoba Bhave [B] Jawarharlal Nehru [C] Lal Bahadur Shastri [D] S. Satyamurti Correct Answer: A [Vinoba Bhave] Explanation: The Congress was in a confused state again after the August Offer. The radicals and leftists wanted to launch a mass Civil Disobedience Movement, but here Gandhi insisted on Individual Satyagraha. The Individual Satyagraha was not to seek independence but to affirm the right of speech. The first Satyagrahi selected was Acharya Vinoba Bhave, who was sent to Jail when he spoke against the war. Second Satyagrahi was Jawahar Lal Nehru. Third was Brahma Datt, one of the inmates of the Gandhi’s Ashram 505. The Lucknow session of Indian National Congress that took place in 1916 was presided by__: [A] Rashbihari Ghosh [B] Ambika Charan Majumdar [C] Bhupendra Nath Bose [D] None of the above Correct Answer: B [Ambika Charan Majumdar] Explanation: The Lucknow Session 1916 {presided by Ambica Charan Majumdar) was special in many respects. Firstly, this session brought the moderates and extremists in Congress on common platform again after nearly a decade, particularly due to efforts of Annie Besant. Secondly, Congress and All India Muslim League signed the historic Lucknow Pact. 506. Who among the following presided the historic Lahore session of 1929 of Indian National Congress? [A] Pandit Jawaharlal Nehru [B] Rajendra Prasad [C] Lala Lajpat Rai [D] Pandit Motilal Nehru Correct Answer: A [Pandit Jawaharlal Nehru] Explanation: In 1928, for the first time a constitution for India was drafted by Nehru Committee that was headed by Moti Lal Nehru. A year later, in the Lahore session of December 1929, Congress passed the Poorna Swaraj resolution. It was the same session in which Jawaharlal Nehru was elected as president of the Congress. Here, please note that though the congress passed the Poorna Swaraj Resolution in December 1929, it was a month later on January 26, 1930, when a Pledge of Indian Independence also known as Declaration of Independence was taken. You must note here that while the Poorna Swaraj Resolution was drafted by Jawaharlal Lal Nehru, the “Declaration of Independence” pledge was drafted by Mahatma Gandhi in 1930 and it echoed the essence of American Declaration of Independence. After this pledge January 26, 1930 was declared as Independence Day by Indian National Congress. 326 | P a g e shop.ssbcrack.com

MCQs

INDIAN HISTORY

MCQs

507. Who among the following leaders joined Gandhiji in the Champaran Satyagraha held on April 10, 1917 ? [A] Bal Gangadhar Tilak [B] Subhash Chandra Bose [C] Rajendra Prasad [D] None of these Correct Answer: C [Rajendra Prasad] Explanation: Gandhi arrived in Champaran 10 April 1917 with a team of eminent lawyers: Brajkishore Prasad, Rajendra Prasad, Anugrah Narayan Sinha and others including Acharya kripalani. 508. Who among the following launched the Home Rule Movement in 1916? [A] Frank Besant [B] Helena Blavatsky [C] Charles Knowlton [D] Annie Besant Correct Answer: D [Annie Besant] Explanation: In 1916, Annie Besant launched the Home Rule Movement along with Lokmanya Tilak. She was married at 19 to Frank Besant but separated from him over religious differences. 509. Who among the following presided the Karachi session of Indian National Congress in 1931? [A] Sardar Vallabhbhai Patel [B] Pandit Jawaharlal Nehru [C] Pandit Madan Mohan Malaviya [D] Dr. M A Ansari 510. On which among the following dates, execution of Bhagat Singh took place ? [A] March 19,1931 [B] April 23, 1931 [C] March 23, 1931 [D] March 14, 1931 Correct Answer: C [March 23, 1931] Explanation: Bhagat Singh was convicted and executed at the young age of 23 for his involvement in the assassination of John Saunders (Lahore Conspiracy Case). Saunders was a British police officer. Bhagat Singh and his comrades sought revenge for the death of Lala Lajpat Rai which was caused by brutal police. They were convicted after a trial by s Special Tribunal which sentenced them to death by hanging. On 23 March, 1931, Bhagat Singh, Rajguru and Sukhdev were executed. 511. On which among the following dates, the Gandhi-Irwin Pact was signed? [A] 5 March 1931 [B] 6 March 1931 [C] 7 March 1931 [D] 8 March 1931 Correct Answer: A [5 March 1931] Explanation: The Gandhi–Irwin Pact was a political agreement signed by Mahatma Gandhi and the then Viceroy of India, Lord Irwin on 5 March 1931 before the second Round Table Conference in London. 512. Who among the following was the author of Anand Math? [A] Bankim Chandra Chatterjee [B] Ravindranath Tagore 327 | P a g e

shop.ssbcrack.com

MCQs

INDIAN HISTORY

MCQs

[C] Raja Ram Mohan Roy [D] Bal Gangadhar Tilak Correct Answer: A [Bankim Chandra Chatterjee] Explanation: Anandmath was a novel by Bankim Chandra Chaterjee and published in 1882. The national song of India, Vande Mataram was originally published in this novel. The Sanyasi Rebellion of the late 1700s was the revolt this novel was based upon and was made famous by Bankim Chandra in this novel. Based upon this novel, Anand Math a Hindi Patriotic film was released in 1952. the film was directed by Hemen Gupta. 513. On which among the following dates, National Development Council was set up? [A] 9th August 1960 [B] 6th August 1952 [C] 15th September 1958 [D] 29th August 1960 Correct Answer: B [6th August 1952] Explanation: The National Development Council was set up on 6th August 1952. 514. The Indian National Association formed in Calcutta by whom among the following? [A] Dwarkanath Tagore [B] Surendranath Banerjee [C] Prasanno Kumar Tagore [D] Debendranath Tagore Correct Answer: B [Surendranath Banerjee] Explanation: On July 26 1876, Surendranath Banerjee, along with Anand Monah Bose, founded the Indian National Association in Calcutta. In 1885 , this organization was merged with the Indian National Congress. 515. Who among the following edited and published the newspaper Indian Mirror in 1861? [A] Amitava Ghosh [B] Ravindranath Tagore [C] Sumit Ganguly [D] Manmohan Ghosh and Devendranath Tagore Correct Answer: D [Manmohan Ghosh and Devendranath Tagore] Explanation: In 1861, the newspaper, Indian Mirror, was edited and published by Manmohan Ghosh and Devendranath Tagore from Calcutta. 516. The first meeting of Indian National Congress was held in Bombay in 1885 A.D. under the leadership of __: [A] Dadabhai Naoroji [B] Sir C. Sankaran Nair [C] Woomesh Chandra Banerjee [D] Badruddin Tyabji Correct Answer: C [Woomesh Chandra Banerjee] Explanation: Under the Presidentship of woomesh Chandra Banerjee, the first meeting of Indian National Congress was held in Bombay in 1885. The meeting was attended by 72 English-educated people that included advocates, traders and zamindars. 517. During the times of Governor-General Lord Ellenborough, which among the following acts declared slavery as illegal? 328 | P a g e shop.ssbcrack.com

MCQs

INDIAN HISTORY

MCQs

[A] Act VI [B] Act V [C] Act III [D] Act VIII Correct Answer: B [Act V] Explanation: The Act V of 1843 declared slavery as illegal during the tenure of Governor-General Lord Ellenborough. 518. Who among the following was called by Jahangir as “English Khan”? [A] Sir Thomas Roe [B] William Hawkins [C] Henry Middleton [D] None of the above Correct Answer: B [William Hawkins] 519. Which among the following was the ONLY session of Indian National Congress that was presided by Mahatma Gandhi? [A] Amaravati [B] Belgaum [C] Nagpur [D] Karachi Correct Answer: B [Belgaum] Explanation: Gandhiji became the president of Indian National Congress in 1924 at the Belgaum session of Indian National Congress. 520. The birth name of which among the following famous mystics was Gadadhar Chattopadhyay? [A] Swami Vivekananda [B] Ram Krishna Paramhansa [C] Sri Aurobindo [D] Swami Prabhupada Correct Answer: B [Ram Krishna Paramhansa] Explanation: The original name of Ramakrishna Paramhansa was Gadadhar Chattopadhyay. 521. The Chauri Chaura Incident paved the way for end of which among the following movements? [A] Civil Disobedience Movement [B] Non-Cooperation Movement [C] Quit India Movement [D] Kheda Satyagraha Correct Answer: B [Non-Cooperation Movement] Explanation: The Non-cooperation movement was on its pinnacle in all of north India. In South, though it was luke warm. On February 4, 1922, a mob of 2000 people gathered to picket a liquor shop at Chauri Chaura, a town near Gorakhpur, Uttar Pradesh. The local administration sent armed police to control the situation. The Police, tried to disperse the crowd by firing two shots in air. So stone pelting started. The police fired and killed 3 people. The result was that outrageous mob set the Police Chauki on fire an d all 23 Police wallas inside got burnt alive. On 12 February 1922, when the Congress leaders met at Bardoli, Gandhi decided to withdraw the Noncooperation movement. It was a bit controversial but by that time Gandhi’s figure was respected by every Congressman. Thus, they accepted this decision, but they got demoralized and disintegrated. 329 | P a g e shop.ssbcrack.com

MCQs

INDIAN HISTORY

MCQs

Gandhi was arrested on 10 March 1922 and trial led at Ahmadabad. A simple prison of 6 years was awarded to him. 522. Which among the following towns / cities of Karnataka is most famous for monuments of Vijayanagar Empire? [A] Gulbarga [B] Belgaum [C] Gadag [D] Hampi Correct Answer: D [Hampi] Explanation: Vijayanagara Empire or Karnata Empire or Kingdom of Bisnegar was established in 1336 by Harihara-I and his brother Bukka Raya I of Sangama Dynasty. Its ruins are located in current day Hampi in Karnataka. Four dynasties – Sangama, Saluva, Tuluva and Aravidu – ruled Vijayanagar from A.D. 1336 to 1672. The most important temples of Vijaynagar Empire include Vitthalaswamy and the Hazara Ramaswamy temples at Hampi. The Tadapatri and Parvati temples at Chidambaram, Varadaraja and Ekambaranatha temples at Kanchipuram. 523. In which among the following years, Sabarmati Ashram was established by Mahatma Gandhi? [A] 1910 [B] 1912 [C] 1915 [D] 1918 Correct Answer: C [1915] 524. At which of the following places, Green Pamphlet was issued by Mahatma Gandhi? [A] Calcutta [B] Surat [C] Rajkot [D] Sabarmati Correct Answer: C [Rajkot] 525. At which among the following sessions, the All India Muslim league passed the Pakistan Resolution on 23 March 1940? [A] Karachi [B] Hyderabad [C] Lahore [D] Lucknow Correct Answer: C [Lahore] Explanation: The Pakistan demand was formalised gradually. On 23 March 1940, the League moved a resolution demanding a measure of autonomy for the Muslim majority areas of the subcontinent. This ambiguous resolution never mentioned partition or Pakistan The name Pakistan or Pak-stan (from Punjab, Afghan, Kashmir, Sind and Baluchistan) was coined by a Punjabi Muslim student at Cambridge, Choudhry Rehmat Ali, who, in pamphlets written in 1933 and 1935, desired a separate national status for this new entity. 526. At which among the following sessions of the Muslim League, Mohammed Ali Jinnah had invented a new slogan “Divide and Quit”? 330 | P a g e shop.ssbcrack.com

MCQs

INDIAN HISTORY

MCQs

[A] Lucknow Session 1931 [B] Karachi Session 1933 [C] Lahore Session 1940 [D] Karachi Session 1943 Correct Answer: D [Karachi Session 1943] 527. Simon Commission had visited India during the times of which among the following Viceroys? [A] Lord Irwin [B] Lord Chelmsford [C] Lord Willingdon [D] Lord Linlithgow Correct Answer: A [Lord Irwin] Explanation: On April 3, 1926 Lord Irwin was appointed 30th Viceroy and Governor-General of India. In 1927, British government appointed a commission under the chairmanship of Sir John Simon. The Commission was appointed to study the reforms of 1919 and suggest further measures for Constitutional reforms. The Commission had no Indian member in it. The Indians boycotted this all-White commission. 528. “Go Back to Vedas“. Who among the following gave this motto? [A] Swami Vivekananda [B] Swami Dayananda Saraswati [C] Ramkrishna Paramahansa [D] None of the above Correct Answer: B [Swami Dayananda Saraswati] Explanation: Swami Dayanand’s greatest asset was his mastery over the Sanskrit language and the Vedas. In 1875, he founded the Arya Samaj. Its main aim was to propagate the true knowledge of the Vedas and discard all evils that had crept into the Hindu society later in its name. He opposed untouchability. He was similarly opposed to polytheism, avataravada and ritualism. His slogan was ‘go back to the Vedas’ whose authority he accepted. For the first time in the history of India, the Vedas were printed in India under his patronage. Personally, his most important work was Satyartha Prakasha (The Light of Truth). His followers started a Dayanand Anglo-Vedic (DAV) School and College in Lahore (now in Pakistan) in 1886. 529. The famous Vidyasagar Setu is located in which among the following cities? [A] Patna [B] Kolkata [C] Lucknow [D] Chennai Correct Answer: B [Kolkata] Explanation: Vidyasagar Setu is also known as Second Hooghly Brdige. It is named after Ishwarchandra Vidyasagar 530. Plassey, which is famous for the Battle of Plassey, is located in which among the following current states of India? [A] West Bengal [B] Bihar [C] Chhattisgarh [D] Jharkhand Correct Answer: A [West Bengal] 331 | P a g e shop.ssbcrack.com

MCQs

INDIAN HISTORY

MCQs

531. In which year and at which place, India’s national anthem was sung for the first time? [A] 1911, Kolkata [B] 1912, Lucknow [C] 1913, Delhi [D] 1914, Bombay Correct Answer: A [1911, Kolkata] 532. Which among the following was the correct title of National Anthem of India, when it was originally penned by Rabindranath Tagore? [A] Jana Gana Mana [B] Bharat Bhagya [C] Bharat Bhagya Vidhata [D] Bharat Vidhata Correct Answer: D [Bharat Vidhata] Explanation: The song Jana-gana-mana, composed originally in Bengali by Rabindranath Tagore, was adopted in its Hindi version by the Constituent Assembly as the national anthem of India on Tuesday, 24 January 1950. National Anthem Jana Gana Mana was first sung in Calcutta Session of the Indian National Congress on 27 December 1911. 533. The 1896 session of Indian National Congress is best known for which among the following? [A] India’s national song was sung for the first time [B] India’s national anthem was sung for the first time [C] India’s tricolor flag was hoisted for the first time [D] None of the above Correct Answer: A [India’s national song was sung for the first time] Explanation: The song Vande Mataram, composed in Sanskrit by Bankimchandra Chatterji, was incorporated in his famous novel Ananda Math (1882). It has an equal status with the National Anthem. Later the song was set to tune by Rabindranath Tagore and sung for the first time before the gathering at the 12th annual session of the Indian National Congress held in 1896 in Calcutta. It was declared as the National Song in 1937 through a resolution. The English translation of the stanza was rendered by Sri Aurobindo. 534. Which among the following was the first state in India in the pre-independence times, when the compulsory primary education was launched for the first time? [A] Baroda [B] Arcot [C] Coorg [D] Hyderabad Correct Answer: A [Baroda] Explanation: The first law on compulsory education was introduced by the State of Baroda in 1906. This law provided for compulsory education for boys and girls in the age groups of 7–12 years and 7–10 years respectively 535. Who among the following is considered to be the last Independent Nawab of Bengal? [A] Siraj ud Daula [B] Mir Kasim 332 | P a g e

shop.ssbcrack.com

MCQs

INDIAN HISTORY

MCQs

[C] Mir Jafar [D] Shujaud daula Correct Answer: A [Siraj ud Daula] 536. Who among the following is known to have founded the “Academic Association”? [A] Henry Vivian Derozio [B] Madhusudan Datta [C] Madan Mohan Tarkalankar [D] Surendranath Tagore Correct Answer: A [Henry Vivian Derozio] 537. The name of P Mitra (Pramathanath Mitra) is associated with the foundation of which among the following revolutionary organizations? [A] Anushilan Samiti [B] India House [C] Jugantar [D] None of the above Correct Answer: A [Anushilan Samiti] Explanation: First proper revolutionary secret society known as Anushilan Samiti (Calcutta) was founded on 24 March 1902 by Pramath Nath Mitra better known as P. Mitra. The people associated with this samiti were Sri Aurobindo, Deshabandhu Chittaranjan Das, Surendranath Tagore, Jatindranath Banerjee, Bagha Jatin, Bhupendra Natha Datta, Barindra Ghosh etc. Out of them Bhupendra Nath Datta was brother of Swami Vivekananda. Barindra Ghosh was sent to Paris to learn the science of Bomb Making and here he came in touch were Madam Kama. Madam Kama was already associated with the India House and the Paris India Society. 538. Ahsan Manzil Palace in Dhaka, is known for which among the following landmarks in Modern Indian History? [A] Establishment of All India Muslim League [B] Pakistan Resolution [C] Jinnah-Gandhi Talks [D] None of the above Correct Answer: A [Establishment of All India Muslim League] 539. At which among the following places, Indian National Congress adopted the resolution of ‘Poorna Swaraj’? [A] Calcutta [B] Lahore [C] Delhi [D] Karachi Correct Answer: B [Lahore] Explanation: In Lahore session of 1929, Congress passed the Poorna Swaraj resolution. It was the same session in which Jawaharlal Nehru was elected as president of the Congress. Congress passed the Poorna Swaraj Resolution in December 1929, it was a month later on January 26, 1930, when a Pledge of Indian Independence also known as Declaration of Independence was taken. You must note here that while the Poorna Swaraj Resolution was drafted by Jawaharlal Lal Nehru, the “Declaration of Independence” pledge was drafted by Mahatma Gandhi in 1930 and it echoed the essence 333 | P a g e

shop.ssbcrack.com

MCQs

INDIAN HISTORY

MCQs

of American Declaration of Independence. After this pledge January 26, 1930 was declared as Independence Day by Indian National Congress. 540. The chief base of Indian Union Muslim League (IUML) is in which among the following states of India? [A] Tamil Nadu [B] Kerala [C] Uttar Pradesh [D] Jammu & Kashmir Correct Answer: B [Kerala] 541. The ‘Tebhega’ movement of 1946 is associated with which among the following current states of India? [A] Andhra Pradesh [B] West Bengal [C] Bihar [D] Odishna Correct Answer: B [West Bengal] Explanation: The Tebhaga movement was led by the share croppers of the Bengal region against the oppressive jotedars in 1946-47. The uprising was due to the share cropping system that prevailed in the Bengal. 542. The name of Barindra Ghosh is associated with which among the following newspapers? [A] Indian Opinion [B] Jugantar [C] Ghadar [D] Maharatta Correct Answer: B [Jugantar] Explanation: Jugantar was a Bengali revolutionary paper founded in 1906 by Barindra kumar Ghosh, Abinash Bhattacharya and Bhupendranath Dutta. 543. Which among the following Sikh Gurus introduced the Gurmukhi Script? [A] Guru Angad [B] Guru Gobind Singh [C] Guru Arjun Singh [D] Guru Harrai Correct Answer: A [Guru Angad] Explanation: Guru Nanak was the founder of Sikhism. Guru Nanak appointed Guru Angad (Lehna) as his successor and he was the second Guru of sikhs. Guru Angad compiled the compositions of Guru Nanak, to which he added his own in a new script known as Gurmukhi. 544. Azad Hind Fauz was formed in which among the following countries? [A] India [B] Singapore [C] Japan [D] Germany Correct Answer: B [Singapore] 334 | P a g e

shop.ssbcrack.com

MCQs

INDIAN HISTORY

MCQs

Explanation: Subhash Chandra Bose and Rash Behari Bose launched the Indian Independence League and the Indian National Army (INA), also called Azad Hind Fauj, at Singapore in 1943. 545. Bhagat Singh, Rajguru and Sukhdev along with Jaigopal were involved in which among the following conspiracy case? [A] kanpur Conspiracy Case [B] Lahore Conspiracy Case [C] Bhagalapur Conspiracy Case [D] Alipore Bomb case Correct Answer: B [Lahore Conspiracy Case] Explanation: To avenge the killing of Lal Lajpat Rai, Bhagat Singh, Raj guru, Jai Gopal and Sukh Dev conspired to kill the police chief, Scott. But they shot on the DSP – J. P. Saunders, who was killed on the spot. Bhagat Singh immediately fled from Lahore and to avoid recognition, he cut his beard and hair. Later he was trailed in this Lahore Conspiracy Case when he was captured after throwing bomb in Delhi Assembly. 546. “Swaraj ha maza janmasidha adhikar aahe ani to mi milavinach”. The word Swaraj was first of all used by Balgangadhar Tilak. Where did Balgangadhar Tilak made the above declaration in Marathi? [A] In Pune Jail [B] In Yavarda Jail [C] In Court [D] In Mandalay Jail Correct Answer: C [In Court] Explanation: In Court, Tilak declared Swaraj ha maza janmasidha adhikar aahe ani to mi milavinach” in court when he was given 7 years rigorous imprisonment in sedition case against him. Later Tilak was sent to Mandalay jail for six years. 547. During the 1857 Mutiny, who among the following was Governor General of India? [A] Lord Canning [B] Lord Elgin [C] Lord Dalhousie [D] Lord Elleborough Correct Answer: A [Lord Canning] 548. Who among the following established Swadesh Bandhab Samiti? [A] Ashwini Kumar Datta [B] Bagha Jatin [C] Bhavabhushan Mitra [D] Narendranath Bhattacharya Correct Answer: A [Ashwini Kumar Datta] Explanation: Swadesh Bandhab Samiti was founded by Ashwini Kumar Dutta. It aimed to promote the consumption of indigenous products and boycott foreign goods. 549. Who among the following was placed on the throne of Nawab of Bengal after the Battle of Buxar? [A] Shiraj-ud-daula [B] Mir Kasim 335 | P a g e

shop.ssbcrack.com

MCQs

INDIAN HISTORY

MCQs

[C] Mir Jafar [D] Najimuddin Ali Khan Correct Answer: C [Mir Jafar] Explanation: Battle of Buxar took place in the year 1764. Mir Jafar became the new Nawab of Bengal after Battle of Buxar. 550. There was one socialist leader, who ran away from the Hazaribagh Prison and joined the Quit India Movement. Identify that leader from the given options: [A] Ram Manohar Lohia [B] Minoo Masani [C] Jayprakash Narayan [D] Achyut Patwardhan Correct Answer: A [Ram Manohar Lohia]

336 | P a g e

shop.ssbcrack.com

MCQs

INDIAN HISTORY

MCQs

PREVIOUS YEAR QUESTIONS ASKED IN CDS EXAMINATION 1. Consider the following statements: [2016-I] 1. In Hind Swaraj, Mahatma Gandhi formulates a conception of part life for the individual as well as the society. 2. Hind Swaraj was the outcome of the experience of Gandhi’s prolonged struggle against colonial Raj in India Which of the statement given above is/ are correct? (a) 1 only (b) 2 only (c) Both 1 and 2 (d) Neither 1 nor 2 2. Who among the following archaeologists was the first to identify similarities between a pre - Harappan culture and the mature Harappan culture? [2016-I] (a) Amalananda Ghosh (b) Rakhaldas Banerji (c) Dava Ram Sahni (d) Sir John Marshall 3. Which one of the following is the common element between the Kailashnatha Temple at Ellora and the Shore Temple at Mamallapuram? [2016-I] (a) Both are examples of Nagara architecture (b) Both are carved out from solid rocks (c) Both are Gupta period temples (d) Both were built under the patronage of Pallava Kings 4. Which of the following is/ are not depicted in the Rajput paintings? 1. The stories of Krishna 2. Ragas and Raginis 3. The deeds of Hamza 4. The deeds of Babur Select the correct answer using the code given below: (a) 1, 2 and 3 (b) 2, 3 and 4 (c) 3 and 4 only (d) 4 only

[2016-I]

5. Consider the following statements about Alauddin Khilji's market policy: [2015-II] 1. He placed markets under the control of a high officer called 'Shahna' for strictly controlling the shopkeepers and prices. 2. In order to ensure a regular supply of cheap food grains, he ordered the land revenue from Doab region to be paid directly to the State. Which of the statements given above is/are correct? (a) 1 only (b) 2 only (c) Both 1 and 2 (d) Neither 1 nor 2

337 | P a g e

shop.ssbcrack.com

MCQs

INDIAN HISTORY

MCQs

6. Match List-I with List-II and select the correct answer using the code given below the Lists: [2015-II] List-I List-II (King) (Region) A. Shashanka 1. Assam B. Kharavela 2. Maharashtra C. Simuka 3. Orissa D. Bhaskara Varman 4. Bengal Code: A B C D (a) 4 2 3 1 (b) 1 3 2 4 (c) 4 3 2 1 (d) 1 2 3 4 7. In ancient India, the 'Yaudheyas' were (a) A sect of the Buddhism (b) A sect of the Jainism (c) A republican tribe (d) Vassals of the Cholas

[2015-II]

8. Which of the following statements about Jyotirao Phule's Satyashodhak Samaj Movement in Maharashtra is/ are true? [2015-II] 1. The Satyashodhak Samaj was set up in 1873. 2. Phule argued that Brahmins were the progeny of 'alien' Aryans. 3. Phule's focus on the Kunbi peasantry in the 1880s and 1890s led to a privileging of Maratha identity. Select the correct answer using the code given below. (a) 1 and 2 only (b) 2 only (c) 1 and 3 only (d) 1, 2 and 3 9. Which of the following statements about Maulvi Ahmadullah Shah, who played an important part in the Revolt of 1857, is/are correct? [2015-II] 1. He was popularly known as Danka Shah or the Maulvi with a drum. 2. He fought in the famous Battle of Chinhat. 3. He was killed by British troops under the command of Henry Lawrence. Select the correct answer using the code given below. (a) 1 only (b) 1 and 3 (c) 2 and 3 (d) 1 and 2 10. Which of the following statements about Mahatma Gandhi's South African experience (1893 - 1914) is/are true? [2015-I] 1. Muslim merchants were actively involved in Gandhian political movements in South Africa. 2. In 1906, Gandhi led a campaign in Cape Town against the ordinance of compulsory registration and passes for Indians. 3. Gandhi began his political career with struggles against the imposition of excessive taxes on Indians in Cape Town. 338 | P a g e

shop.ssbcrack.com

MCQs

INDIAN HISTORY

MCQs

Select the correct answer using the code give below: (a) 1 and 2 only (b) 3 only (c) 1, 2 and 3 (d) 1 only 11. Which of the following sets of newspapers reflected the concerns of educated Indian Muslims during the Khilafat Movement? [2015-I] (a) Comrade and Hamdard (b) Comrade and Hindustan Times (c) Zamindar and Muslim Voice (d) Comrade, Hamdard, Zamindar and Al Hilal 12. Which of the following statements about the social reformer, Raja Ram Mohan Roy, is false? [2015-I] (a) Ram Mohan Roy belonged to the gentry’s class whose power had been diminished because of the imposition of the Permanent Settlement (b) He studied both Vedantic Monism and Christian Unitarianism. (c) He translated the Upanishads into Bengali. (d) His first organization was the Atmiya Sabha, founded in Calcutta in 1815. 13. Which of the following characteristics about the state of Travancore in 18th century Kerala is/are correct? [2015-I] 1. Travancore was ruled by Marthanda Varma from 1729 to 1758. 2. Travancore built a strong army and defeated the Dutch in 1741. 3. Travancore was an important centre of learning. Select the correct answer using the code given below: (a) 1 and 2 only (b) 2 only (c) 1, 2 and 3 (d) 1 only 14. Which of the following is not a central tenet of Socialism? (a)Historical Materialism (b)Dialectical Materialism (c)Alienation and Class Struggle (d)Individual Freedom

[2015-I]

15. Which of the statements given below about the Mughal rule in India is false? [2014-II] (a) Peasant communities were a united and (b) There was an abundance of homogeneous group food grain (c) The State encouraged these crops that (d) Most regions produced two brought in more revenue crops in a year 16. The Ghadar party, formed in the USA, was determined to start a revolt in India. Which among the following provinces did the party choose to begin its armed revolt? [2014-II] (a) Punjab (b) Bengal (c) United Provinces (d) Bihar 17. The ruins of Vijayanagara at Hampi were brought to light in 1800 by (a) Colonel Colin Mackenzie (b) Sir John Shore (c) Andrew Fraser (d) John Marshall

[2014-II]

18. Consider the following statements about colonial economy of Vietnam (Indo-China): 1. The colonial economy in Vietnam was primarily based on rice cultivation and rubber plantations. 2. All the rubber plantations in Vietnam were owned and controlled by small Vietnamese elite. 339 | P a g e shop.ssbcrack.com

MCQs

INDIAN HISTORY

MCQs

3. Indentured Vietnamese labour was widely used in rubber plantations. 4. Indentured labourers worked on the basis of contracts that did not specify any rights of labourers, but gave immense power to the employers. Which of the statements given above is/are correct? [2014-II] (a) 1, 3 and 4 (b) 1 and 4 only (c) 2 and 3 (d) 1 only 19. Which of the four linguistic regions in South India remained unaffected by the Non-Cooperation Movement (1921-22)? [2014-II] (a) Kerala (b) Tamil Nadu (c) Andhra Pradesh (d) Karnataka 20. Which of the following statements about the penetration of English into Bengal is/are correct? 1. Job Charnock arrived in Sutanati in August 1690 and laid the foundation of Calcutta which later became the heart of the British Indian Empire. 2. The French East India Company built a fort near Fort William in Calcutta. Select the correct answer using the code given below: [2014-II] (a) 1 only (b) 2 only (c) Both 1 and 2

(d) Neither 1 nor 2

21. Consider the following statements about Sher Shah’s administration 1. He divided his empire into Sarkars, which were further subdivided into Parganas. 2. The Sarkars and the Parganas were directly administered by Sher Shah without the help of any other officials. Which of the statements given above is/are correct? [2014-I] (a) Only 1 (b) Only 2 (c) Both 1 and 2 (d) Neither 1 nor 2 22. The University of Nalanda was set-up by which Gupta ruler? (a) Kumaragupta II (b) Kumaragupta I (c) Chandragupta II (d) Samudragupta

[2014-I]

23. The Fourth Buddhist Council was held in Kashmir under the leadership of (a) Bindusara (b) Ashoka (c) Kunal (d) Kanishka

[2014-I]

24. Consider the following statements regarding Indian Feudalism in the early medieval period [2014-I] 1. The revenue assignments were called Bhoga. 2. The hereditary chiefs neither collected revenues nor assumed administrative powers. Which of the statements given above is/are correct? (a) Only 1 (b) Only 2 (c) Both 1 and 2

(d) Neither 1 nor 2

25. Which one of the following statements about ancient Indian Mahajanapadas is correct? [2014-I]

340 | P a g e

shop.ssbcrack.com

MCQs

INDIAN HISTORY

(a) All Mahajanapadas were oligarchies, where power was exercised by a group of people (c) No army was maintained by the Mahajanapadas

MCQs

(b) All Mahajanapadas were located in Eastern India (d) Buddhist and Jain texts list sixteen Mahajanapadas

26. Consider the following statements about the causes of success of the American Revolution [2013-II] 1. The remoteness of the American continent and British ignorance of the American continent led to the success of the Americans. 2. The fierce spirit of liberty drove the Americans to success. 3. The American military forces were superior to the British. Which of the statements given above is/are correct? (a) Only 1 (b) 1 and 2 (c) Only 2 (d) 1 and 3 27. Among the precious stones, the most extensive foreign trade during the Gupta age was that of [2013-II] (a) Diamonds (b) Ruby (c) Pearl (d) Sapphire 28. Sangam literature formed a very important source for the reconstruction of the history of South India. It was written in [2013-II] (a) Tamil (b) Kannada (c) Telugu (d) Malayalam 29. Consider the following statements about Satyagraha 1. It was first experimented by Gandhiji in South Africa. 2. It was first experimented in India at Champaran Which of the statements given above is/are correct? (a) Only 1 (b) Only 2 (c) Both 1 and 2 (d) Neither 1 nor 2

[2013-II]

30. Which one among the following events is associated with French Revolution? [2013-II] (a) Calling of the Estates General (b) Guillotine (c) Battle of Concord (d) Tennis Court Oath

341 | P a g e

shop.ssbcrack.com

MCQs

INDIAN HISTORY

MCQs

CDS PREVIOUS YEAR QUESTIONS – SOLUTION 1. (b) Hind Swaraj or Indian Home Rule is a book written by Mahatma Gandhi in which he expresses his views on Swaraj, modern civilisation, mechanisation, etc. 2. (a) Amalananda Ghosh was the first archaeologist to identify similarities between a pre-Harappan culture and the mature Harappan culture. 3. (c) Kailasanatha Temple at Ellora and the Shore temple at Mamallapuram were built around the same time both are Gupta period temples. 4. (c) 'The deeds of Hamza’ and 'The deeds of Babar' are not depicted in the Rajput Paintings. 5. (c) Alauddin Khilji was the second ruler of Khilji dynasty (1296-1316 reigning period). He is famous for market reforms and price control. He did so for maintaining a large army. By controlling the item’s price, he was able to pay salary to soldiers. 6. (c) Shashanka - Bengal, Kharavela - Orissa, Simuka -Maharashtra, Bhaskara - Assam. 7. (c) Yaudheyas were related with Johiya Kshatriyas residing near Muttan (Presently in Pakistan). 8. (b) Jyotirao Phule (11 April 1827- 28 Nov 1890) a social reformer, writer and theologist organised an organisation - Satyashodhak Samaj Movement, He opined that Brahmins are descendants of alien Aryans. 9. (b) Maulvi Ahmadullah Shah was a talukdar in Faizabad (UP). His taluk was confiscated by the British government. He was an important character in the battle of 1857. He was famous as Danka Shah in public. He took part in the battle of Chinhat. 10. (d) In 1893, Mahatma Gandhi accepted an offer from a firm of Muslims to represent them legally in Pretoria, the capital of Transvaal in the Union of South Africa. They were actively involved in Gandhi movement, a political movement in South Africa. 11. (d) Educated Muslims were the editors of Comrade, Hamdard, Zamindar and Al Hilal. Maulana Mohammad Ali Jauhar launched the Urdu weekly Hamdard and in English the Comrade in 1911. The founding editor of this newspaper was Zafar Ali Khan, a poet, intellectual, writer, Muslim nationalist and supporter of All India Muslim League's Pakistan Movement. The Al-Hilal was a weekly Urdu language newspaper established by Maulana Abul Kalam Azad. 12. (a) Raja Ram Mohan Roy belonged to the gentry class whose power had been diminished because of the imposition of the Permanent Settlement. 13. (c) Marthanda Varma was king of Travancore from 1729 to 1758. The Travancore-Dutch War was fought between the Dutch East India Company (VOC) and the Indian kingdom of Travancore, culminating in the Battle of Colachel in 1741. Travancore became the second most prosperous princely state in British India with reputed achievements in education, political administration, public work and social reforms. 14. (c) Alienation and Class Struggle are the tenets of Capitalism. 15. (a) Peasant communities were a heterogeneous group on the basis of caste and other caste like distinctions. 16. (a) The Ghadar Party was an organization founded by Punjabi Indians in the United States and Canada with the aim of securing India's independence from British rule. After the outbreak of World War I, Ghadar party members returned to Punjab to agitate for rebellion alongside the Babbar Akali Movement. 17. (a) The ruins were brought to light by an engineer and antiquarian named Colonel Colin Mackenzie (Surveyor General of India) in 1800 AD. 342 | P a g e

shop.ssbcrack.com

MCQs

INDIAN HISTORY

MCQs

18. (a) Rubber plantation in Vietnam was owned and controlled by French and small Vietnamese elite. 19. (d) Most of the South Indians participated in the movement except Karnataka which remained unaffected. 20. (a) Sutanuti was one of the three villages which were merged to form the city of Kolkata (formerly Calcutta) in India. Job Charnock, an administrator with the British East India Company is traditionally credited with the honour of founding the city. Fort William was established to protect East India Company's trade in the city of Calcutta, the principal town of the Bengal Presidency. In 1756, with the possibility of conflict with French forces, the British began building up the fort's strengths and defenses. 21. (a) Sher Shah divided the entire Kingdom in 47 divisions and called them "Sarkars". These Sarkars were divided into Parganas. Every Pargana was under a Shikqdar, who looked into the law and order of his Pargana. 22. (b) University of Nalanda was set up by Gupta king Kumargupta I in 415 A.D. 23. (d) The Fourth Buddhist Council was held at Kundalvana, Kashmir in 72 AD under the patronage of Kushan king Kanishka and the president of this council was Vasumitra, with Avaghosa as his deputy. This council distinctly divided Buddhists into 2 sects- Mahayan & Hinayan. 24. (a) Bhoga included taxes of fruits, wood, foodgrains etc to the king. They were collected by hereditary chiefs. 25. (d) According to Angunttara Nikaya (Budhist text) and Bhagavati sutra (Jain text) there were 16 Mahajanapadas in ancient India. 26. (b) The causes of the success of American revolution include spirit of freedom of American people and less knowledge about the American continent by the Britishers. 27. (d) Sapphire was the most precious stone in terms of foreign trade during the Gupta period. 28. (a) The poems belonging to the Sangam literature were composed by Tamil poets. 29. (c) The term Satyagraha was coined by Gandhiji. He used it in South Africa for Indian rights. And in India it was first experimented in Champaran District of Bihar. 30. (b) A guillotine is an apparatus designed for carrying out executions by beheading. The device is best known for its use in France, in particular during the French Revolution.

343 | P a g e

shop.ssbcrack.com

MCQs

INDIAN HISTORY

MCQs

PREVIOUS YEAR QUESTIONS ASKED IN NDA EXAMINATION The following 2 (Two) items consist of two statements, Statement I and Statement II. Examine these two statements carefully and select the correct answer using the code given below: [2016-II] Code: (a) Both the statements are individually true and Statement II is the correct explanation of Statement I (b) Both the statements are individually true but Statement II is not the correct explanation of Statement I (c) Statement I is true but Statement II is false (d) Statement I is false but Statement II is true 1. Statement I: The city of Rome revived in a spectacular way in the 15th century. Statement II: From the 15th century onwards, artists were known individually by name, not as member of a group or a guild, in Roman society. 2. Statement I: Chartism was the political reform campaign for democratic rights which swept over Britain between 1838 and 1848. Statement II: The trade union movement declined by the 1830s as it failed to fulfil the aspirations of the working class. 3. Who among the following invented the power loom that revolutionized the cotton textile industry? [2016-II] (a) Edmund Cartwright (b) Samuel Crompton (c) Richard Arkwright (d) James Hargreaves 4. Subsidiary Alliance was a system devised by [2016-II] (a) Lord Wellesley (b) Lord Dalhousie (c) Lord Canning (d) Lord Ripon 5. Consider the following statements: 1. Periplus is a Greek word meaning sailing around. 2. Erythraean was the Greek name for the Mediterranean Sea. Which of the statements give above is/are correct? (a) 1 only (b) 2 only [2016-II] (c) Both 1 and 2 (d) Neither 1 nor 2 6. Which of the following was/were the feature(s) of Lenin’s New Economic Policy (NEP) for the Soviet Union? 1. Private retail trading was strictly forbidden 2. Private enterprise was strictly forbidden 3. Peasants were not allowed to sell their surplus 4. To secure liquid capital, concessions were allowed to foreign capitalists, but the State retained the option of purchasing the products of such concerns Select the correct answer using the code given below: [2016-II] (a) 1 only (b) 2 and 3 (c) 1, 2 and 4 (d) 4 only 7. Which of the following was/were the reasons for the defeat of British during the American War of Independence? 1. The remoteness of the American Continent and the lack of good roads 2. The British authorities failed to rally the loyalist Americans 3. The Americans benefited from the extraordinary military leadership of George Washington 4. The Americans had access to superior arms and ammunition 344 | P a g e

shop.ssbcrack.com

MCQs

INDIAN HISTORY

MCQs

Select the correct answer using the code given below: (a) 1,2 and 4 (b) 3 only (c) 2 and 3 only (d) 1, 2 and 3

[2016-II]

8. Consider the following statements: 1. British colonialism continued to grow steadily in the 18th and 19th centuries 2. Raw cotton for the textile industry in Britain during the industrial revolution needed to be imported. Which of the statements given above is/are correct? (a) 1 only (b) 2 only [2016-II] (c) Both 1 and 2 (d) Neither 1 nor 2 9. Among the following was the author of Neel Darpan, published in 1860? (a) Reverend James Long (b) Michael Madhusudan Dutta (c) Raja Ram Mohan Roy (d) Dinabandhu Mitra 10. Which one of the following organizations was started by Jyotirao Phule in 1873? (a) Prarthana Samaj (b) Bahujan Samaj (c) Justice Party (d) Satyashodhak Samaj

[2016-II]

[2016-II]

11. Although used earlier by French and German writers, the term ‘Industrial Revolution’ in English was first popularized by [2016-II] (a) Adam Smith (b) Arnold Toynbee (c) James Mill (d) Bertrand Russell 12. In the 19th century, the majority of the workers in Japan’s modem industries were mainly [2016-II] (a) Japanese men and children (b) Japanese women and Chinese men (c) women (d) Japanese and Chinese men 13. Consider the following statements about the Constitution of India: 1. A Member of Parliament enjoys freedom of speech in the Parliament as a Parliamentary Privilege protected by the Constitution of India. 2. The Constitution has vested the power to amend the Constitution in the Parliament. Which of the statements given above is/are correct? (a) 1 only (b) 2 only [2016-II] (c) Both 1 and 2 (d) Neither 1 nor 2 14. Which one of the following statements about the Union Executive in India is correct? [2016-II] (a) According to the Constitution of India, the total number of members of the Council of Ministers cannot exceed 20% of the total number of Members of the House of the People (b) The rank of the different Ministers is determined by the President (c) The Ministers are appointed by the President on the advice of the Prime Minister (d) Only a member of either House of Parliament can be appointed as a Minister 15. Which one of the following statements about Money Bill is correct? [2016-II] (a) A Bill shall be deemed to be a Money Bill only (b) A Money Bill shall be introduced in the if it provides for imposition of fines or penalties Rajya Sabha (c) The Rajya Sabha can reject the Money Bill (d) The Speaker of the Lok Sabha finally decides if it is a Money Bill, should any dispute about it arise

345 | P a g e

shop.ssbcrack.com

MCQs

INDIAN HISTORY

MCQs

16. Which one of the following powers of the Prime Minister of India is codified in the Constitution of India itself? [2016-II] (a) The power of advising the President as (b) The power to allocate business amongst regards the appointment of other Ministers the Ministers (c) The power to summon the meeting of the (d) The power to transfer a Minister from one Cabinet Department to another 17. The cylindrical stone seals were used in which civilization? (a) Harappan (b) Egyptian (c) Roman (d) Mesopotamian

[2016-II]

18. Which of the following statements with regards to the Civil Disobedience movement is/are correct? 1. The movement received massive response from business groups and peasantry 2. The movement coincided with large scale labour upsurge in Maharashtra 3. The movement was marked by the mass participation of lawyers and students Select the correct answer using the code given below: [2016-II] (a) 1 only (b) 1 and 2 only (c) 2 and 3 only (d) 1, 2 and 3 19. Which of the following are the features of the ideology of utilitarianism? 1. Utilitarian’s believed that all value derives from land 2. The most celebrated spokesmen of utilitarianism were Jeremy Bentham and John Stuart Mill 3. Utilitarians were advocates of the idea that Idea could be ruled through indigenous laws and customs 4. Utilitarians were advocates of the idea of the ‘greatest good for the greatest number of people’ Select the correct answer using the code given below: [2016-II] (a) 1 ad 4 only (b) 2 and 4 only (c) 1, 2 and 4 (d) 2 and 3 20. He industrial revolution in England had a profound impact on the lives of people. Which of the following statements are correct about that? 1. Women were the main workers in the silk, lacemaking and knitting industries 2. Factory managers were well aware of the health risks of industrial work on children 3. Children were often employed in textile factories 4. The novel ‘Hard Times’ by Charles Dickens was a severe critique of the horrors of industrialization Select the correct answer using the code given below: [2016-II] (a) 1 and 3 only (b) 1 and 2 (c) 1, 3 and 4 (d) 3 and 4 only 21.Which of statements relating to the comptroller and Auditor General (CAG) of India is /are correct? 1. The CAG can attend the sitting of the Committee on public Accounts. 2. The CAG can attend the sittings of Lok Sabha and Rajya Sabha. 3. The jurisdiction of CAG is co-extensive with power of the Union Government. Select the correct answer using the code given below. [2016-I] (a) Only 1 (b) 2 and3 (c) 1 and 2 (d) All of these 22. The citizens of India do not have which of the following Fundamental Rights? (a) Right to reside and settle in any part of India. (b) Right to acquire, hold and dispose property (c) Right to practice any profession (d) Right to form co-operative societies 346 | P a g e

shop.ssbcrack.com

MCQs

INDIAN HISTORY

MCQs

23. Which one among the following Acts for first time allowed Indians, at theoretically, entry to higher posts in British Indian administration? [2016-I] (a) Charter Act, 1813 (b) Charter act, 1833 (c) Charles Wood’s Education Despatch, 1854 (d) Indian Councils Act, 1861 24. Consider the following statements about Rashtrakuta kings: 1. They were ardent patrons of Saivism and not support other forms of religion. 2. They promoted only Sanskrit scholars and gave them large at grants. Which of the statements given above is/are correct? (a) Only 1 (b) Only 2 (c) Both 1 and2 (d) Neither 1 nor 2

[2016-I]

25. Which of the following statements about the first act of the Revolt of 1857 is/are true? 1. It occurred in Meerut when two sepoys stole a superior officer’s rifle. 2. It began when the Rani of Janshi declared war on the British. 3. It began when Mangal pandey fired at a European officer in Meerut. 4. It began when Mangal pandey fired at a European officer in Barrackpore. Select the correct answer using the codes given below (a) Only 2 (b) 1,2 and 3 (c) Only 4 (d) 1 and 4

[2016-I]

26. Which of the following statements above Gandhiji’s Hind. Swaraj written in 1909 is/are true? 1. Hind Swaraj offers a civilization concept of the Indian nation. 2. Hind Swaraj States that parliament democracy was necessary for the amelioration of the sufferings Indians. 3. Hind Swaraj argues that industrial capitalism was responsible for the immorality of society. Select the correct answer using the codes given below. (a)only 3 (b) 1,2 and 3 (c)1 and3 (d) Only 1 27. Division of labour often involves 1. Specialized economic activity. 2. Highly distinct productive roles. 3. Involving everyone in many of the san activities. 4. Individuals engage in only sing activity and are dependent on others meet their various needs. Select the correct answer using the codes given below. [2016-I] (a) 1, and 4 (b) 1,2 and4 (c) Only (d) 2 and 4 28. Who among the following is / are associated with ‘Maharashtra Andhashraddha Nirmoolan Samiti? [2016-I] (a) Chandi Prasad Bhatt (b) Narendra Dabholkar (c) GD Agarwal (d) Kailash Satyarthi 29. Consider the following statements: 1. Warren Hastings setup the Calcutta Madrassa for the study and teaching of Muslim law related subjects. 2. Jonathan Duncan started a Sanskrit College at Varanasi for the study of Hindu law and philosophy. Which of the statements given above is/are correct? (a) Only 1 (b) Only 2 [2016-I] (c) Both 1 and 2 (d) Neither 1 nor 2

347 | P a g e

shop.ssbcrack.com

MCQs

INDIAN HISTORY

30. Consider the following statements about Sufism in India: 1. Pilgrimage called Ziyarat to tombs of Sufi saints is an important feature. 2. The most influential group of Sufis in India were the chishtis. Which of the statements given above is / are correct? (a) Only 1 (b) Only 2 (c) Both 1 and 2 (d) Neither 1 nor 2

MCQs

[2016-I]

31. Which of the following thinker’s movements influenced the development Gandhiji’s political ideas? [2016-I] 1. Henry David Thoreau 2. John Ruskin 3. John Milton 4. Jainism Select the correct answer using the codes given below. (a) Only 1 (b) 1,2 and (c) 1,2 and 4 (d) 2 and 4 32. Consider the following Statements: 1. The Arthashastra is the first Indian treatise to define a state. 2. The Main concerns of the Arthashastra are theoretical issues like the origin of the state. Which of the statements given above is / are correct? [2016-I] (a) Only 1 (b) Only 2 (c) Both 1 and 2 (d) Neither 1 nor 2 33. Which of the following statements about Brahmo Samaj is/are correct? [2016-I] 1. The Brahmo Marriage Act of 1872 allowed inter-caste and widow re-marriage only if the contracting parties declared themselves to be non-Hindus. 2. Keshab Chandra Sen arranged the marriage of his minor daughter with the Maharaja of Burdwan. 3. Keshab Chandra Sen’s followers broke away to form the naba bidhan Brahmo samaj. 4. The Brahmo Samaj grew from a small elite group to a mass movement in the 19th century. Select the correct answer using the codes given below. (a) Only 1 (b) 1 and 2 (c) 2 and 3 (d) All of these. 34. Which of the following statements about the aims of United Nations is /are true? 1. To foster a mutual appreciation of each other’s culture and literature among nations. 2. To achieve international co-operation in solving problems of an economic, social culture or humanitarian character. 3. To foster relations between scholars and academics in different countries. 4. To organize international countries. Select the correct answer using the codes given below. [2016-I] (a) Only 1 (b) 1,2 and 4 (c) Only 2 (d) 1,2 and 3 35. Consider the following statements: 1. The Constitution of India empowers the Parliament to form new States and to alter the areas, boundaries or names of existing States by passing a resolution in simple majority. 2. Jammu and Kashmir has been given special status under Article – 370 of the Constitution of India. Which of the statements given above is / are correct? [2016-I] (a) Only 1 (b) Only 2 348 | P a g e

shop.ssbcrack.com

MCQs

(c) Both 1 and 2

INDIAN HISTORY

MCQs

(d) Neither 1 nor 2

36. The Speaker of the Lok Sabha may resign his office by writing, addressed to the (a) President of India (b) Prime Minister of India (c) Vice – President of India (d) Deputy Speaker of the Lok Sabha 37. Who among the following is the founder of the ‘Bachpan Bachao Andolan’? (a) Shantha Sinha (b)Kailash Satyarthi (c)Aruna Roy (d) Anil Agarwal

[2016-I]

[2016-I]

38. The Election Commission recognizes a political party as a national party if [2016-I] 1. It secures at least 6% of the total valid votes polled in four or more states in a general election to the Lok Sabha or to the State Legislative Assemblies. 2. It wins at least four seats in a general election to the Lok Sabha from any State or States. Select the correct answer using the codes given below (a) Only 1 (b) Only 2 (c) Both 1 and 2 (d) Neither 1 nor 2 39. Which one of the following is not a feature of the Nehru-Mahalanobis model of development strategy? [2016-I] (a) Development of capital goods industries. (b) Major involvement of the State in the economy. (c) Industrial deregulation and disinvestment (d) Enhancing the scope and importance of the public in the public sector. sector. 40. Nabhah Sprsam Diptam is the motto of [2016-I] (a) Central industrial Security Force (b) Indian Air Force (c)ISRO (d) Indian Navy 41. Consider the following the Statements: 1. The Amendment procedure has been provided in Article-368 of the Constitution of India. 2. The consent of the States is mandatory for all Amendments to the Constitution of India. Which of the statements given above is/are correct? (a) Only 1 (b) Only 2 [2016-I] (c) Both 1 and 2 (d) Neither 1 nor 2 42. The word ‘Secular’ was inserted into the Constitution of India by (a) 44th Amendment Act (b) 52nd Amendment Act nd (c) 42 Amendment Act (d) 34th Amendment Act

[2015-II]

43. Which one of the following continents accounts for the maximum share in exports from India? (a) Axis (b) Europe [2015-II] (c) Africa (d) North America 44. The two provisions of the Constitution of India that most clearly express the power of judicial review are [2015-II] (a) Article 21 and Article 446 (b) Article 32 and Article 226 (c) Article 44 and Article 152 (d) Article 17 and Article 143 45. Which one of the following statements about Subsidiary Alliance devised by Lord Wellesley in the year 1728 is not correct? [2015-II] (a) The territories entering into a subsidiary alliance with the British were responsible for their own internal and external protection (b) In the territory of the ally, a British armed contingent would be stationed (c) The ally would have to provide the resources for maintaining the British contingent in the territory 349 | P a g e

shop.ssbcrack.com

MCQs

INDIAN HISTORY

MCQs

(d) The permission of the British was needed for the ally to enter into agreements with other rulers. 46. Who among the following was associated with the Mughal court as a physician to Prince Dara Shikoh? [2015-II] (a) Hakim Afzal Khan (b) Ibn Battuta (c) Francois Bernier (d) Duarte Barbosa 47. During the Eleventh Five Year plan, agriculture sector in India witnessed a growth rate of 3.3% per annum which is higher than 2.4% per annum in the previous Five Year plan. This is largely due to better performance of [2015-II] (a) Crops and livestock (b) Oil seeds and fibres (c) Fishing and oilseeds (d) Fibres and fishing 48. An emergency under Article-352 of the Constitution of India can be declared only during [2015-II] (a) war, external aggression or internal (b) war, external aggression or armed rebellion disturbance (c) failure of constitutional machinery in the (d) financial instability in the country State 49. Which of the following statements about the livestock sector in India is/are correct? 1. Livestock contributed about 25% of gross value added in agriculture 2. It provides self-employment to a large segment of population 3. Rapid growth of livestock sector can be egalitarian and inclusive. Select the correct answer using the code given below: (a) Only 3 (b) 1 and 2 (c) 2 and 3 (d) 1, 2 and 3

[2015-II]

50. Which one of the following statements about Khilafat Movement is not correct? [2015-II] (a) The Khilafat movement demanded that the Khalifat must retain control over Muslim holy places. (b) The radical trend in the khilafat movement was represented by younger leaders like Muhammad Ali, Shaukat Ali, and Maulana Azad. (c) Indian Muslim leaders used khilafat as a symbol that could unite the Indian Muslim community. (d) The Delhi conference of the Central Khilafat committee in 1920 decided to launch a massive Noncooperation Movement. 51.The first Indian satellite, Arya Bhatta, was launched in the year (a) 1972 (b) 1975 (c) 1976 (d) 1979

[2015-II]

52.The European Space Agency (ESA) successfully landed a spacecraft on a spending comet that lies 310 million miles away from Earth. As a result, the scientist may find out [2015-I] (a) the age of the sun (b) the age of the universe (c) new fundamental atomic particle (d) the role of the comets in bringing organic matter and water to Earth 53. The Second Five Year Plan that called for the establishment of Socialist pattern of Society was commonly referred to as the [2015-I] (a) Harrod-Domar Plan (b) Mahalanobis Plan (c) Nehru Plan (d) Peoples Plan 54. Which one of the following is the common element among Rajagriha, Vaishali and Patalipura? 350 | P a g e

shop.ssbcrack.com

MCQs

INDIAN HISTORY

MCQs

(a) Pali canon of the Sthaviravadins was (b) Ashokan Major Rock Edicts were found there compiled there [2015-I] (c) Places where Buddhist Councils were held (d) Places associated with the compilation of Buddhist canons of the Mahasanghikas 55. Which one of the following sects was associated with Gosala Maskariputra? [2015-I] (a) Vajrayana (b) Ajivikas (c) Sthaviravadins (d) (d) Mahasanghghikas 56. Consider the following statements: 1. China won both Thomas Cup and Uber Cup Badminton Tournaments, 2014. 2. The Uber Cup is the World Team Championship for women and the Thomas Cup is for men. Which of the statements given above is/are correct? (a) 1 only (b) 2 only (c) Both 1 and 2 (d) Neither 1 nor 2 57. Consider the following statements about Drishtisystem: 1. It is a sophisticated instrument to assess the runway visual range. 2. This system is developed by Airports Authority of India. Which of the statements given above is/are correct? (a) 1 only (b) 2 only (c) Both 1 and 2 (d) Neither 1 nor 2

[2014-II]

[2014-II]

58.Consider the following statements about Harappa Culture: 1. The Harappa Culture matured in Sind and Punjab. 2. It spread from there to Southwards and Eastwards 3. The area, where it spread, was bigger than Egypt and Mesopotamia. Which of the statements given above is/are correct? (a) 1 and 2 only (b) 2 and 3 only (c) 1, 2 and 3 (d) 3 only 59. Megasthenes was a (a) Greek ambassador to the court of (b) Greek trader during Asoka’s time Chandragupta Maurya (c) Greek trader in the Gupta period (d) Chinese pilgrim during Harsha's time 60. By the late 19th century, India was one of the largest producers and exporters of (a) Cotton yarn and wheat (b) Sugar and rice (c) Sugar and alcohol (d) Iron and steel

[2014-II]

[2014-II]

61. Which one of the following is the correct sequence of appearance of the poet-saints of the Bhakti-Sufi tradition? [2014-II] (a) Basavanna—Appar —Mira Bai —LalDed (b) Appar — Mira Bai- Lal Ded—Basavanna (c) Appar — Basavanna—LalDed—MiraBai (d) Basavanna — Mira Bai— Lal Ded—Appar 62. Who among the following did not bring trading ships to the port of Surat in pre-British times? (a) Portuguese and English (b) Russia and German (c) English and Arab (d) French and Arab

[2014-II]

63.The Factory Act of 1891 in India was enacted to [2014-II] (a) Improve the condition (b) Ensure greater governmental control over of labour in India industry in India. 351 | P a g e

shop.ssbcrack.com

MCQs

c) Provide a level playing field for English manufacturers

INDIAN HISTORY

MCQs

(d) Enable greater political control over Indian industry

64. Which of the following statements is correct? (a) The Presiding Officer of the Rajya Sabha is elected every year (b) The Presiding Officer of the Rajya Sabha is elected for a term of 5 years (c) The Presiding Officer of the Rajya Sabha is elected for a term of 6 years (d) The Vice President of India is the Ex Officio Presiding Officer of the Rajya Sabha

[2014-II]

65. Who among the following was not a member of the Drafting Committee of the Constitution of India? (a) B. R. Ambedkar (b) K. M. Munshi [2014-II] (c) Krishnaswamy lyer (d) M. K Gandhi 66.Who among the following was not a member of the 'Big Four' in the Congress of Vienna (1815)? (a) Great Britain (b) Russia [2014-II] (c) Austria (d) France 67.The Stamp Act Congress consisting of delegates from nine of the thirteen colonies met in 1765 at (a) Philadelphia (b) New York City [2014-II] (c) Boston (d) Providence 68. Which of the following American colonies did not attend the first Continental Congress held in Philadelphia? [2014-II] (a) Rhode Island (b) Connecticut (c) Georgia (d) Maryland 69. The Bolshevik Revolution started in Russia during the reign of (a) Czar Alexander I (b) Czar Alexander II (c) Czar Alexander III (d) Czar Nicholas II

[2014-II]

70.Which of the following is/are not related to fundamental duties? [2014-II] 1. To cherish and follow the noble ideals which inspired our national struggle for freedom 2. To value and preserve the rich heritage of our composite culture 3. To promote the educational and economic interests of the weaker sections of the people, especially the Scheduled Castes and Scheduled Tribes. 4. To protect all monuments of historic interest and national importance. Select the correct answer using the code given below. (a) 1 and 2 (b) 2 and 3 (c) 3 and 4 (d) 4 only 71. During the Civil Disobedience Movement, tributes in Chota Nagpur 1. Gave up opium consumption 2. WoreKhadi 3. Gave up drinking liquor and eating meat. Select the correct answer using the code given below. (a) 1 and 3 only (b) 1 only (c) 1, 2 and 3 (d) 2 and 3

[2014-I]

72. Which of the following was/were the new feature/feature s visible in the Quit India Movement (1942)? 1. Large-scale movements in some of the industrial areas. 2. Large-scale mass involvement in some of the Princely States 3. Increase in the activities of the Kisan Sabha 352 | P a g e

[2014-I]

shop.ssbcrack.com

MCQs

INDIAN HISTORY

Select the correct answer using the code given below. (a) 1, 2 and 3 (c) 1 and 3 only

MCQs

(b) 1 only (d) 3 only

73. The Financial Sector Legislative Reforms Commission report. Submitted recently, among others, contained which of the following proposals? [2014-I] 1. All regulators will have an empowered board, the role of each of the members has been defined. 2. The Commission envisages an important process of judicial review of the regulations. 3. The draft code does not mention about executive powers. Select the correct answer using the code given below. (a) 1 and 2 only (b) 2 only (c) 1 and 3 only (d) 1, 2 and 3 74. Which of the following statements are true for the Fundamental Right to Life and Personal Liberty as guaranteed under Article 21 of the Constitution of India? [2014-I] 1. The Right is available to citizens as well as aliens. 2. It covers protection against arbitrary executive and legislative action. 3. It includes the right to live with human dignity. 4. It can be taken away according to the procedure established by law. Select the correct answer using the code given below. (a) 1,2 and 3 only (b) 2,3 and 4 only (c) 1,2,3 and 4 (d) 1 and 4 only 75. Consider the following statements about 'Sahayog Kaijin': 1. It is a joint exercise of the Coast Guards of India and Japan. 2. Its 13th edition was held recently off Chennai Port. Which of the statements given above is/are correct? (a) 1 only (b) 2 only (c) Both 1 and 2 (d) Neither 1 nor 2 76. Consider the following statements about grand slam tennis tournaments: 1. There are four grand slam annual tennis tournaments. 2. The first grand slam of a year is the US Open. 3. The Australian and the US tournaments are played 00 grass court. Which of the statements given above is/are correct? (a) 1,2 and 3 (b) 1 and 2 only (c) 1 only (d) 2 and 3 only

[2014-I]

[2014-I]

77. The Constitution of India guarantees the Fundamental Right to Freedom of Religion to all its citizens. Which among the following is not true for this Rights? [2014-I] (a) It gives freedom of conscience and freedom to profess, (b) It gives freedom to establish and practice and propagate any religion maintain institution for religious and charitable purposes (c) The Right is subject to public order, morality and (d) The State cannot make any law which health abrogates this Right for citizens 78. The Scheduled Castes and Scheduled Tribes (Prevention of Atrocities) Act, 1989 may be seen as giving effect to which of the following Fundamental Rights? [2014-I] 1. Equality before the Law 2. Right against Discrimination 353 | P a g e

shop.ssbcrack.com

MCQs

INDIAN HISTORY

3. Abolition of Untouchability 4. Right to Freedom of Religion Select the correct answer using the code given below. (a) 1,2 and 3 only (c) 2,3 and 4 only

(b) 2 and 3 only (d) 1,2,3 and 4

79. The Indian National Congress was founded during the viceroyalty of (a) Lord Lytton (b) Lord Ripon (c) Lord Dufferin (d) Lord Mayo 80. Kandh Revolt against the British was led by (a) Chakra Bishoyi (c) Tomma Dora

MCQs

[2013-II]

[2013-II] (b) Dandasena (d) Birsa Munda

81. The implementation of factory legislation in India in the nineteenth century was a result of the pressure on the pressure on the Government of India from [2013-II] (a) the trade unionists in India (b) the socialists In India (c) the manufactures from Lancashire and (d) the Indian National Congress Manchester 82. Who among the following has described the uprising of 1857 as the First Indian war of Independence? [2013-II] (a) Jawaharlal Nehru (b) Vallabhbhai patel (c) Maulana Azad (d) Acharya Kripalani 83. Who among the following has described the uprising of 1857 as the First Indian War of Independence? [2013-II] (a) S.N.sen (b) R.C.Majumdar (c) B.G.Tilak (d) V.D.Savarkar 84. Arrange the following events of Indian freedom movement in correct sequence beginning from the earliest: 1. Second Round Table Conference 2. Common Award 3. Poona Pact 4. Simon Commission Select the correct answer using the code given below: [2013-II] (a) 1-2-3-4 (b) 4-3-2-1 (c) 4-1-2-3 (d) 3-2-1-4 85. Consider the following statements: [2013-II] 1. Morley-Minto reforms ended the official majority in state legislative assemblies. 2. Despite the Government of Indian Act, 1909 the non-officials were elected to local bodies indirectly. Which of the statements given above is/are correct? (a) 1 only (b) 2 only (c)Both 1 and 2 (d)Neither 1 nor 2

354 | P a g e

shop.ssbcrack.com

MCQs

INDIAN HISTORY

MCQs

NDA PREVIOUS YEAR QUESTIONS – SOLUTION 1. (a) Both the statements are individually true and Statement II is the correct explanation of Statement I 2. (a) Both the statements are individually true and Statement II is the correct explanation of Statement I 3. (a) Edmund Cartwright invented the power loom that revolutionized the cotton textile industry 4. (a) Subsidiary Alliance was a system devised by Lord Wellesley, this is a system of ruling a dominated nation. 5. (a) Periplus is a Greek word meaning sailing around. 6. (b) Feature(s) of Lenin’s New Economic Policy (NEP) for the Soviet Union: Private enterprise was strictly forbidden and Peasants were not allowed to sell their surplus 7. (c) The British authorities failed to rally the loyalist Americans and the Americans benefited from the extraordinary military leadership of George Washington were the reasons for the defeat of British during the American War of Independence. 8. (a) British colonialism continued to grow steadily in the 18th and 19th centuries 9. (d) Dinabandhu Mitra was the author of Neel Darpan, published in 1860 10. (d) Satyashodhak Samaj was started by Jyotirao Phule in 1873 11. (b) Although used earlier by French and German writers, the term ‘Industrial Revolution’ in English was first popularized by Arnold Toynbee. 12. (c) In the 19th century, the majority of the workers in Japan’s modem industries were mainly women. 13. (c) The given both statements are true regarding to the constitution of India. 14. (c) The Ministers are appointed by the President on the advice of the Prime Minister. 15. (d) The Speaker of the Lok Sabha finally decides if it is a Money Bill, should any dispute about it arise 16. (a) The power of advising the President as regards the appointment of other Ministers. 17. (d) The cylindrical stone seals were used in Mesopotamian civilization. 18. (d) The given all statements are true to the Civil Disobedience movement. 355 | P a g e

shop.ssbcrack.com

MCQs

INDIAN HISTORY

MCQs

19. (b) The features of the ideology of utilitarianism are the most celebrated spokesmen of utilitarianism were Jeremy Bentham and John Stuart Mill, Utilitarians were advocates of the idea of the ‘greatest good for the greatest number of people’. 20. (c) The correct statements about The industrial revolution in England had a profound impact on the lives of people: Women were the main workers in the silk, lacemaking and knitting industries, Children were often employed in textile factories and the novel ‘Hard Times’ by Charles Dickens was a severe critique of the horrors of industrialization. 21. (a) The CAG is an authority established by Constitution of India under Article-148 to audit all receipts and expenditure of Central and State Governments. CAG assists the public account committee during the examination of audit report of CAG. Further CAG cannot attend the sittings of Lok Sabha and Rajya Sabha. 22. (b) Article -300A in Part-XII deals with right property and deems is to be a legal right rather than Fundamental Right. Originally, the right to property was one of the seven Fundamental Rights &dealt by articles-19(1) (f) and Article-1. Article-19(1) (f) guaranteed to every citizen the right acquire, hold and dispose property. The 44th Amendment Act abolished this right as Fundamental Right. 23. (b) The Charter Act of 18 was the first act which made provision to freely admit the natives of India to share administration in the country. This act also provided that Haile bury College of London should make quota to admit future civil servants. 24. (d) The Rashtrakuta kings were known for their religious tolerance, however, they were more inclined towards Jainism. Many of the scholars who flourished in the court of Rashtrakuta kings wrote in Sanskrit, Kannada, and Apobharamasha and Prakrit languages. 25. (c) Mangal pandey played a key role in events that precede the outbreak of revolt of 187. On March 29, 187, Mangal Pandey fired at a European officer in Barrackpore. 26. (c) Hind Swaraj was written by Gandhiji in 1909 to express his views on Swaraj, modern and industrialization etc. he was in favour for independence not just from British but also from modern civilization. He thought that parliament is simply a costly toy and was not in favour of parliamentary democracy. He was also against industrial capitalism. 27. (b) Divisionof labour is an economic concept that states that dividingproduction process into different stages enables workers to forces on specific tasks. If workers concentration one small aspect of production, their overall efficiency increases. 28. (b) Narendra Dabholkar, a rationalist, founded the Maharashtra Andhashraddha Nirmoolan Samiti in 198. He actively campaigned against superstitions and confronted dubious tantriks. Dabholkar was murdered on August 20, 2013. 29. (c) Madrasah-i-Aliah or Calcutta Madrassa was the first educational institution set up in India in 1780 by Warren Hastings. In 1791 Jonathan Duncan started the Sanskrit College at Varanasi for the study of Hindu law and philosophy. 356 | P a g e

shop.ssbcrack.com

MCQs

INDIAN HISTORY

MCQs

30. (c) One of the most popular rituals in Sufism is visiting of grave-tombs of Sufi saints. This ritual of visiting tombs is called Ziyarart. 31. (c) Gandhiji was greatly influenced by thinkers like John Ruskin, Leo Tolstoy, and Henry David Thoreau. John Ruskin’s Book ‘Undo his last’ had a lasting impact on Gandhiji’s psyche. He paraphrased the book as ‘Sarvodaya’. Gandhi also was influenced by non-violence principle preached by Jainism. 32. (a) The Arthashastra written in Sanskrit was the first Indian work that formally defined state. It is a practical treatise on statecraft, economic policy and military strategy. Its main concern is not deal with theoretical issues like origin of state. 33. (a) The Brahmo Marriage Act was passed in 1872 it allowed inter-caste and widow re-marriage only if the contracting parties declared themselves to be non-Hindus. Keshab Chandra Sen arranged the marriage of his minor daughter with the Maharaja of Cooch Behar. His followers formed Sadhaman Brahmo Samaj. 34. (c) The aims of United Nation Organization includes maintaining international peace and security, developing friendly relations among nations, achieving international co-operation in solving problems of an economic, social culture or humanitarian character and to be a centre for harmonizing actions of nations towards these common ends. 35. (c) Parliament can form new states, alter the area boundaries or names of existing States by passing a law by simple majority. This bill can be introduced in either house on recommendation of the President. Article370 grants special autonomous Status to Jammu and Kashmir. It is a part – XXI of the Constitution. 36. (d) The speaker may resign from office by writing under his hand to the Deputy Speaker. 37. (b) Bachapan Bachao Andolan works in the field of rights of children. It was started in 1980 by Nobel Laureate Kailash Satyarthi. The focus is on ending bonded labour, child labour and human trafficking. 38. (c) A political party is recognized as a national party if it secures at least 6% of valid votes polled in any four or more States. Further it wins at least 4 seats in the house of people from any State or States or wins at least 2% seats in house of the people. 39. (c) The Nehru-mahalanobis model was followed during Second Five year plan. The essence of this model was a shift in the pattern of industrial investment towards building up a domestic consumption goods sector. It was in favour of enhancing public investment and was against industrial deregulation and disinvestment. 40. (b) Nabhah Sprsam Diptum (Touch the sky with glory) is the motto of Indian Air Force. This has taken from eleventh chapter of the Gito, the discourse given by Lord Krishna to Arjuna during the war by Mahabharata. 41. (a) Article-368 of the Indian Constitution lays down the procedure for Amendment of the Constitution. This article provides for two types of amendments 357 | P a g e shop.ssbcrack.com

MCQs

INDIAN HISTORY

MCQs

(i) By a special majority of Parliament (ii) Through ratification of half of the States by a simple majority. Besides, some other articles also provide for constitutional amendment by simple majority of Parliament and no consent of States are required. 42. (c) 44th Amendment Act as was enacted to nullify some of the amendments made by 42nd Amendment Act, 1976. Most of the changes were with regard to declaration on emergency under Article ‘352’. Right to property was deleted from the list of Fundamental Right. It is now only a legal right under the constitution. 52nd Amendment Act added Anti-defection law and the 10th schedule to Indian Constitution. 34th Amendment Act added 9th schedule to Indian Constitution 43. (a) Share of export from India in 2013-2014 Asia 49.6% Europe 18.6% America 17.3% Africa 9.9% 44. (b) The power of judiciary to review and determine the validity of a law or an order may be described as the powers of judicial review. It means that the Constitution is the supreme law of the land and any law consistent therewith is void through judicial review. 45. (a) The Subsidiary Alliance system was used by Wellesley to bring Indian states within the orbit of British power. The system played a very important part in the expansion of the Company’s dominions and many new territories were added to the Company’s possession. 46. (c) Francois Bernier was a French physician and traveler. He was born at Joue - Etiau in Anjou. He was briefly personal physician to Prince Dara Shikoh (28 October 1615-30 August 1659), the elder son of Shah Jahan, and after Dara Shikoh’s fall, he was attached to the court of the Emperor Aurangzeb. 47. (a) 11th plan had some success in reversing the declaration of agriculture growth witnessed during the 9 th and 10th plan but food inflation still remains a major concern. The growth in agriculture in the 11th plan is likely to be around 3.2% year, which is higher than 10th plan growth rate but lower than the target (4.0%) for 11th plan. 48. (b) When a national emergency is declared on the ground of war or external aggression, it is called ‘External Emergency’ and when it is declared on the ground of armed rebellion, it is termed as ‘Internal Emergency’. It should be noted here that the President can declare a national emergency even before that actual occurrence of war or external aggression or armed rebellion, if he is satisfied that there is an imminent danger. 49. (d) Livestock production performance has been more impressive than that of food grain production. Milk, egg, meat, and fish showed impressive growth rates of 5 to 10%. The minimum targeted growth rate for attaining self-sufficiently in milk, fish, meet, and egg by 2001 AD are 5.54%, 6.25 and 5.54% per annum

358 | P a g e

shop.ssbcrack.com

MCQs

INDIAN HISTORY

MCQs

respectively. It provides alternate source of income to small and marginal farmers. Hence, livestock growth leads to egalitarian and inclusive growth. 50. (d) The leaders of the Khilafat Movement joined hands with Indian National Congress for the upcoming Noncooperation Movement. Again March 19, 1920 was observed as Khilafat Day and following that there was an all party conference in June 1920 at Allahabad. The agenda of the Non-cooperation Movement was finalized. 51. (b) Aryabhatta Satellite was launched on 19 th April 1975, with weight of 360 kg. It was launched by Soviet Intercosmos rocket. The objectives of this project were to indigenously design and fabricate a spaceworthy satellite system and evaluate its performance in orbit. 52. (d) The Scientific goals of tye mission focus on “elemental isotopic, molecular and mineral, the characterization of physical properties of the surface and subssurface material. 53. (b) The plan was particularly in the development of the public sector, The plan followed the Mahalanobis model. 54. (c) The First Buddhist council immediately following the death of the Buddha and the second Buddha council in the region of Ashoka occurred in patlipura and Rajgira. 55.(b) Makkhali Gosala as the fou der of the Ajivika sect; other sources state that Gosala was only a leader of a large Ajivika congregation of ascetics, bjut not the founder of the movement himself. 56. (b) Japan won the Thomas Cup 2014 of badminton 2014. China won Uber Cup 2014 of badminton. Thomas Cup is for men and Uber Cup is related to Women. 57. (a) 58. (d) Harappan culture developed and matured in to man urban civilization that developed in Sind and Punjab. From there it spread southwards Southwards and estwards. lts area was 1,299,600 sq km, which is a larger area than that of Pakistan and certainly larger than Egypt and Mesopotamia. 59. (a) Megasthenes was a Greek ambassador to the court of Chandragupta Maurya. He was also the author of the book India. 60. (a) By the late 19th century India was one of the largest producers and exporters of Cotton Yarn and wheat. The export of Indian wheat progressed after Suez Canal opened in 1869. About 17% of India's wheat was exported by 1890s. 61. (c) The correct sequence isAppar- 7th century CE Basavanna- 1134- 1196 CE Lai Ded- 1320-1392 CE Mira Bai- 1498- 1557 CE 62. (b) Russian and German did not bring trading ships to the port of Surat in pre-British times. 359 | P a g e

shop.ssbcrack.com

MCQs

INDIAN HISTORY

MCQs

63. (a) The factory Act of 1891 in India was enacted to improve the condition of labour in India. It provided some specific rules and regulations including-the age of child labour to be established from nine to twelve, child labour could work maximum of six hours, women were given half given half an hour break etc. 64. (a) 64. (d) The Drafting Committee for framing the constitution was appointed on 29th August 1947.The committee comprised of a chairman and six other members. The committee members were:- Dr B. R. Ambedkar(Chairman), K M Munshi, Alladi Krishnaswamy Iyer, N Gopalaswami Ayengar, B L Mitter, Md. Saadullah and D P Khaitan 66. (d) The Congress of Vienna was a conference or ambassadors of European states, and held in Vienna from September 1814 to June 1815. The goal was not simply to restore old boundaries, but to resize the main powers so they could balance each other off and remain at peace. The "Big Four" members included the "Big Four" and France are Austria, England, Prussia and Russia. Later on, France was also invited to join. 67. (b) The Stamp Act Congress met in a Federal I fall building was a meeting held between October 7 and 25, 1765 in New York City. Fit was the first gathering of elected representatives from several of the American colonies to devise a unified protest against new British taxation. 68. (c) The first Continental Congress met in Carpenter's Hall in Philadelphia, from September 5, to October 26, 1774, all of the colonies except Georgia sent delegates. Countries which sent their delegates are New Hampshire, Massachusetts Bay, Rhode Island. Connecticut, New York, New Jersey, Pennsylvania, Delaware, Maryland, Virginia, North and South Carolina. 69. (d) The Bolshevik Revolution started in Russia during the reign of Czar Nicholas 11(1894-1917). Nicholas 11 was the last Emperor and the last Czar of Russia. It was executed along with his family by the Bolsheviks. 70. (c) Article 51(a), Part IV (a) of the Indian Constitution, specifies the list of fundamental duties of the citizens. According to it "to promote the educational and economic interests of the weaker sections of the people, especially the Scheduled castes and Scheduled Tribes" and "To protect all monuments of historic interests and national importance" are not fundamental duties. 71. (c) During the Civil Disobedience Movement. Tribal in Chota Nagpur wore Khadi gave up opium consumption. Drinking liquor and eating meat. 72. (a) On 8 August 1942 at the All-India Congress Committee session in Bombay, Mohandas Karamchand Gandhi launched the 'Quit India' movement. It was a civil disobedience movement. 73. (a) Option (a) is the correct choice. 74. (c) Option (c) is the correct answer. 75. (a)

360 | P a g e

shop.ssbcrack.com

MCQs

INDIAN HISTORY

MCQs

The 13th edition of joint exercise by the Indian Coast Guard and its Japanese counterpart held approximately at 20 nautical miles off Kochi. 76. (c) The Grand Slam itinerary consists of the Australian Open (mid-January). The French Open (May/June). Wimbledon (June/July) and the US Open (August/September.). First grand slam of the year is Australian Open. 77. (d) The state can make laws which abrogate the right in case there is a threat to public order, morality and health. 78. (a) The Scheduled Castes and Tribes (Prevention of Atrocities) Act. 1989, an Act of the Parliament of India enacted to present atrocities against scheduled castes and scheduled tribes. It extends to the whole of India except the State of Jammu & Kashmir. It may be seen as giving effect to equality before law, right against discrimination and abolition of Untouchability. Freedom of religion in India is a fundamental right guaranteed by Article 15 to Article 25 of the Constitution of India. 79. (c) Lord Duffer in was the viceroy of India when the Indian National Congress was founded in 1885. 80. (a) In 1840, Kandh uprising broke out against British efforts to put an end to Kandh’s practice of human sacrifice. This revolt was led by Chakra Bishoyi. 81. (a) 82. (d) J B Kripalani, popularly known as Acharya Kripalani, was an Indian politician, noted particularly for holding the presidency of the Indian National Congress during the transfer of power in, 1947. 83. (d) The term ‘First War of Independence’ was first used by Vinayak Damodar Savarkar in his 1909 book ‘The History of the War of Indian Independece’ which was originally written in Marathi. 84. (c) Simon Commission-November, 1927 Second Round Table Conference-September, 1931 Communal Award-August, 1932 Poona Pact-September, 1932 85. (b)

361 | P a g e

shop.ssbcrack.com

MCQs

INDIAN HISTORY

MCQs

PREVIOUS YEAR QUESTIONS ASKED IN AFCAT EXAMINATION 1. Todar Mal was associated with _______ in Akbar's durbar. (a) Music (b) Literature (c) Finance (d) Law Reforms 2. Mahatma Gandhi left South Africa to return to India in (a) 1911 (b) 1915 (c) 1917 (d) 1919

[2016-II]

[2016-II]

3. Who was the Viceroy when the Simon Commission visited India? [2016-II] (a) Lord Chelmsford (b) Lord Reading (c) Lord Irwin (d) Lord Wellington 4. Think-tank of Government of India that replaced Planning Commission on 01 January 2015 is [2016-II] (a) NITI Vakya (b) NITI Dharma (c) NITI Vajpayee (d) NITI Aayog 5. What is the name of 'Innovative Council' proposed in the Rail Budget 2015 to promote innovation? (a) Rail Tech (b) Sankalp [2016-II] (c) Kayakalp (d) Navachar 6. Who was the person behind conversion of East India Company from a trading company into a regional power? [2016-1] (a) Lord Warren (b) Lord Clive Hastings (c) Lord Dalhousie (d) Lord Wellesley 7. Who was the Viceroy when the Simon Commission visited India? (a) Lord Chelmsford (b) Lord Reading (c) Lord Irwin (d) Lord Wellington 8. Mahatma Gandhi left South Africa to return to India in (a) 1911 (b) 1915 (c) 1917 (d) 1919

[2016-1]

[2016-1]

9. The Battle of Plassey was fought in (a) 1757 (b) 1782 (c) 1748 (d) 1764

[2016-1]

10. Mangalyaan, Mars Orbiter Mission (MOM) was launched in (a) Jan 2013 (b) Oct 2012 (c) Oct 2013 (d) Nov 2013 11. The Indian space program began in (a) 1961 (b) 1962 (c) 1965 (d) 1 969

[2016-1]

[2016-1]

12. The language of discourses of Gautama Buddha was [2016-1] (a) Bhojpuri (b) Magadhi (c) Pali (d) Sanskrit 13. Who among the following was the Congress President at Madras Session of 1927 when it boycotted the Simon Commission? [2015-I] 362 | P a g e

shop.ssbcrack.com

MCQs

INDIAN HISTORY

MCQs

(a) Maulana Abul (b) MA Ansari Kalam Azad (c) Lala Lajpat Rai (d) Subhash Chandra Bose 14. Why did Kalinga prove to be a turning point in the life of Ashoka? (a) Ashoka annexed (b) It was the starting point of the expansion of his empire Kalinga (c) Ashoka became a (d) It enabled Mauryan Empire to reach its climax. zealous Buddhist 15. Alauddin Khilji did not build (a) Siri Fort (b) Tomb of Jalaluddin (c) Hauz-i-Alai (d) Jamaat Khana Masjid

[2015-I]

[2015-I]

16. Which of the following dynasties was ruling over North India at the time of Alexander's invasion? (a) Nanda (b) Maurya [2015-I] (c) Sunga (d) Kanva 17. Which of the following rulers had the title 'Kaviraja' (a) Kumaragupta (b) Chandragupta (c) Skandagupta (d) Samudragupta

[2014-II]

18. The Viceroy who divided Bengal by following the divide and rule policy was: (a) Lord Curzon (b) Lord Ripon (c) Lord Lytton (d) Lord Mayo

[2014-II]

19. Who founded the Asiatic Society of Bengal in Kolkata? (a) Warren Hastings (b) John Shore (c) Sir William jones (d) Lord Cornwallis

[2014-II]

20. Which theory gave birth to the French Revolution and the Revolution in America? (a) Legal theory of (b) Theory of rights natural rights (c) Social welfare (d) Historical theory theory of rights 21. Who was the Governor-General of India during the ‘Sepoy Mutiny’? (a) Lord Dalhousie (b) Lord Harding (c) Lord Canning (d) Lord Lytton

[2014-II]

[2014-I]

22. Who among the following has been called the ‘Napoleon of India’? (a) Ashoka (b) Samudragupta (c) Chandragupta (d) Harshavardhana

[2014-I]

23. Who is known as the ‘Grand Old Man of India’? (a) Dadabhai Naoroji (b) Gopal Krishna Gokhale (c) Bal Gangadhar Tilak (d) A.O. Hume

[2014-I]

24. Kalhana’s ‘Rajatarangini’ is a history of (a) Kashmir (b) Harsha’s reign (c) Rajasthan (d) Chandragupta’s reign 363 | P a g e

[2014-I]

shop.ssbcrack.com

MCQs

INDIAN HISTORY

MCQs

25. Who accorded the title ‘Mahatma” to MK Gandhi? (a) Sardar Patel (b) Nehru (c) Sarojini Naidu (d) Rabindranath Tagore 26. The Non co-operation Movement started in which year? (a) 1900 (b) 1921 (c) 1940 (d) 1935

[2013-I]

27. During the Middle Ages education was confined only to (a) Kshatriyas (b) Brahmins (c) Peasants (d) Shudras

[2013-I]

28. Who among the following was responsible for the spread of Buddhism in Sri Lanka? (a) Ashoka (b) Mahavira (c) Parsavanth (d) Chandra Gupta Maurya

[2013-I]

[2013-I]

29. Who among the following had discovered the Bramhi Script in 1838? (a) Sir William (b) Dr. Rajendra Lal Jones Mitra (c) Dr. Bhaw Dagi (d) Jones Prinsep

[2012-II]

30. Who was known as father of administration in medieval India? (a) Akbar (b) Sher Shah Suri (c) Humayun (d) Aurangzeb

[2012-II]

31. Brahma Samaj was founded by (a) Raja Rammohan (b) Jawaharlal Roy Nehru (c) William Carey (d) Jonathan Duncan 32. The person who is regarded as the greatest law giver of ancient India is (a) Panini (b) Kautilya (c) Manu (d) Patanjali 33. The immortal fame of Ashoka largely rests upon (a) His conversion to (b) His policy for the welfare of his subjects. Buddhism and its propagation. (c) His work in the (d) His extensive conquests. sphere of politics and moral teaching. 34. Gandhiji started Dandi March (a) To demonstrate (b) To break the against the British salt law. Empire. (c) To boycott foreign (d) None of the goods. above. 35. The first war of Indian Independence began on 10 May 1857 at (a) Meerut (b) Jhansi 364 | P a g e

[2012-I]

[2012-I]

[2012-I]

[2012-I] shop.ssbcrack.com

MCQs

(c) Barrackpore

INDIAN HISTORY

MCQs

(d) Delhi

36. The famous Grand Trunk (GT) Road from Peshawar to Kolkata was built by (a) Akbar (b) Ashok (c) Sher Shah Suri (d) Chandragupta

[2011-II]

37. The Ashoka Pillar whose Lion Capitol (Carving) was adopted by the Government of India as National Emblem is situated at [2011-II] (a) Varanasi (b) Puri (c) Prayag (d) Sarnath 38. The First Battle of Panipat was fought between (a) Akbar & Hemu (b) Babur and Ibrahim Lodhi (c) Akbar & Rana (d) Ahmad Shah Abdali Sanga & Marathas

[2011-I]

39. Fa-hien visited India during the reign of (a) Chandra Gupta (b) Bindusara Maurya (c) Chandra Gupta II (d) Bimbisara

[2011-I]

40. Gandhiji’s first experience with Satyagraha came up in (a) Dandi (b) Champaran (c) Bengal (d) Natal

[2011-I]

41. During whose tenure as the viceroy of India were the great Martyrs Bhagat Singh, Sukhdev and Rajguru hanged? [2011-I] (a) Lord Curzon (b) Lord Irwin (c) Lord Minto (d) Lord Chelmsford

365 | P a g e

shop.ssbcrack.com

MCQs

INDIAN HISTORY

MCQs

AFCAT PREVIOUS YEAR QUESTIONS – SOLUTION 1. (c) Raja Todar Mal was the finance minister of the Mughal Empire during Akbar's reign. 2. (b) Mahatma Gandhi returned to India from South Africa in 1915. 3. (c) Lord Irwin was the viceroy of India when Simon Commission visited India. 4. (d) NITI Aayog is the Government of India think tank established by PM Narendra Modi to replace Planning Commission. 5. (c) 'Kayakalp' is the innovative council of Indian Railways for innovation, technology development and manufacturing. 6. (b) Lord Clive converted the East India Company into regional power after winning the battle of Plassey. 7. (c) The Simon commission was established under the guidance of the then vice president Lord Irwin by the council of India. 8. (c) Mahatma Gandhi returned to India from South Africa permanently in 1917. 9. (a) The Battle of Plassey, 23 June 1757, was a decisive British East India Company victory over the Nawab of Bengal and his French allies, establishing Company rule in South Asia which expanded over much of the Indies for the next 190 years. The battle took place at Palashi, Bengal, on the river banks of the Bhagirathi River, about 150 km north of Calcutta, near Murshidabad, then capital of undivided Bengal. The belligerents were Siraj-ud-daulah, the last independent Nawab of Bengal, and the British East India Company. 10. (d) Mangalayan. Mars Orbit Mission was launched on 5 November 2013 by the Indian Space Research Organisation (ISRO). 11. (b) Indian Space programmes began in 1962. 12. (c) The language of discourses of Gautama Buddha was Pali. 13. (b) M. A. Ansari was the Congress President at the Madras session of 1927, when it boycotted the Simon Commission. 14. (c) Ashoka invaded Kalinga in 261 B. C. In this war more than 2 lakh people died, wounded and made prisoners in war. Such a huge carnage and massacre of human lives and the sufferings of the wounded made a deep impression on Ashoka's mind. So, he decided to spare his life to the spread of Buddhism around the world. 15. (d) The Jama 'at-Khana-Masjid or Khilji mosque was built in 1325 by Khizr Khan, son of Alauddin Khilji while all the other monuments were built by Allauddin Khilji. 16. (a) During the invasion Alexander, Nanda dynasty was ruling the North India in the Magadha Empire. 17. (d) Samudragupta, ruler of the Gupta Empire (c. 335 - c. 375 CE), and successor to Chandragupta I, is considered to be one of the greatest military geniuses in Indian history. He was the third ruler of the Gupta Dynasty, who ushered in the Golden Age of India. His title of Kaviraja (King of poets) is justified by various poetical compositions. 18. (a) The decision to effect the Partition of Bengal was announced in July 1905 by the Viceroy of India, Lord Curzon. The partition took place in October 1905 and separated the largely Muslim eastern areas from the largely Hindu western areas. 19. (c) Sir William jones founded the Asiatic Society of Bengal in Kolkata on 15th January 1784. 366 | P a g e

shop.ssbcrack.com

MCQs

INDIAN HISTORY

MCQs

20. (b) Theory of natural rights gave birth to the French Revolution and the Revolution in America. 21. (c) A major cause of resentment that arose ten months prior to the outbreak of the Rebellion was the General Service Enlistment Act of 25 July 1856. As noted above, men of the Bengal Army had been exempted from overseas service. Specifically, they were enlisted only for service in territories to which they could march. Governor-General Lord Dalhousie saw this as an anomaly, since all sepoys of the Madras and Bombay Armies and the six "General Service" battalions of the Bengal Army had accepted an obligation to serve overseas if required. As signed into effect by Lord Canning, Dalhousie's successor as Governor-General, the Act required only new recruits to the Bengal Army to accept a commitment for general service. However, serving high-caste sepoys were fearful that it would be eventually extended to them, as well as preventing sons following fathers into an Army with a strong tradition of family service. 22. (b) Samudragupta, ruler of the Gupta Empire (c. 335 - c. 375 CE), and successor to Chandragupta I, is considered to be one of the greatest military geniuses in Indian history. He was the third ruler of the Gupta Dynasty, who ushered in the Golden Age of India. He was perhaps the greatest king of Gupta dynasty. He was a benevolent ruler, a great warrior and a patron of arts. His name appears in the Javanese text `Tantrikamandaka'. 23. (a) Dadabhai Naoroji (4 September 1825 - 30 June 1917), known as the Grand Old Man of India, was a Parsi intellectual, educator, cotton trader, and an early Indian political and social leader. He was a Member of Parliament (MP) in the United Kingdom House of Commons between 1892 and 1895, and the first Asian to be a British MP. Naoroji is also credited with the founding of the Indian National Congress, along with A.O. Hume and Dinshaw Edulji Wacha. His book Poverty and UnBritish Rule in India brought attention to the draining of India's wealth into Britain. 24. (a) Rajatarangini is a metrical historical chronicle of northwestern Indian subcontinent, particularly the kings of Kashmir, written in Sanskrit by Kashmiri Brahman Kalhana in 12th century CE. The work generally records the heritage of Kashmir, but 120 verses of Rajatarangini describe the misrule prevailing in Kashmir during the reign of King Kalash, son of King Ananta Deva of Kashmir. Although the earlier books are inaccurate in their chronology, they still provide an invaluable source of information about early Kashmir and its neighbors in the north-western parts of the Indian subcontinent, and are widely referenced by later historians and ethnographers. 25. (d) Rabindranath Tagore bestowed the title 'mahatma' to M.K. Gandhi. 26. (b) The Non-cooperation movement was a significant phase of the Indian struggle for freedom from British rule. It was led by Mohandas Gandhi and was supported by the Indian National Congress. 27. (b) In ancient India, during the Vedic period from about 1500 BC to 600 BC, most education was based on the Veda (hymns, formulas, and incantations, recited or chanted by priests of a pre-Hindu tradition) and later Hindu texts and scriptures. Education, at first freely available in Vedic society, became over time more discriminatory as the caste system, originally based on occupation, evolved, with the Brahman (priests) being the most privileged of the castes. 28. (a) Ashoka sent his only daughter Sanghamitra and son Mahindra to spread Buddhism in Sri Lanka (then known as Tamraparni). As a Buddhist emperor, Ashoka believed that Buddhism is beneficial for all human beings as well as animals and plants, so he built a number of stupas, Sangharama, viharas, chaitya, and residences for Buddhist monks all over South Asia and Central Asia. 29. (d) The script was deciphered in 1837 by Jones Prinsep, an archaeologist, philologist, and official of the British East India Company. 30. (a) Akbar was known as father of administration in medieval India. 367 | P a g e

shop.ssbcrack.com

MCQs

INDIAN HISTORY

MCQs

31. (a) Brahma Samaj is the societal component of Brahmoism, a monotheistic reformist and renaissance movement of Hindu religion. It was started at Calcutta on 20 August 1828 by Raja Ram Mohan Roy and Debendranath Tagore. 32. (c) The person who is regarded as the greatest law giver of ancient India is Manu. 33. (a) Ashoka converted gradually to Buddhism beginning about 263 BCE at the latest. He was later dedicated to the propagation of Buddhism across Asia, and established monuments marking several significant sites in the life of Gautama Buddha. 34. (b) The Salt March, also mainly known as the Salt Satyagraha, began with the Dandi March on 12 March 1930. It was a direct-action campaign of tax resistance and nonviolent protest against the British salt monopoly in colonial India, and triggered the wider Civil Disobedience Movement. 35. (a) The Indian Rebellion of 1857 began as a mutiny of sepoys of the East India Company's army on 10 May 1857, in the cantonment of the town of Meerut, Uttar Pradesh. 36. (c) The Grand Trunk Road is one of Asia's oldest and longest major roads. For more than two millennia, it has linked the eastern and western regions of the Indian subcontinent, connecting South Asia with Central Asia. It runs from Chittagong, Bangladesh west to Howrah, West Bengal in India, across north India into Peshawar, up to Kabul, Afghanistan. The predecessor of the modern road was rebuilt by Sher Shah Suri, who renovated and extended the ancient Mauryan route in the 16th century. 37. (d) The Lion Capital of Ashoka is a sculpture of four Indian lions standing back to back, on an elaborate base that includes other animals. A graphic representation of it was adopted as the official Emblem of India in 1950. It was originally placed atop the Ashoka pillar at the important Buddhist site of Sarnath by the Emperor Ashoka, in about 250 BCE. 38. (b) The First Battle of Panipat, on 21 April 1526, was fought between the invading forces of Babur and the Lodi Empire. It took place in north India and marked the beginning of the Mughal Empire. This was one of the earliest battles involving gunpowder firearms and field artillery. Ibrahim Lodi died on the field of battle along with 15,000 of his troops. 39. (c) Fahien visited India in the early fifth century AD. He is said to have walked all the way from China across icy desert and rugged mountain passes. Fahien's visit to India occurred during the reign of Chandragupta II. 40. (b) The first Satyagraha revolutions inspired by Mahatma Gandhi in the Indian Independence Movement occurred in Champaran district of Bihar on 1916. 41. (b) During Lord Irwin, as the viceroy of India were the great Martyrs Bhagat Singh, Sukhdev and Rajguru hanged.

368 | P a g e

shop.ssbcrack.com

More Documents from "Vipendra Singh"